17.07.2024

Как Π½Π°ΠΉΡ‚ΠΈ сопротивлСниС ΠΊΠΎΠ½Ρ‚ΡƒΡ€Π°: Π­Π»Π΅ΠΊΡ‚Ρ€ΠΎΠΌΠ°Π³Π½ΠΈΡ‚Π½Ρ‹Π΅ колСбания — ΠΌΠ°Ρ‚Π΅Ρ€ΠΈΠ°Π»Ρ‹ для ΠΏΠΎΠ΄Π³ΠΎΡ‚ΠΎΠ²ΠΊΠΈ ΠΊ Π•Π“Π­ ΠΏΠΎ Π€ΠΈΠ·ΠΈΠΊΠ΅

Π‘ΠΎΠ΄Π΅Ρ€ΠΆΠ°Π½ΠΈΠ΅

Π Π•Π—ΠžΠΠΠΠ‘ΠΠ«Π• Π―Π’Π›Π•ΠΠ˜Π―, ΠšΠžΠ›Π•Π‘ΠΠ’Π•Π›Π¬ΠΠ«Π™ КОНВУР

Π’Π•ΠžΠ Π˜Π―: ΠŸΠžΠΠ•ΠœΠΠžΠ“Π£ — ΠžΠ‘Πž Π’Π‘Π•Πœ

Β Β Β Β Β Β Β Β 1.8. РСзонансныС явлСния. ΠšΠΎΠ»Π΅Π±Π°Ρ‚Π΅Π»ΡŒΠ½Ρ‹ΠΉ ΠΊΠΎΠ½Ρ‚ΡƒΡ€.

    ЦСпь, ΡΠΎΡΡ‚ΠΎΡΡ‰ΡƒΡŽ ΠΈΠ· ΠΏΠΎΡΠ»Π΅Π΄ΠΎΠ²Π°Ρ‚Π΅Π»ΡŒΠ½ΠΎ Π²ΠΊΠ»ΡŽΡ‡Π΅Π½Π½Ρ‹Ρ… рСзистора, ΠΊΠ°Ρ‚ΡƒΡˆΠΊΠΈ индуктивности
ΠΈ кондСнсатора (рис. 8, Π°), ΠΏΠΎΠ΄ΠΊΠ»ΡŽΡ‡ΠΈΠΌ ΠΊ Π³Π΅Π½Π΅Ρ€Π°Ρ‚ΠΎΡ€Ρƒ ΠΏΠ΅Ρ€Π΅ΠΌΠ΅Π½Π½ΠΎΠ³ΠΎ напряТСния, ΠΏΠΎΠ·Π²ΠΎΠ»ΡΡŽΡ‰Π΅ΠΌΡƒ
Ρ€Π΅Π³ΡƒΠ»ΠΈΡ€ΠΎΠ²Π°Ρ‚ΡŒ частоту ΠΊΠΎΠ»Π΅Π±Π°Π½ΠΈΠΉ (прСдполагаСтся, Ρ‡Ρ‚ΠΎ Π³Π΅Π½Π΅Ρ€Π°Ρ‚ΠΎΡ€ напряТСния ΠΎΠ±Π»Π°Π΄Π°Π΅Ρ‚
бСсконСчно ΠΌΠ°Π»Ρ‹ΠΌ Π²Π½ΡƒΡ‚Ρ€Π΅Π½Π½ΠΈΠΌ сопротивлСниСм ΠΈ поэтому напряТСниС Π½Π° Π΅Π³ΠΎ Π·Π°ΠΆΠΈΠΌΠ°Ρ…
практичСски Π½Π΅ зависит ΠΎΡ‚ Π½Π°Π³Ρ€ΡƒΠ·ΠΊΠΈ). На постоянном Ρ‚ΠΎΠΊΠ΅ (нулСвая частота) ΠΈ
ΠΎΡ‡Π΅Π½ΡŒ Π½ΠΈΠ·ΠΊΠΈΡ… частотах Ρ‚ΠΎΠΊ Π² Ρ†Π΅ΠΏΠΈ практичСски отсутствуСт, Ρ‚Π°ΠΊ ΠΊΠ°ΠΊ СмкостноС
сопротивлСниС кондСнсатора Π²Π΅Π»ΠΈΠΊΠΎ. Π’ΠΎΠΊ Π±ΡƒΠ΄Π΅Ρ‚ ΡΡ‚Ρ€Π΅ΠΌΠΈΡ‚ΡŒΡΡ ΠΊ Π½ΡƒΠ»ΡŽ ΠΈ Π½Π° ΠΎΡ‡Π΅Π½ΡŒ высоких
частотах ΠΈΠ·-Π·Π° возрастания ΠΈΠ½Π΄ΡƒΠΊΡ‚ΠΈΠ²Π½ΠΎΠ³ΠΎ сопротивлСния ΠΊΠ°Ρ‚ΡƒΡˆΠΊΠΈ (см. Π³Ρ€Π°Ρ„ΠΈΠΊΠΈ Π½Π°
рис. 6,Π°).


Рис. 8

    Но Π΅ΡΡ‚ΡŒ ΠΎΠ΄Π½Π° характСрная частота, Π½Π° ΠΊΠΎΡ‚ΠΎΡ€ΠΎΠΉ
Ρ‚ΠΎΠΊ Π² Ρ†Π΅ΠΏΠΈ максималСн ΠΈ Ρ€Π°Π²Π΅Π½ U/R. На этой частотС ΠΈΠ½Π΄ΡƒΠΊΡ‚ΠΈΠ²Π½ΠΎΠ΅ сопротивлСниС
Ρ€Π°Π²Π½ΠΎ Смкостному, Π° ΠΏΠΎΡΠΊΠΎΠ»ΡŒΠΊΡƒ Ρƒ Π½ΠΈΡ… Ρ€Π°Π·Π½Ρ‹Π΅ Π·Π½Π°ΠΊΠΈ, ΠΎΠ½ΠΈ ΠΊΠΎΠΌΠΏΠ΅Π½ΡΠΈΡ€ΡƒΡŽΡ‚ Π΄Ρ€ΡƒΠ³
Π΄Ρ€ΡƒΠ³Π° ΠΈ ΠΏΠΎΠ»Π½ΠΎΠ΅ сопротивлСниС Ρ†Π΅ΠΏΠΈ оказываСтся Π°ΠΊΡ‚ΠΈΠ²Π½Ρ‹ΠΌ ΠΈ Ρ€Π°Π²Π½Ρ‹ΠΌ R. Π­Ρ‚Π°
частота называСтся рСзонансной, Π° Π³Ρ€Π°Ρ„ΠΈΠΊ зависимости Ρ‚ΠΎΠΊΠ° Π² Ρ†Π΅ΠΏΠΈ ΠΎΡ‚
частоты β€” рСзонансной ΠΊΡ€ΠΈΠ²ΠΎΠΉ (рис. 8,Π±). Π—Π½Π°Ρ‡Π΅Π½ΠΈΠ΅ рСзонансной частоты
ΠΌΠΎΠΆΠ½ΠΎ Π½Π°ΠΉΡ‚ΠΈ, приравняв ΠΈΠ½Π΄ΡƒΠΊΡ‚ΠΈΠ²Π½ΠΎΠ΅ ΠΈ СмкостноС сопротивлСния: pL
= 1/Ρ€Π‘, ΡΠ»Π΅Π΄ΠΎΠ²Π°Ρ‚Π΅Π»ΡŒΠ½ΠΎ,
Ρ€2
= 1/LC (рСзонансная частота). НС Π·Π°Π±Ρ‹Π²Π°ΠΉΡ‚Π΅, Ρ‡Ρ‚ΠΎ угловая, ΠΈΠ»ΠΈ
круговая частота Π² 2
ΠΈΠ»ΠΈ Π² 6,28 Ρ€Π°Π·Π° большС ΠΎΠ±Ρ‹Ρ‡Π½ΠΎΠΉ, цикличСской частоты f, измСряСмой Π²
Π³Π΅Ρ€Ρ†Π°Ρ…, Ρ‚.Π΅. = 2f.

Β Β Β Β Π’Π΅ΠΏΠ΅Ρ€ΡŒ ΠΌΡ‹ Π²ΠΏΠ»ΠΎΡ‚Π½ΡƒΡŽ подошли ΠΊ ΠΏΠΎΠ½ΡΡ‚ΠΈΡŽ добротности,
ΠΈΠΌΠ΅ΡŽΡ‰Π΅ΠΌΡƒ Π² Ρ€Π°Π΄ΠΈΠΎΡ‚Π΅Ρ…Π½ΠΈΠΊΠ΅ ΠΎΡ‡Π΅Π½ΡŒ Π²Π°ΠΆΠ½ΠΎΠ΅ Π·Π½Π°Ρ‡Π΅Π½ΠΈΠ΅. Π§Π΅ΠΌ мСньшС Π°ΠΊΡ‚ΠΈΠ²Π½ΠΎΠ΅ сопротивлСниС
R Ρ†Π΅ΠΏΠΈ, ΠΏΠΎΠΊΠ°Π·Π°Π½Π½ΠΎΠΉ Π½Π° рис. 8,Π°, Ρ‚Π΅ΠΌ острСС ΠΈ Π²Ρ‹ΡˆΠ΅ рСзонансная кривая
ΠΈ Ρ‚Π΅ΠΌ большС Ρ‚ΠΎΠΊ Π² Ρ†Π΅ΠΏΠΈ ΠΏΡ€ΠΈ рСзонансС. На самом Π΄Π΅Π»Π΅ Π²Π°ΠΆΠ½ΠΎ Π½Π΅ само ΠΏΠΎ
сСбС Π°ΠΊΡ‚ΠΈΠ²Π½ΠΎΠ΅ сопротивлСниС R, Π° ΠΎΡ‚Π½ΠΎΡˆΠ΅Π½ΠΈΠ΅ Ρ€Π΅Π°ΠΊΡ‚ΠΈΠ²Π½ΠΎΠ³ΠΎ сопротивлСния
r ΠΊΠ°Ρ‚ΡƒΡˆΠΊΠΈ ΠΈΠ»ΠΈ кондСнсатора Π½Π° рСзонансной частотС Ρ€
(Π½Π°ΠΏΠΎΠΌΠ½ΠΈΠΌ, Ρ‡Ρ‚ΠΎ ΠΎΠ½ΠΈ Ρ€Π°Π²Π½Ρ‹) ΠΊ Π°ΠΊΡ‚ΠΈΠ²Π½ΠΎΠΌΡƒ R. Π­Ρ‚ΠΎ ΠΎΡ‚Π½ΠΎΡˆΠ΅Π½ΠΈΠ΅ называСтся Π΄ΠΎΠ±Ρ€ΠΎΡ‚Π½ΠΎΡΡ‚ΡŒΡŽ
ΠΊΠΎΠ»Π΅Π±Π°Ρ‚Π΅Π»ΡŒΠ½ΠΎΠ³ΠΎ ΠΊΠΎΠ½Ρ‚ΡƒΡ€Π°: Q = r/R = pL/R
= 1/pCR (Π΄ΠΎΠ±Ρ€ΠΎΡ‚Π½ΠΎΡΡ‚ΡŒ
ΠΊΠΎΠ½Ρ‚ΡƒΡ€Π°)
. Аналогично Ρ‚ΠΎΠΌΡƒ, ΠΊΠ°ΠΊ ΠΌΡ‹ это сдСлали для рСзонансной частоты,
ΠΌΠΎΠΆΠ½ΠΎ ΠΏΠΎΠ΄ΡΡ‡ΠΈΡ‚Π°Ρ‚ΡŒ, Ρ‡Ρ‚ΠΎ r2 = L/C.

    Если Π½ΡƒΠΆΠ½ΠΎ ΠΏΠΎΠ»ΡƒΡ‡ΠΈΡ‚ΡŒ особСнно Π²Ρ‹ΡΠΎΠΊΡƒΡŽ Π΄ΠΎΠ±Ρ€ΠΎΡ‚Π½ΠΎΡΡ‚ΡŒ,
рСзистор R Π² ΠΊΠΎΠ½Ρ‚ΡƒΡ€, ΠΊΠ°ΠΊ ΠΏΡ€Π°Π²ΠΈΠ»ΠΎ, Π½Π΅ ΡƒΡΡ‚Π°Π½Π°Π²Π»ΠΈΠ²Π°ΡŽΡ‚, Π° Π΅Π³ΠΎ Ρ€ΠΎΠ»ΡŒ выполняСт
Π°ΠΊΡ‚ΠΈΠ²Π½ΠΎΠ΅ сопротивлСниС ΠΏΡ€ΠΎΠ²ΠΎΠ΄Π° ΠΊΠ°Ρ‚ΡƒΡˆΠΊΠΈ. Π”Π°ΠΆΠ΅ Ρƒ Π½Π΅Π±ΠΎΠ»ΡŒΡˆΠΈΡ… радиочастотных
ΠΊΠ°Ρ‚ΡƒΡˆΠ΅ΠΊ ΠΎΠ½ΠΎ составляСт Π΅Π΄ΠΈΠ½ΠΈΡ†Ρ‹, Π° ΠΈΠ½ΠΎΠ³Π΄Π° ΠΈ дСсятки ΠΎΠΌ, ΠΏΠΎΡΠΊΠΎΠ»ΡŒΠΊΡƒ сопротивлСниС
ΠΏΡ€ΠΎΠ²ΠΎΠ΄Π° Π½Π° высокой частотС большС, Ρ‡Π΅ΠΌ Π½Π° постоянном Ρ‚ΠΎΠΊΠ΅. ΠžΠ±ΡŠΡΡΠ½ΡΠ΅Ρ‚ΡΡ
это Ρ‚Π°ΠΊ Π½Π°Π·Ρ‹Π²Π°Π΅ΠΌΡ‹ΠΌ скин-эффСктом, явлСниСм вытСснСния Ρ‚ΠΎΠΊΠ° ΠΊ повСрхности
ΠΏΡ€ΠΎΠ²ΠΎΠ΄Π°. Π’Π°ΠΊ, Π½Π°ΠΏΡ€ΠΈΠΌΠ΅Ρ€, Π² ΠΌΠ΅Π΄Π½ΠΎΠΌ ΠΏΡ€ΠΎΠ²ΠΎΠ΄Π΅ Π½Π° частотС 3 ΠœΠ“Ρ† (3 ΠΌΠΈΠ»Π»ΠΈΠΎΠ½Π°
ΠΊΠΎΠ»Π΅Π±Π°Π½ΠΈΠΉ Π² сСкунду) Ρ‚ΠΎΠΊ Ρ‚Π΅Ρ‡Π΅Ρ‚ Π² повСрхностном слоС Ρ‚ΠΎΠ»Ρ‰ΠΈΠ½ΠΎΠΉ Π½Π΅ Π±ΠΎΠ»Π΅Π΅
0,1 ΠΌΠΌ.

    Для ΡƒΠΌΠ΅Π½ΡŒΡˆΠ΅Π½ΠΈΡ Π°ΠΊΡ‚ΠΈΠ²Π½ΠΎΠ³ΠΎ сопротивлСния ΠΊΠ°Ρ‚ΡƒΡˆΠ΅ΠΊ
Π½Π° радиочастотах часто ΠΈΡΠΏΠΎΠ»ΡŒΠ·ΡƒΡŽΡ‚ ΠΌΠ½ΠΎΠ³ΠΎΠΆΠΈΠ»ΡŒΠ½Ρ‹ΠΉ ΠΎΠ±ΠΌΠΎΡ‚ΠΎΡ‡Π½Ρ‹ΠΉ ΠΏΡ€ΠΎΠ²ΠΎΠ΄ (Π»ΠΈΡ‚Ρ†Π΅Π½Π΄Ρ€Π°Ρ‚),
скручСнный ΠΈΠ· Π½Π΅ΡΠΊΠΎΠ»ΡŒΠΊΠΈΡ… (7β€”21 ΠΈ Π±ΠΎΠ»Π΅Π΅) Ρ‚ΠΎΠ½ΠΊΠΈΡ… ΠΈΠ·ΠΎΠ»ΠΈΡ€ΠΎΠ²Π°Π½Π½Ρ‹Ρ… ΠΏΡ€ΠΎΠ²ΠΎΠ΄Π½ΠΈΠΊΠΎΠ².
ΠŸΡ€ΠΈ Ρ‚ΠΎΠΉ ΠΆΠ΅ ΠΎΠ±Ρ‰Π΅ΠΉ ΠΏΠ»ΠΎΡ‰Π°Π΄ΠΈ сСчСния ΠΈΠ»ΠΈ ΠΎΠ±Ρ‰Π΅ΠΌ Π΄ΠΈΠ°ΠΌΠ΅Ρ‚Ρ€Π΅ ΠΏΡ€ΠΎΠ²ΠΎΠ΄Π° ΠΏΠΎΠ²Π΅Ρ€Ρ…Π½ΠΎΡΡ‚ΡŒ
Ρƒ Π»ΠΈΡ‚Ρ†Π΅Π½Π΄Ρ€Π°Ρ‚Π° (ΠΏΠΎ ΠΊΠΎΡ‚ΠΎΡ€ΠΎΠΉ ΠΈ Ρ‚Π΅ΠΊΡƒΡ‚ высокочастотныС Ρ‚ΠΎΠΊΠΈ) получаСтся Π·Π½Π°Ρ‡ΠΈΡ‚Π΅Π»ΡŒΠ½ΠΎ
большС, Π° сопротивлСниС мСньшС, Ρ‡Π΅ΠΌ Ρƒ одноТильного ΠΏΡ€ΠΎΠ²ΠΎΠ΄Π°.

Β Β Β Β Π’ΠΎΠ»Ρ‰ΠΈΠ½Π° скин-слоя ΠΎΠ±Ρ€Π°Ρ‚Π½ΠΎ ΠΏΡ€ΠΎΠΏΠΎΡ€Ρ†ΠΈΠΎΠ½Π°Π»ΡŒΠ½Π° ΠΊΠΎΡ€Π½ΡŽ
ΠΊΠ²Π°Π΄Ρ€Π°Ρ‚Π½ΠΎΠΌΡƒ ΠΈΠ· частоты, ΠΈ Π½Π° частотС 300 ΠœΠ“Ρ† ΠΎΠ½Π° ΡƒΠΌΠ΅Π½ΡŒΡˆΠ°Π΅Ρ‚ΡΡ Π΄ΠΎ 10 ΠΌΠΊΠΌ.
Π—Π΄Π΅ΡΡŒ ΠΈ Π»ΠΈΡ‚Ρ†Π΅Π½Π΄Ρ€Π°Ρ‚ ΡƒΠΆΠ΅ Π½Π΅ ΠΏΠΎΠΌΠΎΠ³Π°Π΅Ρ‚, ΠΈ приходится ΠΎΠΏΡΡ‚ΡŒ ΠΈΡΠΏΠΎΠ»ΡŒΠ·ΠΎΠ²Π°Ρ‚ΡŒ
ΠΎΠ΄Π½ΠΎΠΆΠΈΠ»ΡŒΠ½Ρ‹Π΅ ΠΏΡ€ΠΎΠ²ΠΎΠ΄Π° Π·Π½Π°Ρ‡ΠΈΡ‚Π΅Π»ΡŒΠ½ΠΎΠ³ΠΎ Π΄ΠΈΠ°ΠΌΠ΅Ρ‚Ρ€Π°, Π±Π»Π°Π³ΠΎ Π½Π° Ρ‚Π°ΠΊΠΈΡ… частотах
ΠΊΠ°Ρ‚ΡƒΡˆΠΊΠΈ ΠΈΠΌΠ΅ΡŽΡ‚ Π½Π΅ Π±ΠΎΠ»Π΅Π΅ Π½Π΅ΡΠΊΠΎΠ»ΡŒΠΊΠΈΡ… Π²ΠΈΡ‚ΠΊΠΎΠ². ΠžΠΊΠΈΡΠ»Π΅Π½Π½Ρ‹Π΅ ΠΈ «ΡˆΠ΅Ρ€ΡˆΠ°Π²Ρ‹Π΅», Ρ‚.Π΅.
ΠΏΠ»ΠΎΡ…ΠΎ ΠΎΠ±Ρ€Π°Π±ΠΎΡ‚Π°Π½Π½Ρ‹Π΅ мСталличСскиС повСрхности Π±ΡƒΠ΄ΡƒΡ‚ Π½Π° этих частотах
ΡƒΠΆΠ΅ ΠΏΠ»ΠΎΡ…ΠΈΠΌΠΈ ΠΏΡ€ΠΎΠ²ΠΎΠ΄Π½ΠΈΠΊΠ°ΠΌΠΈ. Для ΡƒΠ»ΡƒΡ‡ΡˆΠ΅Π½ΠΈΡ проводимости повСрхностного
слоя Π΅Π³ΠΎ часто сСрСбрят, Π° вмСсто ΡΠΏΠ»ΠΎΡˆΠ½Ρ‹Ρ… ΠΊΡ€ΡƒΠ³Π»Ρ‹Ρ… ΠΏΡ€ΠΎΠ²ΠΎΠ΄ΠΎΠ² ΠΈΡΠΏΠΎΠ»ΡŒΠ·ΡƒΡŽΡ‚
тонкостСнныС Ρ‚Ρ€ΡƒΠ±ΠΊΠΈ β€” ΠΈ Π»Π΅Π³Ρ‡Π΅, ΠΈ ΠΌΠ°Ρ‚Π΅Ρ€ΠΈΠ°Π» экономится. А сопротивлСниС
остаСтся Ρ‚Π΅ΠΌ ΠΆΠ΅.

    Если Π²Ρ‹Π²ΠΎΠ΄Ρ‹ Ρ†Π΅ΠΏΠΈ рис. 8,Π° Π·Π°ΠΌΠΊΠ½ΡƒΡ‚ΡŒ Π½Π°ΠΊΠΎΡ€ΠΎΡ‚ΠΊΠΎ,
получится ΠΏΠ°Ρ€Π°Π»Π»Π΅Π»ΡŒΠ½Ρ‹ΠΉ ΠΊΠΎΠ»Π΅Π±Π°Ρ‚Π΅Π»ΡŒΠ½Ρ‹ΠΉ ΠΊΠΎΠ½Ρ‚ΡƒΡ€ (рис. 8,Π²). Он Π³ΠΎΡ€Π°Π·Π΄ΠΎ Ρ‡Π°Ρ‰Π΅
ΠΈΡΠΏΠΎΠ»ΡŒΠ·ΡƒΠ΅Ρ‚ΡΡ Π² Ρ€Π°Π΄ΠΈΠΎΡ‚Π΅Ρ…Π½ΠΈΠΊΠ΅. Π§Ρ‚ΠΎΠ±Ρ‹ Π½Π°Π±Π»ΡŽΠ΄Π°Ρ‚ΡŒ Π² ΠΊΠΎΠ½Ρ‚ΡƒΡ€Π΅ рСзонансныС явлСния,
ΠΊ Π΅Π³ΠΎ Π²Ρ‹Π²ΠΎΠ΄Π°ΠΌ Π½Π°Π΄ΠΎ ΠΏΠΎΠ΄ΠΊΠ»ΡŽΡ‡ΠΈΡ‚ΡŒ ΡƒΠΆΠ΅ Π½Π΅ Π³Π΅Π½Π΅Ρ€Π°Ρ‚ΠΎΡ€ ΠΏΠ΅Ρ€Π΅ΠΌΠ΅Π½Π½ΠΎΠ³ΠΎ напряТСния,
Π° Π³Π΅Π½Π΅Ρ€Π°Ρ‚ΠΎΡ€ Ρ‚ΠΎΠΊΠ°, ΠΎΠ±Π»Π°Π΄Π°ΡŽΡ‰ΠΈΠΉ большим Π²Π½ΡƒΡ‚Ρ€Π΅Π½Π½ΠΈΠΌ сопротивлСниСм ΠΈ поэтому
ΡΠΎΠ·Π΄Π°ΡŽΡ‰ΠΈΠΉ Π² любой Π½Π°Π³Ρ€ΡƒΠ·ΠΊΠ΅ Ρ‚ΠΎΠΊ I, Π½Π΅ зависящий ΠΎΡ‚ Π΅Π΅ сопротивлСния.

Β Β Β Β Π“Π΅Π½Π΅Ρ€Π°Ρ‚ΠΎΡ€ΠΎΠΌ Ρ‚ΠΎΠΊΠ° являСтся, Π½Π°ΠΏΡ€ΠΈΠΌΠ΅Ρ€, короткая
(ΠΏΠΎ ΡΡ€Π°Π²Π½Π΅Π½ΠΈΡŽ с Π΄Π»ΠΈΠ½ΠΎΠΉ Π²ΠΎΠ»Π½Ρ‹) Π°Π½Ρ‚Π΅Π½Π½Π° ΠΈΠ»ΠΈ транзисторный ΡƒΡΠΈΠ»ΠΈΡ‚Π΅Π»ΡŒΠ½Ρ‹ΠΉ
каскад. Π’ этом случаС напряТСниС Π½Π° Π²Ρ‹Π²ΠΎΠ΄Π°Ρ… ΠΏΠ°Ρ€Π°Π»Π»Π΅Π»ΡŒΠ½ΠΎΠ³ΠΎ ΠΊΠΎΠ½Ρ‚ΡƒΡ€Π° Π±ΡƒΠ΄Π΅Ρ‚
ΠΈΠ·ΠΌΠ΅Π½ΡΡ‚ΡŒΡΡ, ΠΏΡ€ΠΈ ΠΈΠ·ΠΌΠ΅Π½Π΅Π½ΠΈΠΈ частоты, Π² соотвСтствии с рСзонансной ΠΊΡ€ΠΈΠ²ΠΎΠΉ,
ΠΏΠΎΠΊΠ°Π·Π°Π½Π½ΠΎΠΉ Π½Π° рис. 8,Π± ΡˆΡ‚Ρ€ΠΈΡ…ΠΎΠ²ΠΎΠΉ Π»ΠΈΠ½ΠΈΠ΅ΠΉ. Как Π²ΠΈΠ΄ΠΈΠΌ, ΠΎΠ½Π° ΠΌΠ°Π»ΠΎ отличаСтся
ΠΎΡ‚ рСзонансной ΠΊΡ€ΠΈΠ²ΠΎΠΉ для ΠΏΠΎΡΠ»Π΅Π΄ΠΎΠ²Π°Ρ‚Π΅Π»ΡŒΠ½ΠΎΠ³ΠΎ ΠΊΠΎΠ½Ρ‚ΡƒΡ€Π°, ΠΏΡ€ΠΈΡ‡Π΅ΠΌ отличия
Π·Π°ΠΌΠ΅Ρ‚Π½Ρ‹ лишь Π½Π° Π±ΠΎΠΊΠΎΠ²Ρ‹Ρ… вСтвях, Π²Π΄Π°Π»ΠΈ ΠΎΡ‚ рСзонансной частоты.

    НапряТСниС Π½Π° Π²Ρ‹Π²ΠΎΠ΄Π°Ρ… ΠΊΠΎΠ½Ρ‚ΡƒΡ€Π° ΠΏΡ€ΠΈ рСзонансной
частотС Ρ€Π°Π²Π½ΠΎ IRΠΎΠ΅, Π³Π΄Π΅ Roe = r2/R
β€” эквивалСнтноС сопротивлСниС ΠΊΠΎΠ½Ρ‚ΡƒΡ€Π° Π½Π° рСзонансной частотС. Оно Ρ‚Π΅ΠΌ
большС, Ρ‡Π΅ΠΌ мСньшС Π°ΠΊΡ‚ΠΈΠ²Π½ΠΎΠ΅ сопротивлСниС, Π²ΠΊΠ»ΡŽΡ‡Π΅Π½Π½ΠΎΠ΅ ΠΏΠΎΡΠ»Π΅Π΄ΠΎΠ²Π°Ρ‚Π΅Π»ΡŒΠ½ΠΎ
с ΠΊΠ°Ρ‚ΡƒΡˆΠΊΠΎΠΉ, ΠΈΠ»ΠΈ сопротивлСниС самой ΠΊΠ°Ρ‚ΡƒΡˆΠΊΠΈ. ΠžΡΡ‚Π°Π΅Ρ‚ΡΡ Π² силС всС Ρ‚ΠΎ,
Ρ‡Ρ‚ΠΎ ΠΌΡ‹ рассказали ΠΎ ΠΊΠΎΠ½Ρ‚ΡƒΡ€Π°Ρ… с высокой Π΄ΠΎΠ±Ρ€ΠΎΡ‚Π½ΠΎΡΡ‚ΡŒΡŽ ΠΈ ΠΎ ΠΌΠ΅Ρ€Π°Ρ… ΡƒΠΌΠ΅Π½ΡŒΡˆΠ΅Π½ΠΈΡ
сопротивлСния ΠΏΡ€ΠΎΠ²ΠΎΠ΄ΠΎΠ² Π½Π° высокой частотС.

    Для Ρ‡Π΅Π³ΠΎ ΠΆΠ΅ Π½ΡƒΠΆΠ΅Π½ ΠΊΠΎΠ»Π΅Π±Π°Ρ‚Π΅Π»ΡŒΠ½Ρ‹ΠΉ ΠΊΠΎΠ½Ρ‚ΡƒΡ€? Π“Π»Π°Π²Π½Ρ‹ΠΌ
ΠΎΠ±Ρ€Π°Π·ΠΎΠΌ, для выдСлСния ΠΊΠΎΠ»Π΅Π±Π°Π½ΠΈΠΉ с Π½ΡƒΠΆΠ½ΠΎΠΉ Π½Π°ΠΌ частотой ΠΈΠ· мноТСства
ΠΊΠΎΠ»Π΅Π±Π°Π½ΠΈΠΉ с Ρ€Π°Π·Π»ΠΈΡ‡Π½Ρ‹ΠΌΠΈ частотами. Π­Ρ‚ΠΎ Ρ‡ΡƒΡ‚ΡŒ Π»ΠΈ Π½Π΅ основная Π·Π°Π΄Π°Ρ‡Π° Ρ€Π°Π΄ΠΈΠΎΡ‚Π΅Ρ…Π½ΠΈΠΊΠΈ.
Π”Π°ΠΆΠ΅ ΠΏΡ€ΠΎΡΡ‚Π΅ΠΉΡˆΠΈΠΉ Π΄Π΅Ρ‚Π΅ΠΊΡ‚ΠΎΡ€Π½Ρ‹ΠΉ Ρ€Π°Π΄ΠΈΠΎΠΏΡ€ΠΈΠ΅ΠΌΠ½ΠΈΠΊ Π±ΡƒΠ΄Π΅Ρ‚ ΠΏΡ€ΠΈΠ½ΠΈΠΌΠ°Ρ‚ΡŒ сигналы сразу
Π½Π΅ΡΠΊΠΎΠ»ΡŒΠΊΠΈΡ… Π½Π°ΠΈΠ±ΠΎΠ»Π΅Π΅ ΠΌΠΎΡ‰Π½Ρ‹Ρ… радиостанций, Ρ€Π°Π±ΠΎΡ‚Π°ΡŽΡ‰ΠΈΡ… Π½Π° Ρ€Π°Π·Π½Ρ‹Ρ… частотах,
Ссли Π΅Π³ΠΎ Π½Π΅ ΠΎΡΠ½Π°ΡΡ‚ΠΈΡ‚ΡŒ ΠΊΠΎΠ»Π΅Π±Π°Ρ‚Π΅Π»ΡŒΠ½Ρ‹ΠΌ ΠΊΠΎΠ½Ρ‚ΡƒΡ€ΠΎΠΌ.

    Когда ΠΊΠΎΠ½Ρ‚ΡƒΡ€ настроСн Π½Π° частоту Π½ΡƒΠΆΠ½ΠΎΠΉ радиостанции,
сигналы всСх ΠΎΡΡ‚Π°Π»ΡŒΠ½Ρ‹Ρ… Π·Π½Π°Ρ‡ΠΈΡ‚Π΅Π»ΡŒΠ½ΠΎ ΠΎΡΠ»Π°Π±Π»ΡΡŽΡ‚ΡΡ, ΠΈ ΠΌΡ‹ ΠΏΡ€ΠΎΡΠ»ΡƒΡˆΠΈΠ²Π°Π΅ΠΌ Ρ‚ΠΎΠ»ΡŒΠΊΠΎ
ΠΎΠ΄Π½Ρƒ Ρ€Π°Π΄ΠΈΠΎΠΏΠ΅Ρ€Π΅Π΄Π°Ρ‡Ρƒ. Π§Ρ‚ΠΎΠ±Ρ‹ ΠΏΠ΅Ρ€Π΅ΡΡ‚Ρ€Π°ΠΈΠ²Π°Ρ‚ΡŒ ΠΊΠΎΠ½Ρ‚ΡƒΡ€ ΠΏΠΎ частотС, Π½Π΅ΠΎΠ±Ρ…ΠΎΠ΄ΠΈΠΌΠΎ
ΠΈΠ·ΠΌΠ΅Π½ΡΡ‚ΡŒ ΠΈΠ½Π΄ΡƒΠΊΡ‚ΠΈΠ²Π½ΠΎΡΡ‚ΡŒ ΠΊΠ°Ρ‚ΡƒΡˆΠΊΠΈ L ΠΈΠ»ΠΈ Π΅ΠΌΠΊΠΎΡΡ‚ΡŒ кондСнсатора Π‘ (ΠΈΠ»ΠΈ ΠΈ Ρ‚ΠΎ
ΠΈ Π΄Ρ€ΡƒΠ³ΠΎΠ΅ ΠΎΠ΄Π½ΠΎΠ²Ρ€Π΅ΠΌΠ΅Π½Π½ΠΎ). Π‘ ΡƒΠ²Π΅Π»ΠΈΡ‡Π΅Π½ΠΈΠ΅ΠΌ индуктивности ΠΈ Смкости рСзонансная
частота ΠΈΠ»ΠΈ частота настройки пониТаСтся. Π§Π°Ρ‰Π΅ всСго ΠΈΡΠΏΠΎΠ»ΡŒΠ·ΡƒΡŽΡ‚ кондСнсатор
ΠΏΠ΅Ρ€Π΅ΠΌΠ΅Π½Π½ΠΎΠΉ Смкости ΠΏΡ€ΠΎΠΌΡ‹ΡˆΠ»Π΅Π½Π½ΠΎΠ³ΠΎ изготовлСния ΠΈ ΠΊΠ°Ρ‚ΡƒΡˆΠΊΡƒ с ΠΎΡ‚Π²ΠΎΠ΄Π°ΠΌΠΈ:
ΠΏΠ΅Ρ€Π΅ΠΊΠ»ΡŽΡ‡Π°Ρ ΠΎΡ‚Π²ΠΎΠ΄Ρ‹, Π²Ρ‹Π±ΠΈΡ€Π°ΡŽΡ‚ Π΄ΠΈΠ°ΠΏΠ°Π·ΠΎΠ½ частот, Π° Π²Π½ΡƒΡ‚Ρ€ΠΈ Π΄ΠΈΠ°ΠΏΠ°Π·ΠΎΠ½Π° частоту
ΡƒΡΡ‚Π°Π½Π°Π²Π»ΠΈΠ²Π°ΡŽΡ‚ кондСнсатором.

Β Β Β Β Π˜Ρ‚Π°ΠΊ, Π½Π΅Π·Π°ΠΌΠ΅Ρ‚Π½ΠΎ ΠΎΡ‚ рассказа ΠΎΠ± элСктротСхникС
ΠΌΡ‹ ΠΏΠ΅Ρ€Π΅ΡˆΠ»ΠΈ ΠΊ Ρ€Π°Π΄ΠΈΠΎΡ‚Π΅Ρ…Π½ΠΈΠΊΠ΅. Но ΠΎ Π½Π΅ΠΉ β€” Π² ΡΠ»Π΅Π΄ΡƒΡŽΡ‰ΠΈΠΉ Ρ€Π°Π·.



Π Π°Π΄ΠΈΠΎ, 1998

ЦСль Ρ€Π°Π±ΠΎΡ‚Ρ‹ ΠΈΠ·ΡƒΡ‡Π΅Π½ΠΈΠ΅ свободных Π·Π°Ρ‚ΡƒΡ…Π°ΡŽΡ‰ΠΈΡ… ΠΊΠΎΠ»Π΅Π±Π°Π½ΠΈΠΉ Π² элСктричСском ΠΊΠΎΠ»Π΅Π±Π°Ρ‚Π΅Π»ΡŒΠ½ΠΎΠΌ ΠΊΠΎΠ½Ρ‚ΡƒΡ€Π΅

ЦСль Ρ€Π°Π±ΠΎΡ‚Ρ‹: ΠΈΠ·ΡƒΡ‡Π΅Π½ΠΈΠ΅
свободных Π·Π°Ρ‚ΡƒΡ…Π°ΡŽΡ‰ΠΈΡ… ΠΊΠΎΠ»Π΅Π±Π°Π½ΠΈΠΉ Π²
элСктричСском ΠΊΠΎΠ»Π΅Π±Π°Ρ‚Π΅Π»ΡŒΠ½ΠΎΠΌ ΠΊΠΎΠ½Ρ‚ΡƒΡ€Π΅.

Π—Π°Π΄Π°Ρ‡Π°: ΠΎΠΏΡ€Π΅Π΄Π΅Π»Π΅Π½ΠΈΠ΅
характСристик Π·Π°Ρ‚ΡƒΡ…Π°ΡŽΡ‰ΠΈΡ… ΠΊΠΎΠ»Π΅Π±Π°Π½ΠΈΠΉ.

ΠŸΡ€ΠΈΠ±ΠΎΡ€Ρ‹ ΠΈ принадлСТности:
источник питания,
ΠΊΠΎΠ»Π΅Π±Π°Ρ‚Π΅Π»ΡŒΠ½Ρ‹ΠΉ ΠΊΠΎΠ½Ρ‚ΡƒΡ€, осциллограф,
Π³Π΅Π½Π΅Ρ€Π°Ρ‚ΠΎΡ€ Π·Π²ΡƒΠΊΠΎΠ²ΠΎΠΉ частоты, ΠΌΠ°Π³Π°Π·ΠΈΠ½
сопротивлСний, ΠΏΡ€Π΅ΠΎΠ±Ρ€Π°Π·ΠΎΠ²Π°Ρ‚Π΅Π»ΡŒ ΠΈΠΌΠΏΡƒΠ»ΡŒΡΠΎΠ².

Π’Π΅Ρ…Π½ΠΈΠΊΠ° бСзопасности:
ΡΠΊΡΠΏΠ΅Ρ€ΠΈΠΌΠ΅Π½Ρ‚Π°Π»ΡŒΠ½Π°Ρ установка
питаСтся напряТСниСм 220 Π’, поэтому
Ρ‚ΠΎΠΊΠΎΠ²Π΅Π΄ΡƒΡ‰ΠΈΠ΅ части Π΄ΠΎΠ»ΠΆΠ½Ρ‹ Π±Ρ‹Ρ‚ΡŒ Π·Π°ΠΊΡ€Ρ‹Ρ‚Ρ‹.

Π’Π’Π•Π”Π•ΠΠ˜Π•

Π’ Ρ†Π΅ΠΏΠΈ, содСрТащСй ΠΈΠ½Π΄ΡƒΠΊΡ‚ΠΈΠ²Π½ΠΎΡΡ‚ΡŒ
ΠΈ Π΅ΠΌΠΊΠΎΡΡ‚ΡŒ, ΠΌΠΎΠ³ΡƒΡ‚ Π²ΠΎΠ·Π½ΠΈΠΊΠ°Ρ‚ΡŒ элСктричСскиС
колСбания. ΠŸΠΎΡΡ‚ΠΎΠΌΡƒ Ρ‚Π°ΠΊΡƒΡŽ Ρ†Π΅ΠΏΡŒ Π½Π°Π·Ρ‹Π²Π°ΡŽΡ‚
ΠΊΠΎΠ»Π΅Π±Π°Ρ‚Π΅Π»ΡŒΠ½Ρ‹ΠΌ ΠΊΠΎΠ½Ρ‚ΡƒΡ€ΠΎΠΌ.

Рассмотрим элСктричСский
ΠΊΠΎΠ»Π΅Π±Π°Ρ‚Π΅Π»ΡŒΠ½Ρ‹ΠΉ ΠΊΠΎΠ½Ρ‚ΡƒΡ€, состоящий ΠΈΠ·
ΠΏΠΎΡΠ»Π΅Π΄ΠΎΠ²Π°Ρ‚Π΅Π»ΡŒΠ½ΠΎ соСдинСнных ΠΊΠ°Ρ‚ΡƒΡˆΠΊΠΈ
индуктивности L, кондСнсатора C ΠΈ
сопротивлСния R (рис. 1).

Рис. 1. ΠšΠΎΠ»Π΅Π±Π°Ρ‚Π΅Π»ΡŒΠ½Ρ‹ΠΉ ΠΊΠΎΠ½Ρ‚ΡƒΡ€.

Π’ состоянии «равновСсия» Ρ‚ΠΎΠΊ Π²
ΠΊΠΎΠ½Ρ‚ΡƒΡ€Π΅ ΠΈ заряд Π½Π° ΠΎΠ±ΠΊΠ»Π°Π΄ΠΊΠ°Ρ… кондСнсатора
Ρ€Π°Π²Π½Ρ‹ Π½ΡƒΠ»ΡŽ. КолСбания Π² ΠΊΠΎΠ½Ρ‚ΡƒΡ€Π΅ ΠΌΠΎΠΆΠ½ΠΎ
Π²Ρ‹Π·Π²Π°Ρ‚ΡŒ, Π»ΠΈΠ±ΠΎ сообщив ΠΎΠ±ΠΊΠ»Π°Π΄ΠΊΠ°ΠΌ
кондСнсатора Π½Π°Ρ‡Π°Π»ΡŒΠ½Ρ‹ΠΉ заряд (Π½Π°ΠΏΡ€ΠΈΠΌΠ΅Ρ€,
с ΠΏΠΎΠΌΠΎΡ‰ΡŒΡŽ Π³Π΅Π½Π΅Ρ€Π°Ρ‚ΠΎΡ€Π° ΠΈΠΌΠΏΡƒΠ»ΡŒΡΠΎΠ²), Π»ΠΈΠ±ΠΎ
Π²ΠΎΠ·Π±ΡƒΠ΄ΠΈΠ² Π² индуктивности Ρ‚ΠΎΠΊ (Π½Π°ΠΏΡ€ΠΈΠΌΠ΅Ρ€,
ΠΏΡƒΡ‚Π΅ΠΌ Π²Ρ‹ΠΊΠ»ΡŽΡ‡Π΅Π½ΠΈΡ внСшнСго ΠΌΠ°Π³Π½ΠΈΡ‚Π½ΠΎΠ³ΠΎ
поля, ΠΏΡ€ΠΎΠ½ΠΈΠ·Ρ‹Π²Π°Π²ΡˆΠ΅Π³ΠΎ Π²ΠΈΡ‚ΠΊΠΈ ΠΊΠ°Ρ‚ΡƒΡˆΠΊΠΈ). Π’
Π΄Π°Π½Π½ΠΎΠΉ Ρ€Π°Π±ΠΎΡ‚Π΅ ΠΈΡΠΏΠΎΠ»ΡŒΠ·ΡƒΠ΅Ρ‚ΡΡ ΠΏΠ΅Ρ€Π²Ρ‹ΠΉ
способ возбуТдСния ΠΊΠΎΠ»Π΅Π±Π°Π½ΠΈΠΉ.

ΠŸΡƒΡΡ‚ΡŒ Π² ΠΌΠΎΠΌΠ΅Π½Ρ‚ Π²Ρ€Π΅ΠΌΠ΅Π½ΠΈ t = 0 Π½Π°
ΠΎΠ±ΠΊΠ»Π°Π΄ΠΊΠ°Ρ… кондСнсатора Π²ΠΎΠ·Π½ΠΈΠΊΠ»ΠΈ заряды
+q ΠΈ –q. ΠšΠΎΠ½Π΄Π΅Π½ΡΠ°Ρ‚ΠΎΡ€ C Π½Π°Ρ‡Π½Π΅Ρ‚ Ρ€Π°Π·Ρ€ΡΠΆΠ°Ρ‚ΡŒΡΡ,
ΠΈ Π² ΠΊΠΎΠ½Ρ‚ΡƒΡ€Π΅ Π²ΠΎΠ·Π½ΠΈΠΊΠ°Π΅Ρ‚ Ρ‚ΠΎΠΊ i = dq / dt. ИзмСнСниС
Π²Π΅Π»ΠΈΡ‡ΠΈΠ½Ρ‹ Ρ‚ΠΎΠΊΠ°, проходящСго Ρ‡Π΅Ρ€Π΅Π· ΠΊΠ°Ρ‚ΡƒΡˆΠΊΡƒ
индуктивности L, Π²Ρ‹Π·Ρ‹Π²Π°Π΅Ρ‚ Π² Π½Π΅ΠΉ
Π²ΠΎΠ·Π½ΠΈΠΊΠ½ΠΎΠ²Π΅Π½ΠΈΠ΅ Π­Π”Π‘ самоиндукции
,
ΠΏΡ€Π΅ΠΏΡΡ‚ΡΡ‚Π²ΡƒΡŽΡ‰Π΅ΠΉ измСнСнию Ρ‚ΠΎΠΊΠ°. ΠŸΠΎΡΡ‚ΠΎΠΌΡƒ,
Π΄Π°ΠΆΠ΅ Π² ΠΌΠΎΠΌΠ΅Π½Ρ‚ Π²Ρ€Π΅ΠΌΠ΅Π½ΠΈ, ΠΊΠΎΠ³Π΄Π° заряд q Π½Π°
Π΅ΠΌΠΊΠΎΡΡ‚ΡŒ C становится Ρ€Π°Π²Π½Ρ‹ΠΌ Π½ΡƒΠ»ΡŽ, Ρ‚ΠΎΠΊ Π²
ΠΊΠΎΠ½Ρ‚ΡƒΡ€Π΅ Π½Π΅ исчСзаСт, Π² слСдствиС Π­Π”Π‘
самоиндукции ΠΏΡ€ΠΎΠ΄ΠΎΠ»ΠΆΠ°Π΅Ρ‚ ΠΈΠ΄Ρ‚ΠΈ Π² ΠΏΡ€Π΅ΠΆΠ½Π΅ΠΌ
Π½Π°ΠΏΡ€Π°Π²Π»Π΅Π½ΠΈΠΈ. ΠšΠΎΠ½Π΄Π΅Π½ΡΠ°Ρ‚ΠΎΡ€ C
ΠΎΠΏΡΡ‚ΡŒ заряТаСтся, Π½ΠΎ Π·Π½Π°ΠΊ зарядов Π½Π°
Π΅Π³ΠΎ ΠΎΠ±ΠΊΠ»Π°Π΄ΠΊΠ°Ρ… измСняСтся Π½Π° ΠΏΡ€ΠΎΡ‚ΠΈΠ²ΠΎΠΏΠΎΠ»ΠΎΠΆΠ½Ρ‹ΠΉ.
ПослС Ρ‚ΠΎΠ³ΠΎ ΠΊΠ°ΠΊ Ρ‚ΠΎΠΊ i становится Ρ€Π°Π²Π½Ρ‹ΠΌ
Π½ΡƒΠ»ΡŽ, вСсь Ρ†ΠΈΠΊΠ» разряда повторяСтся Π²
ΠΎΠ±Ρ€Π°Ρ‚Π½ΠΎΠΌ Π½Π°ΠΏΡ€Π°Π²Π»Π΅Π½ΠΈΠΈ.

ΠŸΡ€ΠΈ отсутствии Π² ΠΊΠΎΠ½Ρ‚ΡƒΡ€Π΅ Π°ΠΊΡ‚ΠΈΠ²Π½ΠΎΠ³ΠΎ
сопротивлСния (R=0) колСбания заряда q ΠΈ
Ρ‚ΠΎΠΊΠ° i Π½ΠΈΠΊΠΎΠ³Π΄Π° Π±Ρ‹ Π½Π΅ ΠΏΡ€Π΅ΠΊΡ€Π°Ρ‚ΠΈΠ»ΠΈΡΡŒ. Но Π²ΠΎ
всяком Ρ€Π΅Π°Π»ΡŒΠ½ΠΎΠΌ ΠΊΠΎΠ½Ρ‚ΡƒΡ€Π΅ Rβ‰ 0 (сопротивлСниС
ΡΠΎΠ΅Π΄ΠΈΠ½ΠΈΡ‚Π΅Π»ΡŒΠ½Ρ‹Ρ… ΠΏΡ€ΠΎΠ²ΠΎΠ΄ΠΎΠ², ΠΎΠ±ΠΌΠΎΡ‚ΠΊΠΈ ΠΊΠ°Ρ‚ΡƒΡˆΠΊΠΈ
индуктивности…). ΠŸΠΎΡΡ‚ΠΎΠΌΡƒ энСргия
элСктричСских ΠΊΠΎΠ»Π΅Π±Π°Π½ΠΈΠΉ постСпСнно
ΡƒΠΌΠ΅Π½ΡŒΡˆΠ°Π΅Ρ‚ΡΡ, пСрСходя Π² Ρ‚Π΅ΠΏΠ»ΠΎΠ²ΡƒΡŽ, ΠΈ
колСбания Π·Π°Ρ‚ΡƒΡ…Π°ΡŽΡ‚.

НайдСм ΡƒΡ€Π°Π²Π½Π΅Π½ΠΈΠ΅ свободных
ΠΊΠΎΠ»Π΅Π±Π°Π½ΠΈΠΉ Π² ΠΊΠΎΠ½Ρ‚ΡƒΡ€Π΅. Π‘ΡƒΠΌΠΌΠ°Ρ€Π½ΠΎΠ΅ ΠΏΠ°Π΄Π΅Π½ΠΈΠ΅
напряТСния Π½Π° кондСнсаторС UC
ΠΈ сопротивлСния UR
Ρ€Π°Π²Π½ΠΎ Π­Π”Π‘ самоиндукции
:
UC+ UR
=
.

Учитывая, Ρ‡Ρ‚ΠΎ UC
= q/C,
UR
= i
R,

= -L
di/dt,
I =
dq/dt,
ΠΏΠΎΠ»ΡƒΡ‡ΠΈΠΌ Π΄ΠΈΡ„Ρ„Π΅Ρ€Π΅Π½Ρ†ΠΈΠ°Π»ΡŒΠ½ΠΎΠ΅ ΡƒΡ€Π°Π²Π½Π΅Π½ΠΈΠ΅
ΠΊΠΎΠ»Π΅Π±Π°Π½ΠΈΠΉ Π² ΠΊΠΎΠ½Ρ‚ΡƒΡ€Π΅:

.
(1)

Π’Π²Π΅Π΄Π΅ΠΌ понятиС собствСнной
частоты ΠΊΠΎΠ½Ρ‚ΡƒΡ€Π°

(1Π°)

ΠΈ коэффициСнта затухания
.
(1Π±)

Π’ΠΎΠ³Π΄Π° ΡƒΡ€Π°Π²Π½Π΅Π½ΠΈΠ΅ (1) ΠΌΠΎΠΆΠ½ΠΎ Π·Π°ΠΏΠΈΡΠ°Ρ‚ΡŒ
Π² стандартном Π²ΠΈΠ΄Π΅:

.
(2)

АналогичныС ΡƒΡ€Π°Π²Π½Π΅Π½ΠΈΠ΅ ΠΏΠΎΠ»ΡƒΡ‡Π°ΡŽΡ‚ΡΡ
ΠΈ для Π²Π΅Π»ΠΈΡ‡ΠΈΠ½ i, UC,
UR.

РСшСниС уравнСния (2) ΠΈΠΌΠ΅Π΅Ρ‚
ΡΠ»Π΅Π΄ΡƒΡŽΡ‰ΠΈΠΉ Π²ΠΈΠ΄:

,
(3)

Π³Π΄Π΅

(4)

опрСдСляСт частоту ΠΊΠΎΠ»Π΅Π±Π°Π½ΠΈΠΉ,
Π° Ο† – Π½Π°Ρ‡Π°Π»ΡŒΠ½ΡƒΡŽ Ρ„Π°Π·Ρƒ ΠΊΠΎΠ»Π΅Π±Π°Π½ΠΈΠΉ. Π’Π΅Π»ΠΈΡ‡ΠΈΠ½Ρ‹
Ο† ΠΈ q0, ΠΊΠ°ΠΊ
ΠΏΡ€Π°Π²ΠΈΠ»ΠΎ, ΠΎΠΏΡ€Π΅Π΄Π΅Π»ΡΡŽΡ‚ΡΡ Π½Π°Ρ‡Π°Π»ΡŒΠ½Ρ‹ΠΌΠΈ
условиями. Амплитуда ΠΊΠΎΠ»Π΅Π±Π°Π½ΠΈΠΉ A(t), ΠΊΠ°ΠΊ
Π²ΠΈΠ΄Π½ΠΎ ΠΈΠ· (3), даСтся Π²Π΅Π»ΠΈΡ‡ΠΈΠ½ΠΎΠΉ

ΠΈ, ΡΠ»Π΅Π΄ΠΎΠ²Π°Ρ‚Π΅Π»ΡŒΠ½ΠΎ, зависит ΠΎΡ‚ Π²Ρ€Π΅ΠΌΠ΅Π½ΠΈ.
ΠŸΠΎΡΡ‚ΠΎΠΌΡƒ Π·Π°Ρ‚ΡƒΡ…Π°ΡŽΡ‰ΠΈΠ΅ колСбания Π½Π΅ ΡΠ²Π»ΡΡŽΡ‚ΡΡ
гармоничСскими.

Π₯Π°Ρ€Π°ΠΊΡ‚Π΅Ρ€ зависимости q(t)
опрСдСляСтся (см.(3)) ΡΠΎΠΎΡ‚Π½ΠΎΡˆΠ΅Π½ΠΈΠ΅ΠΌ ΠΌΠ΅ΠΆΠ΄Ρƒ
Π²Π΅Π»ΠΈΡ‡ΠΈΠ½Π°ΠΌΠΈ Ο‰0
ΠΈ Ξ΄, Ρ‚.Π΅. ΠΏΠ°Ρ€Π°ΠΌΠ΅Ρ‚Ρ€Π°ΠΌΠΈ ΠΊΠΎΠ½Ρ‚ΡƒΡ€Π° R, L, C. Когда
Π°ΠΊΡ‚ΠΈΠ²Π½ΠΎΠ΅ сопротивлСниС ΠΊΠΎΠ½Ρ‚ΡƒΡ€Π° ΠΌΠ°Π»ΠΎ
(R <<
,
Ξ΄ <<
),
частота ΠΊΠΎΠ»Π΅Π±Π°Π½ΠΈΠΉ Ο‰ Π±Π»ΠΈΠ·ΠΊΠ° ΠΊ частотС
Ο‰0.

Π‘Π²ΠΎΠ±ΠΎΠ΄Π½Ρ‹Π΅ Π·Π°Ρ‚ΡƒΡ…Π°ΡŽΡ‰ΠΈΠΉ колСбания
ΠΈΠ·ΠΎΠ±Ρ€Π°ΠΆΠ΅Π½Ρ‹ Π½Π° рис. 2.

Рис. 2. Π‘Π²ΠΎΠ±ΠΎΠ΄Π½Ρ‹Π΅ Π·Π°Ρ‚ΡƒΡ…Π°ΡŽΡ‰ΠΈΠ΅
колСбания (Ξ΄ β€Ήβ€Ή Ο‰0)

ΠŸΠ΅Ρ€ΠΈΠΎΠ΄ ΠΊΠΎΠ»Π΅Π±Π°Π½ΠΈΠΉ T опрСдСляСтся
Π²Ρ‹Ρ€Π°ΠΆΠ΅Π½ΠΈΠ΅ΠΌ

.
(5)

Π—Π°Ρ‚ΡƒΡ…Π°Π½ΠΈΠ΅ ΠΊΠΎΠ»Π΅Π±Π°Π½ΠΈΠΉ принято
Ρ…Π°Ρ€Π°ΠΊΡ‚Π΅Ρ€ΠΈΠ·ΠΎΠ²Π°Ρ‚ΡŒ логарифмичСским
Π΄Π΅ΠΊΡ€Π΅ΠΌΠ΅Π½Ρ‚ΠΎΠΌ затухания

,
(6)

ΠΎΠΏΡ€Π΅Π΄Π΅Π»ΡΡŽΡ‰ΠΈΠΌ ΠΎΡ‚Π½ΠΎΡΠΈΡ‚Π΅Π»ΡŒΠ½ΠΎΠ΅
ΡƒΠΌΠ΅Π½ΡŒΡˆΠ΅Π½ΠΈΠ΅ Π°ΠΌΠΏΠ»ΠΈΡ‚ΡƒΠ΄Ρ‹ ΠΊΠΎΠ»Π΅Π±Π°Π½ΠΈΠΉ Π·Π° ΠΎΠ΄ΠΈΠ½
ΠΏΠ΅Ρ€ΠΈΠΎΠ΄. Π§Π΅ΠΌ мСньшС логарифмичСский
Π΄Π΅ΠΊΡ€Π΅ΠΌΠ΅Π½Ρ‚ затухания, Ρ‚Π΅ΠΌ Π±Π»ΠΈΠΆΠ΅ колСбания
Π² ΠΊΠΎΠ½Ρ‚ΡƒΡ€Π΅ ΠΊ гармоничСским, Ρ‚Π΅ΠΌ большС
число ΠΊΠΎΠ»Π΅Π±Π°Π½ΠΈΠΉ Π² систСмС ΠΏΡ€ΠΈ Π·Π°Π΄Π°Π½Π½Ρ‹Ρ…
Π½Π°Ρ‡Π°Π»ΡŒΠ½Ρ‹Ρ… условиях:

,
(7)

Π³Π΄Π΅ Ne
– число ΠΊΠΎΠ»Π΅Π±Π°Π½ΠΈΠΉ, ΡΠΎΠ²Π΅Ρ€ΡˆΠ°Π΅ΠΌΡ‹Ρ… Π·Π° врСмя,
Π² Ρ‚Π΅Ρ‡Π΅Π½ΠΈΠ΅ ΠΊΠΎΡ‚ΠΎΡ€ΠΎΠ³ΠΎ Π°ΠΌΠΏΠ»ΠΈΡ‚ΡƒΠ΄Π° ΡƒΠΌΠ΅Π½ΡŒΡˆΠ°Π΅Ρ‚ΡΡ
Π² e Ρ€Π°Π· (e = 2,718 – основаниС Π½Π°Ρ‚ΡƒΡ€Π°Π»ΡŒΠ½ΠΎΠ³ΠΎ
Π»ΠΎΠ³Π°Ρ€ΠΈΡ„ΠΌΠ°).

ЭнСргСтичСскиС ΠΏΠΎΡ‚Π΅Ρ€ΠΈ Π² ΠΊΠΎΠ½Ρ‚ΡƒΡ€Π΅
принято Ρ…Π°Ρ€Π°ΠΊΡ‚Π΅Ρ€ΠΈΠ·ΠΎΠ²Π°Ρ‚ΡŒ Π΄ΠΎΠ±Ρ€ΠΎΡ‚Π½ΠΎΡΡ‚ΡŒΡŽ
ΠΊΠΎΠ½Ρ‚ΡƒΡ€Π° Q:

,
(8)

Π³Π΄Π΅ W – энСргия, запасСнная Π²
ΠΊΠΎΠ½Ρ‚ΡƒΡ€Π΅, Π° Ξ”W
– ΡƒΠΌΠ΅Π½ΡŒΡˆΠ΅Π½ΠΈΠ΅ энСргии Π·Π° ΠΏΠ΅Ρ€ΠΈΠΎΠ΄ Ξ”W
= W(t)
– W(t+T).

МоТно ΠΏΠΎΠΊΠ°Π·Π°Ρ‚ΡŒ, Ρ‡Ρ‚ΠΎ Π΄ΠΎΠ±Ρ€ΠΎΡ‚Π½ΠΎΡΡ‚ΡŒ
ΠΊΠΎΠ½Ρ‚ΡƒΡ€Π° ΠΎΠ±Ρ€Π°Ρ‚Π½ΠΎ ΠΏΡ€ΠΎΠΏΠΎΡ€Ρ†ΠΈΠΎΠ½Π°Π»ΡŒΠ½Π°
логарифмичСскому Π΄Π΅ΠΊΡ€Π΅ΠΌΠ΅Π½Ρ‚Ρƒ затухания:

.
(9)

Π‘ ΡƒΠ²Π΅Π»ΠΈΡ‡Π΅Π½ΠΈΠ΅ΠΌ Π°ΠΊΡ‚ΠΈΠ²Π½ΠΎΠ³ΠΎ
сопротивлСния R логарифмичСский Π΄Π΅ΠΊΡ€Π΅ΠΌΠ΅Π½Ρ‚
затухания Ξ» растСт, Π° Π΄ΠΎΠ±Ρ€ΠΎΡ‚Π½ΠΎΡΡ‚ΡŒ ΠΊΠΎΠ½Ρ‚ΡƒΡ€Π°
Q ΡƒΠΌΠ΅Π½ΡŒΡˆΠ°Π΅Ρ‚ΡΡ.

БоотвСтствСнно ΡƒΠΌΠ΅Π½ΡŒΡˆΠ°Π΅Ρ‚ΡΡ
число ΠΊΠΎΠ»Π΅Π±Π°Π½ΠΈΠΉ Ne.
ΠŸΡ€ΠΈ R > 2
колСбания Π² ΠΊΠΎΠ½Ρ‚ΡƒΡ€Π΅ Π²ΠΎΠΎΠ±Ρ‰Π΅ Π½Π΅ Π½Π°Π±Π»ΡŽΠ΄Π°ΡŽΡ‚ΡΡ
(рис. 3).

Π—Π½Π°Ρ‡Π΅Π½ΠΈΠ΅

,
(10)

ΠΏΡ€ΠΈ ΠΊΠΎΡ‚ΠΎΡ€ΠΎΠΌ ΠΊΠΎΠ»Π΅Π±Π°Ρ‚Π΅Π»ΡŒΠ½Ρ‹ΠΉ Ρ€Π΅ΠΆΠΈΠΌ
ΠΏΠ΅Ρ€Π΅Ρ…ΠΎΠ΄ΠΈΡ‚ Π² апСриодичСский, называСтся
критичСским сопротивлСниСм RΠΊΡ€
ΠΊΠΎΠ½Ρ‚ΡƒΡ€Π°.

Рис. 3. АпСриодичСский Ρ€Π΅ΠΆΠΈΠΌ
(Ξ΄>Ο‰0, R>RΠΊΡ€)

ΠžΠŸΠ˜Π‘ΠΠΠ˜Π• Π£Π‘Π’ΠΠΠžΠ’ΠšΠ˜
И ΠœΠ•Π’ΠžΠ”Π Π˜Π—ΠœΠ•Π Π•ΠΠ˜Π™

Π’ Π΄Π°Π½Π½ΠΎΠΉ Ρ€Π°Π±ΠΎΡ‚Π΅ для исслСдования
Π·Π°Ρ‚ΡƒΡ…Π°ΡŽΡ‰ΠΈΡ… ΠΊΠΎΠ»Π΅Π±Π°Π½ΠΈΠΉ Π² Ρ€Π΅Π°Π»ΡŒΠ½ΠΎΠΌ
ΠΊΠΎΠ»Π΅Π±Π°Ρ‚Π΅Π»ΡŒΠ½ΠΎΠΌ ΠΊΠΎΠ½Ρ‚ΡƒΡ€Π΅, Π²ΠΊΠ»ΡŽΡ‡Π°ΡŽΡ‰Π΅ΠΌ
Π°ΠΊΡ‚ΠΈΠ²Π½ΠΎΠ΅ сопротивлСниС R, примСняСтся
осциллограф. ΠŸΡ€ΠΈ этом Ρ‡Π΅Ρ€Π΅Π· Π³Π΅Π½Π΅Ρ€Π°Ρ‚ΠΎΡ€
Π·Π²ΡƒΠΊΠΎΠ²Ρ‹Ρ… ΠΊΠΎΠ»Π΅Π±Π°Π½ΠΈΠΉ производится
пСриодичСская подзарядка кондСнсатора,
Ρ‚.Π΅. кривая Π·Π°Ρ‚ΡƒΡ…Π°ΡŽΡ‰ΠΈΡ… ΠΊΠΎΠ»Π΅Π±Π°Π½ΠΈΠΉ
пСриодичСски повторяСтся.

Рис. 4.

ΠŸΡ€ΠΈ Π½Π΅ ΠΎΡ‡Π΅Π½ΡŒ Π±ΠΎΠ»ΡŒΡˆΠΈΡ… значСниях
сопротивлСния ΠΊΠΎΠ½Ρ‚ΡƒΡ€Π° (R < 2,
Π³Π΄Π΅ L
– ΠΈΠ½Π΄ΡƒΠΊΡ‚ΠΈΠ²Π½ΠΎΡΡ‚ΡŒ ΠΊΠ°Ρ‚ΡƒΡˆΠΊΠΈ, C – Π΅ΠΌΠΊΠΎΡΡ‚ΡŒ
кондСнсатора), Π½Π° экранС осциллографа
Π½Π°Π±Π»ΡŽΠ΄Π°Π΅Ρ‚ΡΡ ΠΊΠ°Ρ€Ρ‚ΠΈΠ½Π° Π·Π°Ρ‚ΡƒΡ…Π°ΡŽΡ‰ΠΈΡ… ΠΊΠΎΠ»Π΅Π±Π°Π½ΠΈΠΉ,
ΠΊΠ°ΠΊ это ΠΏΠΎΠΊΠ°Π·Π°Π½ΠΎ Π½Π° рис. 4, Ρ‡Ρ‚ΠΎ соотвСтствуСт
Π·Π°ΠΊΠΎΠ½Ρƒ измСнСния напряТСния

(11)

Если Π³Π΅Π½Π΅Ρ€Π°Ρ‚ΠΎΡ€ Π·Π°Π΄Π°Π΅Ρ‚ частоту
Ξ½, Ρ‚ΠΎ Ρ†ΠΈΠΊΠ» подзарядки кондСнсатора
длится (1/Ξ½) сСкунд, этому Π²Ρ€Π΅ΠΌΠ΅Π½ΠΈ Π½Π°
экранС осциллографа соотвСтствуСт
ΠΎΡ‚Ρ€Π΅Π·ΠΎΠΊ L1.
ΠŸΠ΅Ρ€ΠΈΠΎΠ΄Ρƒ ΠΊΠΎΠ»Π΅Π±Π°Π½ΠΈΠΉ T соотвСтствуСт
ΠΎΡ‚Ρ€Π΅Π·ΠΎΠΊ L. Π‘Π»Π΅Π΄ΠΎΠ²Π°Ρ‚Π΅Π»ΡŒΠ½ΠΎ, ΠΏΠ΅Ρ€ΠΈΠΎΠ΄ Π·Π°Ρ‚ΡƒΡ…Π°ΡŽΡ‰ΠΈΡ…
ΠΊΠΎΠ»Π΅Π±Π°Π½ΠΈΠΉ ΠΌΠΎΠΆΠ΅Ρ‚ Π±Ρ‹Ρ‚ΡŒ ΠΎΠΏΡ€Π΅Π΄Π΅Π»Π΅Π½ ΠΏΠΎ
Ρ„ΠΎΡ€ΠΌΡƒΠ»Π΅

.
(12)

Π˜Π·ΠΌΠ΅Ρ€ΠΈΠ² Π°ΠΌΠΏΠ»ΠΈΡ‚ΡƒΠ΄Ρƒ ΠΊΠΎΠ»Π΅Π±Π°Π½ΠΈΠΉ,
отстоящиС Π΄Ρ€ΡƒΠ³ ΠΎΡ‚ Π΄Ρ€ΡƒΠ³Π° Π½Π° врСмя, Ρ€Π°Π²Π½ΠΎΠ΅
ΠΏΠ΅Ρ€ΠΈΠΎΠ΄Ρƒ

ΠΌΠΎΠΆΠ½ΠΎ ΠΎΠΏΡ€Π΅Π΄Π΅Π»ΠΈΡ‚ΡŒ логарифмичСский
Π΄Π΅ΠΊΡ€Π΅ΠΌΠ΅Π½Ρ‚ затухания

ΠΈΠ»ΠΈ

(13)

ΠΈ Π΅Π³ΠΎ срСднСС Π·Π½Π°Ρ‡Π΅Π½ΠΈΠ΅

.
(14)

Π’ΠΎΠ³Π΄Π° коэффициСнт затухания
ΠΌΠΎΠΆΠ½ΠΎ Ρ€Π°ΡΡΡ‡ΠΈΡ‚Π°Ρ‚ΡŒ ΠΊΠ°ΠΊ

.
(15)

Π—Π½Π°Ρ‡Π΅Π½ΠΈΠ΅ сопротивлСния Π² ΠΊΠΎΠ½Ρ‚ΡƒΡ€Π΅
ΠΌΠΎΠΆΠ½ΠΎ ΠΈΠ·ΠΌΠ΅Π½ΡΡ‚ΡŒ, Π½Π°ΠΏΡ€ΠΈΠΌΠ΅Ρ€, с ΠΏΠΎΠΌΠΎΡ‰ΡŒΡŽ
ΠΌΠ°Π³Π°Π·ΠΈΠ½Π° сопротивлСний (RΠΌ).
Π—Π°Π²ΠΈΡΠΈΠΌΠΎΡΡ‚ΡŒ логарифмичСского коэффициСнта
затухания

ΠΎΡ‚ сопротивлСния RΠΌ
Π² ΠΊΠΎΠ½Ρ‚ΡƒΡ€Π΅ ΠΏΠΎΠΊΠ°Π·Π°Π½Π° Π½Π° рис. 5.

Рис. 5.

ПолноС Π°ΠΊΡ‚ΠΈΠ²Π½ΠΎΠ΅ сопротивлСниС
ΠΊΠΎΠ½Ρ‚ΡƒΡ€Π° R складываСтся ΠΈΠ· Π°ΠΊΡ‚ΠΈΠ²Π½ΠΎΠ³ΠΎ
сопротивлСния ΠΊΠ°Ρ‚ΡƒΡˆΠΊΠΈ индуктивности
Rk
ΠΈ сопротивлСния ΠΌΠ°Π³Π°Π·ΠΈΠ½Π° RΠΌ:

R = Rk
+ RΠΌ

Π—Π½Π°Ρ‡Π΅Π½ΠΈΠ΅ Rk
ΠΌΠΎΠΆΠ½ΠΎ ΠΎΠΏΡ€Π΅Π΄Π΅Π»ΠΈΡ‚ΡŒ, экстраполируя Π³Ρ€Π°Ρ„ΠΈΠΊ
Π΄ΠΎ значСния
.
Π’ΠΎΠ³Π΄Π° согласно Ρ„ΠΎΡ€ΠΌΡƒΠ»Π΅ для коэффициСнта
затухания Ξ΄ = R/2L, ΠΌΠΎΠΆΠ½ΠΎ Ρ€Π°ΡΡΡ‡ΠΈΡ‚Π°Ρ‚ΡŒ
ΠΈΠ½Π΄ΡƒΠΊΡ‚ΠΈΠ²Π½ΠΎΡΡ‚ΡŒ L ΠΊΠ°Ρ‚ΡƒΡˆΠΊΠΈ

(16)

ΠΈ, считая Ξ΄ << Ο‰0,
Π΅ΠΌΠΊΠΎΡΡ‚ΡŒ кондСнсатора ΠΈΠ· Ρ„ΠΎΡ€ΠΌΡƒΠ»Ρ‹ Вомсона

.
(17)

ΠŸΡ€ΠΈ Π±ΠΎΠ»ΡŒΡˆΠΈΡ… значСниях сопротивлСния
ΠΊΠΎΠ½Ρ‚ΡƒΡ€Π° (R
> 2)
Π½Π° экранС осциллографа Π±ΡƒΠ΄Π΅Ρ‚ Π½Π°Π±Π»ΡŽΠ΄Π°Ρ‚ΡŒΡΡ
апСриодичСский процСсс, ΠΏΠΎΠΊΠ°Π·Π°Π½Π½Ρ‹ΠΉ Π½Π°
рис. 6.

Рис. 6.

Π˜Π·ΠΌΠ΅Ρ€Π΅Π½ΠΈΡ логарифмичСского
Π΄Π΅ΠΊΡ€Π΅ΠΌΠ΅Π½Ρ‚Π° затухания Ξ» ΠΌΠΎΠΆΠ½ΠΎ ΠΏΡ€ΠΎΠ²ΠΎΠ΄ΠΈΡ‚ΡŒ
Ρ‚Π°ΠΊΠΆΠ΅ с ΠΏΠΎΠΌΠΎΡ‰ΡŒΡŽ Ρ„Π°Π·ΠΎΠ²ΠΎΠΉ ΠΊΡ€ΠΈΠ²ΠΎΠΉ u = f(i).
Если сопротивлСниС ΠΊΠΎΠ½Ρ‚ΡƒΡ€Π° R < 2,
Ρ‚ΠΎ Ρ„Π°Π·ΠΎΠ²Ρ‹Π΅ ΠΊΡ€ΠΈΠ²Ρ‹Π΅ ΠΈΠΌΠ΅ΡŽΡ‚ Π²ΠΈΠ΄, ΠΏΠΎΠΊΠ°Π·Π°Π½Π½Ρ‹ΠΉ
Π½Π° рис. 7.

Рис. 7.

Π˜Π·ΠΌΠ΅Ρ€ΡΡ Π·Π½Π°Ρ‡Π΅Π½ΠΈΠ΅ напряТСния,
Ρ€Π°Π·Π΄Π΅Π»Π΅Π½Π½Ρ‹Π΅ ΠΏΡ€ΠΎΠΌΠ΅ΠΆΡƒΡ‚ΠΊΠΎΠΌ Π²Ρ€Π΅ΠΌΠ΅Π½ΠΈ, Ρ€Π°Π²Π½Ρ‹ΠΌ
ΠΏΠ΅Ρ€ΠΈΠΎΠ΄Ρƒ, ΠΌΠΎΠΆΠ½ΠΎ ΠΏΠΎ Ρ„ΠΎΡ€ΠΌΡƒΠ»Π°ΠΌ (13) ΠΈ (14)
ΠΎΠΏΡ€Π΅Π΄Π΅Π»ΠΈΡ‚ΡŒ логарифмичСский Π΄Π΅ΠΊΡ€Π΅ΠΌΠ΅Π½Ρ‚
затухания Ξ». АналогичныС измСрСния
ΠΌΠΎΠΆΠ½ΠΎ провСсти ΠΈ ΠΏΠΎ значСниям Ρ‚ΠΎΠΊΠ°

ΠΈΠ»ΠΈ
.
(18)

ΠŸΡ€ΠΈ Π±ΠΎΠ»ΡŒΡˆΠΈΡ… значСниях сопротивлСния
ΠΊΠΎΠ½Ρ‚ΡƒΡ€Π° (R > 2)
фазовая кривая для апСриодичСского
разряда ΠΏΡ€ΠΈΠ½ΠΈΠΌΠ°Π΅Ρ‚ Π²ΠΈΠ΄, ΠΏΠΎΠΊΠ°Π·Π°Π½Π½Ρ‹ΠΉ Π½Π°
рис. 8.

Рис. 8.

Π­ΠšΠ‘ΠŸΠ•Π Π˜ΠœΠ•ΠΠ’ΠΠ›Π¬ΠΠΠ―
Π£Π‘Π’ΠΠΠžΠ’ΠšΠ

Для изучСния Ρ€Π΅Π°Π»ΡŒΠ½ΠΎΠ³ΠΎ
ΠΊΠΎΠ»Π΅Π±Π°Ρ‚Π΅Π»ΡŒΠ½ΠΎΠ³ΠΎ ΠΊΠΎΠ½Ρ‚ΡƒΡ€Π° ΠΏΡ€Π΅Π΄Π½Π°Π·Π½Π°Ρ‡Π΅Π½Π°
ΡΠΊΡΠΏΠ΅Ρ€ΠΈΠΌΠ΅Π½Ρ‚Π°Π»ΡŒΠ½Π°Ρ установка, ΠΎΠ±Ρ‰ΠΈΠΉ Π²ΠΈΠ΄
ΠΊΠΎΡ‚ΠΎΡ€ΠΎΠΉ ΠΏΡ€ΠΈΠ²Π΅Π΄Π΅Π½ Π½Π° рис. 9.

  1. ЀПЭ-10 – ΠΌΠΎΠ΄ΡƒΠ»ΡŒ

  2. ЀПЭ-ИП – источник питания

  3. ЀПЭ-08 – ΠΌΠΎΠ΄ΡƒΠ»ΡŒ

  4. ЀПЭ-МБ – ΠΌΠ°Π³Π°Π·ΠΈΠ½ сопротивлСний

  5. PO
    – осциллограф

  6. PQ
    – Π³Π΅Π½Π΅Ρ€Π°Ρ‚ΠΎΡ€ Π·Π²ΡƒΠΊΠΎΠ²ΠΎΠΉ частоты

НапряТСниС ΠΎΡ‚ источника питания
ЀПЭ-ИП ΠΈ ΠΎΡ‚ Π³Π΅Π½Π΅Ρ€Π°Ρ‚ΠΎΡ€Π° подаСтся Π½Π° ΠΌΠΎΠ΄ΡƒΠ»ΡŒ
ЀПЭ-08 (ΠΏΡ€Π΅ΠΎΠ±Ρ€Π°Π·ΠΎΠ²Π°Ρ‚Π΅Π»ΡŒ ΠΈΠΌΠΏΡƒΠ»ΡŒΡΠΎΠ²), ΠΈ
Π΄Π°Π»Π΅Π΅ – Π½Π° Π²Ρ…ΠΎΠ΄ ΠΊΠΎΠ»Π΅Π±Π°Ρ‚Π΅Π»ΡŒΠ½ΠΎΠ³ΠΎ ΠΊΠΎΠ½Ρ‚ΡƒΡ€Π°
(ΠΌΠΎΠ΄ΡƒΠ»ΡŒ ЀПЭ-10) для цикличСской подзарядка.
Π’Ρ‹Ρ…ΠΎΠ΄Ρ‹ »X» ΠΈ »Y» модуля ЀПЭ-10 ΡΠΎΠ΅Π΄ΠΈΠ½ΡΡŽΡ‚ΡΡ
с ΡΠΎΠΎΡ‚Π²Π΅Ρ‚ΡΡ‚Π²ΡƒΡŽΡ‰ΠΈΠΌΠΈ Π³Π½Π΅Π·Π΄Π°ΠΌΠΈ осциллографа
PO. ΠšΡ€ΠΎΠΌΠ΅ Ρ‚ΠΎΠ³ΠΎ, ΠΊ ΠΊΠΎΠ»Π΅Π±Π°Ρ‚Π΅Π»ΡŒΠ½ΠΎΠΌΡƒ ΠΊΠΎΠ½Ρ‚ΡƒΡ€Ρƒ
модуля ЀПЭ-10 подсоСдиняСтся ΠΌΠ°Π³Π°Π·ΠΈΠ½
сопротивлСния ЀПЭ-МБ, Ρ‡Ρ‚ΠΎ позволяСт
ΠΈΠ·ΠΌΠ΅Π½ΡΡ‚ΡŒ Π²Π΅Π»ΠΈΡ‡ΠΈΠ½Ρƒ Π°ΠΊΡ‚ΠΈΠ²Π½ΠΎΠ³ΠΎ сопротивлСния
Π² ΠΊΠΎΠ½Ρ‚ΡƒΡ€Π΅.

ΠŸΠžΠ Π―Π”ΠžΠš Π’Π«ΠŸΠžΠ›ΠΠ•ΠΠ˜Π―
Π ΠΠ‘ΠžΠ’Π«

Π—Π°Π΄Π°Π½ΠΈΠ΅ 1. Π˜Π·ΠΌΠ΅Ρ€Π΅Π½ΠΈΠ΅
ΠΏΠ΅Ρ€ΠΈΠΎΠ΄Π° T
Π·Π°Ρ‚ΡƒΡ…Π°ΡŽΡ‰ΠΈΡ… ΠΊΠΎΠ»Π΅Π±Π°Π½ΠΈΠΉ, логарифмичСского
Π΄Π΅ΠΊΡ€Π΅ΠΌΠ΅Π½Ρ‚Π° Ξ» ΠΈ ΠΏΠ°Ρ€Π°ΠΌΠ΅Ρ‚Ρ€ΠΎΠ²

L, C, R ΠΊΠΎΠ»Π΅Π±Π°Ρ‚Π΅Π»ΡŒΠ½ΠΎΠ³ΠΎ ΠΊΠΎΠ½Ρ‚ΡƒΡ€Π°.

1. На ΠΌΠΎΠ΄ΡƒΠ»Π΅ ЀПэ-08 (ΠΏΡ€Π΅ΠΎΠ±Ρ€Π°Π·ΠΎΠ²Π°Ρ‚Π΅Π»Π΅
ΠΈΠΌΠΏΡƒΠ»ΡŒΡΠΎΠ²) Π½Π°ΠΆΠ°Ρ‚ΡŒ ΡΡ€Π΅Π΄Π½ΡŽΡŽ ΠΊΠ»Π°Π²ΠΈΡˆΡƒ

(см. рис. 9) ΠΈ ΠΏΡ€Π°Π²ΡƒΡŽ ΠΊΠ»Π°Π²ΠΈΡˆΡƒ β€œΡΠΊΠ²Π°ΠΆΠ½ΠΎΡΡ‚ΡŒ
грубо”.

2. На ΠΌΠ°Π³Π°Π·ΠΈΠ½Π΅ сопротивлСний
ЀПЭ-МБ ΡƒΡΡ‚Π°Π½ΠΎΠ²ΠΈΡ‚ΡŒ Π·Π½Π°Ρ‡Π΅Π½ΠΈΠ΅ сопротивлСния
Rм = 100 Ом.

3. На Π³Π΅Π½Π΅Ρ€Π°Ρ‚ΠΎΡ€Π΅ ΡƒΡΡ‚Π°Π½ΠΎΠ²ΠΈΡ‚ΡŒ
частоту Ξ½ = 250 Π“Ρ† ΠΈ напряТСниС 2-3 Π’.

4. Π’ΠΊΠ»ΡŽΡ‡ΠΈΡ‚ΡŒ Ρ‚ΡƒΠΌΠ±Π»Π΅Ρ€ β€œΠ‘Π΅Ρ‚ΡŒβ€ Π½Π°
ΠΏΡ€ΠΈΠ±ΠΎΡ€Π°Ρ….

5. ΠŸΠΎΠ»ΡƒΡ‡ΠΈΡ‚ΡŒ Π½Π° экранС осциллографа
ΡƒΡΡ‚ΠΎΠΉΡ‡ΠΈΠ²ΡƒΡŽ ΠΊΠ°Ρ€Ρ‚ΠΈΠ½Ρƒ Π·Π°Ρ‚ΡƒΡ…Π°ΡŽΡ‰ΠΈΡ… ΠΊΠΎΠ»Π΅Π±Π°Π½ΠΈΠΉ
(см. рис. 4).

6. Π˜Π·ΠΌΠ΅Ρ€ΠΈΡ‚ΡŒ Π½Π° экранС ΠΎΡ‚Ρ€Π΅Π·ΠΊΠΈ L ΠΈ
L1 (см. рис.
4). По Ρ„ΠΎΡ€ΠΌΡƒΠ»Π΅ (12) Ρ€Π°ΡΡΡ‡ΠΈΡ‚Π°Ρ‚ΡŒ ΠΏΠ΅Ρ€ΠΈΠΎΠ΄
ΠΊΠΎΠ»Π΅Π±Π°Π½ΠΈΠΉ ΠΈ Π΅Π³ΠΎ Π·Π½Π°Ρ‡Π΅Π½ΠΈΠ΅ Π·Π°ΠΏΠΈΡΠ°Ρ‚ΡŒ Π²
Ρ‚Π°Π±Π». 1.

Π’Π°Π±Π»ΠΈΡ†Π° 1

β„– ΠΏ.ΠΏ.

Π’

с

RΠΌ

Ом

Ξ»/

Ξ»//

Ξ²

с1

Rk

Ом

L

Π“Π½

Π“Π½

C

Π€

ДСлСния

1

100

2

300

3

500

4

600

  1. Π˜Π·ΠΌΠ΅Ρ€ΠΈΡ‚ΡŒ Π² дСлСниях Π°ΠΌΠΏΠ»ΠΈΡ‚ΡƒΠ΄Ρƒ
    ΠΊΠΎΠ»Π΅Π±Π°Π½ΠΈΠΉ
    ,
    ,

    Π½Π° экранС осциллографа. По Ρ„ΠΎΡ€ΠΌΡƒΠ»Π°ΠΌ
    (13) ΠΈ (14) Ρ€Π°ΡΡΡ‡ΠΈΡ‚Π°Ρ‚ΡŒ для ΠΊΠ°ΠΆΠ΄ΠΎΠΉ ΠΏΠ°Ρ€Ρ‹
    ΠΈΠ·ΠΌΠ΅Ρ€Π΅Π½ΠΈΠΉ логарифмичСский Π΄Π΅ΠΊΡ€Π΅ΠΌΠ΅Π½Ρ‚
    затухания Ξ» ΠΈ Π³ΠΎ срСднСС Π·Π½Π°Ρ‡Π΅Π½ΠΈΠ΅
    .
    Π Π΅Π·ΡƒΠ»ΡŒΡ‚Π°Ρ‚Ρ‹ занСсти Π² Ρ‚Π°Π±Π». 1.

  2. Π’Ρ‹Ρ‡ΠΈΡΠ»ΠΈΡ‚ΡŒ коэффициСнт затухания
    ΠΏΠΎ Ρ„ΠΎΡ€ΠΌΡƒΠ»Π΅ (15).

  3. ΠŸΠΎΠ²Ρ‚ΠΎΡ€ΠΈΡ‚ΡŒ ΠΈΠ·ΠΌΠ΅Ρ€Π΅Π½ΠΈΠ΅ ΠΏΠΎ ΠΏ.ΠΏ. 9, 10
    для Π·Π½Π°Ρ‡Π΅Π½ΠΈΠΉ Π½Π° ΠΌΠ°Π³Π°Π·ΠΈΠ½Π΅ сопротивлСний
    RΠΌ = 300, 500,
    600 Ом.

  4. ΠŸΠΎΡΡ‚Ρ€ΠΎΠΈΡ‚ΡŒ Π³Ρ€Π°Ρ„ΠΈΠΊ зависимости
    логарифмичСского Π΄Π΅ΠΊΡ€Π΅ΠΌΠ΅Π½Ρ‚Π° затухания
    ΠΎΡ‚ Π·Π½Π°Ρ‡Π΅Π½ΠΈΠΉ ΠΌΠ°Π³Π°Π·ΠΈΠ½Π° сопротивлСний

    = f(RΠΌ), ΠΊΠ°ΠΊ
    это ΠΏΠΎΠΊΠ°Π·Π°Π½ΠΎ Π½Π° рис. 5.

  5. Экстраполируя Π³Ρ€Π°Ρ„ΠΈΠΊ Π΄ΠΎ значСния
    0,
    ΠΏΠΎΠ»ΡƒΡ‡ΠΈΡ‚ΡŒ Π·Π½Π°Ρ‡Π΅Π½ΠΈΠ΅ Π°ΠΊΡ‚ΠΈΠ²Π½ΠΎΠ³ΠΎ сопротивлСния
    Rk
    ΠΊΠ°Ρ‚ΡƒΡˆΠΊΠΈ индуктивности.

  6. Для ΠΊΠ°ΠΆΠ΄ΠΎΠ³ΠΎ значСния RΠΌ
    Π²Ρ‹Ρ‡ΠΈΡΠ»ΠΈΡ‚ΡŒ ΠΈΠ½Π΄ΡƒΠΊΡ‚ΠΈΠ²Π½ΠΎΡΡ‚ΡŒ L ΠΊΠ°Ρ‚ΡƒΡˆΠΊΠΈ ΠΏΠΎ
    Ρ„ΠΎΡ€ΠΌΡƒΠ»Π΅ (16) ΠΈ Ρ€Π°ΡΡΡ‡ΠΈΡ‚Π°Ρ‚ΡŒ срСднСС
    арифмСтичСскоС Π·Π½Π°Ρ‡Π΅Π½ΠΈΠ΅

    для всСх ΠΈΠ·ΠΌΠ΅Ρ€Π΅Π½ΠΈΠΉ.

  7. ΠžΠΏΡ€Π΅Π΄Π΅Π»ΠΈΡ‚ΡŒ Π΅ΠΌΠΊΠΎΡΡ‚ΡŒ C кондСнсатора
    ΠΏΠΎ Ρ„ΠΎΡ€ΠΌΡƒΠ»Π΅ (17).

  8. ΠŸΠΎΠ΄ΠΎΠ±Ρ€Π°Ρ‚ΡŒ минимальноС
    (критичСскоС) Π·Π½Π°Ρ‡Π΅Π½ΠΈΠ΅ ΠΌΠ°Π³Π°Π·ΠΈΠ½Π°
    сопротивлСний
    ,
    ΠΏΡ€ΠΈ ΠΊΠΎΡ‚ΠΎΡ€ΠΎΠΌ Π½Π°Π±Π»ΡŽΠ΄Π°Π΅Ρ‚ΡΡ апСриодичСский
    разряд кондСнсатора (см. рис. 6). Π­Ρ‚ΠΎ
    Π·Π½Π°Ρ‡Π΅Π½ΠΈΠ΅ Π½Π΅ ΠΏΡ€Π΅Π²Ρ‹ΡˆΠ°Π΅Ρ‚ 2000 Ом. Π Π΅Π·ΡƒΠ»ΡŒΡ‚Π°Ρ‚
    Π·Π°ΠΏΠΈΡΠ°Ρ‚ΡŒ Π² Ρ‚Π°Π±Π». 2.

Π’Π°Π±Π»ΠΈΡ†Π° 2

  1. ΠŸΡ€ΠΎΠ²Π΅Ρ€ΠΈΡ‚ΡŒ Π²Ρ‹ΠΏΠΎΠ»Π½Π΅Π½ΠΈΠ΅ равСнства

ΠΈ Π½Π°ΠΉΡ‚ΠΈ ΠΎΡ‚Π½ΠΎΡΠΈΡ‚Π΅Π»ΡŒΠ½ΡƒΡŽ ΠΎΡˆΠΈΠ±ΠΊΡƒ

Π Π΅Π·ΡƒΠ»ΡŒΡ‚Π°Ρ‚Ρ‹
занСсти Π² Ρ‚Π°Π±Π». 2.

Π—Π°Π΄Π°Π½ΠΈΠ΅ 2.
ИсслСдованиС Ρ„Π°Π·ΠΎΠ²Ρ‹Ρ… ΠΊΡ€ΠΈΠ²Ρ‹Ρ….

  1. Π£ΡΡ‚Π°Π½ΠΎΠ²ΠΈΡ‚ΡŒ Π½Π° ΠΌΠ°Π³Π°Π·ΠΈΠ½Π΅
    сопротивлСний ЀПЭ-МБ Π·Π½Π°Ρ‡Π΅Π½ΠΈΠ΅ RΠΌ
    = 100 Ом.

  2. Π’ΠΊΠ»ΡŽΡ‡ΠΈΡ‚ΡŒ осциллограф. ΠŸΠΎΠΌΠ΅ΡΡ‚ΠΈΡ‚ΡŒ
    Ρ„Π°Π·ΠΎΠ²ΡƒΡŽ ΠΊΡ€ΠΈΠ²ΡƒΡŽ Π² Ρ†Π΅Π½Ρ‚Ρ€Π΅ экрана.

  3. Π˜Π·ΠΌΠ΅Ρ€ΠΈΡ‚ΡŒ Π°ΠΌΠΏΠ»ΠΈΡ‚ΡƒΠ΄Ρƒ ΠΊΠΎΠ»Π΅Π±Π°Π½ΠΈΠΉ

    ΠΈ ΠΏΠΎ Ρ„ΠΎΡ€ΠΌΡƒΠ»Π°ΠΌ (13) ΠΈ (14) Ρ€Π°ΡΡΡ‡ΠΈΡ‚Π°Ρ‚ΡŒ для
    ΠΊΠ°ΠΆΠ΄ΠΎΠΉ ΠΏΠ°Ρ€Ρ‹ Π·Π½Π°Ρ‡Π΅Π½ΠΈΠΉ логарифмичСский
    Π΄Π΅ΠΊΡ€Π΅ΠΌΠ΅Π½Ρ‚ затухания Ξ» ΠΈ Π΅Π³ΠΎ срСднСС
    Π·Π½Π°Ρ‡Π΅Π½ΠΈΠ΅
    .
    Π Π΅Π·ΡƒΠ»ΡŒΡ‚Π°Ρ‚Ρ‹ занСсти Π² Ρ‚Π°Π±Π». 3.

Π’Π°Π±Π»ΠΈΡ†Π° 3

β„– ΠΏ.ΠΏ.

RΠΌ

Ом

Ξ»/

Ξ»//

Ξ»/

Ξ»//

ДСлСния

ДСлСния

  1. Π˜Π·ΠΌΠ΅Ρ€ΠΈΡ‚ΡŒ Π°ΠΌΠΏΠ»ΠΈΡ‚ΡƒΠ΄Ρƒ ΠΊΠΎΠ»Π΅Π±Π°Π½ΠΈΠΉ
    ,
    ,

    ;
    ΠΏΠΎ Ρ„ΠΎΡ€ΠΌΡƒΠ»Π°ΠΌ Ρ€Π°ΡΡΡ‡ΠΈΡ‚Π°Ρ‚ΡŒ для ΠΊΠ°ΠΆΠ΄ΠΎΠΉ ΠΏΠ°Ρ€Ρ‹
    ΠΈΠ·ΠΌΠ΅Ρ€Π΅Π½ΠΈΠΉ логарифмичСски Π΄Π΅ΠΊΡ€Π΅ΠΌΠ΅Π½Ρ‚
    затухания Ξ» ΠΈ Π½Π°ΠΉΡ‚ΠΈ Π΅Π³ΠΎ срСднСС Π·Π½Π°Ρ‡Π΅Π½ΠΈΠ΅
    .
    Π Π΅Π·ΡƒΠ»ΡŒΡ‚Π°Ρ‚Ρ‹ занСсти Π² Ρ‚Π°Π±Π». 3.

  2. ΠŸΠΎΠ²Ρ‚ΠΎΡ€ΠΈΡ‚ΡŒ измСрСния ΠΏΠΎ ΠΏ.ΠΏ. 4,5
    для Π·Π½Π°Ρ‡Π΅Π½ΠΈΠΉ сопротивлСния ΠΌΠ°Π³Π°Π·ΠΈΠ½Π°
    RΠΌ = 200, 300,
    400 Ом.

  3. Π£Π²Π΅Π»ΠΈΡ‡ΠΈΡ‚ΡŒ сопротивлСниС Π½Π°
    ΠΌΠ°Π³Π°Π·ΠΈΠ½Π΅ сопротивлСний, ΠΏΠΎΠ»ΡƒΡ‡ΠΈΡ‚ΡŒ
    Ρ„Π°Π·ΠΎΠ²ΡƒΡŽ ΠΊΡ€ΠΈΠ²ΡƒΡŽ для апСриодичСского
    разряда кондСнсатора, ΠΊΠ°ΠΊ ΠΏΠΎΠΊΠ°Π·Π°Π½ΠΎ Π½Π°
    рис. 8. Π’ΠΈΠ΄ ΠΏΠΎΠ»ΡƒΡ‡Π΅Π½Π½ΠΎΠΉ ΠΊΡ€ΠΈΠ²ΠΎΠΉ Π·Π°Ρ€ΠΈΡΠΎΠ²Π°Ρ‚ΡŒ
    Π² ΠΆΡƒΡ€Π½Π°Π» наблюдСний.

ΠšΠžΠΠ’Π ΠžΠ›Π¬ΠΠ«Π•
Π’ΠžΠŸΠ ΠžΠ‘Π«

  1. ΠΠ°Ρ‡Π΅Ρ€Ρ‚ΠΈΡ‚ΡŒ схСму ΠΊΠΎΠ»Π΅Π±Π°Ρ‚Π΅Π»ΡŒΠ½ΠΎΠ³ΠΎ
    ΠΊΠΎΠ½Ρ‚ΡƒΡ€Π° для Π·Π°Ρ‚ΡƒΡ…Π°ΡŽΡ‰ΠΈΡ… ΠΊΠΎΠ»Π΅Π±Π°Π½ΠΈΠΉ ΠΈ
    ΠΎΠ±ΠΎΠ·Π½Π°Ρ‡ΠΈΡ‚ΡŒ входящиС Π² Π½Π΅Π³ΠΎ элСмСнты.

  2. ΠΠ°Ρ‡Π΅Ρ€Ρ‚ΠΈΡ‚ΡŒ Π³Ρ€Π°Ρ„ΠΈΠΊ Π·Π°Ρ‚ΡƒΡ…Π°ΡŽΡ‰ΠΈΡ…
    ΠΊΠΎΠ»Π΅Π±Π°Π½ΠΈΠΉ ΠΈ Π΄Π°Ρ‚ΡŒ ΠΎΠΏΡ€Π΅Π΄Π΅Π»Π΅Π½ΠΈΠ΅ Π΄Π΅ΠΊΡ€Π΅ΠΌΠ΅Π½Ρ‚Π°
    Π·Π°Ρ‚ΡƒΡ…Π°ΡŽΡ‰ΠΈΡ… ΠΊΠΎΠ»Π΅Π±Π°Π½ΠΈΠΉ, логарифмичСского
    Π΄Π΅ΠΊΡ€Π΅ΠΌΠ΅Π½Ρ‚Π° затухания.

  3. ΠŸΠΎΡΡΠ½ΠΈΡ‚ΡŒ физичСский смысл
    добротности ΠΊΠΎΠ½Ρ‚ΡƒΡ€Π°.

  4. ΠΠ°ΠΏΠΈΡΠ°Ρ‚ΡŒ Π·Π°ΠΊΠΎΠ½ измСнСния
    Π°ΠΌΠΏΠ»ΠΈΡ‚ΡƒΠ΄Ρ‹ Π·Π°Ρ‚ΡƒΡ…Π°ΡŽΡ‰ΠΈΡ… ΠΊΠΎΠ»Π΅Π±Π°Π½ΠΈΠΉ со
    Π²Ρ€Π΅ΠΌΠ΅Π½Π΅ΠΌ ΠΈ Π½Π°Ρ‡Π΅Ρ€Ρ‚ΠΈΡ‚ΡŒ Π³Ρ€Π°Ρ„ΠΈΠΊ этой
    зависимости.

  5. Π―Π²Π»ΡΡŽΡ‚ΡΡ Π»ΠΈ Π·Π°Ρ‚ΡƒΡ…Π°ΡŽΡ‰ΠΈΠ΅ колСбания
    гармоничСскими?

  6. ВывСсти Π΄ΠΈΡ„Ρ„Π΅Ρ€Π΅Π½Ρ†ΠΈΠ°Π»ΡŒΠ½ΠΎΠ΅
    ΡƒΡ€Π°Π²Π½Π΅Π½ΠΈΠ΅ Π·Π°Ρ‚ΡƒΡ…Π°ΡŽΡ‰ΠΈΡ… ΠΊΠΎΠ»Π΅Π±Π°Π½ΠΈΠΉ ΠΈ
    ΠΏΠΎΡΡΠ½ΠΈΡ‚ΡŒ смысл ΠΊΠ°ΠΆΠ΄ΠΎΠ³ΠΎ слагаСмого Π²
    уравнСниях.

  7. Π Π΅ΡˆΠΈΡ‚ΡŒ Π΄ΠΈΡ„Ρ„Π΅Ρ€Π΅Π½Ρ†ΠΈΠ°Π»ΡŒΠ½ΠΎΠ΅ ΡƒΡ€Π°Π²Π½Π΅Π½ΠΈΠ΅
    Π·Π°Ρ‚ΡƒΡ…Π°ΡŽΡ‰ΠΈΡ… ΠΊΠΎΠ»Π΅Π±Π°Π½ΠΈΠΉ, ΠΏΠΎΠ»ΡƒΡ‡ΠΈΡ‚ΡŒ Π²Ρ‹Ρ€Π°ΠΆΠ΅Π½ΠΈΠ΅
    для частоты Π·Π°Ρ‚ΡƒΡ…Π°ΡŽΡ‰ΠΈΡ… ΠΊΠΎΠ»Π΅Π±Π°Π½ΠΈΠΉ. Как
    зависит эта частота ΠΎΡ‚ L?

  8. РСшив Π΄ΠΈΡ„Ρ„Π΅Ρ€Π΅Π½Ρ†ΠΈΠ°Π»ΡŒΠ½ΠΎΠ΅ ΡƒΡ€Π°Π²Π½Π΅Π½ΠΈΠ΅
    Π·Π°Ρ‚ΡƒΡ…Π°ΡŽΡ‰ΠΈΡ… ΠΊΠΎΠ»Π΅Π±Π°Π½ΠΈΠΉ, ΠΏΠΎΠ»ΡƒΡ‡ΠΈΡ‚ΡŒ Π²Ρ‹Ρ€Π°ΠΆΠ΅Π½ΠΈΠ΅
    для Π°ΠΌΠΏΠ»ΠΈΡ‚ΡƒΠ΄Ρ‹ этих ΠΊΠΎΠ»Π΅Π±Π°Π½ΠΈΠΉ.

  9. Как Π±ΡƒΠ΄Π΅Ρ‚ ΠΌΠ΅Π½ΡΡ‚ΡŒΡΡ Ρ…Π°Ρ€Π°ΠΊΡ‚Π΅Ρ€
    ΠΊΠΎΠ»Π΅Π±Π°Π½ΠΈΠΉ ΠΏΡ€ΠΈ ΡƒΠ²Π΅Π»ΠΈΡ‡Π΅Π½ΠΈΠΈ Π°ΠΊΡ‚ΠΈΠ²Π½ΠΎΠ³ΠΎ
    сопротивлСния ΠΊΠΎΠ½Ρ‚ΡƒΡ€Π°? Анализ ΠΏΠΎΠ΄Ρ‚Π²Π΅Ρ€Π΄ΠΈΡ‚ΡŒ
    расчСтом.

  10. Как ΠΈΠ·ΠΌΠ΅Π½ΠΈΡ‚ΡŒΡΡ частота ΠΊΠΎΠ»Π΅Π±Π°Π½ΠΈΠΉ
    ΠΊΠΎΠ½Ρ‚ΡƒΡ€Π°, Ссли Π² Π΅Π³ΠΎ ΠΊΠ°Ρ‚ΡƒΡˆΠΊΡƒ ввСсти
    ΠΆΠ΅Π»Π΅Π·Π½Ρ‹ΠΉ (ΠΌΠ΅Π΄Π½Ρ‹ΠΉ) сСрдСчник? Ссли
    ΡƒΠ²Π΅Π»ΠΈΡ‡ΠΈΡ‚ΡŒ расстояниС ΠΌΠ΅ΠΆΠ΄Ρƒ пластинами
    кондСнсатора?

  11. ΠœΠΎΠ³ΡƒΡ‚ Π»ΠΈ ΡΡƒΡ‰Π΅ΡΡ‚Π²ΠΎΠ²Π°Ρ‚ΡŒ колСбания
    Π² ΠΊΠΎΠ½Ρ‚ΡƒΡ€Π΅ ΠΏΡ€ΠΈ R0?
    ΠŸΡ€ΠΈ L0?

  12. ΠŸΠΎΠΊΠ°Π·Π°Ρ‚ΡŒ, Ρ‡Ρ‚ΠΎ ΠΏΡ€ΠΈ ΠΌΠ°Π»ΠΎΠΌ Π·Π°Ρ‚ΡƒΡ…Π°Π½ΠΈΠΈ
    (Ξ΄<<Ο‰0)
    Π΄ΠΎΠ±Ρ€ΠΎΡ‚Π½ΠΎΡΡ‚ΡŒ опрСдСляСтся Π²Ρ‹Ρ€Π°ΠΆΠ΅Π½ΠΈΠ΅ΠΌ
    Q
    Ο‰0L/R
    = Ο‰0/2Ξ΄.

  13. ΠŸΠΎΠΊΠ°Π·Π°Ρ‚ΡŒ, Ρ‡Ρ‚ΠΎ ΠΏΡ€ΠΈ ΠΌΠ°Π»ΠΎΠΌ Π·Π°Ρ‚ΡƒΡ…Π°Π½ΠΈΠΈ
    Q

    RΠΊΡ€/2R.

  14. ΠœΠΎΠ³ΡƒΡ‚ Π»ΠΈ Π² ΠΊΠΎΠ½Ρ‚ΡƒΡ€Π΅, состоящСм
    ΠΈΠ· кондСнсатора ΠΈ Π°ΠΊΡ‚ΠΈΠ²Π½ΠΎΠ³ΠΎ сопротивлСния,
    Π²ΠΎΠ·Π½ΠΈΠΊΠ°Ρ‚ΡŒ свободныС колСбания?

  15. Какова энСргия кондСнсатора Π²
    ΠΊΠΎΠ»Π΅Π±Π°Ρ‚Π΅Π»ΡŒΠ½ΠΎΠΌ ΠΊΠΎΠ½Ρ‚ΡƒΡ€Π΅ Π² ΠΌΠΎΠΌΠ΅Π½Ρ‚Ρ‹
    максимумов Ρ‚ΠΎΠΊΠ° Π² ΠΊΠ°Ρ‚ΡƒΡˆΠΊΠ΅ Π² случаС,
    ΠΊΠΎΠ³Π΄Π° сопротивлСниС Π½ΠΈΡ‡Ρ‚ΠΎΠΆΠ½ΠΎ ΠΌΠ°Π»ΠΎ?

БПИБОК Π›Π˜Π’Π•Π ΠΠ’Π£Π Π«

  1. БавСльСв И.Π’. ΠšΡƒΡ€Ρ ΠΎΠ±Ρ‰Π΅ΠΉ Ρ„ΠΈΠ·ΠΈΠΊΠΈ:
    Π’ 3 Ρ‚. – М.: Наука, 1988.–В.2. – 496 с.

  2. Π’ΡƒΠ»ΡŒΡ‡ΠΈΠ½ΡΠΊΠΈΠΉ М.Π•. ΠšΠ°Ρ‡Π΅ΡΡ‚Π²Π΅Π½Π½Ρ‹Π΅
    Π·Π°Π΄Π°Ρ‡ΠΈ ΠΏΠΎ Ρ„ΠΈΠ·ΠΈΠΊΠ΅. – М.: ΠŸΡ€ΠΎΡΠ²Π΅Ρ‰Π΅Π½ΠΈΠ΅,
    1972. – 240 с.

  3. Π’Ρ€ΠΎΡ„ΠΈΠΌΠΎΠ²Π° Π’.И. ΠšΡƒΡ€Ρ Ρ„ΠΈΠ·ΠΈΠΊΠΈ. –
    М.: Π’Ρ‹ΡΡˆΠ°Ρ школа, 1998. – 543 с.

Π‘ΠΎΠΏΡ€ΠΎΡ‚ΠΈΠ²Π»Π΅Π½ΠΈΠ΅ кондСнсатора, тСория ΠΈ ΠΏΡ€ΠΈΠΌΠ΅Ρ€Ρ‹

Π‘ΠΎΠΏΡ€ΠΎΡ‚ΠΈΠ²Π»Π΅Π½ΠΈΠ΅ кондСнсатора постоянному Ρ‚ΠΎΠΊΡƒ

ΠšΠΎΠ½Π΄Π΅Π½ΡΠ°Ρ‚ΠΎΡ€ Ρ…Π°Ρ€Π°ΠΊΡ‚Π΅Ρ€ΠΈΠ·ΡƒΡŽΡ‚ ΠΏΡ€ΠΈ ΠΏΠΎΠΌΠΎΡ‰ΠΈ ряда ΠΏΠ°Ρ€Π°ΠΌΠ΅Ρ‚Ρ€ΠΎΠ² (Π΅ΠΌΠΊΠΎΡΡ‚ΡŒ, Ρ€Π°Π±ΠΎΡ‡Π΅Π΅ напряТСниС ΠΈ Ρ‚. Π΄), ΠΎΠ΄Π½ΠΎΠΉ ΠΈΠ· Ρ‚Π°ΠΊΠΈΡ… характСристик являСтся сопротивлСниС. ΠšΠΎΠ½Π΄Π΅Π½ΡΠ°Ρ‚ΠΎΡ€ практичСски Π½Π΅ пропускаСт постоянный элСктричСский Ρ‚ΠΎΠΊ. Π’ΠΎ Π΅ΡΡ‚ΡŒ сопротивлСниС кондСнсатора являСтся бСсконСчно большим для постоянного Ρ‚ΠΎΠΊΠ°, Π½ΠΎ это ΠΈΠ΄Π΅Π°Π»ΡŒΠ½Ρ‹ΠΉ случай. Π§Π΅Ρ€Π΅Π· Ρ€Π΅Π°Π»ΡŒΠ½Ρ‹ΠΉ диэлСктрик ΠΎΡ‡Π΅Π½ΡŒ ΠΌΠ°Π»Ρ‹ΠΉ Ρ‚ΠΎΠΊ ΠΏΡ€ΠΎΡ‚Π΅ΠΊΠ°Ρ‚ΡŒ ΠΌΠΎΠΆΠ΅Ρ‚. Π­Ρ‚ΠΎΡ‚ Ρ‚ΠΎΠΊ Π½Π°Π·Ρ‹Π²Π°ΡŽΡ‚ Ρ‚ΠΎΠΊΠΎΠΌ ΡƒΡ‚Π΅Ρ‡ΠΊΠΈ. Π’ΠΎΠΊ ΡƒΡ‚Π΅Ρ‡ΠΊΠΈ являСтся ΠΏΠΎΠΊΠ°Π·Π°Ρ‚Π΅Π»Π΅ΠΌ качСства диэлСктрика, ΠΊΠΎΡ‚ΠΎΡ€Ρ‹ΠΉ примСняСтся ΠΏΡ€ΠΈ ΠΈΠ·Π³ΠΎΡ‚ΠΎΠ²Π»Π΅Π½ΠΈΠΈ кондСнсатора. Π£ соврСмСнных кондСнсаторов Ρ‚ΠΎΠΊ ΡƒΡ‚Π΅Ρ‡ΠΊΠΈ составляСт Π½Π΅ΠΊΠΎΡ‚ΠΎΡ€Ρ‹Π΅ Π΄ΠΎΠ»ΠΈ ΠΌΠΈΠΊΡ€ΠΎΠ°ΠΌΠΏΠ΅Ρ€Π°. Π‘ΠΎΠΏΡ€ΠΎΡ‚ΠΈΠ²Π»Π΅Π½ΠΈΠ΅ кондСнсатора Π² Ρ‚Π°ΠΊΠΎΠΌ случаС ΠΌΠΎΠΆΠ½ΠΎ Π²Ρ‹Ρ‡ΠΈΡΠ»ΠΈΡ‚ΡŒ, ΠΈΡΠΏΠΎΠ»ΡŒΠ·ΡƒΡ Π·Π°ΠΊΠΎΠ½ Ома для участка Ρ†Π΅ΠΏΠΈ, зная Π²Π΅Π»ΠΈΡ‡ΠΈΠ½Ρƒ напряТСния, Π΄ΠΎ ΠΊΠΎΡ‚ΠΎΡ€ΠΎΠΉ заряТСн кондСнсатор ΠΈ Ρ‚ΠΎΠΊ ΡƒΡ‚Π΅Ρ‡ΠΊΠΈ. Но ΠΎΠ±Ρ‹Ρ‡Π½ΠΎ ΠΏΡ€ΠΈ Ρ€Π΅ΡˆΠ΅Π½ΠΈΠΈ ΡƒΡ‡Π΅Π±Π½Ρ‹Ρ… Π·Π°Π΄Π°Ρ‡ сопротивлСниС кондСнсатора постоянному Ρ‚ΠΎΠΊΡƒ ΡΡ‡ΠΈΡ‚Π°ΡŽΡ‚ бСсконСчно большим.

Π‘ΠΎΠΏΡ€ΠΎΡ‚ΠΈΠ²Π»Π΅Π½ΠΈΠ΅ кондСнсатора ΠΏΠ΅Ρ€Π΅ΠΌΠ΅Π½Π½ΠΎΠΌΡƒ Π½Π°ΠΏΡ€ΡΠΆΠ΅Π½ΠΈΡŽ

ΠŸΡ€ΠΈ Π²ΠΊΠ»ΡŽΡ‡Π΅Π½ΠΈΠΈ кондСнсатора Π² Ρ†Π΅ΠΏΡŒ с ΠΏΠ΅Ρ€Π΅ΠΌΠ΅Π½Π½Ρ‹ΠΌ Ρ‚ΠΎΠΊΠΎΠΌ, Ρ‚ΠΎΠΊ свободно ΠΏΡ€ΠΎΡ…ΠΎΠ΄ΠΈΡ‚ Ρ‡Π΅Ρ€Π΅Π· кондСнсатор. Π­Ρ‚ΠΎ ΠΎΠ±ΡŠΡΡΠ½ΡΠ΅Ρ‚ΡΡ ΠΎΡ‡Π΅Π½ΡŒ просто: происходит процСсс постоянной зарядки ΠΈ разрядки кондСнсатора. ΠŸΡ€ΠΈ этом говорят, Ρ‡Ρ‚ΠΎ Π² Ρ†Π΅ΠΏΠΈ присутствуСт СмкостноС сопротивлСниС кондСнсатора, ΠΏΠΎΠΌΠΈΠΌΠΎ Π°ΠΊΡ‚ΠΈΠ²Π½ΠΎΠ³ΠΎ сопротивлСния.

И Ρ‚Π°ΠΊ, кондСнсатор, ΠΊΠΎΡ‚ΠΎΡ€Ρ‹ΠΉ Π²ΠΊΠ»ΡŽΡ‡Π΅Π½ Π² Ρ†Π΅ΠΏΡŒ ΠΏΠ΅Ρ€Π΅ΠΌΠ΅Π½Π½ΠΎΠ³ΠΎ Ρ‚ΠΎΠΊΠ°, Π²Π΅Π΄Π΅Ρ‚ сСбя ΠΊΠ°ΠΊ сопротивлСниС, Ρ‚ΠΎ Π΅ΡΡ‚ΡŒ ΠΎΠΊΠ°Π·Ρ‹Π²Π°Π΅Ρ‚ влияниС Π½Π° силу Ρ‚ΠΎΠΊΠ°, Ρ‚Π΅ΠΊΡƒΡ‰ΡƒΡŽ Π² Ρ†Π΅ΠΏΠΈ. Π’Π΅Π»ΠΈΡ‡ΠΈΠ½Ρƒ Смкостного сопротивлСния ΠΎΠ±ΠΎΠ·Π½Π°Ρ‡ΠΈΠΌ ΠΊΠ°ΠΊ , Π΅Π³ΠΎ Π²Π΅Π»ΠΈΡ‡ΠΈΠ½Π° связана с частотой Ρ‚ΠΎΠΊΠ° ΠΈ ΠΎΠΏΡ€Π΅Π΄Π΅Π»Π΅Π½Π° Ρ„ΠΎΡ€ΠΌΡƒΠ»ΠΎΠΉ:

Β  Β 

Π³Π΄Π΅ – частота ΠΏΠ΅Ρ€Π΅ΠΌΠ΅Π½Π½ΠΎΠ³ΠΎ Ρ‚ΠΎΠΊΠ°; – угловая частота Ρ‚ΠΎΠΊΠ°; C – Π΅ΠΌΠΊΠΎΡΡ‚ΡŒ кондСнсатора.

Если кондСнсатор Π²ΠΊΠ»ΡŽΡ‡Π΅Π½ Π² Ρ†Π΅ΠΏΡŒ ΠΏΠ΅Ρ€Π΅ΠΌΠ΅Π½Π½ΠΎΠ³ΠΎ Ρ‚ΠΎΠΊΠ°, Ρ‚ΠΎ Π² Π½Π΅ΠΌ Π½Π΅ затрачиваСтся ΠΌΠΎΡ‰Π½ΠΎΡΡ‚ΡŒ, ΠΏΠΎΡ‚ΠΎΠΌΡƒ Ρ‡Ρ‚ΠΎ Ρ„Π°Π·Π° Ρ‚ΠΎΠΊΠ° сдвинута ΠΏΠΎ ΠΎΡ‚Π½ΠΎΡˆΠ΅Π½ΠΈΡŽ ΠΊ Π½Π°ΠΏΡ€ΡΠΆΠ΅Π½ΠΈΡŽ Π½Π° . Если Ρ€Π°ΡΡΠΌΠΎΡ‚Ρ€Π΅Ρ‚ΡŒ ΠΎΠ΄ΠΈΠ½ ΠΏΠ΅Ρ€ΠΈΠΎΠ΄ колСбания Ρ‚ΠΎΠΊΠ° Π² Ρ†Π΅ΠΏΠΈ (T), Ρ‚ΠΎ происходит ΡΠ»Π΅Π΄ΡƒΡŽΡ‰Π΅Π΅: ΠΏΡ€ΠΈ зарядС кондСнсатора (это составляСт ) энСргия Π² ΠΏΠΎΠ»Π΅ кондСнсатора запасаСтся; Π½Π° ΡΠ»Π΅Π΄ΡƒΡŽΡ‰Π΅ΠΌ ΠΎΡ‚Ρ€Π΅Π·ΠΊΠ΅ Π²Ρ€Π΅ΠΌΠ΅Π½ΠΈ () кондСнсатор разряТаСтся ΠΈ ΠΎΡ‚Π΄Π°Π΅Ρ‚ ΡΠ½Π΅Ρ€Π³ΠΈΡŽ Π² Ρ†Π΅ΠΏΡŒ. ΠŸΠΎΡΡ‚ΠΎΠΌΡƒ ёмкостноС сопротивлСниС Π½Π°Π·Ρ‹Π²Π°ΡŽΡ‚ Ρ€Π΅Π°ΠΊΡ‚ΠΈΠ²Π½Ρ‹ΠΌ (Π±Π΅Π·Π²Π°Ρ‚Ρ‚Π½Ρ‹ΠΌ).

Π‘Π»Π΅Π΄ΡƒΠ΅Ρ‚ Π·Π°ΠΌΠ΅Ρ‚ΠΈΡ‚ΡŒ, Ρ‡Ρ‚ΠΎ Π² ΠΊΠ°ΠΆΠ΄ΠΎΠΌ Ρ€Π΅Π°Π»ΡŒΠ½ΠΎΠΌ кондСнсаторС Ρ€Π΅Π°Π»ΡŒΠ½Π°Ρ ΠΌΠΎΡ‰Π½ΠΎΡΡ‚ΡŒ (ΠΌΠΎΡ‰Π½ΠΎΡΡ‚ΡŒ ΠΏΠΎΡ‚Π΅Ρ€ΡŒ) всС ΠΆΠ΅ тратится, ΠΏΡ€ΠΈ Ρ‚Π΅Ρ‡Π΅Π½ΠΈΠΈ Ρ‡Π΅Ρ€Π΅Π· Π½Π΅Π³ΠΎ ΠΏΠ΅Ρ€Π΅ΠΌΠ΅Π½Π½ΠΎΠ³ΠΎ Ρ‚ΠΎΠΊΠ°. Π­Ρ‚ΠΎ Π²Ρ‹Π·Π²Π°Π½ΠΎ Ρ‚Π΅ΠΌ, Ρ‡Ρ‚ΠΎ происходят измСнСния Π² состоянии диэлСктрика кондСнсатора.{ — \beta t} \sin \omega t$. Найти напряТСниС Π½Π° кондСнсаторС Π² зависимости ΠΎΡ‚ Π²Ρ€Π΅ΠΌΠ΅Π½ΠΈ ΠΈ, Π² частности, Π² ΠΌΠΎΠΌΠ΅Π½Ρ‚ $t = 0$.


ΠŸΠΎΠ΄Ρ€ΠΎΠ±Π½Π΅Π΅

Π—Π°Π΄Π°Ρ‡Π° ΠΏΠΎ Ρ„ΠΈΠ·ΠΈΠΊΠ΅ — 7855

ΠšΠΎΠ»Π΅Π±Π°Ρ‚Π΅Π»ΡŒΠ½Ρ‹ΠΉ ΠΊΠΎΠ½Ρ‚ΡƒΡ€ состоит ΠΈΠ· кондСнсатора Смкости $C = 4,0 ΠΌΠΊΠ€$ ΠΈ ΠΊΠ°Ρ‚ΡƒΡˆΠΊΠΈ с ΠΈΠ½Π΄ΡƒΠΊΡ‚ΠΈΠ²Π½ΠΎΡΡ‚ΡŒΡŽ $L = 2,0 ΠΌΠ“$ ΠΈ Π°ΠΊΡ‚ΠΈΠ²Π½Ρ‹ΠΌ сопротивлСниСм $R = 10 Ом$. Найти ΠΎΡ‚Π½ΠΎΡˆΠ΅Π½ΠΈΠ΅ энСргии ΠΌΠ°Π³Π½ΠΈΡ‚Π½ΠΎΠ³ΠΎ поля ΠΊΠ°Ρ‚ΡƒΡˆΠΊΠΈ ΠΊ энСргии элСктричСского поля кондСнсатора Π² ΠΌΠΎΠΌΠ΅Π½Ρ‚ максимума Ρ‚ΠΎΠΊΠ°.


ΠŸΠΎΠ΄Ρ€ΠΎΠ±Π½Π΅Π΅

Π—Π°Π΄Π°Ρ‡Π° ΠΏΠΎ Ρ„ΠΈΠ·ΠΈΠΊΠ΅ — 7856

НСкоторый ΠΊΠΎΠ»Π΅Π±Π°Ρ‚Π΅Π»ΡŒΠ½Ρ‹ΠΉ ΠΊΠΎΠ½Ρ‚ΡƒΡ€ содСрТит Π΄Π²Π΅ ΠΏΠΎΡΠ»Π΅Π΄ΠΎΠ²Π°Ρ‚Π΅Π»ΡŒΠ½ΠΎ соСдинСнныС ΠΊΠ°Ρ‚ΡƒΡˆΠΊΠΈ с Π°ΠΊΡ‚ΠΈΠ²Π½Ρ‹ΠΌΠΈ сопротивлСниями $R_{1}$ ΠΈ $R_{2}$ ΠΈ индуктивностями $L_{1}$ ΠΈ $L_{2}$, ΠΏΡ€ΠΈΡ‡Π΅ΠΌ взаимная ΠΈΠ½Π΄ΡƒΠΊΡ‚ΠΈΠ²Π½ΠΎΡΡ‚ΡŒ ΠΈΡ… ΠΏΡ€Π΅Π½Π΅Π±Ρ€Π΅ΠΆΠΈΠΌΠΎ ΠΌΠ°Π»Π°. Π­Ρ‚ΠΈ ΠΊΠ°Ρ‚ΡƒΡˆΠΊΠΈ Π½Π°Π΄ΠΎ Π·Π°ΠΌΠ΅Π½ΠΈΡ‚ΡŒ ΠΎΠ΄Π½ΠΎΠΉ Ρ‚Π°ΠΊ, Ρ‡Ρ‚ΠΎΠ±Ρ‹ частота ΠΈ Π΄ΠΎΠ±Ρ€ΠΎΡ‚Π½ΠΎΡΡ‚ΡŒ ΠΊΠΎΠ½Ρ‚ΡƒΡ€Π° Π½Π΅ измСнились. Найти ΠΈΠ½Π΄ΡƒΠΊΡ‚ΠΈΠ²Π½ΠΎΡΡ‚ΡŒ ΠΈ Π°ΠΊΡ‚ΠΈΠ²Π½ΠΎΠ΅ сопротивлСниС Ρ‚Π°ΠΊΠΎΠΉ ΠΊΠ°Ρ‚ΡƒΡˆΠΊΠΈ.


ΠŸΠΎΠ΄Ρ€ΠΎΠ±Π½Π΅Π΅

Π—Π°Π΄Π°Ρ‡Π° ΠΏΠΎ Ρ„ΠΈΠ·ΠΈΠΊΠ΅ — 7857

Найти врСмя, Π·Π° ΠΊΠΎΡ‚ΠΎΡ€ΠΎΠ΅ Π°ΠΌΠΏΠ»ΠΈΡ‚ΡƒΠ΄Π° ΠΊΠΎΠ»Π΅Π±Π°Π½ΠΈΠΉ Ρ‚ΠΎΠΊΠ° Π² ΠΊΠΎΠ½Ρ‚ΡƒΡ€Π΅ с Π΄ΠΎΠ±Ρ€ΠΎΡ‚Π½ΠΎΡΡ‚ΡŒΡŽ $Q = 5000$ ΡƒΠΌΠ΅Π½ΡŒΡˆΠΈΡ‚ΡΡ Π² $\eta = 2,0$ Ρ€Π°Π·Π°, Ссли частота ΠΊΠΎΠ»Π΅Π±Π°Π½ΠΈΠΉ $\nu = 2,2 ΠœΠ“Ρ†$.


ΠŸΠΎΠ΄Ρ€ΠΎΠ±Π½Π΅Π΅

Π—Π°Π΄Π°Ρ‡Π° ΠΏΠΎ Ρ„ΠΈΠ·ΠΈΠΊΠ΅ — 7858

ΠšΠΎΠ»Π΅Π±Π°Ρ‚Π΅Π»ΡŒΠ½Ρ‹ΠΉ ΠΊΠΎΠ½Ρ‚ΡƒΡ€ ΠΈΠΌΠ΅Π΅Ρ‚ Π΅ΠΌΠΊΠΎΡΡ‚ΡŒ $C = 10 ΠΌΠΊΠ€$, ΠΈΠ½Π΄ΡƒΠΊΡ‚ΠΈΠ²Π½ΠΎΡΡ‚ΡŒ $L = 25 ΠΌΠ“$ ΠΈ Π°ΠΊΡ‚ΠΈΠ²Π½ΠΎΠ΅ сопротивлСниС $R = 1,0 Ом$. Π§Π΅Ρ€Π΅Π· сколько ΠΊΠΎΠ»Π΅Π±Π°Π½ΠΈΠΉ Π°ΠΌΠΏΠ»ΠΈΡ‚ΡƒΠ΄Π° Ρ‚ΠΎΠΊΠ° Π² этом ΠΊΠΎΠ½Ρ‚ΡƒΡ€Π΅ ΡƒΠΌΠ΅Π½ΡŒΡˆΠΈΡ‚ΡΡ Π² $e$ Ρ€Π°Π·?


ΠŸΠΎΠ΄Ρ€ΠΎΠ±Π½Π΅Π΅

Π—Π°Π΄Π°Ρ‡Π° ΠΏΠΎ Ρ„ΠΈΠ·ΠΈΠΊΠ΅ — 7859

На сколько ΠΏΡ€ΠΎΡ†Π΅Π½Ρ‚ΠΎΠ² отличаСтся частота со свободных ΠΊΠΎΠ»Π΅Π±Π°Π½ΠΈΠΉ ΠΊΠΎΠ½Ρ‚ΡƒΡ€Π° с Π΄ΠΎΠ±Ρ€ΠΎΡ‚Π½ΠΎΡΡ‚ΡŒΡŽ $Q = 5,0$ ΠΎΡ‚ собствСнной частоты $\omega_{0}$ ΠΊΠΎΠ»Π΅Π±Π°Π½ΠΈΠΉ этого ΠΊΠΎΠ½Ρ‚ΡƒΡ€Π°?


ΠŸΠΎΠ΄Ρ€ΠΎΠ±Π½Π΅Π΅

Π—Π°Π΄Π°Ρ‡Π° ΠΏΠΎ Ρ„ΠΈΠ·ΠΈΠΊΠ΅ — 7860

Π’ схСмС (рис.) э. Π΄. с. элСмСнта $\mathcal{E} = 2,0 Π’$, Π΅Π³ΠΎ Π²Π½ΡƒΡ‚Ρ€Π΅Π½Π½Π΅Π΅ сопротивлСниС $r = 9,0 Ом$, Π΅ΠΌΠΊΠΎΡΡ‚ΡŒ кондСнсатора $C = 10 ΠΌΠΊΠ€$, ΠΈΠ½Π΄ΡƒΠΊΡ‚ΠΈΠ²Π½ΠΎΡΡ‚ΡŒ ΠΊΠ°Ρ‚ΡƒΡˆΠΊΠΈ $L = 100 ΠΌΠ“$ ΠΈ сопротивлСниС $R = 1,0 Ом$. Π’ Π½Π΅ΠΊΠΎΡ‚ΠΎΡ€Ρ‹ΠΉ ΠΌΠΎΠΌΠ΅Π½Ρ‚ ΠΊΠ»ΡŽΡ‡ К Ρ€Π°Π·ΠΎΠΌΠΊΠ½ΡƒΠ»ΠΈ. Найти ΡΠ½Π΅Ρ€Π³ΠΈΡŽ ΠΊΠΎΠ»Π΅Π±Π°Π½ΠΈΠΉ Π² ΠΊΠΎΠ½Ρ‚ΡƒΡ€Π΅:

Π°) нСпосрСдствСнно послС размыкания ΠΊΠ»ΡŽΡ‡Π°;

Π±) Ρ‡Π΅Ρ€Π΅Π· $t = 0,30 с$ послС размыкания ΠΊΠ»ΡŽΡ‡Π°.


ΠŸΠΎΠ΄Ρ€ΠΎΠ±Π½Π΅Π΅

Π—Π°Π΄Π°Ρ‡Π° ΠΏΠΎ Ρ„ΠΈΠ·ΠΈΠΊΠ΅ — 7861

Π’ ΠΊΠΎΠ½Ρ‚ΡƒΡ€Π΅, Π΄ΠΎΠ±Ρ€ΠΎΡ‚Π½ΠΎΡΡ‚ΡŒ ΠΊΠΎΡ‚ΠΎΡ€ΠΎΠ³ΠΎ $Q = 50$ ΠΈ собствСнная частота ΠΊΠΎΠ»Π΅Π±Π°Π½ΠΈΠΉ $\nu_{0} = 5,5 ΠΊΠ“Ρ†$, Π²ΠΎΠ·Π±ΡƒΠΆΠ΄Π°ΡŽΡ‚ΡΡ Π·Π°Ρ‚ΡƒΡ…Π°ΡŽΡ‰ΠΈΠ΅ колСбания. Π§Π΅Ρ€Π΅Π· сколько Π²Ρ€Π΅ΠΌΠ΅Π½ΠΈ энСргия, запасСнная Π² ΠΊΠΎΠ½Ρ‚ΡƒΡ€Π΅, ΡƒΠΌΠ΅Π½ΡŒΡˆΠΈΡ‚ΡΡ Π² $\eta = 2,0$ Ρ€Π°Π·Π°?


ΠŸΠΎΠ΄Ρ€ΠΎΠ±Π½Π΅Π΅

Π—Π°Π΄Π°Ρ‡Π° ΠΏΠΎ Ρ„ΠΈΠ·ΠΈΠΊΠ΅ — 7862

ΠšΠΎΠ»Π΅Π±Π°Ρ‚Π΅Π»ΡŒΠ½Ρ‹ΠΉ ΠΊΠΎΠ½Ρ‚ΡƒΡ€ содСрТит кондСнсатор с ΡƒΡ‚Π΅Ρ‡ΠΊΠΎΠΉ. Π•ΠΌΠΊΠΎΡΡ‚ΡŒ кондСнсатора $C$, Π΅Π³ΠΎ Π°ΠΊΡ‚ΠΈΠ²Π½ΠΎΠ΅ сопротивлСниС $R$. Π˜Π½Π΄ΡƒΠΊΡ‚ΠΈΠ²Π½ΠΎΡΡ‚ΡŒ ΠΊΠ°Ρ‚ΡƒΡˆΠΊΠΈ $L$. Π‘ΠΎΠΏΡ€ΠΎΡ‚ΠΈΠ²Π»Π΅Π½ΠΈΠ΅ ΠΊΠ°Ρ‚ΡƒΡˆΠΊΠΈ ΠΈ ΠΏΡ€ΠΎΠ²ΠΎΠ΄ΠΎΠ² ΠΏΡ€Π΅Π½Π΅Π±Ρ€Π΅ΠΆΠΈΠΌΠΎ ΠΌΠ°Π»ΠΎ. Найти:

Π°) частоту Π·Π°Ρ‚ΡƒΡ…Π°ΡŽΡ‰ΠΈΡ… ΠΊΠΎΠ»Π΅Π±Π°Π½ΠΈΠΉ Ρ‚Π°ΠΊΠΎΠ³ΠΎ ΠΊΠΎΠ½Ρ‚ΡƒΡ€Π°;

Π±) Π΅Π³ΠΎ Π΄ΠΎΠ±Ρ€ΠΎΡ‚Π½ΠΎΡΡ‚ΡŒ.


ΠŸΠΎΠ΄Ρ€ΠΎΠ±Π½Π΅Π΅

Π—Π°Π΄Π°Ρ‡Π° ΠΏΠΎ Ρ„ΠΈΠ·ΠΈΠΊΠ΅ — 7863

Найти Π΄ΠΎΠ±Ρ€ΠΎΡ‚Π½ΠΎΡΡ‚ΡŒ ΠΊΠΎΠ½Ρ‚ΡƒΡ€Π° с Π΅ΠΌΠΊΠΎΡΡ‚ΡŒΡŽ $C = 2,0 ΠΌΠΊΠ€$ ΠΈ ΠΈΠ½Π΄ΡƒΠΊΡ‚ΠΈΠ²Π½ΠΎΡΡ‚ΡŒΡŽ $L = 5,0 ΠΌΠ“$, Ссли Π½Π° ΠΏΠΎΠ΄Π΄Π΅Ρ€ΠΆΠ°Π½ΠΈΠ΅ Π² Π½Π΅ΠΌ Π½Π΅Π·Π°Ρ‚ΡƒΡ…Π°ΡŽΡ‰ΠΈΡ… ΠΊΠΎΠ»Π΅Π±Π°Π½ΠΈΠΉ с Π°ΠΌΠΏΠ»ΠΈΡ‚ΡƒΠ΄ΠΎΠΉ напряТСния Π½Π° кондСнсаторС $U_{m} = 1,0 Π’$ Π½Π΅ΠΎΠ±Ρ…ΠΎΠ΄ΠΈΠΌΠΎ ΠΏΠΎΠ΄Π²ΠΎΠ΄ΠΈΡ‚ΡŒ ΠΌΠΎΡ‰Π½ΠΎΡΡ‚ΡŒ $\langle P \rangle = 0,10 ΠΌΠ’Ρ‚$. Π—Π°Ρ‚ΡƒΡ…Π°Π½ΠΈΠ΅ ΠΊΠΎΠ»Π΅Π±Π°Π½ΠΈΠΉ Π² ΠΊΠΎΠ½Ρ‚ΡƒΡ€Π΅ достаточно ΠΌΠ°Π»ΠΎ.


ΠŸΠΎΠ΄Ρ€ΠΎΠ±Π½Π΅Π΅

Π—Π°Π΄Π°Ρ‡Π° ΠΏΠΎ Ρ„ΠΈΠ·ΠΈΠΊΠ΅ — 7864

ΠšΠ°ΠΊΡƒΡŽ ΡΡ€Π΅Π΄Π½ΡŽΡŽ ΠΌΠΎΡ‰Π½ΠΎΡΡ‚ΡŒ Π΄ΠΎΠ»ΠΆΠ΅Π½ ΠΏΠΎΡ‚Ρ€Π΅Π±Π»ΡΡ‚ΡŒ ΠΊΠΎΠ»Π΅Π±Π°Ρ‚Π΅Π»ΡŒΠ½Ρ‹ΠΉ ΠΊΠΎΠ½Ρ‚ΡƒΡ€ с Π°ΠΊΡ‚ΠΈΠ²Π½Ρ‹ΠΌ сопротивлСниСм $R = 0,45 Ом$, Ρ‡Ρ‚ΠΎΠ±Ρ‹ Π² Π½Π΅ΠΌ ΠΏΠΎΠ΄Π΄Π΅Ρ€ΠΆΠΈΠ²Π°Π»ΠΈΡΡŒ Π½Π΅Π·Π°Ρ‚ΡƒΡ…Π°ΡŽΡ‰ΠΈΠ΅ гармоничСскиС колСбания с Π°ΠΌΠΏΠ»ΠΈΡ‚ΡƒΠ΄ΠΎΠΉ Ρ‚ΠΎΠΊΠ° $I_{m} = 30 мА$?


ΠŸΠΎΠ΄Ρ€ΠΎΠ±Π½Π΅Π΅

Π—Π°Π΄Π°Ρ‡Π° ΠΏΠΎ Ρ„ΠΈΠ·ΠΈΠΊΠ΅ — 7865

ΠšΠΎΠ»Π΅Π±Π°Ρ‚Π΅Π»ΡŒΠ½Ρ‹ΠΉ ΠΊΠΎΠ½Ρ‚ΡƒΡ€ содСрТит кондСнсатор Π΅ΠΌΠΊΠΎΡΡ‚ΡŒΡŽ $C = 1,2 Π½Π€$ ΠΈ ΠΊΠ°Ρ‚ΡƒΡˆΠΊΡƒ с ΠΈΠ½Π΄ΡƒΠΊΡ‚ΠΈΠ²Π½ΠΎΡΡ‚ΡŒΡŽ $L = 6,0 ΠΌΠΊΠ“$ ΠΈ Π°ΠΊΡ‚ΠΈΠ²Π½Ρ‹ΠΌ сопротивлСниСм $R = 0,50 Ом$. ΠšΠ°ΠΊΡƒΡŽ ΡΡ€Π΅Π΄Π½ΡŽΡŽ ΠΌΠΎΡ‰Π½ΠΎΡΡ‚ΡŒ Π½ΡƒΠΆΠ½ΠΎ ΠΏΠΎΠ΄Π²ΠΎΠ΄ΠΈΡ‚ΡŒ ΠΊ ΠΊΠΎΠ½Ρ‚ΡƒΡ€Ρƒ, Ρ‡Ρ‚ΠΎΠ±Ρ‹ ΠΏΠΎΠ΄Π΄Π΅Ρ€ΠΆΠΈΠ²Π°Ρ‚ΡŒ Π² Π½Π΅ΠΌ Π½Π΅Π·Π°Ρ‚ΡƒΡ…Π°ΡŽΡ‰ΠΈΠ΅ гармоничСскиС колСбания с Π°ΠΌΠΏΠ»ΠΈΡ‚ΡƒΠ΄ΠΎΠΉ напряТСния Π½Π° кондСнсаторС $U_{m} = 10 Π’$?


ΠŸΠΎΠ΄Ρ€ΠΎΠ±Π½Π΅Π΅

Π—Π°Π΄Π°Ρ‡Π° ΠΏΠΎ Ρ„ΠΈΠ·ΠΈΠΊΠ΅ — 7866

Найти частоту Π·Π°Ρ‚ΡƒΡ…Π°ΡŽΡ‰ΠΈΡ… ΠΊΠΎΠ»Π΅Π±Π°Π½ΠΈΠΉ ΠΊΠΎΠ½Ρ‚ΡƒΡ€Π°, ΠΏΠΎΠΊΠ°Π·Π°Π½Π½ΠΎΠ³ΠΎ Π½Π° рис. Π•ΠΌΠΊΠΎΡΡ‚ΡŒ $C$, ΠΈΠ½Π΄ΡƒΠΊΡ‚ΠΈΠ²Π½ΠΎΡΡ‚ΡŒ $L$ ΠΈ Π°ΠΊΡ‚ΠΈΠ²Π½ΠΎΠ΅ сопротивлСниС $R$ ΠΏΡ€Π΅Π΄ΠΏΠΎΠ»Π°Π³Π°ΡŽΡ‚ΡΡ извСстными. Π’Ρ‹ΡΡΠ½ΠΈΡ‚ΡŒ, ΠΏΡ€ΠΈ ΠΊΠ°ΠΊΠΎΠΌ ΡΠΎΠΎΡ‚Π½ΠΎΡˆΠ΅Π½ΠΈΠΈ ΠΌΠ΅ΠΆΠ΄Ρƒ $C,L$ ΠΈ $R$ колСбания Π²ΠΎΠ·ΠΌΠΎΠΆΠ½Ρ‹.


ΠŸΠΎΠ΄Ρ€ΠΎΠ±Π½Π΅Π΅

Π—Π°Π΄Π°Ρ‡Π° ΠΏΠΎ Ρ„ΠΈΠ·ΠΈΠΊΠ΅ — 7867

Π˜ΠΌΠ΅ΡŽΡ‚ΡΡ Π΄Π²Π° ΠΊΠΎΠ»Π΅Π±Π°Ρ‚Π΅Π»ΡŒΠ½Ρ‹Ρ… ΠΊΠΎΠ½Ρ‚ΡƒΡ€Π° (рис.) с кондСнсаторами ΠΎΠ΄ΠΈΠ½Π°ΠΊΠΎΠ²ΠΎΠΉ Смкости. ΠŸΡ€ΠΈ ΠΊΠ°ΠΊΠΎΠΌ ΡΠΎΠΎΡ‚Π½ΠΎΡˆΠ΅Π½ΠΈΠΈ ΠΌΠ΅ΠΆΠ΄Ρƒ индуктивностями ΠΈ Π°ΠΊΡ‚ΠΈΠ²Π½Ρ‹ΠΌΠΈ сопротивлСниями ΠΊΠ°Ρ‚ΡƒΡˆΠ΅ΠΊ частоты ΠΈ Π·Π°Ρ‚ΡƒΡ…Π°Π½ΠΈΠ΅ свободных ΠΊΠΎΠ»Π΅Π±Π°Π½ΠΈΠΉ Π² ΠΎΠ±ΠΎΠΈΡ… ΠΊΠΎΠ½Ρ‚ΡƒΡ€Π°Ρ… Π±ΡƒΠ΄ΡƒΡ‚ ΠΎΠ΄ΠΈΠ½Π°ΠΊΠΎΠ²Ρ‹ΠΌΠΈ? Взаимная ΠΈΠ½Π΄ΡƒΠΊΡ‚ΠΈΠ²Π½ΠΎΡΡ‚ΡŒ ΠΊΠ°Ρ‚ΡƒΡˆΠ΅ΠΊ Π»Π΅Π²ΠΎΠ³ΠΎ ΠΊΠΎΠ½Ρ‚ΡƒΡ€Π° ΠΏΡ€Π΅Π½Π΅Π±Ρ€Π΅ΠΆΠΈΠΌΠΎ ΠΌΠ°Π»Π°.


ΠŸΠΎΠ΄Ρ€ΠΎΠ±Π½Π΅Π΅

Π—Π°Π΄Π°Ρ‡Π° ΠΏΠΎ Ρ„ΠΈΠ·ΠΈΠΊΠ΅ — 7868

ΠšΠΎΠ½Ρ‚ΡƒΡ€ состоит ΠΈΠ· ΠΏΠΎΡΠ»Π΅Π΄ΠΎΠ²Π°Ρ‚Π΅Π»ΡŒΠ½ΠΎ Π²ΠΊΠ»ΡŽΡ‡Π΅Π½Π½Ρ‹Ρ… кондСнсатора Смкости $C$, ΠΊΠ°Ρ‚ΡƒΡˆΠΊΠΈ индуктивности $L$, ΠΊΠ»ΡŽΡ‡Π° ΠΈ сопротивлСния, Ρ€Π°Π²Π½ΠΎΠ³ΠΎ критичСскому для Π΄Π°Π½Π½ΠΎΠ³ΠΎ ΠΊΠΎΠ½Ρ‚ΡƒΡ€Π°. ΠŸΡ€ΠΈ Ρ€Π°Π·ΠΎΠΌΠΊΠ½ΡƒΡ‚ΠΎΠΌ ΠΊΠ»ΡŽΡ‡Π΅ кондСнсатор зарядили Π΄ΠΎ напряТСния $U_{0}$ ΠΈ Π² ΠΌΠΎΠΌΠ΅Π½Ρ‚ $t = 0$ ΠΊΠ»ΡŽΡ‡ Π·Π°ΠΌΠΊΠ½ΡƒΠ»ΠΈ. Найти Ρ‚ΠΎΠΊ $I$ Π² ΠΊΠΎΠ½Ρ‚ΡƒΡ€Π΅ ΠΊΠ°ΠΊ Ρ„ΡƒΠ½ΠΊΡ†ΠΈΡŽ Π²Ρ€Π΅ΠΌΠ΅Π½ΠΈ $t$. Π§Π΅ΠΌΡƒ Ρ€Π°Π²Π΅Π½ $I_{макс}$?


ΠŸΠΎΠ΄Ρ€ΠΎΠ±Π½Π΅Π΅

РСзонанс Ρ‚ΠΎΠΊΠΎΠ² ΠΈ напряТСний: условия возникновСния ΠΈ ΠΏΡ€ΠΈΠΌΠ΅Π½Π΅Π½ΠΈΠ΅

Π―Π²Π»Π΅Π½ΠΈΠ΅ рСзонанса Ρ‚ΠΎΠΊΠΎΠ² ΠΈ напряТСний Π½Π°Π±Π»ΡŽΠ΄Π°Π΅Ρ‚ΡΡ Π² цСпях ΠΈΠ½Π΄ΡƒΠΊΡ‚ΠΈΠ²Π½ΠΎ-Смкостного Ρ…Π°Ρ€Π°ΠΊΡ‚Π΅Ρ€Π°. Π­Ρ‚ΠΎ явлСниС нашло ΠΏΡ€ΠΈΠΌΠ΅Π½Π΅Π½ΠΈΠ΅ Π² радиоэлСктроникС, став основным способов настройки ΠΏΡ€ΠΈΠ΅ΠΌΠ½ΠΈΠΊΠ° Π½Π° ΠΎΠΏΡ€Π΅Π΄Π΅Π»Π΅Π½Π½ΡƒΡŽ Π²ΠΎΠ»Π½Ρƒ. К соТалСнию, рСзонанс ΠΌΠΎΠΆΠ΅Ρ‚ нанСсти Π²Ρ€Π΅Π΄ ΡΠ»Π΅ΠΊΡ‚Ρ€ΠΎΠΎΠ±ΠΎΡ€ΡƒΠ΄ΠΎΠ²Π°Π½ΠΈΡŽ ΠΈ ΠΊΠ°Π±Π΅Π»ΡŒΠ½Ρ‹ΠΌ линиям. Π’ Ρ„ΠΈΠ·ΠΈΠΊΠ΅ рСзонансом являСтся совпадСниС частот Π½Π΅ΡΠΊΠΎΠ»ΡŒΠΊΠΈΡ… систСм. Π”Π°Π²Π°ΠΉΡ‚Π΅ рассмотрим, Ρ‡Ρ‚ΠΎ Ρ‚Π°ΠΊΠΎΠ΅ рСзонанс напряТСний ΠΈ Ρ‚ΠΎΠΊΠΎΠ², ΠΊΠ°ΠΊΠΎΠ΅ Π·Π½Π°Ρ‡Π΅Π½ΠΈΠ΅ ΠΎΠ½ ΠΈΠΌΠ΅Π΅Ρ‚ ΠΈ Π³Π΄Π΅ ΠΈΡΠΏΠΎΠ»ΡŒΠ·ΡƒΠ΅Ρ‚ΡΡ Π² элСктротСхникС.

Π Π΅Π°ΠΊΡ‚ΠΈΠ²Π½Ρ‹Π΅ сопротивлСния индуктивности ΠΈ Смкости

Π˜Π½Π΄ΡƒΠΊΡ‚ΠΈΠ²Π½ΠΎΡΡ‚ΡŒΡŽ называСтся ΡΠΏΠΎΡΠΎΠ±Π½ΠΎΡΡ‚ΡŒ Ρ‚Π΅Π»Π° Π½Π°ΠΊΠ°ΠΏΠ»ΠΈΠ²Π°Ρ‚ΡŒ ΡΠ½Π΅Ρ€Π³ΠΈΡŽ Π² ΠΌΠ°Π³Π½ΠΈΡ‚Π½ΠΎΠΌ ΠΏΠΎΠ»Π΅. Для Π½Π΅Π΅ Ρ…Π°Ρ€Π°ΠΊΡ‚Π΅Ρ€Π½ΠΎ отставаниС Ρ‚ΠΎΠΊΠ° ΠΎΡ‚ напряТСния ΠΏΠΎ Ρ„Π°Π·Π΅. Π₯Π°Ρ€Π°ΠΊΡ‚Π΅Ρ€Π½Ρ‹Π΅ ΠΈΠ½Π΄ΡƒΠΊΡ‚ΠΈΠ²Π½Ρ‹Π΅ элСмСнты – дроссСля, ΠΊΠ°Ρ‚ΡƒΡˆΠΊΠΈ, трансформаторы, элСктродвигатСли.

Π•ΠΌΠΊΠΎΡΡ‚ΡŒΡŽ Π½Π°Π·Ρ‹Π²Π°ΡŽΡ‚ΡΡ элСмСнты, ΠΊΠΎΡ‚ΠΎΡ€Ρ‹Π΅ Π½Π°ΠΊΠ°ΠΏΠ»ΠΈΠ²Π°ΡŽΡ‚ ΡΠ½Π΅Ρ€Π³ΠΈΡŽ с ΠΏΠΎΠΌΠΎΡ‰ΡŒΡŽ элСктричСского поля. Для Смкостных элСмСнтов Ρ…Π°Ρ€Π°ΠΊΡ‚Π΅Ρ€Π½ΠΎ отставаниС ΠΏΠΎ Ρ„Π°Π·Π΅ напряТСния ΠΎΡ‚ Ρ‚ΠΎΠΊΠ°. ЕмкостныС элСмСнты: кондСнсаторы, Π²Π°Ρ€ΠΈΠΊΠ°ΠΏΡ‹.

ΠŸΡ€ΠΈΠ²Π΅Π΄Π΅Π½Ρ‹ ΠΈΡ… основныС свойства, Π½ΡŽΠ°Π½ΡΡ‹ Π² ΠΏΡ€Π΅Π΄Π΅Π»Π°Ρ… этой ΡΡ‚Π°Ρ‚ΡŒΠΈ Π²ΠΎ Π²Π½ΠΈΠΌΠ°Π½ΠΈΠ΅ Π½Π΅ бСрутся.

ΠšΡ€ΠΎΠΌΠ΅ пСрСчислСнных элСмСнтов Π΄Ρ€ΡƒΠ³ΠΈΠ΅ Ρ‚Π°ΠΊΠΆΠ΅ ΠΈΠΌΠ΅ΡŽΡ‚ ΠΎΠΏΡ€Π΅Π΄Π΅Π»Π΅Π½Π½ΡƒΡŽ ΠΈΠ½Π΄ΡƒΠΊΡ‚ΠΈΠ²Π½ΠΎΡΡ‚ΡŒ ΠΈ Π΅ΠΌΠΊΠΎΡΡ‚ΡŒ, Π½Π°ΠΏΡ€ΠΈΠΌΠ΅Ρ€ Π² элСктричСских кабСлях распрСдСлСнныС ΠΏΠΎ Π΅Π³ΠΎ Π΄Π»ΠΈΠ½Π΅.

Π•ΠΌΠΊΠΎΡΡ‚ΡŒ ΠΈ ΠΈΠ½Π΄ΡƒΠΊΡ‚ΠΈΠ²Π½ΠΎΡΡ‚ΡŒ Π² Ρ†Π΅ΠΏΠΈ ΠΏΠ΅Ρ€Π΅ΠΌΠ΅Π½Π½ΠΎΠ³ΠΎ Ρ‚ΠΎΠΊΠ°

Если Π² цСпях постоянного Ρ‚ΠΎΠΊΠ° Π΅ΠΌΠΊΠΎΡΡ‚ΡŒ Π² ΠΎΠ±Ρ‰Π΅ΠΌ смыслС прСдставляСт собой Ρ€Π°Π·ΠΎΡ€Π²Π°Π½Π½Ρ‹ΠΉ участок Ρ†Π΅ΠΏΠΈ, Π° ΠΈΠ½Π΄ΡƒΠΊΡ‚ΠΈΠ²Π½ΠΎΡΡ‚ΡŒ – ΠΏΡ€ΠΎΠ²ΠΎΠ΄Π½ΠΈΠΊ, Ρ‚ΠΎ Π² ΠΏΠ΅Ρ€Π΅ΠΌΠ΅Π½Π½ΠΎΠΌ кондСнсаторы ΠΈ ΠΊΠ°Ρ‚ΡƒΡˆΠΊΠΈ ΠΏΡ€Π΅Π΄ΡΡ‚Π°Π²Π»ΡΡŽΡ‚ собой Ρ€Π΅Π°ΠΊΡ‚ΠΈΠ²Π½Ρ‹ΠΉ Π°Π½Π°Π»ΠΎΠ³ рСзистора.

Π Π΅Π°ΠΊΡ‚ΠΈΠ²Π½ΠΎΠ΅ сопротивлСниС ΠΊΠ°Ρ‚ΡƒΡˆΠΊΠΈ индуктивности опрСдСляСтся ΠΏΠΎ Ρ„ΠΎΡ€ΠΌΡƒΠ»Π΅:

ВСкторная Π΄ΠΈΠ°Π³Ρ€Π°ΠΌΠΌΠ°:

Β 

Π Π΅Π°ΠΊΡ‚ΠΈΠ²Π½ΠΎΠ΅ сопротивлСниС кондСнсатора:

Π—Π΄Π΅ΡΡŒ w – угловая частота, f – частота Π² Ρ†Π΅ΠΏΠΈ ΡΠΈΠ½ΡƒΡΠΎΠΈΠ΄Π°Π»ΡŒΠ½ΠΎΠ³ΠΎ Ρ‚ΠΎΠΊΠ°, L – ΠΈΠ½Π΄ΡƒΠΊΡ‚ΠΈΠ²Π½ΠΎΡΡ‚ΡŒ, C – Π΅ΠΌΠΊΠΎΡΡ‚ΡŒ.

ВСкторная Π΄ΠΈΠ°Π³Ρ€Π°ΠΌΠΌΠ°:

Π‘Ρ‚ΠΎΠΈΡ‚ ΠΎΡ‚ΠΌΠ΅Ρ‚ΠΈΡ‚ΡŒ, Ρ‡Ρ‚ΠΎ ΠΏΡ€ΠΈ расчСтС соСдинСнных ΠΏΠΎΡΠ»Π΅Π΄ΠΎΠ²Π°Ρ‚Π΅Π»ΡŒΠ½ΠΎ Ρ€Π΅Π°ΠΊΡ‚ΠΈΠ²Π½Ρ‹Ρ… элСмСнтов ΠΈΡΠΏΠΎΠ»ΡŒΠ·ΡƒΡŽΡ‚ Ρ„ΠΎΡ€ΠΌΡƒΠ»Ρƒ:

ΠžΠ±Ρ€Π°Ρ‚ΠΈΡ‚Π΅ Π²Π½ΠΈΠΌΠ°Π½ΠΈΠ΅, Ρ‡Ρ‚ΠΎ Смкостная ΡΠΎΡΡ‚Π°Π²Π»ΡΡŽΡ‰Π°Ρ принимаСтся со Π·Π½Π°ΠΊΠΎΠΌ минус. Если Π² Ρ†Π΅ΠΏΠΈ присутствуСт Π΅Ρ‰Π΅ ΠΈ активная ΡΠΎΡΡ‚Π°Π²Π»ΡΡŽΡ‰Π°Ρ (рСзистор), Ρ‚ΠΎ ΡΠΊΠ»Π°Π΄Ρ‹Π²Π°ΡŽΡ‚ ΠΏΠΎ Ρ„ΠΎΡ€ΠΌΡƒΠ»Π΅ Ρ‚Π΅ΠΎΡ€Π΅ΠΌΡ‹ ΠŸΠΈΡ„Π°Π³ΠΎΡ€Π° (исходя ΠΈΠ· Π²Π΅ΠΊΡ‚ΠΎΡ€Π½ΠΎΠΉ Π΄ΠΈΠ°Π³Ρ€Π°ΠΌΠΌΡ‹):

ΠžΡ‚ Ρ‡Π΅Π³ΠΎ зависит Ρ€Π΅Π°ΠΊΡ‚ΠΈΠ²Π½ΠΎΠ΅ сопротивлСниС?Β Π Π΅Π°ΠΊΡ‚ΠΈΠ²Π½Ρ‹Π΅ характСристики зависят ΠΎΡ‚ Π²Π΅Π»ΠΈΡ‡ΠΈΠ½Ρ‹ Смкости ΠΈΠ»ΠΈ индуктивности, Π° Ρ‚Π°ΠΊΠΆΠ΅ ΠΎΡ‚ частоты ΠΏΠ΅Ρ€Π΅ΠΌΠ΅Π½Π½ΠΎΠ³ΠΎ Ρ‚ΠΎΠΊΠ°.

Если ΠΏΠΎΡΠΌΠΎΡ‚Ρ€Π΅Ρ‚ΡŒ Π½Π° Ρ„ΠΎΡ€ΠΌΡƒΠ»Ρƒ Ρ€Π΅Π°ΠΊΡ‚ΠΈΠ²Π½ΠΎΠΉ ΡΠΎΡΡ‚Π°Π²Π»ΡΡŽΡ‰Π΅ΠΉ, Ρ‚ΠΎ ΠΌΠΎΠΆΠ½ΠΎ Π·Π°ΠΌΠ΅Ρ‚ΠΈΡ‚ΡŒ, Ρ‡Ρ‚ΠΎ ΠΏΡ€ΠΈ ΠΎΠΏΡ€Π΅Π΄Π΅Π»Π΅Π½Π½Ρ‹Ρ… значСниях Смкостной ΠΈΠ»ΠΈ ΠΈΠ½Π΄ΡƒΠΊΡ‚ΠΈΠ²Π½ΠΎΠΉ ΡΠΎΡΡ‚Π°Π²Π»ΡΡŽΡ‰Π΅ΠΉ ΠΈΡ… Ρ€Π°Π·Π½ΠΎΡΡ‚ΡŒ Π±ΡƒΠ΄Π΅Ρ‚ Ρ€Π°Π²Π½Π° Π½ΡƒΠ»ΡŽ, Ρ‚ΠΎΠ³Π΄Π° Π² Ρ†Π΅ΠΏΠΈ останСтся Ρ‚ΠΎΠ»ΡŒΠΊΠΎ Π°ΠΊΡ‚ΠΈΠ²Π½ΠΎΠ΅ сопротивлСниС. Но это Π½Π΅ всС особСнности Ρ‚Π°ΠΊΠΎΠΉ ситуации.

РСзонанс напряТСний

Если ΠΏΠΎΡΠ»Π΅Π΄ΠΎΠ²Π°Ρ‚Π΅Π»ΡŒΠ½ΠΎ с Π³Π΅Π½Π΅Ρ€Π°Ρ‚ΠΎΡ€ΠΎΠΌ ΡΠΎΠ΅Π΄ΠΈΠ½ΠΈΡ‚ΡŒ кондСнсатор ΠΈ ΠΊΠ°Ρ‚ΡƒΡˆΠΊΡƒ индуктивности, Ρ‚ΠΎ, ΠΏΡ€ΠΈ условии равСнства ΠΈΡ… Ρ€Π΅Π°ΠΊΡ‚ΠΈΠ²Π½Ρ‹Ρ… сопротивлСний, Π²ΠΎΠ·Π½ΠΈΠΊΠ½Π΅Ρ‚ рСзонанс напряТСний. ΠŸΡ€ΠΈ этом активная Ρ‡Π°ΡΡ‚ΡŒ Z Π΄ΠΎΠ»ΠΆΠ½ΠΎ Π±Ρ‹Ρ‚ΡŒ ΠΊΠ°ΠΊ ΠΌΠΎΠΆΠ½ΠΎ мСньшСй.

Π‘Ρ‚ΠΎΠΈΡ‚ ΠΎΡ‚ΠΌΠ΅Ρ‚ΠΈΡ‚ΡŒ, Ρ‡Ρ‚ΠΎ ΠΈΠ½Π΄ΡƒΠΊΡ‚ΠΈΠ²Π½ΠΎΡΡ‚ΡŒ ΠΈ Π΅ΠΌΠΊΠΎΡΡ‚ΡŒ ΠΎΠ±Π»Π°Π΄Π°Π΅Ρ‚ Ρ‚ΠΎΠ»ΡŒΠΊΠΎ Ρ€Π΅Π°ΠΊΡ‚ΠΈΠ²Π½Ρ‹ΠΌΠΈ качСствами лишь Π² ΠΈΠ΄Π΅Π°Π»ΠΈΠ·ΠΈΡ€ΠΎΠ²Π°Π½Π½Ρ‹Ρ… ΠΏΡ€ΠΈΠΌΠ΅Ρ€Π°Ρ…. Π’ Ρ€Π΅Π°Π»ΡŒΠ½Ρ‹Ρ… ΠΆΠ΅ цСпях ΠΈ элСмСнтах всСгда присутствуСт Π°ΠΊΡ‚ΠΈΠ²Π½ΠΎΠ΅ сопротивлСниС ΠΏΡ€ΠΎΠ²ΠΎΠ΄Π½ΠΈΠΊΠΎΠ², Ρ…ΠΎΡ‚ΡŒ ΠΎΠ½ΠΎ ΠΈ ΠΊΡ€Π°ΠΉΠ½Π΅ ΠΌΠ°Π»ΠΎ.

ΠŸΡ€ΠΈ рСзонансС происходит ΠΎΠ±ΠΌΠ΅Π½ энСргиСй ΠΌΠ΅ΠΆΠ΄Ρƒ дроссСлСм ΠΈ кондСнсатором. Π’ ΠΈΠ΄Π΅Π°Π»ΡŒΠ½Ρ‹Ρ… ΠΏΡ€ΠΈΠΌΠ΅Ρ€Π°Ρ… ΠΏΡ€ΠΈ ΠΏΠ΅Ρ€Π²ΠΎΠ½Π°Ρ‡Π°Π»ΡŒΠ½ΠΎΠΌ ΠΏΠΎΠ΄ΠΊΠ»ΡŽΡ‡Π΅Π½ΠΈΠΈ источника энСргии (Π³Π΅Π½Π΅Ρ€Π°Ρ‚ΠΎΡ€Π°) энСргия накапливаСтся Π² кондСнсаторС (ΠΈΠ»ΠΈ дроссСлС) ΠΈ послС Π΅Π³ΠΎ ΠΎΡ‚ΠΊΠ»ΡŽΡ‡Π΅Π½ΠΈΡ происходят Π½Π΅Π·Π°Ρ‚ΡƒΡ…Π°ΡŽΡ‰ΠΈΠ΅ колСбания Π·Π° счСт этого ΠΎΠ±ΠΌΠ΅Π½Π°.

НапряТСния Π½Π° индуктивности ΠΈ Смкости ΠΏΡ€ΠΈΠΌΠ΅Ρ€Π½ΠΎ ΠΎΠ΄ΠΈΠ½Π°ΠΊΠΎΠ²Ρ‹, согласно Π·Π°ΠΊΠΎΠ½Ρƒ Ома:

U=I/X

Π“Π΄Π΅ X – это Xc СмкостноС ΠΈΠ»ΠΈ XL ΠΈΠ½Π΄ΡƒΠΊΡ‚ΠΈΠ²Π½ΠΎΠ΅ сопротивлСниС соотвСтствСнно.

ЦСпь, ΡΠΎΡΡ‚ΠΎΡΡ‰ΡƒΡŽ ΠΈΠ· индуктивности ΠΈ Смкости, Π½Π°Π·Ρ‹Π²Π°ΡŽΡ‚ ΠΊΠΎΠ»Π΅Π±Π°Ρ‚Π΅Π»ΡŒΠ½Ρ‹ΠΌ ΠΊΠΎΠ½Ρ‚ΡƒΡ€ΠΎΠΌ. Π•Π³ΠΎ частота вычисляСтся ΠΏΠΎ Ρ„ΠΎΡ€ΠΌΡƒΠ»Π΅:

ΠŸΠ΅Ρ€ΠΈΠΎΠ΄ ΠΊΠΎΠ»Π΅Π±Π°Π½ΠΈΠΉ опрСдСляСтся ΠΏΠΎ Ρ„ΠΎΡ€ΠΌΡƒΠ»Π΅ Вомпсона:

Π’Π°ΠΊ ΠΊΠ°ΠΊ Ρ€Π΅Π°ΠΊΡ‚ΠΈΠ²Π½ΠΎΠ΅ сопротивлСниС зависит ΠΎΡ‚ частоты, Ρ‚ΠΎ сопротивлСниС индуктивности с ростом частоты увСличиваСтся, Π° Ρƒ ёмкости ΠΏΠ°Π΄Π°Π΅Ρ‚. Когда сопротивлСния Ρ€Π°Π²Π½Ρ‹, Ρ‚ΠΎ ΠΎΠ±Ρ‰Π΅Π΅ сопротивлСниС сильно сниТаСтся, Ρ‡Ρ‚ΠΎ ΠΎΡ‚Ρ€Π°ΠΆΠ΅Π½ΠΎ Π½Π° Π³Ρ€Π°Ρ„ΠΈΠΊΠ΅:

ΠžΡΠ½ΠΎΠ²Π½Ρ‹ΠΌΠΈ характСристиками ΠΊΠΎΠ½Ρ‚ΡƒΡ€Π° ΡΠ²Π»ΡΡŽΡ‚ΡΡ Π΄ΠΎΠ±Ρ€ΠΎΡ‚Π½ΠΎΡΡ‚ΡŒ (Q) ΠΈ частота. Если Ρ€Π°ΡΡΠΌΠΎΡ‚Ρ€Π΅Ρ‚ΡŒ ΠΊΠΎΠ½Ρ‚ΡƒΡ€ Π² качСствС Ρ‡Π΅Ρ‚Ρ‹Ρ€Π΅Ρ…ΠΏΠΎΠ»ΡŽΡΠ½ΠΈΠΊΠ°, Ρ‚ΠΎ Π΅Π³ΠΎ коэффициСнт ΠΏΠ΅Ρ€Π΅Π΄Π°Ρ‡ΠΈ послС нСслоТных вычислСний сводится ΠΊ добротности:

K=Q

А напряТСниС Π½Π° Π²Ρ‹Π²ΠΎΠ΄Π°Ρ… Ρ†Π΅ΠΏΠΈ увСличиваСтся ΠΏΡ€ΠΎΠΏΠΎΡ€Ρ†ΠΈΠΎΠ½Π°Π»ΡŒΠ½ΠΎ коэффициСнту ΠΏΠ΅Ρ€Π΅Π΄Π°Ρ‡ΠΈ (добротности) ΠΊΠΎΠ½Ρ‚ΡƒΡ€Π°.

UΠΊ=UΠ²Ρ…*Q

ΠŸΡ€ΠΈ рСзонансС напряТСний, Ρ‡Π΅ΠΌ Π²Ρ‹ΡˆΠ΅ Π΄ΠΎΠ±Ρ€ΠΎΡ‚Π½ΠΎΡΡ‚ΡŒ, Ρ‚Π΅ΠΌ большС напряТСниС Π½Π° элСмСнтах ΠΊΠΎΠ½Ρ‚ΡƒΡ€Π° Π±ΡƒΠ΄Π΅Ρ‚ ΠΏΡ€Π΅Π²Ρ‹ΡˆΠ°Ρ‚ΡŒ напряТСниС ΠΏΠΎΠ΄ΠΊΠ»ΡŽΡ‡Π΅Π½Π½ΠΎΠ³ΠΎ Π³Π΅Π½Π΅Ρ€Π°Ρ‚ΠΎΡ€Π°. НапряТСниС ΠΌΠΎΠΆΠ΅Ρ‚ ΠΏΠΎΠ²Ρ‹ΡˆΠ°Ρ‚ΡŒΡΡ Π² дСсятки ΠΈ сотни Ρ€Π°Π·. Π­Ρ‚ΠΎ ΠΎΡ‚ΠΎΠ±Ρ€Π°ΠΆΠ΅Π½ΠΎ Π½Π° Π³Ρ€Π°Ρ„ΠΈΠΊΠ΅:

ΠŸΠΎΡ‚Π΅Ρ€ΠΈ мощности Π² ΠΊΠΎΠ½Ρ‚ΡƒΡ€Π΅ обусловлСны Ρ‚ΠΎΠ»ΡŒΠΊΠΎ Π½Π°Π»ΠΈΡ‡ΠΈΠ΅ΠΌ Π°ΠΊΡ‚ΠΈΠ²Π½ΠΎΠ³ΠΎ сопротивлСния. ЭнСргия ΠΈΠ· источника питания бСрСтся Ρ‚ΠΎΠ»ΡŒΠΊΠΎ для поддСрТания ΠΊΠΎΠ»Π΅Π±Π°Π½ΠΈΠΉ.

ΠšΠΎΡΡ„Ρ„ΠΈΡ†ΠΈΠ΅Π½Ρ‚ мощности Π±ΡƒΠ΄Π΅Ρ‚ Ρ€Π°Π²Π΅Π½:

cosΠ€=1

Π­Ρ‚Π° Ρ„ΠΎΡ€ΠΌΡƒΠ»Π° ΠΏΠΎΠΊΠ°Π·Ρ‹Π²Π°Π΅Ρ‚, Ρ‡Ρ‚ΠΎ ΠΏΠΎΡ‚Π΅Ρ€ΠΈ происходят Π·Π° счСт Π°ΠΊΡ‚ΠΈΠ²Π½ΠΎΠΉ мощности:

S=P/CosΡ„

РСзонанс Ρ‚ΠΎΠΊΠΎΠ²

РСзонанс Ρ‚ΠΎΠΊΠΎΠ² Π½Π°Π±Π»ΡŽΠ΄Π°Π΅Ρ‚ΡΡ Π² цСпях, Π³Π΄Π΅ ΠΈΠ½Π΄ΡƒΠΊΡ‚ΠΈΠ²Π½ΠΎΡΡ‚ΡŒ ΠΈ Π΅ΠΌΠΊΠΎΡΡ‚ΡŒ соСдинСны ΠΏΠ°Ρ€Π°Π»Π»Π΅Π»ΡŒΠ½ΠΎ.

Π―Π²Π»Π΅Π½ΠΈΠ΅ Π·Π°ΠΊΠ»ΡŽΡ‡Π°Π΅Ρ‚ΡΡ Π² ΠΏΡ€ΠΎΡ‚Π΅ΠΊΠ°Π½ΠΈΠΈ Ρ‚ΠΎΠΊΠΎΠ² большой Π²Π΅Π»ΠΈΡ‡ΠΈΠ½Ρ‹ ΠΌΠ΅ΠΆΠ΄Ρƒ кондСнсатором ΠΈ ΠΊΠ°Ρ‚ΡƒΡˆΠΊΠΎΠΉ, ΠΏΡ€ΠΈ Π½ΡƒΠ»Π΅Π²ΠΎΠΌ Ρ‚ΠΎΠΊΠ΅ Π² Π½Π΅Ρ€Π°Π·Π²Π΅Ρ‚Π²Π»Π΅Π½Π½ΠΎΠΉ части Ρ†Π΅ΠΏΠΈ. Π­Ρ‚ΠΎ ΠΎΠ±ΡŠΡΡΠ½ΡΠ΅Ρ‚ΡΡ Ρ‚Π΅ΠΌ, Ρ‡Ρ‚ΠΎ ΠΏΡ€ΠΈ достиТСнии рСзонансной частоты ΠΎΠ±Ρ‰Π΅Π΅ сопротивлСниС Z возрастаСт. Или простым языком Π·Π²ΡƒΡ‡ΠΈΡ‚ Ρ‚Π°ΠΊ – Π² Ρ‚ΠΎΡ‡ΠΊΠ΅ рСзонанса достигаСтся максимальноС ΠΎΠ±Ρ‰Π΅Π΅ Π·Π½Π°Ρ‡Π΅Π½ΠΈΠ΅ сопротивлСния Z, послС Ρ‡Π΅Π³ΠΎ ΠΎΠ΄Π½ΠΎ ΠΈΠ· сопротивлСний увСличиваСтся, Π° Π΄Ρ€ΡƒΠ³ΠΎΠ΅ сниТаСтся Π² зависимости ΠΎΡ‚ Ρ‚ΠΎΠ³ΠΎ растСт ΠΈΠ»ΠΈ сниТаСтся частота. Π­Ρ‚ΠΎ наглядно ΠΎΡ‚ΠΎΠ±Ρ€Π°ΠΆΠ΅Π½ΠΎ Π½Π° Π³Ρ€Π°Ρ„ΠΈΠΊΠ΅:

Π’ ΠΎΠ±Ρ‰Π΅ΠΌ, всё Π°Π½Π°Π»ΠΎΠ³ΠΈΡ‡Π½ΠΎ ΠΏΡ€Π΅Π΄Ρ‹Π΄ΡƒΡ‰Π΅ΠΌΡƒ явлСнию, условия возникновСния рСзонанса Ρ‚ΠΎΠΊΠΎΠ² ΡΠ»Π΅Π΄ΡƒΡŽΡ‰ΠΈΠ΅:

  1. Частота питания Π°Π½Π°Π»ΠΎΠ³ΠΈΡ‡Π½Π° рСзонансной Ρƒ ΠΊΠΎΠ½Ρ‚ΡƒΡ€Π°.
  2. ΠŸΡ€ΠΎΠ²ΠΎΠ΄ΠΈΠΌΠΎΡΡ‚ΠΈ Ρƒ индуктивности ΠΈ ёмкости ΠΏΠΎ ΠΏΠ΅Ρ€Π΅ΠΌΠ΅Π½Π½ΠΎΠΌΡƒ Ρ‚ΠΎΠΊΡƒ Ρ€Π°Π²Π½Ρ‹ BL=Bc, B=1/X.

ΠŸΡ€ΠΈΠΌΠ΅Π½Π΅Π½ΠΈΠ΅ Π½Π° ΠΏΡ€Π°ΠΊΡ‚ΠΈΠΊΠ΅

Рассмотрим, какая польза ΠΈ Π²Ρ€Π΅Π΄ рСзонанса Ρ‚ΠΎΠΊΠΎΠ² ΠΈ напряТСний. ΠΠ°ΠΈΠ±ΠΎΠ»ΡŒΡˆΡƒΡŽ ΠΏΠΎΠ»ΡŒΠ·Ρƒ явлСния рСзонанса принСсли Π² Ρ€Π°Π΄ΠΈΠΎΠΏΠ΅Ρ€Π΅Π΄Π°ΡŽΡ‰Π΅ΠΉ Π°ΠΏΠΏΠ°Ρ€Π°Ρ‚ΡƒΡ€Π΅. ΠŸΡ€ΠΎΡΡ‚Ρ‹ΠΌΠΈ словами, Π° схСмС ΠΏΡ€ΠΈΠ΅ΠΌΠ½ΠΈΠΊΠ° установлСны ΠΊΠ°Ρ‚ΡƒΡˆΠΊΠ° ΠΈ кондСнсатор, ΠΏΠΎΠ΄ΠΊΠ»ΡŽΡ‡Π΅Π½Π½Ρ‹Π΅ ΠΊ Π°Π½Ρ‚Π΅Π½Π½Π΅. Π‘ ΠΏΠΎΠΌΠΎΡ‰ΡŒΡŽ измСнСния индуктивности (Π½Π°ΠΏΡ€ΠΈΠΌΠ΅Ρ€, пСрСмСщая сСрдСчник) ΠΈΠ»ΠΈ Π²Π΅Π»ΠΈΡ‡ΠΈΠ½Ρ‹ Смкости (Π½Π°ΠΏΡ€ΠΈΠΌΠ΅Ρ€, Π²ΠΎΠ·Π΄ΡƒΡˆΠ½Ρ‹ΠΌ ΠΏΠ΅Ρ€Π΅ΠΌΠ΅Π½Π½Ρ‹ΠΌ кондСнсатором) Π²Ρ‹ настраиваСтС Ρ€Π΅Π·ΠΎΠ½Π°Π½ΡΠ½ΡƒΡŽ частоту. Π’ Ρ€Π΅Π·ΡƒΠ»ΡŒΡ‚Π°Ρ‚Π΅ Ρ‡Π΅Π³ΠΎ напряТСниС Π½Π° ΠΊΠ°Ρ‚ΡƒΡˆΠΊΠ΅ ΠΏΠΎΠ²Ρ‹ΡˆΠ°Π΅Ρ‚ΡΡ ΠΈ ΠΏΡ€ΠΈΠ΅ΠΌΠ½ΠΈΠΊ Π»ΠΎΠ²ΠΈΡ‚ ΠΎΠΏΡ€Π΅Π΄Π΅Π»Π΅Π½Π½ΡƒΡŽ Ρ€Π°Π΄ΠΈΠΎΠ²ΠΎΠ»Π½Ρƒ.

Π’Ρ€Π΅Π΄ эти явлСния ΠΌΠΎΠ³ΡƒΡ‚ Π½Π° нСсти Π² элСктротСхникС, Π½Π°ΠΏΡ€ΠΈΠΌΠ΅Ρ€, Π½Π° ΠΊΠ°Π±Π΅Π»ΡŒΠ½Ρ‹Ρ… линиях. КабСль прСдставляСт собой Ρ€Π°ΡΠΏΡ€Π΅Π΄Π΅Π»Π΅Π½Π½ΡƒΡŽ ΠΏΠΎ Π΄Π»ΠΈΠ½Π΅ ΠΈΠ½Π΄ΡƒΠΊΡ‚ΠΈΠ²Π½ΠΎΡΡ‚ΡŒ ΠΈ Π΅ΠΌΠΊΠΎΡΡ‚ΡŒ, Ссли Π½Π° Π΄Π»ΠΈΠ½Π½ΡƒΡŽ линию ΠΏΠΎΠ΄Π°Ρ‚ΡŒ напряТСниС Π² Ρ€Π΅ΠΆΠΈΠΌΠ΅ холостого Ρ…ΠΎΠ΄Π° (ΠΊΠΎΠ³Π΄Π° Π½Π° ΠΏΡ€ΠΎΡ‚ΠΈΠ²ΠΎΠΏΠΎΠ»ΠΎΠΆΠ½ΠΎΠΌ ΠΎΡ‚ источника питания ΠΊΠΎΠ½Ρ†Π΅ кабСля Π½Π°Π³Ρ€ΡƒΠ·ΠΊΠ° Π½Π΅ ΠΏΠΎΠ΄ΠΊΠ»ΡŽΡ‡Π΅Π½Π°). ΠŸΠΎΡΡ‚ΠΎΠΌΡƒ Π΅ΡΡ‚ΡŒ ΠΎΠΏΠ°ΡΠ½ΠΎΡΡ‚ΡŒ Ρ‚ΠΎΠ³ΠΎ, Ρ‡Ρ‚ΠΎ ΠΏΡ€ΠΎΠΈΠ·ΠΎΠΉΠ΄Π΅Ρ‚ ΠΏΡ€ΠΎΠ±ΠΎΠΉ изоляции, Π²ΠΎ ΠΈΠ·Π±Π΅ΠΆΠ°Π½ΠΈΠ΅ этого ΠΏΠΎΠ΄ΠΊΠ»ΡŽΡ‡Π°Π΅Ρ‚ΡΡ Π½Π°Π³Ρ€ΡƒΠ·ΠΎΡ‡Π½Ρ‹ΠΉ балласт. Π’Π°ΠΊΠΆΠ΅ аналогичная ситуация ΠΌΠΎΠΆΠ΅Ρ‚ привСсти ΠΊ Π²Ρ‹Ρ…ΠΎΠ΄Ρƒ ΠΈΠ· строя элСктронных ΠΊΠΎΠΌΠΏΠΎΠ½Π΅Π½Ρ‚ΠΎΠ², ΠΈΠ·ΠΌΠ΅Ρ€ΠΈΡ‚Π΅Π»ΡŒΠ½Ρ‹Ρ… ΠΏΡ€ΠΈΠ±ΠΎΡ€ΠΎΠ² ΠΈ Π΄Ρ€ΡƒΠ³ΠΎΠ³ΠΎ элСктрооборудования – это опасныС послСдствия возникновСния этого явлСния.

Π—Π°ΠΊΠ»ΡŽΡ‡Π΅Π½ΠΈΠ΅

РСзонанс напряТСний ΠΈ Ρ‚ΠΎΠΊΠΎΠ² – интСрСсноС явлСниС, ΠΎ ΠΊΠΎΡ‚ΠΎΡ€ΠΎΠΌ Π½ΡƒΠΆΠ½ΠΎ Π·Π½Π°Ρ‚ΡŒ. Он Π½Π°Π±Π»ΡŽΠ΄Π°Π΅Ρ‚ΡΡ Ρ‚ΠΎΠ»ΡŒΠΊΠΎ Π² ΠΈΠ½Π΄ΡƒΠΊΡ‚ΠΈΠ²Π½ΠΎ-Смкостных цСпях. Π’ цСпях с большим Π°ΠΊΡ‚ΠΈΠ²Π½Ρ‹ΠΌ сопротивлСниям ΠΎΠ½ Π½Π΅ ΠΌΠΎΠΆΠ΅Ρ‚ Π²ΠΎΠ·Π½ΠΈΠΊΠ½ΡƒΡ‚ΡŒ. ПодвСдСм ΠΈΡ‚ΠΎΠ³ΠΈ, ΠΊΡ€Π°Ρ‚ΠΊΠΎ ΠΎΡ‚Π²Π΅Ρ‚ΠΈΠ² Π½Π° основныС вопросы ΠΏΠΎ этой Ρ‚Π΅ΠΌΠ΅:

  1. Π“Π΄Π΅ ΠΈ Π² ΠΊΠ°ΠΊΠΈΡ… цСпях Π½Π°Π±Π»ΡŽΠ΄Π°Π΅Ρ‚ΡΡ явлСниС рСзонанса?

Π’ ΠΈΠ½Π΄ΡƒΠΊΡ‚ΠΈΠ²Π½ΠΎ-Смкостных цСпях.

  1. КакиС условия возникновСния рСзонанса Ρ‚ΠΎΠΊΠΎΠ² ΠΈ напряТСний?

Π’ΠΎΠ·Π½ΠΈΠΊΠ°Π΅Ρ‚ ΠΏΡ€ΠΈ условии равСнства Ρ€Π΅Π°ΠΊΡ‚ΠΈΠ²Π½Ρ‹Ρ… сопротивлСний. Π’ Ρ†Π΅ΠΏΠΈ Π΄ΠΎΠ»ΠΆΠ½ΠΎ Π±Ρ‹Ρ‚ΡŒ минимальноС Π°ΠΊΡ‚ΠΈΠ²Π½ΠΎΠ΅ сопротивлСниС, Π° частота источника питания ΡΠΎΠ²ΠΏΠ°Π΄Π°Ρ‚ΡŒ с рСзонансной частотой ΠΊΠΎΠ½Ρ‚ΡƒΡ€Π°.

  1. Как Π½Π°ΠΉΡ‚ΠΈ Ρ€Π΅Π·ΠΎΠ½Π°Π½ΡΠ½ΡƒΡŽ частоту?

Π’ ΠΎΠ±ΠΎΠΈΡ… случаях ΠΏΠΎ Ρ„ΠΎΡ€ΠΌΡƒΠ»Π΅:Β w=(1/LC)^(1/2)

  1. Как ΡƒΡΡ‚Ρ€Π°Π½ΠΈΡ‚ΡŒ явлСниС?

Π£Π²Π΅Π»ΠΈΡ‡ΠΈΠ² Π°ΠΊΡ‚ΠΈΠ²Π½ΠΎΠ΅ сопротивлСниС Π² Ρ†Π΅ΠΏΠΈ ΠΈΠ»ΠΈ ΠΈΠ·ΠΌΠ΅Π½ΠΈΠ² частоту.

Π’Π΅ΠΏΠ΅Ρ€ΡŒ Π²Ρ‹ Π·Π½Π°Π΅Ρ‚Π΅, Ρ‡Ρ‚ΠΎ Ρ‚Π°ΠΊΠΎΠ΅ рСзонанс Ρ‚ΠΎΠΊΠΎΠ² ΠΈ напряТСний, ΠΊΠ°ΠΊΠΎΠ²Ρ‹ условия Π΅Π³ΠΎ возникновСния ΠΈ Π²Π°Ρ€ΠΈΠ°Π½Ρ‚Ρ‹ примСнСния Π½Π° ΠΏΡ€Π°ΠΊΡ‚ΠΈΠΊΠ΅. Для закрСплСния ΠΌΠ°Ρ‚Π΅Ρ€ΠΈΠ°Π»Π° Ρ€Π΅ΠΊΠΎΠΌΠ΅Π½Π΄ΡƒΠ΅ΠΌ ΠΏΡ€ΠΎΡΠΌΠΎΡ‚Ρ€Π΅Ρ‚ΡŒ ΠΏΠΎΠ»Π΅Π·Π½ΠΎΠ΅ Π²ΠΈΠ΄Π΅ΠΎ ΠΏΠΎ Ρ‚Π΅ΠΌΠ΅:

ΠœΠ°Ρ‚Π΅Ρ€ΠΈΠ°Π»Ρ‹ ΠΏΠΎ Ρ‚Π΅ΠΌΠ΅:

Π—Π°Ρ‚ΡƒΡ…Π°ΡŽΡ‰ΠΈΠ΅ колСбания Π² ΠΊΠΎΠ½Ρ‚ΡƒΡ€Π΅ ΠΈ ΠΈΡ… ΡƒΡ€Π°Π²Π½Π΅Π½ΠΈΠ΅

ΠžΠΏΡ€Π΅Π΄Π΅Π»Π΅Π½ΠΈΠ΅ 1

Π‘ΡƒΡ‰Π΅ΡΡ‚Π²ΡƒΡŽΡ‚ колСбания Π² систСмС Π±Π΅Π· источника энСргии, Π½Π°Π·Ρ‹Π²Π°Π΅ΠΌΡ‹Π΅ Π·Π°Ρ‚ΡƒΡ…Π°ΡŽΡ‰ΠΈΠΌΠΈ. Рассмотрим Ρ€Π΅Π°Π»ΡŒΠ½Ρ‹ΠΉ ΠΊΠΎΠ½Ρ‚ΡƒΡ€ с сопротивлСниСм Π½Π΅ Ρ€Π°Π²Π½Ρ‹ΠΌ Π½ΡƒΠ»ΡŽ. Для ΠΏΡ€ΠΈΠΌΠ΅Ρ€Π° ΠΈΡΠΏΠΎΠ»ΡŒΠ·ΡƒΡŽΡ‚ ΠΊΠΎΠ½Ρ‚ΡƒΡ€ с Π²ΠΊΠ»ΡŽΡ‡Π΅Π½Π½Ρ‹ΠΌ сопротивлСниСм R, с Π΅ΠΌΠΊΠΎΡΡ‚ΡŒΡŽ кондСнсатора C, с ΠΊΠ°Ρ‚ΡƒΡˆΠΊΠΎΠΉ индуктивности L, ΠΈΠ·ΠΎΠ±Ρ€Π°ΠΆΠ΅Π½Π½Ρ‹ΠΉ Π½Π° рисункС 1. КолСбания, происходящиС Π² Π½Π΅ΠΌ, — Π·Π°Ρ‚ΡƒΡ…Π°ΡŽΡ‰ΠΈΠ΅.

Рисунок 1

ИмСнно Π½Π°Π»ΠΈΡ‡ΠΈΠ΅ сопротивлСния становится Π³Π»Π°Π²Π½ΠΎΠΉ ΠΏΡ€ΠΈΡ‡ΠΈΠ½ΠΎΠΉ ΠΈΡ… затухания. Π”Π°Π½Π½Ρ‹ΠΉ процСсс Π²ΠΎΠ·ΠΌΠΎΠΆΠ΅Π½ посрСдствам ΠΏΠΎΡ‚Π΅Ρ€ΡŒ энСргии Π½Π° Π²Ρ‹Π΄Π΅Π»Π΅Π½ΠΈΠ΅ Π΄ΠΆΠΎΡƒΠ»Π΅Π²Π° Ρ‚Π΅ΠΏΠ»Π°. Аналог сопротивлСния Π² ΠΌΠ΅Ρ…Π°Π½ΠΈΠΊΠ΅ – дСйствиС сил трСния.

Π₯арактСристики Π·Π°Ρ‚ΡƒΡ…Π°ΡŽΡ‰ΠΈΡ… ΠΊΠΎΠ»Π΅Π±Π°Π½ΠΈΠΉ

Π—Π°Ρ‚ΡƒΡ…Π°ΡŽΡ‰ΠΈΠ΅ колСбания Ρ…Π°Ρ€Π°ΠΊΡ‚Π΅Ρ€ΠΈΠ·ΡƒΡŽΡ‚ коэффициСнтом затухания Ξ². ΠŸΡ€ΠΈΠΌΠ΅Π½ΠΈΠ² Π²Ρ‚ΠΎΡ€ΠΎΠΉ Π·Π°ΠΊΠΎΠ½ ΠΡŒΡŽΡ‚ΠΎΠ½Π°, ΠΏΠΎΠ»ΡƒΡ‡ΠΈΠΌ:

ma=-kx-yv,d2xdt2+rmdxdt+kmx=0,Ο‰02=km,Ξ²=r2m.

Из записи Π²ΠΈΠ΄Π½ΠΎ, Ρ‡Ρ‚ΠΎ Ξ²Β Π΄Π΅ΠΉΡΡ‚Π²ΠΈΡ‚Π΅Π»ΡŒΠ½ΠΎ являСтся характСристикой ΠΊΠΎΠ½Ρ‚ΡƒΡ€Π°. Π Π΅ΠΆΠ΅ вмСсто Ξ²Β ΠΏΡ€ΠΈΠΌΠ΅Π½ΡΡŽΡ‚ Π΄Π΅ΠΊΡ€Π΅ΠΌΠ΅Π½Ρ‚ затухания Ξ΄,

Π—Π½Π°Ρ‡Π΅Π½ΠΈΠ΅ aΒ (t) являСтся Π°ΠΌΠΏΠ»ΠΈΡ‚ΡƒΠ΄ΠΎΠΉ заряда, силы Ρ‚ΠΎΠΊΠ° ΠΈ Ρ‚Π°ΠΊ Π΄Π°Π»Π΅Π΅, δ равняСтся количСству ΠΊΠΎΠ»Π΅Π±Π°Π½ΠΈΠΉ, Π° NeΒ — ΠΏΠ΅Ρ€ΠΈΠΎΠ΄ Π²Ρ€Π΅ΠΌΠ΅Π½ΠΈ ΡƒΠΌΠ΅Π½ΡŒΡˆΠ΅Π½ΠΈΡ Π°ΠΌΠΏΠ»ΠΈΡ‚ΡƒΠ΄Ρ‹ Π² e Ρ€Π°Π·.

Для RLCΒ ΠΊΠΎΠ½Ρ‚ΡƒΡ€Π° ΠΏΡ€ΠΈΠΌΠ΅Π½ΠΈΠΌΠ° Ρ„ΠΎΡ€ΠΌΡƒΠ»Π° с ω частотой.

ΠŸΡ€ΠΈ нСбольшой Ξ΄β‰ͺ1 говорят, Ρ‡Ρ‚ΠΎ Ξ²β‰ͺΟ‰0Β Ο‰0=1LCΒ — собствСнная частота, ΠΎΡ‚ΡΡŽΠ΄Π° Ο‰β‰ˆΟ‰0.

ΠŸΡ€ΠΈ рассмотрСнии Π·Π°Ρ‚ΡƒΡ…Π°ΡŽΡ‰ΠΈΡ… ΠΊΠΎΠ»Π΅Π±Π°Π½ΠΈΠΉ ΠΏΠΎΡΠ»Π΅Π΄ΠΎΠ²Π°Ρ‚Π΅Π»ΡŒΠ½ΠΎΠ³ΠΎ ΠΊΠΎΠ½Ρ‚ΡƒΡ€Π° ΠΊΠΎΠ»Π΅Π±Π°Ρ‚Π΅Π»ΡŒΠ½Ρ‹ΠΉ ΠΊΠΎΠ½Ρ‚ΡƒΡ€ характСризуСтся Π΄ΠΎΠ±Ρ€ΠΎΡ‚Π½ΠΎΡΡ‚ΡŒΡŽ Q :

Q=1RLC=Ο‰0LR, Π³Π΄Π΅ R,Β L ΠΈΒ C — сопротивлСниС, ΠΈΠ½Π΄ΡƒΠΊΡ‚ΠΈΠ²Π½ΠΎΡΡ‚ΡŒ, Π΅ΠΌΠΊΠΎΡΡ‚ΡŒ, Π° Ο‰0- частота рСзонанса. Π’Ρ‹Ρ€Π°ΠΆΠ΅Π½ΠΈΠ΅ LC Π½Π°Π·Ρ‹Π²Π°ΡŽΡ‚ характСристичСским ΠΈΠ»ΠΈ Π²ΠΎΠ»Π½ΠΎΠ²Ρ‹ΠΌ сопротивлСниСм. Для ΠΏΠ°Ρ€Π°Π»Π»Π΅Π»ΡŒΠ½ΠΎΠ³ΠΎ ΠΊΠΎΠ½Ρ‚ΡƒΡ€Π° Ρ„ΠΎΡ€ΠΌΡƒΠ»Π° ΠΏΡ€ΠΈΠΌΠ΅Ρ‚ Π²ΠΈΠ΄:

Q=RLC=Rω0L.

R являСтся Π²Ρ…ΠΎΠ΄Π½Ρ‹ΠΌ сопротивлСниСм ΠΏΠ°Ρ€Π°Π»Π»Π΅Π»ΡŒΠ½ΠΎΠ³ΠΎ ΠΊΠΎΠ½Ρ‚ΡƒΡ€Π°.

ΠžΠΏΡ€Π΅Π΄Π΅Π»Π΅Π½ΠΈΠ΅ 2

Π­ΠΊΠ²ΠΈΠ²Π°Π»Π΅Π½Ρ‚Π½ΠΎΠ΅ ΠΎΠΏΡ€Π΅Π΄Π΅Π»Π΅Π½ΠΈΠ΅ добротности примСняСтся ΠΏΡ€ΠΈ слабых затуханиях. Π•Π³ΠΎ Π²Ρ‹Ρ€Π°ΠΆΠ°ΡŽΡ‚ Ρ‡Π΅Ρ€Π΅Π· ΠΎΡ‚Π½ΠΎΡˆΠ΅Π½ΠΈΠ΅ энСргий:

Q=Ο‰0WPd=2Ο€f0WPd, Π½Π°Π·Ρ‹Π²Π°Π΅ΠΌΠΎΠ΅ ΠΎΠ±Ρ‰Π΅ΠΉ Ρ„ΠΎΡ€ΠΌΡƒΠ»ΠΎΠΉ.

НуТна ΠΏΠΎΠΌΠΎΡ‰ΡŒ прСподаватСля?

Опиши Π·Π°Π΄Π°Π½ΠΈΠ΅Β β€” и наши экспСрты Ρ‚Π΅Π±Π΅ ΠΏΠΎΠΌΠΎΠ³ΡƒΡ‚!

ΠžΠΏΠΈΡΠ°Ρ‚ΡŒ Π·Π°Π΄Π°Π½ΠΈΠ΅

УравнСния Π·Π°Ρ‚ΡƒΡ…Π°ΡŽΡ‰ΠΈΡ… ΠΊΠΎΠ»Π΅Π±Π°Π½ΠΈΠΉ

Рассмотрим рисунок 1. ИзмСнСниС заряда q Π½Π° кондСнсаторС Π² Ρ‚Π°ΠΊΠΎΠΌ ΠΊΠΎΠ½Ρ‚ΡƒΡ€Π΅ описываСтся Π΄ΠΈΡ„Ρ„Π΅Ρ€Π΅Π½Ρ†ΠΈΠ°Π»ΡŒΠ½Ρ‹ΠΌ ΡƒΡ€Π°Π²Π½Π΅Π½ΠΈΠ΅ΠΌ:

q(t)=q0e(-Ξ²t)cosΟ‰t+a’0=q0e-Ξ²tcos(Ο‰t).

Если t=0, Ρ‚ΠΎ заряд кондСнсатора становится Ρ€Π°Π²Π½Ρ‹ΠΌ q0, ΠΈ Ρ‚ΠΎΠΊ Π² Ρ†Π΅ΠΏΠΈ отсутствуСт.

Если R>2LC измСнСния заряда Π½Π΅ относят ΠΊ колСбаниям, разряд Π½Π°Π·Ρ‹Π²Π°ΡŽΡ‚ апСриодичСским.

Π—Π½Π°Ρ‡Π΅Π½ΠΈΠ΅ сопротивлСния, ΠΏΡ€ΠΈ ΠΊΠΎΡ‚ΠΎΡ€ΠΎΠΌ колСбания ΠΏΡ€Π΅Π²Ρ€Π°Ρ‰Π°ΡŽΡ‚ΡΡ Π² апСриодичСский разряд кондСнсатора, критичСскоС Rk.

rΠΊΡ€=2LC.

Ѐункция изобраТаСтся Π°Π½Π°Π»ΠΎΠ³ΠΈΡ‡Π½ΠΎ рисунку 2.

Рисунок 2

ΠŸΡ€ΠΈΠΌΠ΅Ρ€ 1

Π—Π°ΠΏΠΈΡΠ°Ρ‚ΡŒ Π·Π°ΠΊΠΎΠ½ убывания энСргии, запасСнной Π² ΠΊΠΎΠ½Ρ‚ΡƒΡ€Π΅ WΒ (t)Β ΠΏΡ€ΠΈ WΒ (t=0)=W0 с Π·Π°Ρ‚ΡƒΡ…Π°ΡŽΡ‰ΠΈΠΌΠΈ колСбаниями. ΠžΠ±ΠΎΠ·Π½Π°Ρ‡ΠΈΡ‚ΡŒ коэффициСнт затухания Π² ΠΊΠΎΠ½Ρ‚ΡƒΡ€Π΅ Ξ², Π° ΡΠΎΠ±ΡΡ‚Π²Π΅Π½Π½ΡƒΡŽ частоту — Ο‰0.

РСшСниС

ΠžΡ‚ΠΏΡ€Π°Π²Π½Π°Ρ Ρ‚ΠΎΡ‡ΠΊΠ° Ρ€Π΅ΡˆΠ΅Π½ΠΈΡ – это ΠΏΡ€ΠΈΠΌΠ΅Π½Π΅Π½ΠΈΠ΅ Ρ„ΠΎΡ€ΠΌΡƒΠ»Ρ‹ измСнСния заряда Π½Π° кондСнсаторС Π² RLC — ΠΊΠΎΠ½Ρ‚ΡƒΡ€Π΅:

q(t)=q0e(-Ξ²t)cosΟ‰t+a’0=q0e-Ξ²tcos(Ο‰t).

ΠŸΡ€Π΅Π΄ΠΏΠΎΠ»ΠΎΠΆΠΈΠΌ, Ρ‡Ρ‚ΠΎ ΠΏΡ€ΠΈ t=0,Β a’0=0. Π’ΠΎΠ³Π΄Π° ΠΏΡ€ΠΈΠΌΠ΅Π½ΠΈΠΌ Π²Ρ‹Ρ€Π°ΠΆΠ΅Π½ΠΈΠ΅

I=dqdt.

Для нахоТдСния I(t):

I(t)=-Ο‰0q0e(-2Ξ²t)sin(Ο‰t+Ξ±), Π³Π΄Π΅ tgΒ Ξ±=Ξ²Ο‰.

ΠžΡ‡Π΅Π²ΠΈΠ΄Π½ΠΎ, Ρ‡Ρ‚ΠΎ элСктричСская энСргия WqΒ Π·Π°ΠΏΠΈΡˆΠ΅Ρ‚ΡΡ ΠΊΠ°ΠΊ:

Wq=q22C=q022Ce(-2Ξ²t)cos2(Ο‰t)=W0e(-2Ξ²t)cos2(Ο‰t).

Π’ΠΎΠ³Π΄Π° Π·Π½Π°Ρ‡Π΅Π½ΠΈΠ΅ ΠΌΠ°Π³Π½ΠΈΡ‚Π½ΠΎΠΉ энСргии ΠΊΠΎΠ½Ρ‚ΡƒΡ€Π° Wm равняСтся:

Wm=L2Ο‰02q02e(-2Ξ²t)sin2Ο‰t+a=W0e-2Ξ²tsin2Ο‰t+a.

Π—Π°ΠΏΠΈΡΡŒ ΠΏΠΎΠ»Π½ΠΎΠΉ энСргии Π±ΡƒΠ΄Π΅Ρ‚ ΠΈΠΌΠ΅Ρ‚ΡŒ Π²ΠΈΠ΄:

W=Wq+Wm=W0e(-2Ξ²t)(cos2(Ο‰t)+sin2(Ο‰t+a))==W0e(-2Ξ²t)1+Ξ²Ο‰0sin(2Ο‰t+Ξ±).

Π“Π΄Π΅ sinΒ Ξ±=Ξ²Ο‰0.

ΠžΡ‚Π²Π΅Ρ‚: WΒ (t)=W0e(-2Ξ²t)1+Ξ²Ο‰0sinΒ (2Ο‰t+a).

ΠŸΡ€ΠΈΠΌΠ΅Ρ€ 2

ΠŸΡ€ΠΈΠΌΠ΅Π½ΠΈΠ² Ρ€Π΅Π·ΡƒΠ»ΡŒΡ‚Π°Ρ‚ ΠΏΡ€Π΅Π΄Ρ‹Π΄ΡƒΡ‰Π΅Π³ΠΎ ΠΏΡ€ΠΈΠΌΠ΅Ρ€Π°, Π·Π°ΠΏΠΈΡΠ°Ρ‚ΡŒ Π²Ρ‹Ρ€Π°ΠΆΠ΅Π½ΠΈΠ΅ для энСргии, запасСнной Π² ΠΊΠΎΠ½Ρ‚ΡƒΡ€Π΅ WΒ (t), ΠΏΡ€ΠΈ ΠΌΠ΅Π΄Π»Π΅Π½Π½ΠΎ Π·Π°Ρ‚ΡƒΡ…Π°ΡŽΡ‰ΠΈΡ… колСбаниях. ΠΠ°Ρ‡Π΅Ρ€Ρ‚ΠΈΡ‚ΡŒ Π³Ρ€Π°Ρ„ΠΈΠΊ убывания энСргии.

РСшСниС

Если колСбания Π² ΠΊΠΎΠ½Ρ‚ΡƒΡ€Π΅ Π·Π°Ρ‚ΡƒΡ…Π°ΡŽΡ‚ ΠΌΠ΅Π΄Π»Π΅Π½Π½ΠΎ, Ρ‚ΠΎ:

Ξ²Ο‰0β‰ͺ1.

ΠžΡ‡Π΅Π²ΠΈΠ΄Π½ΠΎ, Π²Ρ‹Ρ€Π°ΠΆΠ΅Π½ΠΈΠ΅ энСргии, запасСнной Π² ΠΊΠΎΠ½Ρ‚ΡƒΡ€Π΅, вычислим ΠΈΠ·

WΒ (t)=W0e(-2Ξ²t)1+Ξ²Ο‰0sinΒ (2Ο‰t+a), ΠΏΡ€Π΅Π΄Π²Π°Ρ€ΠΈΡ‚Π΅Π»ΡŒΠ½ΠΎ ΠΏΡ€Π΅ΠΎΠ±Ρ€Π°Π·ΠΎΠ²Π°Π² Π΄ΠΎ WΒ (t)=W0e(-2Ξ²t).

Π’Π°ΠΊΠΎΠ΅ ΡƒΠΏΡ€ΠΎΡ‰Π΅Π½ΠΈΠ΅ Π²ΠΎΠ·ΠΌΠΎΠΆΠ½ΠΎ ΠΏΠΎ ΠΏΡ€ΠΈΡ‡ΠΈΠ½Π΅ выполнСния условия Ξ²Ο‰0β‰ͺ1, sinΒ (2Ο‰t+a)≀1, Ρ‡Ρ‚ΠΎ ΠΎΠ·Π½Π°Ρ‡Π°Π΅Ρ‚ Ξ²Ο‰0sinΒ (2Ο‰t+a)β‰ͺ1.

Рисунок 3

ΠžΡ‚Π²Π΅Ρ‚: WΒ (t)=W0e(-2Ξ²t). ЭнСргия Π² ΠΊΠΎΠ½Ρ‚ΡƒΡ€Π΅ ΡƒΠ±Ρ‹Π²Π°Π΅Ρ‚ ΠΏΠΎ экспонСнтС.

ΠžΠΏΡ€Π΅Π΄Π΅Π»ΠΈΡ‚ΡŒ Π°ΠΊΡ‚ΠΈΠ²Π½ΠΎΠ΅ сопротивлСниС ΠΊΠΎΠ»Π΅Π±Π°Ρ‚Π΅Π»ΡŒΠ½ΠΎΠ³ΠΎ ΠΊΠΎΠ½Ρ‚ΡƒΡ€Π° ΠΈΠ½Π΄ΡƒΠΊΡ‚ΠΈΠ²Π½ΠΎΡΡ‚ΡŒ

Для ΠΊΠΎΠ»Π΅Π±Π°Ρ‚Π΅Π»ΡŒΠ½ΠΎΠ³ΠΎ ΠΊΠΎΠ½Ρ‚ΡƒΡ€Π° рСкомСндуСтся Π²Ρ‹Π±Ρ€Π°Ρ‚ΡŒ ΠΊΠ°Ρ‚ΡƒΡˆΠΊΡƒ 220/127 ΠΎΡ‚ школьного трансформатора, ΠΌΠ°Π³Π°Π·ΠΈΠ½ кондСнсаторов Π 544, выставив Π΅ΠΌΠΊΠΎΡΡ‚ΡŒ порядка 0,5 ΠΌΠΊΠ€, ΠΈ ΠΌΠ°Π³Π°Π·ΠΈΠ½ сопротивлСний Π 33.

ΠžΠΏΡ€Π΅Π΄Π΅Π»ΠΈΡ‚ΡŒ ΠΈΠ½Π΄ΡƒΠΊΡ‚ΠΈΠ²Π½ΠΎΡΡ‚ΡŒ ΠΈ Π°ΠΊΡ‚ΠΈΠ²Π½ΠΎΠ΅ сопротивлСниС ΠΊΠ°Ρ‚ΡƒΡˆΠΊΠΈ ΠΌΠ΅Ρ‚ΠΎΠ΄ΠΎΠΌ Π°ΠΌΠΏΠ΅Ρ€ΠΌΠ΅Ρ‚Ρ€Π° ΠΈ Π²ΠΎΠ»ΡŒΡ‚ΠΌΠ΅Ρ‚Ρ€Π°. Для этого ΡΠΎΠ±Ρ€Π°Ρ‚ΡŒ схСму, ΡƒΠΊΠ°Π·Π°Π½Π½ΡƒΡŽ Π½Π° (Рис. 4). ΠŸΡ€ΠΈ ΠΏΠΎΠ΄ΠΊΠ»ΡŽΡ‡Π΅Π½ΠΈΠΈ Π² качСствС источника Ρ‚ΠΎΠΊΠ° постоянного Π²Ρ‹Ρ…ΠΎΠ΄Π° Π’Π‘-24 сопротивлСниС rL Ρ€Π°Π²Π½ΠΎ

,

Π° ΠΏΡ€ΠΈ ΠΏΠΎΠ΄ΠΊΠ»ΡŽΡ‡Π΅Π½ΠΈΠΈ ΠΏΠ΅Ρ€Π΅ΠΌΠ΅Π½Π½ΠΎΠ³ΠΎ Π²Ρ‹Ρ…ΠΎΠ΄Π° – сопротивлСниС Z Ρ€Π°Π²Π½ΠΎ:

,

ΠΎΡ‚ΠΊΡƒΠ΄Π° ΠΌΠΎΠΆΠ½ΠΎ Ρ€Π°ΡΡΡ‡ΠΈΡ‚Π°Ρ‚ΡŒ ΠΈΠ½Π΄ΡƒΠΊΡ‚ΠΈΠ²Π½ΠΎΡΡ‚ΡŒ L:

.

РасчСт ΠΏΠ°Ρ€Π°ΠΌΠ΅Ρ‚Ρ€ΠΎΠ² ΠΊΠΎΠ»Π΅Π±Π°Ρ‚Π΅Π»ΡŒΠ½ΠΎΠ³ΠΎ ΠΊΠΎΠ½Ρ‚ΡƒΡ€Π° ΠΈ ΡΠΊΡΠΏΠ΅Ρ€ΠΈΠΌΠ΅Π½Ρ‚Π°Π»ΡŒΠ½ΠΎΠ΅ ΠΏΠΎΠ»ΡƒΡ‡Π΅Π½ΠΈΠ΅ Π·Π°Ρ‚ΡƒΡ…Π°ΡŽΡ‰ΠΈΡ… ΠΊΠΎΠ»Π΅Π±Π°Π½ΠΈΠΉ.

Π—Π°Π΄Π°Π² Π΅ΠΌΠΊΠΎΡΡ‚ΡŒ кондСнсатора порядка 0,1 ΠΌΠΊΠ€ ΠΈ Π°ΠΊΡ‚ΠΈΠ²Π½ΠΎΠ΅ сопротивлСниС R Ρ€Π°Π²Π½ΠΎΠ΅ Π½ΡƒΠ»ΡŽ, Ρ€Π°ΡΡΡ‡ΠΈΡ‚Π°Ρ‚ΡŒ ΠΏΠ°Ρ€Π°ΠΌΠ΅Ρ‚Ρ€Ρ‹ ΠΏΠΎΠ»ΡƒΡ‡ΠΈΠ²ΡˆΠ΅Π³ΠΎΡΡ ΠΊΠΎΠ»Π΅Π±Π°Ρ‚Π΅Π»ΡŒΠ½ΠΎΠ³ΠΎ ΠΊΠΎΠ½Ρ‚ΡƒΡ€Π°: частоту n (ΠΈΠ»ΠΈ w), коэффициСнт затухания b, ΠΏΠ΅Ρ€ΠΈΠΎΠ΄ Π·Π°Ρ‚ΡƒΡ…Π°ΡŽΡ‰ΠΈΡ… ΠΊΠΎΠ»Π΅Π±Π°Π½ΠΈΠΉ Π’, логарифмичСский Π΄Π΅ΠΊΡ€Π΅ΠΌΠ΅Π½Ρ‚ затухания d, Π΄ΠΎΠ±Ρ€ΠΎΡ‚Π½ΠΎΡΡ‚ΡŒ ΠΊΠΎΠ»Π΅Π±Π°Ρ‚Π΅Π»ΡŒΠ½ΠΎΠ³ΠΎ ΠΊΠΎΠ½Ρ‚ΡƒΡ€Π° Q, критичСскоС сопротивлСниС RΠΊΡ€.

Π‘ΠΎΠ±Ρ€Π°Ρ‚ΡŒ схСму (Рис. 5), выставив Π½Π° ΠΌΠ°Π³Π°Π·ΠΈΠ½Π΅ СмкостСй 0,1 ΠΌΠΊΠ€, Π½Π° ΠΌΠ°Π³Π°Π·ΠΈΠ½Π΅ сопротивлСний – 0 Ом. Для Ρ‚ΠΎΠ³ΠΎ, Ρ‡Ρ‚ΠΎΠ±Ρ‹ ΠΊΠ°Ρ€Ρ‚ΠΈΠ½Π° Π·Π°Ρ‚ΡƒΡ…Π°ΡŽΡ‰ΠΈΡ… ΠΊΠΎΠ»Π΅Π±Π°Π½ΠΈΠΉ Π±Ρ‹Π»Π° постоянно Π²ΠΈΠ΄Π½Π° Π½Π° экранС осциллографа, Π½Π΅ΠΎΠ±Ρ…ΠΎΠ΄ΠΈΠΌΠΎ пСриодичСски Π΄ΠΎΠ±Π°Π²Π»ΡΡ‚ΡŒ ΡΠ½Π΅Ρ€Π³ΠΈΡŽ Π² ΠΊΠΎΠ»Π΅Π±Π°Ρ‚Π΅Π»ΡŒΠ½Ρ‹ΠΉ ΠΊΠΎΠ½Ρ‚ΡƒΡ€ подзаряТая кондСнсатор. Π’ качСствС пСриодичСского источника энСргии ΠΈΡΠΏΠΎΠ»ΡŒΠ·ΡƒΠ΅Ρ‚ΡΡ Π²Ρ‹Ρ…ΠΎΠ΄ ΠΏΠΈΠ»ΠΎΠΎΠ±Ρ€Π°Π·Π½ΠΎΠ³ΠΎ напряТСния Π½Π° ΠΏΡ€Π°Π²ΠΎΠΉ Π±ΠΎΠΊΠΎΠ²ΠΎΠΉ ΠΏΠ°Π½Π΅Π»ΠΈ осциллографа. Частоту Ρ€Π°Π·Π²Π΅Ρ€Ρ‚ΠΊΠΈ осциллографа Π½Π°Π΄ΠΎ ΠΏΠΎΠ΄ΠΎΠ±Ρ€Π°Ρ‚ΡŒ Ρ‚Π°ΠΊ, Ρ‡Ρ‚ΠΎΠ±Ρ‹ Π½Π° ΠΎΠ΄ΠΈΠ½ ΠΏΠ΅Ρ€ΠΈΠΎΠ΄ Ρ€Π°Π·Π²Π΅Ρ€Ρ‚ΠΊΠΈ ΠΏΡ€ΠΈΡ…ΠΎΠ΄ΠΈΠ»ΠΎΡΡŒ нСсколько ΠΏΠ΅Ρ€ΠΈΠΎΠ΄ΠΎΠ² Π·Π°Ρ‚ΡƒΡ…Π°ΡŽΡ‰ΠΈΡ… ΠΊΠΎΠ»Π΅Π±Π°Π½ΠΈΠΉ.

ΠšΠΎΠ½Π΄Π΅Π½ΡΠ°Ρ‚ΠΎΡ€ Π‘Π΄ΠΈΡ„ ΠΈ Π²Ρ…ΠΎΠ΄Π½ΠΎΠ΅ сопротивлСниС осциллографа RΠ²Ρ… ΠΏΡ€Π΅Π΄ΡΡ‚Π°Π²Π»ΡΡŽΡ‚ ΠΈΠ· сСбя Π΄ΠΈΡ„Ρ„Π΅Ρ€Π΅Π½Ρ†ΠΈΡ€ΡƒΡŽΡ‰ΡƒΡŽ Ρ†Π΅ΠΏΡŒ, ΠΏΡ€Π΅Π²Ρ€Π°Ρ‰Π°ΡŽΡ‰ΡƒΡŽ ΠΏΠΈΠ»ΠΎΠΎΠ±Ρ€Π°Π·Π½Ρ‹ΠΉ сигнал Π² ΠΈΠΌΠΏΡƒΠ»ΡŒΡΠ½Ρ‹ΠΉ (Рис. 6). ΠŸΡ€ΠΈ ΠΏΠ»Π°Π²Π½ΠΎΠΌ нарастании напряТСния кондСнсатор успСваСт Π·Π°Ρ€ΡΠΆΠ°Ρ‚ΡŒΡΡ, напряТСниС Π½Π° Π½Π΅ΠΌ Π² ΠΊΠ°ΠΆΠ΄Ρ‹ΠΉ ΠΌΠΎΠΌΠ΅Π½Ρ‚ Π²Ρ€Π΅ΠΌΠ΅Π½ΠΈ практичСски Ρ€Π°Π²Π½ΠΎ Π½Π°ΠΏΡ€ΡΠΆΠ΅Π½ΠΈΡŽ источника ΠΏΠΈΠ»ΠΎΠΎΠ±Ρ€Π°Π·Π½ΠΎΠ³ΠΎ сигнала, ΠΈ Ρ‚ΠΎΠΊ Π² Ρ†Π΅ΠΏΠΈ отсутствуСт. ΠŸΡ€ΠΈ Ρ€Π΅Π·ΠΊΠΎΠΌ ΡƒΠΌΠ΅Π½ΡŒΡˆΠ΅Π½ΠΈΠΈ напряТСния Π² Ρ†Π΅ΠΏΠΈ Π½Π°Π±Π»ΡŽΠ΄Π°Π΅Ρ‚ΡΡ ΠΈΠΌΠΏΡƒΠ»ΡŒΡ Ρ‚ΠΎΠΊΠ° разрядки кондСнсатора. Π’Ρ‹Ρ…ΠΎΠ΄Π½ΠΎΠ΅ напряТСниС являСтся Π΄ΠΈΡ„Ρ„Π΅Ρ€Π΅Π½Ρ†ΠΈΠ°Π»ΠΎΠΌ Π²Ρ…ΠΎΠ΄Π½ΠΎΠ³ΠΎ напряТСния ΠΏΠΎ Π²Ρ€Π΅ΠΌΠ΅Π½ΠΈ. ΠŸΠΎΠ΄ΠΎΠ±Ρ€Π°Ρ‚ΡŒ Π‘Π΄ΠΈΡ„ порядка 100 ΒΈ 1000 ΠΏΠ€.

По ΠΏΠΎΠ»ΡƒΡ‡Π΅Π½Π½ΠΎΠΉ ΠΊΠ°Ρ€Ρ‚ΠΈΠ½Π΅ ΠΎΠΏΡ€Π΅Π΄Π΅Π»ΠΈΡ‚ΡŒ ΠΏΠ°Ρ€Π°ΠΌΠ΅Ρ‚Ρ€Ρ‹ ΠΊΠΎΠ»Π΅Π±Π°Ρ‚Π΅Π»ΡŒΠ½ΠΎΠ³ΠΎ ΠΊΠΎΠ½Ρ‚ΡƒΡ€Π° ΠΈ ΡΡ€Π°Π²Π½ΠΈΡ‚ΡŒ ΠΈΡ… с рассчитанными Ρ€Π°Π½Π΅Π΅. МСняя ΠΈΠ½Π΄ΡƒΠΊΡ‚ΠΈΠ²Π½ΠΎΡΡ‚ΡŒ ΠΊΠ°Ρ‚ΡƒΡˆΠΊΠΈ, вводя Π² Π½Π΅Π΅ сСрдСчник, ΠΈ Π΅ΠΌΠΊΠΎΡΡ‚ΡŒ кондСнсатора, ΠΏΡ€ΠΎΠ½Π°Π±Π»ΡŽΠ΄Π°Ρ‚ΡŒ ΠΈ ΠΎΠ±ΡŠΡΡΠ½ΠΈΡ‚ΡŒ ΠΈΠ·ΠΌΠ΅Π½Π΅Π½ΠΈΠ΅ ΠΊΠ°Ρ€Ρ‚ΠΈΠ½Ρ‹ Π·Π°Ρ‚ΡƒΡ…Π°ΡŽΡ‰ΠΈΡ… ΠΊΠΎΠ»Π΅Π±Π°Π½ΠΈΠΉ.

ΠŸΡ€ΠΎΠ½Π°Π±Π»ΡŽΠ΄Π°Ρ‚ΡŒ ΠΈΠ·ΠΌΠ΅Π½Π΅Π½ΠΈΠ΅ ΠΊΠ°Ρ€Ρ‚ΠΈΠ½Ρ‹ ΠΏΡ€ΠΈ ΡƒΠ²Π΅Π»ΠΈΡ‡Π΅Π½ΠΈΠΈ Π°ΠΊΡ‚ΠΈΠ²Π½ΠΎΠ³ΠΎ сопротивлСния R. Π’Ρ‹ΡΡ‚Π°Π²ΠΈΡ‚ΡŒ Π½Π° ΠΌΠ°Π³Π°Π·ΠΈΠ½Π΅ сопротивлСний Ρ‚Π°ΠΊΠΎΠ΅ R, Ρ‡Ρ‚ΠΎΠ±Ρ‹ Π²Ρ‹ΠΏΠΎΠ»Π½ΡΠ»ΠΎΡΡŒ условиС:

,

ΠΈ ΡƒΠ±Π΅Π΄ΠΈΡ‚ΡŒΡΡ, Ρ‡Ρ‚ΠΎ колСбания Π² ΠΊΠΎΠ½Ρ‚ΡƒΡ€Π΅ ΠΎΡ‚ΡΡƒΡ‚ΡΡ‚Π²ΡƒΡŽΡ‚.

Вопросы ΠΊ Π·Π°Ρ‡Π΅Ρ‚Ρƒ ΠΏΠΎ Ρ€Π°Π±ΠΎΡ‚Π΅.

– ΠžΠ±ΡŠΡΡΠ½ΠΈΡ‚ΡŒ физичСский ΠΌΠ΅Ρ…Π°Π½ΠΈΠ·ΠΌ элСктромагнитных ΠΊΠΎΠ»Π΅Π±Π°Π½ΠΈΠΉ Π² ΠΊΠΎΠ»Π΅Π±Π°Ρ‚Π΅Π»ΡŒΠ½ΠΎΠΌ ΠΊΠΎΠ½Ρ‚ΡƒΡ€Π΅.

– Как прСобразуСтся энСргия ΠΏΡ€ΠΈ элСктромагнитных колСбаниях ΠΈ Ρ‡Π΅ΠΌΡƒ Ρ€Π°Π²Π½Π° полная энСргия?

– Как влияСт Π½Π°Π»ΠΈΡ‡ΠΈΠ΅ Π°ΠΊΡ‚ΠΈΠ²Π½ΠΎΠ³ΠΎ сопротивлСния ΠΊΠΎΠ»Π΅Π±Π°Ρ‚Π΅Π»ΡŒΠ½ΠΎΠ³ΠΎ ΠΊΠΎΠ½Ρ‚ΡƒΡ€Π° Π½Π° элСктромагнитныС колСбания? Π§Ρ‚ΠΎ Ρ‚Π°ΠΊΠΎΠ΅ Π·Π°Ρ‚ΡƒΡ…Π°ΡŽΡ‰ΠΈΠ΅ элСктромагнитныС колСбания?

– КакиС ΠΏΠ°Ρ€Π°ΠΌΠ΅Ρ‚Ρ€Ρ‹ ΠΊΠΎΠ½Ρ‚ΡƒΡ€Π° ΠΎΠΏΡ€Π΅Π΄Π΅Π»ΡΡŽΡ‚ Ρ…Π°Ρ€Π°ΠΊΡ‚Π΅Ρ€ элСктромагнитных ΠΊΠΎΠ»Π΅Π±Π°Π½ΠΈΠΉ Π² ΠΊΠΎΠ½Ρ‚ΡƒΡ€Π΅?

– ΠžΠ±ΡŠΡΡΠ½ΠΈΡ‚ΡŒ, ΠΏΠΎΡ‡Π΅ΠΌΡƒ Π½Π°Π»ΠΈΡ‡ΠΈΠ΅ критичСского сопротивлСния Π² Ρ†Π΅ΠΏΠΈ прСпятствуСт возникновСнию элСктромагнитных ΠΊΠΎΠ»Π΅Π±Π°Π½ΠΈΠΉ Π² ΠΊΠΎΠ½Ρ‚ΡƒΡ€Π΅.

Лабораторная Ρ€Π°Π±ΠΎΡ‚Π° β„– 10

ЯвлСния Π² цСпях ΠΏΠ΅Ρ€Π΅ΠΌΠ΅Π½Π½ΠΎΠ³ΠΎ Ρ‚ΠΎΠΊΠ°

ЦСль Ρ€Π°Π±ΠΎΡ‚Ρ‹.

Π˜Π·ΡƒΡ‡ΠΈΡ‚ΡŒ закономСрности явлСний, Π½Π°Π±Π»ΡŽΠ΄Π°Π΅ΠΌΡ‹Ρ… Π² цСпях ΠΏΠ΅Ρ€Π΅ΠΌΠ΅Π½Π½ΠΎΠ³ΠΎ Ρ‚ΠΎΠΊΠ°.

Знания, Π½Π΅ΠΎΠ±Ρ…ΠΎΠ΄ΠΈΠΌΡ‹Π΅ для допуска ΠΊ Ρ€Π°Π±ΠΎΡ‚Π΅.

– Π˜Π½Π΄ΡƒΠΊΡ‚ΠΈΠ²Π½ΠΎΡΡ‚ΡŒ ΠΈ Π΅ΠΌΠΊΠΎΡΡ‚ΡŒ Π² цСпях ΠΏΠ΅Ρ€Π΅ΠΌΠ΅Π½Π½ΠΎΠ³ΠΎ Ρ‚ΠΎΠΊΠ°;

– Π—Π°ΠΊΠΎΠ½ Ома для ΠΏΠ΅Ρ€Π΅ΠΌΠ΅Π½Π½ΠΎΠ³ΠΎ Ρ‚ΠΎΠΊΠ°;

– РСзонансныС явлСния Π² цСпях ΠΏΠ΅Ρ€Π΅ΠΌΠ΅Π½Π½ΠΎΠ³ΠΎ Ρ‚ΠΎΠΊΠ°.

ΠšΡ€Π°Ρ‚ΠΊΠΈΠ΅ свСдСния ΠΈΠ· Ρ‚Π΅ΠΎΡ€ΠΈΠΈ.

ΠŸΠ΅Ρ€Π΅ΠΌΠ΅Π½Π½Ρ‹ΠΌ Ρ‚ΠΎΠΊΠΎΠΌ называСтся любой Ρ‚ΠΎΠΊ, Π²Π΅Π»ΠΈΡ‡ΠΈΠ½Π° ΠΊΠΎΡ‚ΠΎΡ€ΠΎΠ³ΠΎ пСриодичСски мСняСтся со Π²Ρ€Π΅ΠΌΠ΅Π½Π΅ΠΌ. Но Ρ‡Π°Ρ‰Π΅ всСго ΠΏΠΎΠ΄ ΠΏΠ΅Ρ€Π΅ΠΌΠ΅Π½Π½Ρ‹ΠΌ Ρ‚ΠΎΠΊΠΎΠΌ подразумСваСтся Ρ‚ΠΎΠΊ, ΠΌΠ΅Π½ΡΡŽΡ‰ΠΈΠΉΡΡ ΠΏΠΎ Π·Π°ΠΊΠΎΠ½Ρƒ синуса (ΠΈΠ»ΠΈ косинуса):

,

Π³Π΄Π΅ I – Π°ΠΌΠΏΠ»ΠΈΡ‚ΡƒΠ΄Π° Ρ‚ΠΎΠΊΠ°, – цикличСская частота, Π° – Ρ„Π°Π·Π° ΠΊΠΎΠ»Π΅Π±Π°Π½ΠΈΠΉ, Ρ…Π°Ρ€Π°ΠΊΡ‚Π΅Ρ€ΠΈΠ·ΡƒΡŽΡ‰Π°Ρ состояниС ΠΊΠΎΠ»Π΅Π±Π°Ρ‚Π΅Π»ΡŒΠ½ΠΎΠΉ систСмы Π² Π΄Π°Π½Π½Ρ‹ΠΉ ΠΌΠΎΠΌΠ΅Π½Ρ‚ Π²Ρ€Π΅ΠΌΠ΅Π½ΠΈ t.

Рассмотрим ΡΠ»Π΅ΠΊΡ‚Ρ€ΠΈΡ‡Π΅ΡΠΊΡƒΡŽ Ρ†Π΅ΠΏΡŒ, ΡΠΎΠ΄Π΅Ρ€ΠΆΠ°Ρ‰ΡƒΡŽ ΠΏΠΎΡΠ»Π΅Π΄ΠΎΠ²Π°Ρ‚Π΅Π»ΡŒΠ½ΠΎ соСдинСнныС рСзистор, кондСнсатор ΠΈ ΠΊΠ°Ρ‚ΡƒΡˆΠΊΡƒ индуктивности, ΠΏΠΎΠ΄ΠΊΠ»ΡŽΡ‡Π΅Π½Π½ΡƒΡŽ ΠΊ источнику ΠΏΠ΅Ρ€Π΅ΠΌΠ΅Π½Π½ΠΎΠ³ΠΎ напряТСния (Рис. 1). По этой Ρ†Π΅ΠΏΠΈ ΠΏΡ€ΠΎΡ‚Π΅ΠΊΠ°Π΅Ρ‚ Ρ‚ΠΎΠΊ, ΠΌΠ΅Π½ΡΡŽΡ‰ΠΈΠΉΡΡ ΠΏΠΎ ΡΠΈΠ½ΡƒΡΠΎΠΈΠ΄Π°Π»ΡŒΠ½ΠΎΠΌΡƒ Π·Π°ΠΊΠΎΠ½Ρƒ

.

Π’Ρ‹Ρ…ΠΎΠ΄Π½ΠΎΠ΅ напряТСниС источника Ρ‚ΠΎΠΊΠ° ΠΏΡ€ΠΈ ΠΏΡ€ΠΎΡ‚Π΅ΠΊΠ°Π½ΠΈΠΈ постоянного Ρ‚ΠΎΠΊΠ° ΠΏΠΎ ΠΏΠΎΡΠ»Π΅Π΄ΠΎΠ²Π°Ρ‚Π΅Π»ΡŒΠ½ΠΎΠΌΡƒ соСдинСнию ΠΏΡ€ΠΎΠ²ΠΎΠ΄Π½ΠΈΠΊΠΎΠ² Π΄ΠΎΠ»ΠΆΠ½ΠΎ Π±Ρ‹Ρ‚ΡŒ Ρ€Π°Π²Π½ΠΎ суммС ΠΏΠ°Π΄Π΅Π½ΠΈΠΉ напряТСния Π½Π° ΠΊΠ°ΠΆΠ΄ΠΎΠΌ ΠΏΡ€ΠΎΠ²ΠΎΠ΄Π½ΠΈΠΊΠ΅:

,

Π½ΠΎ ΠΏΡ€ΠΈ ΠΏΠ΅Ρ€Π΅ΠΌΠ΅Π½Π½ΠΎΠΌ Ρ‚ΠΎΠΊΠ΅ Π² Ρ†Π΅ΠΏΠΈ, содСрТащСй Π΅ΠΌΠΊΠΎΡΡ‚ΡŒ ΠΈ ΠΈΠ½Π΄ΡƒΠΊΡ‚ΠΈΠ²Π½ΠΎΡΡ‚ΡŒ, Π΅ΡΡ‚ΡŒ Π½Π΅ΠΊΠΎΡ‚ΠΎΡ€Ρ‹Π΅ отличия.

ПадСниС напряТСния Π½Π° рСзисторС колСблСтся ΠΏΠΎ Ρ‚Π°ΠΊΠΎΠΌΡƒ ΠΆΠ΅ Π·Π°ΠΊΠΎΠ½Ρƒ, ΠΊΠ°ΠΊ ΠΈ Ρ‚ΠΎΠΊ

,

ΠΈ ΠΈΡ… Ρ„Π°Π·Ρ‹ ΠΊΠΎΠ»Π΅Π±Π°Π½ΠΈΠΉ ΡΠΎΠ²ΠΏΠ°Π΄Π°ΡŽΡ‚.

НапряТСниС Π½Π° ΠΎΠ±ΠΊΠ»Π°Π΄ΠΊΠ°Ρ… кондСнсатора ΠΏΡ€ΠΎΠΏΠΎΡ€Ρ†ΠΈΠΎΠ½Π°Π»ΡŒΠ½ΠΎ заряду Π½Π° Π½ΠΈΡ… Π² ΠΊΠ°ΠΆΠ΄Ρ‹ΠΉ ΠΌΠΎΠΌΠ΅Π½Ρ‚ Π²Ρ€Π΅ΠΌΠ΅Π½ΠΈ

,

Π° заряд ΠΌΠΎΠΆΠ½ΠΎ ΠΎΠΏΡ€Π΅Π΄Π΅Π»ΠΈΡ‚ΡŒ ΠΊΠ°ΠΊ ΠΈΠ½Ρ‚Π΅Π³Ρ€Π°Π» Ρ‚ΠΎΠΊΠ° ΠΏΠΎ Π²Ρ€Π΅ΠΌΠ΅Π½ΠΈ

.

.

Из этого выраТСния ΡΠ»Π΅Π΄ΡƒΡŽΡ‚ Π΄Π²Π° Π²Ρ‹Π²ΠΎΠ΄Π°: Π²ΠΎ-ΠΏΠ΅Ρ€Π²Ρ‹Ρ…, колСбания напряТСния Π½Π° кондСнсаторС ΠΎΡ‚ΡΡ‚Π°ΡŽΡ‚ ΠΎΡ‚ ΠΊΠΎΠ»Π΅Π±Π°Π½ΠΈΠΉ Ρ‚ΠΎΠΊΠ° Π½Π° , Π° Π²ΠΎ-Π²Ρ‚ΠΎΡ€Ρ‹Ρ…, Π°ΠΌΠΏΠ»ΠΈΡ‚ΡƒΠ΄Π½ΠΎΠ΅ Π·Π½Π°Ρ‡Π΅Π½ΠΈΠ΅ напряТСниС связано с Π°ΠΌΠΏΠ»ΠΈΡ‚ΡƒΠ΄Π½Ρ‹ΠΌ Π·Π½Π°Ρ‡Π΅Π½ΠΈΠ΅ΠΌ Ρ‚ΠΎΠΊΠ° ΡΠΎΠΎΡ‚Π½ΠΎΡˆΠ΅Π½ΠΈΠ΅ΠΌ:

,

Π³Π΄Π΅ называСтся Смкостным сопротивлСниСм.

ΠŸΡ€ΠΈ ΠΏΡ€ΠΎΡ‚Π΅ΠΊΠ°Π½ΠΈΠΈ ΠΏΠ΅Ρ€Π΅ΠΌΠ΅Π½Π½ΠΎΠ³ΠΎ Ρ‚ΠΎΠΊΠ° Ρ‡Π΅Ρ€Π΅Π· ΠΊΠ°Ρ‚ΡƒΡˆΠΊΡƒ Π² Π½Π΅ΠΉ Π²ΠΎΠ·Π½ΠΈΠΊΠ°Π΅Ρ‚ Π­Π”Π‘ самоиндукции, ΠΏΡ€Π΅ΠΏΡΡ‚ΡΡ‚Π²ΡƒΡŽΡ‰Π°Ρ измСнСнию Ρ‚ΠΎΠΊΠ°

.

Π’ этом случаС для участка Ρ†Π΅ΠΏΠΈ, содСрТащСго ΠΊΠ°Ρ‚ΡƒΡˆΠΊΡƒ (Ρ‚.Π΅. источник Π­Π”Π‘, Π²ΠΊΠ»ΡŽΡ‡Π΅Π½Π½Ρ‹ΠΉ навстрСчу Ρ‚ΠΎΠΊΡƒ) ΠΏΠ°Π΄Π΅Π½ΠΈΠ΅ напряТСния Ρ€Π°Π²Π½ΠΎ

,

Ρ‚Π°ΠΊ ΠΊΠ°ΠΊ ΠΏΠΎΠΌΠΈΠΌΠΎ Π­Π”Π‘ самоиндукции происходит ΠΏΠ°Π΄Π΅Π½ΠΈΠ΅ напряТСния Π½Π° сопротивлСнии ΠΏΡ€ΠΎΠ²ΠΎΠ΄Π° r, ΠΈΠ· ΠΊΠΎΡ‚ΠΎΡ€ΠΎΠ³ΠΎ ΠΈΠ·Π³ΠΎΡ‚ΠΎΠ²Π»Π΅Π½Π° ΠΊΠ°Ρ‚ΡƒΡˆΠΊΠ°. Если ΠΏΡ€Π΅Π΄ΠΏΠΎΠ»ΠΎΠΆΠΈΡ‚ΡŒ Π΅Π³ΠΎ ΠΌΠ°Π»Ρ‹ΠΌ, Ρ‚ΠΎ ΠΈ

.

ΠžΡ‡Π΅Π²ΠΈΠ΄Π½ΠΎ, Ρ‡Ρ‚ΠΎ колСбания напряТСния Π½Π° ΠΊΠ°Ρ‚ΡƒΡˆΠΊΠ΅ ΠΎΠΏΠ΅Ρ€Π΅ΠΆΠ°ΡŽΡ‚ колСбания Ρ‚ΠΎΠΊΠ° Π½Π° , Π° ΠΈΡ… Π°ΠΌΠΏΠ»ΠΈΡ‚ΡƒΠ΄Ρ‹ связаны ΡΠΎΠΎΡ‚Π½ΠΎΡˆΠ΅Π½ΠΈΠ΅ΠΌ

,

Π³Π΄Π΅ – ΠΈΠ½Π΄ΡƒΠΊΡ‚ΠΈΠ²Π½ΠΎΠ΅ сопротивлСниС ΠΊΠ°Ρ‚ΡƒΡˆΠΊΠΈ.

БопротивлСния R, r Π½Π°Π·Ρ‹Π²Π°ΡŽΡ‚ΡΡ Π°ΠΊΡ‚ΠΈΠ²Π½Ρ‹ΠΌΠΈ (ΠΈΠ»ΠΈ омичСскими), Π° сопротивлСния XL ΠΈ XC – Ρ€Π΅Π°ΠΊΡ‚ΠΈΠ²Π½Ρ‹ΠΌΠΈ.

Π‘ΠΎΠΎΡ‚Π½ΠΎΡˆΠ΅Π½ΠΈΡ Ρ„Π°Π· ΠΊΠΎΠ»Π΅Π±Π°Π½ΠΈΠΉ напряТСний Π½Π° Π°ΠΊΡ‚ΠΈΠ²Π½Ρ‹Ρ… ΠΈ Ρ€Π΅Π°ΠΊΡ‚ΠΈΠ²Π½Ρ‹Ρ… сопротивлСниях ΠΌΠΎΠΆΠ½ΠΎ ΠΏΡ€ΠΎΠΈΠ»Π»ΡŽΡΡ‚Ρ€ΠΈΡ€ΠΎΠ²Π°Ρ‚ΡŒ Π½Π° Π²Π΅ΠΊΡ‚ΠΎΡ€Π½ΠΎΠΉ Π΄ΠΈΠ°Π³Ρ€Π°ΠΌΠΌΠ΅ (Рис. 2). Π—Π° основноС Π½Π°ΠΏΡ€Π°Π²Π»Π΅Π½ΠΈΠ΅ Π½Π°Π΄ΠΎ Π²Π·ΡΡ‚ΡŒ силу Ρ‚ΠΎΠΊΠ°, Ρ‚Π°ΠΊ ΠΊΠ°ΠΊ ΠΎΠ½ являСтся ΠΎΠ±Ρ‰ΠΈΠΌ для ΠΏΠΎΡΠ»Π΅Π΄ΠΎΠ²Π°Ρ‚Π΅Π»ΡŒΠ½ΠΎ соСдинСнных элСмСнтов схСмы. Π’Π΅Π»ΠΈΡ‡ΠΈΠ½Ρƒ Π°ΠΌΠΏΠ»ΠΈΡ‚ΡƒΠ΄Ρ‹ Π²Ρ‹Ρ…ΠΎΠ΄Π½ΠΎΠ³ΠΎ напряТСния ΠΌΠΎΠΆΠ½ΠΎ ΠΎΠΏΡ€Π΅Π΄Π΅Π»ΠΈΡ‚ΡŒ, ΠΈΡΠΏΠΎΠ»ΡŒΠ·ΡƒΡ Π·Π°ΠΊΠΎΠ½ слоТСния Π²Π΅ΠΊΡ‚ΠΎΡ€ΠΎΠ²:

.

Π’ΠΈΠ΄Π½ΠΎ, Ρ‡Ρ‚ΠΎ колСбания напряТСния ΠΈ Ρ‚ΠΎΠΊΠ° сдвинуты ΠΏΠΎ Ρ„Π°Π·Π΅ Π΄Ρ€ΡƒΠ³ ΠΎΡ‚Π½ΠΎΡΠΈΡ‚Π΅Π»ΡŒΠ½ΠΎ Π΄Ρ€ΡƒΠ³Π° Π½Π° j. ВынСся ΠΎΠ±Ρ‰ΠΈΠΉ ΠΌΠ½ΠΎΠΆΠΈΡ‚Π΅Π»ΡŒ – силу Ρ‚ΠΎΠΊΠ° – ΠΈΠ·-ΠΏΠΎΠ΄ корня, ΠΏΠΎΠ»ΡƒΡ‡Π°Π΅ΠΌ Π²Ρ‹Ρ€Π°ΠΆΠ΅Π½ΠΈΠ΅:

,

Π³Π΄Π΅ R – всС Π°ΠΊΡ‚ΠΈΠ²Π½ΠΎΠ΅ сопротивлСниС элСктричСской Ρ†Π΅ΠΏΠΈ. Π­Ρ‚ΠΎ Π²Ρ‹Ρ€Π°ΠΆΠ΅Π½ΠΈΠ΅ являСтся матСматичСской Ρ„ΠΎΡ€ΠΌΡƒΠ»ΠΈΡ€ΠΎΠ²ΠΊΠΎΠΉ Π·Π°ΠΊΠΎΠ½Π° Ома для ΠΏΠ΅Ρ€Π΅ΠΌΠ΅Π½Π½ΠΎΠΉ Ρ†Π΅ΠΏΠΈ. ΠžΠ±Ρ‰Π΅Π΅ сопротивлСниС Ρ†Π΅ΠΏΠΈ Z ΠΈ тангСнс сдвига Ρ„Π°Π· ΠΌΠ΅ΠΆΠ΄Ρƒ колСбаниями Ρ‚ΠΎΠΊΠ° ΠΈ напряТСния tgj опрСдСляСтся ΠΏΠΎ Ρ„ΠΎΡ€ΠΌΡƒΠ»Π°ΠΌ:

.

Как Π²ΠΈΠ΄Π½ΠΎ ΠΈΠ· этих Ρ„ΠΎΡ€ΠΌΡƒΠ», ΠΏΠΎΠ»Π½ΠΎΠ΅ сопротивлСниС Ρ†Π΅ΠΏΠΈ ΠΏΠ΅Ρ€Π΅ΠΌΠ΅Π½Π½ΠΎΠ³ΠΎ Ρ‚ΠΎΠΊΠ° зависит Π½Π΅ Ρ‚ΠΎΠ»ΡŒΠΊΠΎ ΠΎΡ‚ Π²Π΅Π»ΠΈΡ‡ΠΈΠ½ Π°ΠΊΡ‚ΠΈΠ²Π½ΠΎΠ³ΠΎ сопротивлСния, индуктивности ΠΈ Смкости, Π½ΠΎ ΠΈ ΠΎΡ‚ частоты ΠΏΠ΅Ρ€Π΅ΠΌΠ΅Π½Π½ΠΎΠ³ΠΎ Ρ‚ΠΎΠΊΠ°. ΠŸΡ€ΠΈ частотС Π±Π»ΠΈΠ·ΠΊΠΎΠΉ ΠΊ Π½ΡƒΠ»ΡŽ ΠΏΠΎΠ»Π½ΠΎΠ΅ сопротивлСниС Ρ†Π΅ΠΏΠΈ опрСдСляСтся Смкостным сопротивлСниСм ΠΈ стрСмится ΠΊ бСсконСчности, Π° сдвиг Ρ„Π°Π· . ΠŸΡ€ΠΈ высокой частотС ΠΏΠ΅Ρ€Π΅ΠΌΠ΅Π½Π½ΠΎΠ³ΠΎ Ρ‚ΠΎΠΊΠ° соотвСтствСнно ΠΈ .

Π˜Π½Ρ‚Π΅Ρ€Π΅ΡΠ½Π°Ρ ситуация Π½Π°Π±Π»ΡŽΠ΄Π°Π΅Ρ‚ΡΡ, ΠΊΠΎΠ³Π΄Π° частота ΠΏΠ΅Ρ€Π΅ΠΌΠ΅Π½Π½ΠΎΠ³ΠΎ Ρ‚ΠΎΠΊΠ° удовлСтворяСт ΡƒΡΠ»ΠΎΠ²ΠΈΡŽ:

.

Π’ этом случаС рСактивная ΡΠΎΡΡ‚Π°Π²Π»ΡΡŽΡ‰Π°Ρ ΠΏΠΎΠ»Π½ΠΎΠ³ΠΎ сопротивлСния Ρ€Π°Π²Π½Π° Π½ΡƒΠ»ΡŽ ΠΈ, соотвСтствСнно, ΠΏΠΎΠ»Π½ΠΎΠ΅ сопротивлСниС минимально ΠΈ Ρ€Π°Π²Π½ΠΎ Π°ΠΊΡ‚ΠΈΠ²Π½ΠΎΠΌΡƒ ΡΠΎΠΏΡ€ΠΎΡ‚ΠΈΠ²Π»Π΅Π½ΠΈΡŽ , Π° сдвиг Ρ„Π°Π· – Π½ΡƒΠ»ΡŽ. Π’ΠΎΠΊ Π² этом случаС ΠΏΡ€ΠΈΠΎΠ±Ρ€Π΅Ρ‚Π°Π΅Ρ‚ максимальноС Π·Π½Π°Ρ‡Π΅Π½ΠΈΠ΅ . Π’Π°ΠΊΠΎΠ΅ состояниС Ρ†Π΅ΠΏΠΈ ΠΏΠ΅Ρ€Π΅ΠΌΠ΅Π½Π½ΠΎΠ³ΠΎ Ρ‚ΠΎΠΊΠ° называСтся рСзонансом напряТСний, Π° частота – рСзонансной частотой wΡ€Π΅Π·.

Π˜Π½Ρ‚Π΅Ρ€Π΅ΡΠ΅Π½ Ρ‚Π°ΠΊΠΆΠ΅ Ρ„Π°ΠΊΡ‚ ΠΏΡ€Π΅Π²Ρ‹ΡˆΠ΅Π½ΠΈΡ напряТСния Π½Π° Ρ€Π΅Π°ΠΊΡ‚ΠΈΠ²Π½Ρ‹Ρ… элСмСнтах схСмы Π²Ρ‹Ρ…ΠΎΠ΄Π½ΠΎΠ³ΠΎ напряТСния источника Ρ‚ΠΎΠΊΠ°. Если Π² ΠΌΠΎΠΌΠ΅Π½Ρ‚ рСзонанса ΠΈΠ½Π΄ΡƒΠΊΡ‚ΠΈΠ²Π½ΠΎΠ΅ ΠΈ СмкостноС сопротивлСния большС Π°ΠΊΡ‚ΠΈΠ²Π½ΠΎΠ³ΠΎ сопротивлСния Ρ†Π΅ΠΏΠΈ , Ρ‚ΠΎ напряТСния Π½Π° Π½ΠΈΡ… .

ΠŸΡ€Π°ΠΊΡ‚ΠΈΡ‡Π΅ΡΠΊΠΈΠ΅ задания

ПослСднСС ΠΈΠ·ΠΌΠ΅Π½Π΅Π½ΠΈΠ΅ этой страницы: 2016-09-20; ΠΠ°Ρ€ΡƒΡˆΠ΅Π½ΠΈΠ΅ авторского ΠΏΡ€Π°Π²Π° страницы

Π’ ΡΡ‚Π°Ρ‚ΡŒΠ΅ расскаТСм Ρ‡Ρ‚ΠΎ Ρ‚Π°ΠΊΠΎΠ΅ ΠΊΠΎΠ»Π΅Π±Π°Ρ‚Π΅Π»ΡŒΠ½Ρ‹ΠΉ ΠΊΠΎΠ½Ρ‚ΡƒΡ€. ΠŸΠΎΡΠ»Π΅Π΄ΠΎΠ²Π°Ρ‚Π΅Π»ΡŒΠ½Ρ‹ΠΉ ΠΈ ΠΏΠ°Ρ€Π°Π»Π»Π΅Π»ΡŒΠ½Ρ‹ΠΉ ΠΊΠΎΠ»Π΅Π±Π°Ρ‚Π΅Π»ΡŒΠ½Ρ‹ΠΉ ΠΊΠΎΠ½Ρ‚ΡƒΡ€.

ΠšΠΎΠ»Π΅Π±Π°Ρ‚Π΅Π»ΡŒΠ½Ρ‹ΠΉ ΠΊΠΎΠ½Ρ‚ΡƒΡ€ β€” устройство ΠΈΠ»ΠΈ элСктричСская Ρ†Π΅ΠΏΡŒ, содСрТащСС Π½Π΅ΠΎΠ±Ρ…ΠΎΠ΄ΠΈΠΌΡ‹Π΅ радиоэлСктронныС элСмСнты для создания элСктромагнитных ΠΊΠΎΠ»Π΅Π±Π°Π½ΠΈΠΉ. РаздСляСтся Π½Π° Π΄Π²Π° Ρ‚ΠΈΠΏΠ° Π² зависимости ΠΎΡ‚ соСдинСния элСмСнтов: ΠΏΠΎΡΠ»Π΅Π΄ΠΎΠ²Π°Ρ‚Π΅Π»ΡŒΠ½Ρ‹ΠΉ ΠΈ ΠΏΠ°Ρ€Π°Π»Π»Π΅Π»ΡŒΠ½Ρ‹ΠΉ.

Основная радиоэлСмСнтная Π±Π°Π·Π° ΠΊΠΎΠ»Π΅Π±Π°Ρ‚Π΅Π»ΡŒΠ½ΠΎΠ³ΠΎ ΠΊΠΎΠ½Ρ‚ΡƒΡ€Π°: ΠšΠΎΠ½Π΄Π΅Π½ΡΠ°Ρ‚ΠΎΡ€, источник питания ΠΈ ΠΊΠ°Ρ‚ΡƒΡˆΠΊΠ° индуктивности.

ΠŸΠΎΡΠ»Π΅Π΄ΠΎΠ²Π°Ρ‚Π΅Π»ΡŒΠ½Ρ‹ΠΉ ΠΊΠΎΠ»Π΅Π±Π°Ρ‚Π΅Π»ΡŒΠ½Ρ‹ΠΉ ΠΊΠΎΠ½Ρ‚ΡƒΡ€

ΠŸΠΎΡΠ»Π΅Π΄ΠΎΠ²Π°Ρ‚Π΅Π»ΡŒΠ½Ρ‹ΠΉ ΠΊΠΎΠ»Π΅Π±Π°Ρ‚Π΅Π»ΡŒΠ½Ρ‹ΠΉ ΠΊΠΎΠ½Ρ‚ΡƒΡ€ являСтся ΠΏΡ€ΠΎΡΡ‚Π΅ΠΉΡˆΠ΅ΠΉ рСзонансной (ΠΊΠΎΠ»Π΅Π±Π°Ρ‚Π΅Π»ΡŒΠ½ΠΎΠΉ) Ρ†Π΅ΠΏΡŒΡŽ. Бостоит ΠΏΠΎΡΠ»Π΅Π΄ΠΎΠ²Π°Ρ‚Π΅Π»ΡŒΠ½Ρ‹ΠΉ ΠΊΠΎΠ»Π΅Π±Π°Ρ‚Π΅Π»ΡŒΠ½Ρ‹ΠΉ ΠΊΠΎΠ½Ρ‚ΡƒΡ€, ΠΈΠ· ΠΏΠΎΡΠ»Π΅Π΄ΠΎΠ²Π°Ρ‚Π΅Π»ΡŒΠ½ΠΎ Π²ΠΊΠ»ΡŽΡ‡Π΅Π½Π½Ρ‹Ρ… ΠΊΠ°Ρ‚ΡƒΡˆΠΊΠΈ индуктивности ΠΈ кондСнсатора. ΠŸΡ€ΠΈ воздСйствии Π½Π° Ρ‚Π°ΠΊΡƒΡŽ Ρ†Π΅ΠΏΡŒ ΠΏΠ΅Ρ€Π΅ΠΌΠ΅Π½Π½ΠΎΠ³ΠΎ (гармоничСского) напряТСния, Ρ‡Π΅Ρ€Π΅Π· ΠΊΠ°Ρ‚ΡƒΡˆΠΊΡƒ ΠΈ кондСнсатор Π±ΡƒΠ΄Π΅Ρ‚ ΠΏΡ€ΠΎΡ‚Π΅ΠΊΠ°Ρ‚ΡŒ ΠΏΠ΅Ρ€Π΅ΠΌΠ΅Π½Π½Ρ‹ΠΉ Ρ‚ΠΎΠΊ, Π²Π΅Π»ΠΈΡ‡ΠΈΠ½Π° ΠΊΠΎΡ‚ΠΎΡ€ΠΎΠ³ΠΎ вычисляСтся ΠΏΠΎ Π·Π°ΠΊΠΎΠ½Ρƒ Ома: I = U / Π₯Ξ£ , Π³Π΄Π΅ Π₯Ξ£ β€” сумма Ρ€Π΅Π°ΠΊΡ‚ΠΈΠ²Π½Ρ‹Ρ… сопротивлСний ΠΏΠΎΡΠ»Π΅Π΄ΠΎΠ²Π°Ρ‚Π΅Π»ΡŒΠ½ΠΎ Π²ΠΊΠ»ΡŽΡ‡Π΅Π½Π½Ρ‹Ρ… ΠΊΠ°Ρ‚ΡƒΡˆΠΊΠΈ ΠΈ кондСнсатора (ΠΈΡΠΏΠΎΠ»ΡŒΠ·ΡƒΠ΅Ρ‚ΡΡ ΠΌΠΎΠ΄ΡƒΠ»ΡŒ суммы).

Для освСТСния памяти, вспомним ΠΊΠ°ΠΊ зависят Ρ€Π΅Π°ΠΊΡ‚ΠΈΠ²Π½Ρ‹Π΅ сопротивлСния кондСнсатора ΠΈ ΠΊΠ°Ρ‚ΡƒΡˆΠΊΠΈ индуктивности ΠΎΡ‚ частоты ΠΏΡ€ΠΈΠ»ΠΎΠΆΠ΅Π½Π½ΠΎΠ³ΠΎ ΠΏΠ΅Ρ€Π΅ΠΌΠ΅Π½Π½ΠΎΠ³ΠΎ напряТСния. Для ΠΊΠ°Ρ‚ΡƒΡˆΠΊΠΈ индуктивности, эта Π·Π°Π²ΠΈΡΠΈΠΌΠΎΡΡ‚ΡŒ Π±ΡƒΠ΄Π΅Ρ‚ ΠΈΠΌΠ΅Ρ‚ΡŒ Π²ΠΈΠ΄:

Из Ρ„ΠΎΡ€ΠΌΡƒΠ»Ρ‹ Π²ΠΈΠ΄Π½ΠΎ, Ρ‡Ρ‚ΠΎ ΠΏΡ€ΠΈ ΡƒΠ²Π΅Π»ΠΈΡ‡Π΅Π½ΠΈΠΈ частоты, Ρ€Π΅Π°ΠΊΡ‚ΠΈΠ²Π½ΠΎΠ΅ сопротивлСниС ΠΊΠ°Ρ‚ΡƒΡˆΠΊΠΈ индуктивности увСличиваСтся. Для кондСнсатора Π·Π°Π²ΠΈΡΠΈΠΌΠΎΡΡ‚ΡŒ Π΅Π³ΠΎ Ρ€Π΅Π°ΠΊΡ‚ΠΈΠ²Π½ΠΎΠ³ΠΎ сопротивлСния ΠΎΡ‚ частоты Π±ΡƒΠ΄Π΅Ρ‚ Π²Ρ‹Π³Π»ΡΠ΄Π΅Ρ‚ΡŒ ΡΠ»Π΅Π΄ΡƒΡŽΡ‰ΠΈΠΌ ΠΎΠ±Ρ€Π°Π·ΠΎΠΌ:

Π’ ΠΎΡ‚Π»ΠΈΡ‡ΠΈΠΈ ΠΎΡ‚ индуктивности, Ρƒ кондСнсатора всё происходит Π½Π°ΠΎΠ±ΠΎΡ€ΠΎΡ‚ β€” ΠΏΡ€ΠΈ ΡƒΠ²Π΅Π»ΠΈΡ‡Π΅Π½ΠΈΠΈ частоты, Ρ€Π΅Π°ΠΊΡ‚ΠΈΠ²Π½ΠΎΠ΅ сопротивлСниС ΡƒΠΌΠ΅Π½ΡŒΡˆΠ°Π΅Ρ‚ΡΡ. На ΡΠ»Π΅Π΄ΡƒΡŽΡ‰Π΅ΠΌ рисункС графичСски прСдставлСны зависимости Ρ€Π΅Π°ΠΊΡ‚ΠΈΠ²Π½Ρ‹Ρ… сопротивлСний ΠΊΠ°Ρ‚ΡƒΡˆΠΊΠΈ XL ΠΈ кондСнсатора Π₯C ΠΎΡ‚ цикличСской (ΠΊΡ€ΡƒΠ³ΠΎΠ²ΠΎΠΉ) частоты Ο‰, Π° Ρ‚Π°ΠΊΠΆΠ΅ Π³Ρ€Π°Ρ„ΠΈΠΊ зависимости ΠΎΡ‚ частоты Ο‰ ΠΈΡ… алгСбраичСской суммы Π₯Ξ£. Π“Ρ€Π°Ρ„ΠΈΠΊ, ΠΏΠΎ сути, ΠΏΠΎΠΊΠ°Π·Ρ‹Π²Π°Π΅Ρ‚ Π·Π°Π²ΠΈΡΠΈΠΌΠΎΡΡ‚ΡŒ ΠΎΡ‚ частоты ΠΎΠ±Ρ‰Π΅Π³ΠΎ Ρ€Π΅Π°ΠΊΡ‚ΠΈΠ²Π½ΠΎΠ³ΠΎ сопротивлСния ΠΏΠΎΡΠ»Π΅Π΄ΠΎΠ²Π°Ρ‚Π΅Π»ΡŒΠ½ΠΎΠ³ΠΎ ΠΊΠΎΠ»Π΅Π±Π°Ρ‚Π΅Π»ΡŒΠ½ΠΎΠ³ΠΎ ΠΊΠΎΠ½Ρ‚ΡƒΡ€Π°.

Из Π³Ρ€Π°Ρ„ΠΈΠΊΠ° Π²ΠΈΠ΄Π½ΠΎ, Ρ‡Ρ‚ΠΎ Π½Π° Π½Π΅ΠΊΠΎΡ‚ΠΎΡ€ΠΎΠΉ частотС Ο‰=Ο‰Ρ€ , Π½Π° ΠΊΠΎΡ‚ΠΎΡ€ΠΎΠΉ Ρ€Π΅Π°ΠΊΡ‚ΠΈΠ²Π½Ρ‹Π΅ сопротивлСния ΠΊΠ°Ρ‚ΡƒΡˆΠΊΠΈ ΠΈ кондСнсатора Ρ€Π°Π²Π½Ρ‹ ΠΏΠΎ ΠΌΠΎΠ΄ΡƒΠ»ΡŽ (Ρ€Π°Π²Π½Ρ‹ ΠΏΠΎ Π·Π½Π°Ρ‡Π΅Π½ΠΈΡŽ, Π½ΠΎ ΠΏΡ€ΠΎΡ‚ΠΈΠ²ΠΎΠΏΠΎΠ»ΠΎΠΆΠ½Ρ‹ ΠΏΠΎ Π·Π½Π°ΠΊΡƒ), ΠΎΠ±Ρ‰Π΅Π΅ сопротивлСниС Ρ†Π΅ΠΏΠΈ обращаСтся Π² ноль. На этой частотС Π² Ρ†Π΅ΠΏΠΈ Π½Π°Π±Π»ΡŽΠ΄Π°Π΅Ρ‚ΡΡ максимум Ρ‚ΠΎΠΊΠ°, ΠΊΠΎΡ‚ΠΎΡ€Ρ‹ΠΉ ΠΎΠ³Ρ€Π°Π½ΠΈΡ‡Π΅Π½ Ρ‚ΠΎΠ»ΡŒΠΊΠΎ омичСскими потСрями Π² ΠΊΠ°Ρ‚ΡƒΡˆΠΊΠ΅ индуктивности (Ρ‚.Π΅. Π°ΠΊΡ‚ΠΈΠ²Π½Ρ‹ΠΌ сопротивлСниСм ΠΏΡ€ΠΎΠ²ΠΎΠ΄Π° ΠΎΠ±ΠΌΠΎΡ‚ΠΊΠΈ ΠΊΠ°Ρ‚ΡƒΡˆΠΊΠΈ) ΠΈ Π²Π½ΡƒΡ‚Ρ€Π΅Π½Π½ΠΈΠΌ сопротивлСниСм источника Ρ‚ΠΎΠΊΠ° (Π³Π΅Π½Π΅Ρ€Π°Ρ‚ΠΎΡ€Π°). Π’Π°ΠΊΡƒΡŽ частоту, ΠΏΡ€ΠΈ ΠΊΠΎΡ‚ΠΎΡ€ΠΎΠΉ Π½Π°Π±Π»ΡŽΠ΄Π°Π΅Ρ‚ΡΡ рассмотрСнноС явлСниС, Π½Π°Π·Ρ‹Π²Π°Π΅ΠΌΠΎΠ΅ Π² Ρ„ΠΈΠ·ΠΈΠΊΠ΅ рСзонансом, Π½Π°Π·Ρ‹Π²Π°ΡŽΡ‚ рСзонансной частотой ΠΈΠ»ΠΈ собствСнной частотой ΠΊΠΎΠ»Π΅Π±Π°Π½ΠΈΠΉ Ρ†Π΅ΠΏΠΈ. Π’Π°ΠΊΠΆΠ΅ ΠΈΠ· Π³Ρ€Π°Ρ„ΠΈΠΊΠ° Π²ΠΈΠ΄Π½ΠΎ, Ρ‡Ρ‚ΠΎ Π½Π° частотах, Π½ΠΈΠΆΠ΅ частоты рСзонанса Ρ€Π΅Π°ΠΊΡ‚ΠΈΠ²Π½ΠΎΠ΅ сопротивлСниС ΠΏΠΎΡΠ»Π΅Π΄ΠΎΠ²Π°Ρ‚Π΅Π»ΡŒΠ½ΠΎΠ³ΠΎ ΠΊΠΎΠ»Π΅Π±Π°Ρ‚Π΅Π»ΡŒΠ½ΠΎΠ³ΠΎ ΠΊΠΎΠ½Ρ‚ΡƒΡ€Π° носит Смкостной Ρ…Π°Ρ€Π°ΠΊΡ‚Π΅Ρ€, Π° Π½Π° Π±ΠΎΠ»Π΅Π΅ высоких частотах β€” ΠΈΠ½Π΄ΡƒΠΊΡ‚ΠΈΠ²Π½Ρ‹ΠΉ. Π§Ρ‚ΠΎ касаСтся самой рСзонансной частоты, Ρ‚ΠΎ ΠΎΠ½Π° ΠΌΠΎΠΆΠ΅Ρ‚ Π±Ρ‹Ρ‚ΡŒ вычислСна ΠΏΡ€ΠΈ ΠΏΠΎΠΌΠΎΡ‰ΠΈ Ρ„ΠΎΡ€ΠΌΡƒΠ»Ρ‹ Вомсона, ΠΊΠΎΡ‚ΠΎΡ€ΡƒΡŽ ΠΌΡ‹ ΠΌΠΎΠΆΠ΅ΠΌ вывСсти ΠΈΠ· Ρ„ΠΎΡ€ΠΌΡƒΠ» Ρ€Π΅Π°ΠΊΡ‚ΠΈΠ²Π½Ρ‹Ρ… сопротивлСний ΠΊΠ°Ρ‚ΡƒΡˆΠΊΠΈ индуктивности ΠΈ кондСнсатора, приравняв ΠΈΡ… Ρ€Π΅Π°ΠΊΡ‚ΠΈΠ²Π½Ρ‹Π΅ сопротивлСния Π΄Ρ€ΡƒΠ³ ΠΊ Π΄Ρ€ΡƒΠ³Ρƒ:

На рисункС справа, ΠΈΠ·ΠΎΠ±Ρ€Π°ΠΆΠ΅Π½Π° эквивалСнтная схСма ΠΏΠΎΡΠ»Π΅Π΄ΠΎΠ²Π°Ρ‚Π΅Π»ΡŒΠ½ΠΎΠ³ΠΎ рСзонансного ΠΊΠΎΠ½Ρ‚ΡƒΡ€Π° с ΡƒΡ‡Π΅Ρ‚ΠΎΠΌ омичСских ΠΏΠΎΡ‚Π΅Ρ€ΡŒ R, ΠΏΠΎΠ΄ΠΊΠ»ΡŽΡ‡Π΅Π½Π½ΠΎΠ³ΠΎ ΠΊ ΠΈΠ΄Π΅Π°Π»ΡŒΠ½ΠΎΠΌΡƒ Π³Π΅Π½Π΅Ρ€Π°Ρ‚ΠΎΡ€Ρƒ гармоничСского напряТСния с Π°ΠΌΠΏΠ»ΠΈΡ‚ΡƒΠ΄ΠΎΠΉ U. ПолноС сопротивлСниС (импСданс) Ρ‚Π°ΠΊΠΎΠΉ Ρ†Π΅ΠΏΠΈ опрСдСляСтся: Z = √(R 2 +XΞ£ 2 ), Π³Π΄Π΅ XΞ£ = Ο‰ L-1/Ο‰C. На рСзонансной частотС, ΠΊΠΎΠ³Π΄Π° Π²Π΅Π»ΠΈΡ‡ΠΈΠ½Ρ‹ Ρ€Π΅Π°ΠΊΡ‚ΠΈΠ²Π½Ρ‹Ρ… сопротивлСний ΠΊΠ°Ρ‚ΡƒΡˆΠΊΠΈ XL = Ο‰L ΠΈ кондСнсатора Π₯Π‘= 1/Ο‰Π‘ Ρ€Π°Π²Π½Ρ‹ ΠΏΠΎ ΠΌΠΎΠ΄ΡƒΠ»ΡŽ, Π²Π΅Π»ΠΈΡ‡ΠΈΠ½Π° XΞ£ обращаСтся Π² Π½ΡƒΠ»ΡŒ (ΡΠ»Π΅Π΄ΠΎΠ²Π°Ρ‚Π΅Π»ΡŒΠ½ΠΎ, сопротивлСниС Ρ†Π΅ΠΏΠΈ чисто Π°ΠΊΡ‚ΠΈΠ²Π½ΠΎΠ΅), Π° Ρ‚ΠΎΠΊ Π² Ρ†Π΅ΠΏΠΈ опрСдСлятся ΠΎΡ‚Π½ΠΎΡˆΠ΅Π½ΠΈΠ΅ΠΌ Π°ΠΌΠΏΠ»ΠΈΡ‚ΡƒΠ΄Ρ‹ напряТСния Π³Π΅Π½Π΅Ρ€Π°Ρ‚ΠΎΡ€Π° ΠΊ ΡΠΎΠΏΡ€ΠΎΡ‚ΠΈΠ²Π»Π΅Π½ΠΈΡŽ омичСских ΠΏΠΎΡ‚Π΅Ρ€ΡŒ: I= U/R. ΠŸΡ€ΠΈ этом Π½Π° ΠΊΠ°Ρ‚ΡƒΡˆΠΊΠ΅ ΠΈ Π½Π° кондСнсаторС, Π² ΠΊΠΎΡ‚ΠΎΡ€Ρ‹Ρ… запасСна рСактивная элСктричСская энСргия, ΠΏΠ°Π΄Π°Π΅Ρ‚ ΠΎΠ΄ΠΈΠ½Π°ΠΊΠΎΠ²ΠΎΠ΅ напряТСниС UL = UΠ‘ = IXL = IXΠ‘.

На любой Π΄Ρ€ΡƒΠ³ΠΎΠΉ частотС, ΠΎΡ‚Π»ΠΈΡ‡Π½ΠΎΠΉ ΠΎΡ‚ рСзонансной, напряТСния Π½Π° ΠΊΠ°Ρ‚ΡƒΡˆΠΊΠ΅ ΠΈ кондСнсаторС Π½Π΅ΠΎΠ΄ΠΈΠ½Π°ΠΊΠΎΠ²Ρ‹ β€” ΠΎΠ½ΠΈ ΠΎΠΏΡ€Π΅Π΄Π΅Π»ΡΡŽΡ‚ΡΡ Π°ΠΌΠΏΠ»ΠΈΡ‚ΡƒΠ΄ΠΎΠΉ Ρ‚ΠΎΠΊΠ° Π² Ρ†Π΅ΠΏΠΈ ΠΈ Π²Π΅Π»ΠΈΡ‡ΠΈΠ½Π°ΠΌΠΈ ΠΌΠΎΠ΄ΡƒΠ»Π΅ΠΉ Ρ€Π΅Π°ΠΊΡ‚ΠΈΠ²Π½Ρ‹Ρ… сопротивлСний XL ΠΈ XΠ‘.ΠŸΠΎΡΡ‚ΠΎΠΌΡƒ рСзонанс Π² ΠΏΠΎΡΠ»Π΅Π΄ΠΎΠ²Π°Ρ‚Π΅Π»ΡŒΠ½ΠΎΠΌ ΠΊΠΎΠ»Π΅Π±Π°Ρ‚Π΅Π»ΡŒΠ½ΠΎΠΌ ΠΊΠΎΠ½Ρ‚ΡƒΡ€Π΅ принято Π½Π°Π·Ρ‹Π²Π°Ρ‚ΡŒ рСзонансом напряТСний. РСзонансной частотой ΠΊΠΎΠ½Ρ‚ΡƒΡ€Π° Π½Π°Π·Ρ‹Π²Π°ΡŽΡ‚ Ρ‚Π°ΠΊΡƒΡŽ частоту, Π½Π° ΠΊΠΎΡ‚ΠΎΡ€ΠΎΠΉ сопротивлСниС ΠΊΠΎΠ½Ρ‚ΡƒΡ€Π° ΠΈΠΌΠ΅Π΅Ρ‚ чисто Π°ΠΊΡ‚ΠΈΠ²Π½Ρ‹ΠΉ (рСзистивный) Ρ…Π°Ρ€Π°ΠΊΡ‚Π΅Ρ€. УсловиС рСзонанса β€” это равСнство Π²Π΅Π»ΠΈΡ‡ΠΈΠ½ Ρ€Π΅Π°ΠΊΡ‚ΠΈΠ²Π½Ρ‹Ρ… сопротивлСний ΠΊΠ°Ρ‚ΡƒΡˆΠΊΠΈ индуктивности ΠΈ ёмкости.

Одними ΠΈΠ· Π½Π°ΠΈΠ±ΠΎΠ»Π΅Π΅ Π²Π°ΠΆΠ½Ρ‹Ρ… ΠΏΠ°Ρ€Π°ΠΌΠ΅Ρ‚Ρ€ΠΎΠ² ΠΊΠΎΠ»Π΅Π±Π°Ρ‚Π΅Π»ΡŒΠ½ΠΎΠ³ΠΎ ΠΊΠΎΠ½Ρ‚ΡƒΡ€Π° (ΠΊΡ€ΠΎΠΌΠ΅, разумССтся, рСзонансной частоты) ΡΠ²Π»ΡΡŽΡ‚ΡΡ Π΅Π³ΠΎ характСристичСскоС (ΠΈΠ»ΠΈ Π²ΠΎΠ»Π½ΠΎΠ²ΠΎΠ΅) сопротивлСниС ρ ΠΈ Π΄ΠΎΠ±Ρ€ΠΎΡ‚Π½ΠΎΡΡ‚ΡŒ ΠΊΠΎΠ½Ρ‚ΡƒΡ€Π° Q. Π₯арактСристичСским (Π²ΠΎΠ»Π½ΠΎΠ²Ρ‹ΠΌ) сопротивлСниСм ΠΊΠΎΠ½Ρ‚ΡƒΡ€Π° ρ называСтся Π²Π΅Π»ΠΈΡ‡ΠΈΠ½Π° Ρ€Π΅Π°ΠΊΡ‚ΠΈΠ²Π½ΠΎΠ³ΠΎ сопротивлСния Смкости ΠΈ индуктивности ΠΊΠΎΠ½Ρ‚ΡƒΡ€Π° Π½Π° рСзонансной частотС: ρ = Π₯L = Π₯C ΠΏΡ€ΠΈ Ο‰ =Ο‰Ρ€ . Π₯арактСристичСскоС сопротивлСниС ΠΌΠΎΠΆΠ΅Ρ‚ Π±Ρ‹Ρ‚ΡŒ вычислСно ΡΠ»Π΅Π΄ΡƒΡŽΡ‰ΠΈΠΌ ΠΎΠ±Ρ€Π°Π·ΠΎΠΌ: ρ = √(L/C). Π₯арактСристичСскоС сопротивлСниС ρ являСтся количСствСнной ΠΌΠ΅Ρ€ΠΎΠΉ ΠΎΡ†Π΅Π½ΠΊΠΈ энСргии, запасСнной Ρ€Π΅Π°ΠΊΡ‚ΠΈΠ²Π½Ρ‹ΠΌΠΈ элСмСнтами ΠΊΠΎΠ½Ρ‚ΡƒΡ€Π° β€” ΠΊΠ°Ρ‚ΡƒΡˆΠΊΠΎΠΉ (энСргия ΠΌΠ°Π³Π½ΠΈΡ‚Π½ΠΎΠ³ΠΎ поля) WL = (LI 2 )/2 ΠΈ кондСнсатором (энСргия элСктричСского поля) WC=(CU 2 )/2. ΠžΡ‚Π½ΠΎΡˆΠ΅Π½ΠΈΠ΅ энСргии, запасСнной Ρ€Π΅Π°ΠΊΡ‚ΠΈΠ²Π½Ρ‹ΠΌΠΈ элСмСнтами ΠΊΠΎΠ½Ρ‚ΡƒΡ€Π°, ΠΊ энСргии омичСских (рСзистивных) ΠΏΠΎΡ‚Π΅Ρ€ΡŒ Π·Π° ΠΏΠ΅Ρ€ΠΈΠΎΠ΄ принято Π½Π°Π·Ρ‹Π²Π°Ρ‚ΡŒ Π΄ΠΎΠ±Ρ€ΠΎΡ‚Π½ΠΎΡΡ‚ΡŒΡŽ Q ΠΊΠΎΠ½Ρ‚ΡƒΡ€Π°, Ρ‡Ρ‚ΠΎ Π² Π±ΡƒΠΊΠ²Π°Π»ΡŒΠ½ΠΎΠΌ ΠΏΠ΅Ρ€Π΅Π²ΠΎΠ΄Π΅ с английского языка ΠΎΠ±ΠΎΠ·Π½Π°Ρ‡Π°Π΅Ρ‚ «качСство».

Π”ΠΎΠ±Ρ€ΠΎΡ‚Π½ΠΎΡΡ‚ΡŒ ΠΊΠΎΠ»Π΅Π±Π°Ρ‚Π΅Π»ΡŒΠ½ΠΎΠ³ΠΎ ΠΊΠΎΠ½Ρ‚ΡƒΡ€Π° β€” характСристика, ΠΎΠΏΡ€Π΅Π΄Π΅Π»ΡΡŽΡ‰Π°Ρ Π°ΠΌΠΏΠ»ΠΈΡ‚ΡƒΠ΄Ρƒ ΠΈ ΡˆΠΈΡ€ΠΈΠ½Ρƒ АЧΠ₯ рСзонанса ΠΈ ΠΏΠΎΠΊΠ°Π·Ρ‹Π²Π°ΡŽΡ‰Π°Ρ, Π²ΠΎ сколько Ρ€Π°Π· запасы энСргии Π² ΠΊΠΎΠ½Ρ‚ΡƒΡ€Π΅ большС, Ρ‡Π΅ΠΌ ΠΏΠΎΡ‚Π΅Ρ€ΠΈ энСргии Π·Π° ΠΎΠ΄ΠΈΠ½ ΠΏΠ΅Ρ€ΠΈΠΎΠ΄ ΠΊΠΎΠ»Π΅Π±Π°Π½ΠΈΠΉ. Π”ΠΎΠ±Ρ€ΠΎΡ‚Π½ΠΎΡΡ‚ΡŒ ΡƒΡ‡ΠΈΡ‚Ρ‹Π²Π°Π΅Ρ‚ Π½Π°Π»ΠΈΡ‡ΠΈΠ΅ Π°ΠΊΡ‚ΠΈΠ²Π½ΠΎΠ³ΠΎ сопротивлСния Π½Π°Π³Ρ€ΡƒΠ·ΠΊΠΈ R.

Для ΠΏΠΎΡΠ»Π΅Π΄ΠΎΠ²Π°Ρ‚Π΅Π»ΡŒΠ½ΠΎΠ³ΠΎ ΠΊΠΎΠ»Π΅Π±Π°Ρ‚Π΅Π»ΡŒΠ½ΠΎΠ³ΠΎ ΠΊΠΎΠ½Ρ‚ΡƒΡ€Π° Π² RLC цСпях, Π² ΠΊΠΎΡ‚ΠΎΡ€ΠΎΠΌ всС Ρ‚Ρ€ΠΈ элСмСнта Π²ΠΊΠ»ΡŽΡ‡Π΅Π½Ρ‹ ΠΏΠΎΡΠ»Π΅Π΄ΠΎΠ²Π°Ρ‚Π΅Π»ΡŒΠ½ΠΎ, Π΄ΠΎΠ±Ρ€ΠΎΡ‚Π½ΠΎΡΡ‚ΡŒ вычисляСтся:

Π³Π΄Π΅ R, L ΠΈ C β€” сопротивлСниС, ΠΈΠ½Π΄ΡƒΠΊΡ‚ΠΈΠ²Π½ΠΎΡΡ‚ΡŒ ΠΈ Ρ‘ΠΌΠΊΠΎΡΡ‚ΡŒ рСзонансной Ρ†Π΅ΠΏΠΈ, соотвСтствСнно.

Π’Π΅Π»ΠΈΡ‡ΠΈΠ½Ρƒ, ΠΎΠ±Ρ€Π°Ρ‚Π½ΡƒΡŽ добротности d = 1 / Q Π½Π°Π·Ρ‹Π²Π°ΡŽΡ‚ Π·Π°Ρ‚ΡƒΡ…Π°Π½ΠΈΠ΅ΠΌ ΠΊΠΎΠ½Ρ‚ΡƒΡ€Π°. Для опрСдСлСния добротности ΠΎΠ±Ρ‹Ρ‡Π½ΠΎ ΠΏΠΎΠ»ΡŒΠ·ΡƒΡŽΡ‚ΡΡ Ρ„ΠΎΡ€ΠΌΡƒΠ»ΠΎΠΉ Q = ρ / R, Π³Π΄Π΅ R-сопротивлСниС омичСских ΠΏΠΎΡ‚Π΅Ρ€ΡŒ ΠΊΠΎΠ½Ρ‚ΡƒΡ€Π°, Ρ…Π°Ρ€Π°ΠΊΡ‚Π΅Ρ€ΠΈΠ·ΡƒΡŽΡ‰Π΅Π΅ ΠΌΠΎΡ‰Π½ΠΎΡΡ‚ΡŒ рСзистивных (Π°ΠΊΡ‚ΠΈΠ²Π½Ρ‹Ρ… ΠΏΠΎΡ‚Π΅Ρ€ΡŒ) ΠΊΠΎΠ½Ρ‚ΡƒΡ€Π° Π  = I 2 R. Π”ΠΎΠ±Ρ€ΠΎΡ‚Π½ΠΎΡΡ‚ΡŒ Ρ€Π΅Π°Π»ΡŒΠ½Ρ‹Ρ… ΠΊΠΎΠ»Π΅Π±Π°Ρ‚Π΅Π»ΡŒΠ½Ρ‹Ρ… ΠΊΠΎΠ½Ρ‚ΡƒΡ€ΠΎΠ², Π²Ρ‹ΠΏΠΎΠ»Π½Π΅Π½Π½Ρ‹Ρ… Π½Π° дискрСтных ΠΊΠ°Ρ‚ΡƒΡˆΠΊΠ°Ρ… индуктивности ΠΈ кондСнсаторах, составляСт ΠΎΡ‚ Π½Π΅ΡΠΊΠΎΠ»ΡŒΠΊΠΈΡ… Π΅Π΄ΠΈΠ½ΠΈΡ† Π΄ΠΎ сотни ΠΈ Π±ΠΎΠ»Π΅Π΅. Π”ΠΎΠ±Ρ€ΠΎΡ‚Π½ΠΎΡΡ‚ΡŒ Ρ€Π°Π·Π»ΠΈΡ‡Π½Ρ‹Ρ… ΠΊΠΎΠ»Π΅Π±Π°Ρ‚Π΅Π»ΡŒΠ½Ρ‹Ρ… систСм, построСнных Π½Π° ΠΏΡ€ΠΈΠ½Ρ†ΠΈΠΏΠ΅ ΠΏΡŒΠ΅Π·ΠΎΡΠ»Π΅ΠΊΡ‚Ρ€ΠΈΡ‡Π΅ΡΠΊΠΈΡ… ΠΈ Π΄Ρ€ΡƒΠ³ΠΈΡ… эффСктов (Π½Π°ΠΏΡ€ΠΈΠΌΠ΅Ρ€, ΠΊΠ²Π°Ρ€Ρ†Π΅Π²Ρ‹Π΅ Ρ€Π΅Π·ΠΎΠ½Π°Ρ‚ΠΎΡ€Ρ‹) ΠΌΠΎΠΆΠ΅Ρ‚ Π΄ΠΎΡΡ‚ΠΈΠ³Π°Ρ‚ΡŒ Π½Π΅ΡΠΊΠΎΠ»ΡŒΠΊΠΈΡ… тысяч ΠΈ Π±ΠΎΠ»Π΅Π΅.

ЧастотныС свойства Ρ€Π°Π·Π»ΠΈΡ‡Π½Ρ‹Ρ… Ρ†Π΅ΠΏΠ΅ΠΉ Π² Ρ‚Π΅Ρ…Π½ΠΈΠΊΠ΅ принято ΠΎΡ†Π΅Π½ΠΈΠ²Π°Ρ‚ΡŒ с ΠΏΠΎΠΌΠΎΡ‰ΡŒΡŽ Π°ΠΌΠΏΠ»ΠΈΡ‚ΡƒΠ΄Π½ΠΎ-частотных характСристик (АЧΠ₯), ΠΏΡ€ΠΈ этом сами Ρ†Π΅ΠΏΠΈ Ρ€Π°ΡΡΠΌΠ°Ρ‚Ρ€ΠΈΠ²Π°ΡŽΡ‚ ΠΊΠ°ΠΊ Ρ‡Π΅Ρ‚Ρ‹Ρ€Ρ‘Ρ…ΠΏΠΎΠ»ΡŽΡΠ½ΠΈΠΊΠΈ. На рисунках Π½ΠΈΠΆΠ΅ прСдставлСны Π΄Π²Π° ΠΏΡ€ΠΎΡΡ‚Π΅ΠΉΡˆΠΈΡ… Ρ‡Π΅Ρ‚Ρ‹Ρ€Π΅Ρ…ΠΏΠΎΠ»ΡŽΡΠ½ΠΈΠΊΠ°, содСрТащих ΠΏΠΎΡΠ»Π΅Π΄ΠΎΠ²Π°Ρ‚Π΅Π»ΡŒΠ½Ρ‹ΠΉ ΠΊΠΎΠ»Π΅Π±Π°Ρ‚Π΅Π»ΡŒΠ½Ρ‹ΠΉ ΠΊΠΎΠ½Ρ‚ΡƒΡ€ ΠΈ АЧΠ₯ этих Ρ†Π΅ΠΏΠ΅ΠΉ, ΠΊΠΎΡ‚ΠΎΡ€Ρ‹Π΅ ΠΏΡ€ΠΈΠ²Π΅Π΄Π΅Π½Ρ‹ (ΠΏΠΎΠΊΠ°Π·Π°Π½Ρ‹ ΡΠΏΠ»ΠΎΡˆΠ½Ρ‹ΠΌΠΈ линями). По Π²Π΅Ρ€Ρ‚ΠΈΠΊΠ°Π»ΡŒΠ½ΠΎΠΉ оси Π³Ρ€Π°Ρ„ΠΈΠΊΠΎΠ² АЧΠ₯ ΠΎΡ‚Π»ΠΎΠΆΠ΅Π½Π° Π²Π΅Π»ΠΈΡ‡ΠΈΠ½Π° коэффициСнта ΠΏΠ΅Ρ€Π΅Π΄Π°Ρ‡ΠΈ Ρ†Π΅ΠΏΠΈ ΠΏΠΎ Π½Π°ΠΏΡ€ΡΠΆΠ΅Π½ΠΈΡŽ К, ΠΏΠΎΠΊΠ°Π·Ρ‹Π²Π°ΡŽΡ‰Π°Ρ ΠΎΡ‚Π½ΠΎΡˆΠ΅Π½ΠΈΠ΅ Π²Ρ‹Ρ…ΠΎΠ΄Π½ΠΎΠ³ΠΎ напряТСния Ρ†Π΅ΠΏΠΈ ΠΊ Π²Ρ…ΠΎΠ΄Π½ΠΎΠΌΡƒ.

Для пассивных Ρ†Π΅ΠΏΠ΅ΠΉ (Ρ‚.Π΅. Π½Π΅ содСрТащих ΡƒΡΠΈΠ»ΠΈΡ‚Π΅Π»ΡŒΠ½Ρ‹Ρ… элСмСнтов ΠΈ источников энСргии), Π²Π΅Π»ΠΈΡ‡ΠΈΠ½Π° К Π½ΠΈΠΊΠΎΠ³Π΄Π° Π½Π΅ ΠΏΡ€Π΅Π²Ρ‹ΡˆΠ°Π΅Ρ‚ Π΅Π΄ΠΈΠ½ΠΈΡ†Ρƒ. Π‘ΠΎΠΏΡ€ΠΎΡ‚ΠΈΠ²Π»Π΅Π½ΠΈΠ΅ ΠΏΠ΅Ρ€Π΅ΠΌΠ΅Π½Π½ΠΎΠΌΡƒ Ρ‚ΠΎΠΊΡƒ ΠΈΠ·ΠΎΠ±Ρ€Π°ΠΆΡ‘Π½Π½ΠΎΠΉ Π½Π° рисункС Ρ†Π΅ΠΏΠΈ, Π±ΡƒΠ΄Π΅Ρ‚ минимально ΠΏΡ€ΠΈ частотС воздСйствия, Ρ€Π°Π²Π½ΠΎΠΉ рСзонансной частотС ΠΊΠΎΠ½Ρ‚ΡƒΡ€Π°. Π’ этом случаС коэффициСнт ΠΏΠ΅Ρ€Π΅Π΄Π°Ρ‡ΠΈ Ρ†Π΅ΠΏΠΈ Π±Π»ΠΈΠ·ΠΎΠΊ ΠΊ Π΅Π΄ΠΈΠ½ΠΈΡ†Π΅ (опрСдСляСтся омичСскими потСрями Π² ΠΊΠΎΠ½Ρ‚ΡƒΡ€Π΅). На частотах, сильно ΠΎΡ‚Π»ΠΈΡ‡Π°ΡŽΡ‰ΠΈΡ…ΡΡ ΠΎΡ‚ рСзонансной, сопротивлСниС ΠΊΠΎΠ½Ρ‚ΡƒΡ€Π° ΠΏΠ΅Ρ€Π΅ΠΌΠ΅Π½Π½ΠΎΠΌΡƒ Ρ‚ΠΎΠΊΡƒ достаточно Π²Π΅Π»ΠΈΠΊΠΎ, Π° ΡΠ»Π΅Π΄ΠΎΠ²Π°Ρ‚Π΅Π»ΡŒΠ½ΠΎ, ΠΈ коэффициСнт ΠΏΠ΅Ρ€Π΅Π΄Π°Ρ‡ΠΈ Ρ†Π΅ΠΏΠΈ Π±ΡƒΠ΄Π΅Ρ‚ ΠΏΠ°Π΄Π°Ρ‚ΡŒ практичСски Π΄ΠΎ нуля.

ΠŸΡ€ΠΈ рСзонансС Π² этой Ρ†Π΅ΠΏΠΈ, источник Π²Ρ…ΠΎΠ΄Π½ΠΎΠ³ΠΎ сигнала оказываСтся фактичСски Π·Π°ΠΌΠΊΠ½ΡƒΡ‚Ρ‹ΠΌ Π½Π°ΠΊΠΎΡ€ΠΎΡ‚ΠΊΠΎ ΠΌΠ°Π»Ρ‹ΠΌ сопротивлСниСм ΠΊΠΎΠ½Ρ‚ΡƒΡ€Π°, благодаря Ρ‡Π΅ΠΌΡƒ коэффициСнт ΠΏΠ΅Ρ€Π΅Π΄Π°Ρ‡ΠΈ Ρ‚Π°ΠΊΠΎΠΉ Ρ†Π΅ΠΏΠΈ Π½Π° рСзонансной частотС ΠΏΠ°Π΄Π°Π΅Ρ‚ практичСски Π΄ΠΎ нуля (ΠΎΠΏΡΡ‚ΡŒ-Ρ‚Π°ΠΊΠΈ Π² силу наличия ΠΊΠΎΠ½Π΅Ρ‡Π½ΠΎΠ³ΠΎ сопротивлСния ΠΏΠΎΡ‚Π΅Ρ€ΡŒ). Наоборот, ΠΏΡ€ΠΈ частотах Π²Ρ…ΠΎΠ΄Π½ΠΎΠ³ΠΎ воздСйствия, Π·Π½Π°Ρ‡ΠΈΡ‚Π΅Π»ΡŒΠ½ΠΎ отстоящих ΠΎΡ‚ рСзонансной, коэффициСнт ΠΏΠ΅Ρ€Π΅Π΄Π°Ρ‡ΠΈ Ρ†Π΅ΠΏΠΈ оказываСтся Π±Π»ΠΈΠ·ΠΊΠΈΠΌ ΠΊ Π΅Π΄ΠΈΠ½ΠΈΡ†Π΅. Бвойство ΠΊΠΎΠ»Π΅Π±Π°Ρ‚Π΅Π»ΡŒΠ½ΠΎΠ³ΠΎ ΠΊΠΎΠ½Ρ‚ΡƒΡ€Π° Π² Π·Π½Π°Ρ‡ΠΈΡ‚Π΅Π»ΡŒΠ½ΠΎΠΉ стСпСни ΠΈΠ·ΠΌΠ΅Π½ΡΡ‚ΡŒ коэффициСнт ΠΏΠ΅Ρ€Π΅Π΄Π°Ρ‡ΠΈ Π½Π° частотах, Π±Π»ΠΈΠ·ΠΊΠΈΡ… ΠΊ рСзонансной, ΡˆΠΈΡ€ΠΎΠΊΠΎ ΠΈΡΠΏΠΎΠ»ΡŒΠ·ΡƒΠ΅Ρ‚ΡΡ Π½Π° ΠΏΡ€Π°ΠΊΡ‚ΠΈΠΊΠ΅, ΠΊΠΎΠ³Π΄Π° трСбуСтся Π²Ρ‹Π΄Π΅Π»ΠΈΡ‚ΡŒ сигнал с ΠΊΠΎΠ½ΠΊΡ€Π΅Ρ‚Π½ΠΎΠΉ частотой ΠΈΠ· мноТСства Π½Π΅Π½ΡƒΠΆΠ½Ρ‹Ρ… сигналов, располоТСнных Π½Π° Π΄Ρ€ΡƒΠ³ΠΈΡ… частотах. Π’Π°ΠΊ, Π² любом Ρ€Π°Π΄ΠΈΠΎΠΏΡ€ΠΈΠ΅ΠΌΠ½ΠΈΠΊΠ΅ ΠΏΡ€ΠΈ ΠΏΠΎΠΌΠΎΡ‰ΠΈ ΠΊΠΎΠ»Π΅Π±Π°Ρ‚Π΅Π»ΡŒΠ½Ρ‹Ρ… Ρ†Π΅ΠΏΠ΅ΠΉ обСспСчиваСтся настройка Π½Π° частоту Π½ΡƒΠΆΠ½ΠΎΠΉ радиостанции. Бвойство ΠΊΠΎΠ»Π΅Π±Π°Ρ‚Π΅Π»ΡŒΠ½ΠΎΠ³ΠΎ ΠΊΠΎΠ½Ρ‚ΡƒΡ€Π° Π²Ρ‹Π΄Π΅Π»ΡΡ‚ΡŒ ΠΈΠ· мноТСства частот ΠΎΠ΄Π½Ρƒ принято Π½Π°Π·Ρ‹Π²Π°Ρ‚ΡŒ ΡΠ΅Π»Π΅ΠΊΡ‚ΠΈΠ²Π½ΠΎΡΡ‚ΡŒΡŽ ΠΈΠ»ΠΈ ΠΈΠ·Π±ΠΈΡ€Π°Ρ‚Π΅Π»ΡŒΠ½ΠΎΡΡ‚ΡŒΡŽ. ΠŸΡ€ΠΈ этом ΠΈΠ½Ρ‚Π΅Π½ΡΠΈΠ²Π½ΠΎΡΡ‚ΡŒ измСнСния коэффициСнта ΠΏΠ΅Ρ€Π΅Π΄Π°Ρ‡ΠΈ Ρ†Π΅ΠΏΠΈ ΠΏΡ€ΠΈ отстройкС частоты воздСйствия ΠΎΡ‚ рСзонанса принято ΠΎΡ†Π΅Π½ΠΈΠ²Π°Ρ‚ΡŒ ΠΏΡ€ΠΈ ΠΏΠΎΠΌΠΎΡ‰ΠΈ ΠΏΠ°Ρ€Π°ΠΌΠ΅Ρ‚Ρ€Π°, Π½Π°Π·Ρ‹Π²Π°Π΅ΠΌΠΎΠ³ΠΎ полосой пропускания. Π—Π° полосу пропускания принимаСтся Π΄ΠΈΠ°ΠΏΠ°Π·ΠΎΠ½ частот, Π² ΠΏΡ€Π΅Π΄Π΅Π»Π°Ρ… ΠΊΠΎΡ‚ΠΎΡ€ΠΎΠ³ΠΎ ΡƒΠΌΠ΅Π½ΡŒΡˆΠ΅Π½ΠΈΠ΅ (ΠΈΠ»ΠΈ ΡƒΠ²Π΅Π»ΠΈΡ‡Π΅Π½ΠΈΠ΅ β€” Π² зависимости ΠΎΡ‚ Π²ΠΈΠ΄Π° Ρ†Π΅ΠΏΠΈ) коэффициСнта ΠΏΠ΅Ρ€Π΅Π΄Π°Ρ‡ΠΈ ΠΎΡ‚Π½ΠΎΡΠΈΡ‚Π΅Π»ΡŒΠ½ΠΎ Π΅Π³ΠΎ значСния Π½Π° рСзонансной частотС, Π½Π΅ ΠΏΡ€Π΅Π²Ρ‹ΡˆΠ°Π΅Ρ‚ Π²Π΅Π»ΠΈΡ‡ΠΈΠ½Ρ‹ 0,7 (3Π΄Π‘).

ΠŸΡƒΠ½ΠΊΡ‚ΠΈΡ€Π½Ρ‹ΠΌΠΈ линиями Π½Π° Π³Ρ€Π°Ρ„ΠΈΠΊΠ°Ρ… ΠΏΠΎΠΊΠ°Π·Π°Π½Ρ‹ АЧΠ₯ Ρ‚ΠΎΡ‡Π½ΠΎ Ρ‚Π°ΠΊΠΈΡ… ΠΆΠ΅ Ρ†Π΅ΠΏΠ΅ΠΉ, ΠΊΠΎΠ»Π΅Π±Π°Ρ‚Π΅Π»ΡŒΠ½Ρ‹Π΅ ΠΊΠΎΠ½Ρ‚ΡƒΡ€Ρ‹ ΠΊΠΎΡ‚ΠΎΡ€Ρ‹Ρ… ΠΈΠΌΠ΅ΡŽΡ‚ Ρ‚Π°ΠΊΠΈΠ΅ ΠΆΠ΅ рСзонансныС частоты, ΠΊΠ°ΠΊ ΠΈ для случая рассмотрСнного Π²Ρ‹ΡˆΠ΅, Π½ΠΎ ΠΎΠ±Π»Π°Π΄Π°ΡŽΡ‰ΠΈΠ΅ мСньшСй Π΄ΠΎΠ±Ρ€ΠΎΡ‚Π½ΠΎΡΡ‚ΡŒΡŽ (Π½Π°ΠΏΡ€ΠΈΠΌΠ΅Ρ€, ΠΊΠ°Ρ‚ΡƒΡˆΠΊΠ° индуктивности Π½Π°ΠΌΠΎΡ‚Π°Π½Π° ΠΏΡ€ΠΎΠ²ΠΎΠ΄ΠΎΠΌ, ΠΎΠ±Π»Π°Π΄Π°ΡŽΡ‰ΠΈΠΌ большим сопротивлСниСм постоянному Ρ‚ΠΎΠΊΡƒ). Как Π²ΠΈΠ΄Π½ΠΎ ΠΈΠ· рисунков, ΠΏΡ€ΠΈ этом Ρ€Π°ΡΡˆΠΈΡ€ΡΠ΅Ρ‚ΡΡ полоса пропускания Ρ†Π΅ΠΏΠΈ ΠΈ ΡƒΡ…ΡƒΠ΄ΡˆΠ°ΡŽΡ‚ΡΡ Π΅Π΅ сСлСктивныС (ΠΈΠ·Π±ΠΈΡ€Π°Ρ‚Π΅Π»ΡŒΠ½Ρ‹Π΅) свойства. Π˜ΡΡ…ΠΎΠ΄Ρ ΠΈΠ· этого, ΠΏΡ€ΠΈ расчСтС ΠΈ конструировании ΠΊΠΎΠ»Π΅Π±Π°Ρ‚Π΅Π»ΡŒΠ½Ρ‹Ρ… ΠΊΠΎΠ½Ρ‚ΡƒΡ€ΠΎΠ² Π½ΡƒΠΆΠ½ΠΎ ΡΡ‚Ρ€Π΅ΠΌΠΈΡ‚ΡŒΡΡ ΠΊ ΠΏΠΎΠ²Ρ‹ΡˆΠ΅Π½ΠΈΡŽ ΠΈΡ… добротности. Однако, Π² рядС случаСв, Π΄ΠΎΠ±Ρ€ΠΎΡ‚Π½ΠΎΡΡ‚ΡŒ ΠΊΠΎΠ½Ρ‚ΡƒΡ€Π°, Π½Π°ΠΎΠ±ΠΎΡ€ΠΎΡ‚, приходится Π·Π°Π½ΠΈΠΆΠ°Ρ‚ΡŒ (Π½Π°ΠΏΡ€ΠΈΠΌΠ΅Ρ€, Π²ΠΊΠ»ΡŽΡ‡Π°Ρ ΠΏΠΎΡΠ»Π΅Π΄ΠΎΠ²Π°Ρ‚Π΅Π»ΡŒΠ½ΠΎ с ΠΊΠ°Ρ‚ΡƒΡˆΠΊΠΎΠΉ индуктивности рСзистор нСбольшой Π²Π΅Π»ΠΈΡ‡ΠΈΠ½Ρ‹ сопротивлСния), Ρ‡Ρ‚ΠΎ позволяСт ΠΈΠ·Π±Π΅ΠΆΠ°Ρ‚ΡŒ искаТСний ΡˆΠΈΡ€ΠΎΠΊΠΎΠΏΠΎΠ»ΠΎΡΠ½Ρ‹Ρ… сигналов. Π₯отя, Ссли Π½Π° ΠΏΡ€Π°ΠΊΡ‚ΠΈΠΊΠ΅ трСбуСтся Π²Ρ‹Π΄Π΅Π»ΠΈΡ‚ΡŒ достаточно ΡˆΠΈΡ€ΠΎΠΊΠΎΠΏΠΎΠ»ΠΎΡΠ½Ρ‹ΠΉ сигнал, сСлСктивныС Ρ†Π΅ΠΏΠΈ, ΠΊΠ°ΠΊ ΠΏΡ€Π°Π²ΠΈΠ»ΠΎ, строятся Π½Π΅ Π½Π° ΠΎΠ΄ΠΈΠ½ΠΎΡ‡Π½Ρ‹Ρ… ΠΊΠΎΠ»Π΅Π±Π°Ρ‚Π΅Π»ΡŒΠ½Ρ‹Ρ… ΠΊΠΎΠ½Ρ‚ΡƒΡ€Π°Ρ…, Π° Π½Π° Π±ΠΎΠ»Π΅Π΅ слоТных связанных (ΠΌΠ½ΠΎΠ³ΠΎΠΊΠΎΠ½Ρ‚ΡƒΡ€Π½Ρ‹Ρ…) ΠΊΠΎΠ»Π΅Π±Π°Ρ‚Π΅Π»ΡŒΠ½Ρ‹Ρ… систСмах, Π² Ρ‚.Ρ‡. ΠΌΠ½ΠΎΠ³ΠΎΠ·Π²Π΅Π½Π½Ρ‹Ρ… Ρ„ΠΈΠ»ΡŒΡ‚Ρ€Π°Ρ….

ΠŸΠ°Ρ€Π°Π»Π»Π΅Π»ΡŒΠ½Ρ‹ΠΉ ΠΊΠΎΠ»Π΅Π±Π°Ρ‚Π΅Π»ΡŒΠ½Ρ‹ΠΉ ΠΊΠΎΠ½Ρ‚ΡƒΡ€

Π’ Ρ€Π°Π·Π»ΠΈΡ‡Π½Ρ‹Ρ… радиотСхничСских устройствах наряду с ΠΏΠΎΡΠ»Π΅Π΄ΠΎΠ²Π°Ρ‚Π΅Π»ΡŒΠ½Ρ‹ΠΌΠΈ ΠΊΠΎΠ»Π΅Π±Π°Ρ‚Π΅Π»ΡŒΠ½Ρ‹ΠΌΠΈ ΠΊΠΎΠ½Ρ‚ΡƒΡ€Π°ΠΌΠΈ часто (Π΄Π°ΠΆΠ΅ Ρ‡Π°Ρ‰Π΅, Ρ‡Π΅ΠΌ ΠΏΠΎΡΠ»Π΅Π΄ΠΎΠ²Π°Ρ‚Π΅Π»ΡŒΠ½Ρ‹Π΅) ΠΏΡ€ΠΈΠΌΠ΅Π½ΡΡŽΡ‚ ΠΏΠ°Ρ€Π°Π»Π»Π΅Π»ΡŒΠ½Ρ‹Π΅ ΠΊΠΎΠ»Π΅Π±Π°Ρ‚Π΅Π»ΡŒΠ½Ρ‹Π΅ ΠΊΠΎΠ½Ρ‚ΡƒΡ€Ρ‹ На рисункС ΠΏΡ€ΠΈΠ²Π΅Π΄Π΅Π½Π° ΠΏΡ€ΠΈΠ½Ρ†ΠΈΠΏΠΈΠ°Π»ΡŒΠ½Π°Ρ схСма ΠΏΠ°Ρ€Π°Π»Π»Π΅Π»ΡŒΠ½ΠΎΠ³ΠΎ ΠΊΠΎΠ»Π΅Π±Π°Ρ‚Π΅Π»ΡŒΠ½ΠΎΠ³ΠΎ ΠΊΠΎΠ½Ρ‚ΡƒΡ€Π°. Π—Π΄Π΅ΡΡŒ ΠΏΠ°Ρ€Π°Π»Π»Π΅Π»ΡŒΠ½ΠΎ Π²ΠΊΠ»ΡŽΡ‡Π΅Π½Ρ‹ Π΄Π²Π° Ρ€Π΅Π°ΠΊΡ‚ΠΈΠ²Π½Ρ‹Ρ… элСмСнта с Ρ€Π°Π·Π½Ρ‹ΠΌ Ρ…Π°Ρ€Π°ΠΊΡ‚Π΅Ρ€ΠΎΠΌ рСактивности Как извСстно, ΠΏΡ€ΠΈ ΠΏΠ°Ρ€Π°Π»Π»Π΅Π»ΡŒΠ½ΠΎΠΌ Π²ΠΊΠ»ΡŽΡ‡Π΅Π½ΠΈΠΈ элСмСнтов ΡΠΊΠ»Π°Π΄Ρ‹Π²Π°Ρ‚ΡŒ ΠΈΡ… сопротивлСния нСльзя β€” ΠΌΠΎΠΆΠ½ΠΎ лишь ΡΠΊΠ»Π°Π΄Ρ‹Π²Π°Ρ‚ΡŒ проводимости. На рисункС ΠΏΡ€ΠΈΠ²Π΅Π΄Π΅Π½Ρ‹ графичСскиС зависимости Ρ€Π΅Π°ΠΊΡ‚ΠΈΠ²Π½Ρ‹Ρ… проводимостСй ΠΊΠ°Ρ‚ΡƒΡˆΠΊΠΈ индуктивности BL = 1/Ο‰L, кондСнсатора Π’C = -Ο‰C, Π° Ρ‚Π°ΠΊΠΆΠ΅ суммарной проводимости Π’Ξ£, этих Π΄Π²ΡƒΡ… элСмСнтов, ΡΠ²Π»ΡΡŽΡ‰Π°ΡΡΡ Ρ€Π΅Π°ΠΊΡ‚ΠΈΠ²Π½ΠΎΠΉ ΠΏΡ€ΠΎΠ²ΠΎΠ΄ΠΈΠΌΠΎΡΡ‚ΡŒΡŽ ΠΏΠ°Ρ€Π°Π»Π»Π΅Π»ΡŒΠ½ΠΎΠ³ΠΎ ΠΊΠΎΠ»Π΅Π±Π°Ρ‚Π΅Π»ΡŒΠ½ΠΎΠ³ΠΎ ΠΊΠΎΠ½Ρ‚ΡƒΡ€Π°. Аналогично, ΠΊΠ°ΠΊ ΠΈ для ΠΏΠΎΡΠ»Π΅Π΄ΠΎΠ²Π°Ρ‚Π΅Π»ΡŒΠ½ΠΎΠ³ΠΎ ΠΊΠΎΠ»Π΅Π±Π°Ρ‚Π΅Π»ΡŒΠ½ΠΎΠ³ΠΎ ΠΊΠΎΠ½Ρ‚ΡƒΡ€Π°, имССтся нСкоторая частота, называСмая рСзонансной, Π½Π° ΠΊΠΎΡ‚ΠΎΡ€ΠΎΠΉ Ρ€Π΅Π°ΠΊΡ‚ΠΈΠ²Π½Ρ‹Π΅ сопротивлСния (Π° Π·Π½Π°Ρ‡ΠΈΡ‚ ΠΈ проводимости) ΠΊΠ°Ρ‚ΡƒΡˆΠΊΠΈ ΠΈ кондСнсатора ΠΎΠ΄ΠΈΠ½Π°ΠΊΠΎΠ²Ρ‹. На этой частотС суммарная ΠΏΡ€ΠΎΠ²ΠΎΠ΄ΠΈΠΌΠΎΡΡ‚ΡŒ ΠΏΠ°Ρ€Π°Π»Π»Π΅Π»ΡŒΠ½ΠΎΠ³ΠΎ ΠΊΠΎΠ»Π΅Π±Π°Ρ‚Π΅Π»ΡŒΠ½ΠΎΠ³ΠΎ ΠΊΠΎΠ½Ρ‚ΡƒΡ€Π° Π±Π΅Π· ΠΏΠΎΡ‚Π΅Ρ€ΡŒ обращаСтся Π² Π½ΡƒΠ»ΡŒ. Π­Ρ‚ΠΎ Π·Π½Π°Ρ‡ΠΈΡ‚, Ρ‡Ρ‚ΠΎ Π½Π° этой частотС ΠΊΠΎΠ»Π΅Π±Π°Ρ‚Π΅Π»ΡŒΠ½Ρ‹ΠΉ ΠΊΠΎΠ½Ρ‚ΡƒΡ€ ΠΎΠ±Π»Π°Π΄Π°Π΅Ρ‚ бСсконСчно большим сопротивлСниСм ΠΏΠ΅Ρ€Π΅ΠΌΠ΅Π½Π½ΠΎΠΌΡƒ Ρ‚ΠΎΠΊΡƒ.

Если ΠΏΠΎΡΡ‚Ρ€ΠΎΠΈΡ‚ΡŒ Π·Π°Π²ΠΈΡΠΈΠΌΠΎΡΡ‚ΡŒ Ρ€Π΅Π°ΠΊΡ‚ΠΈΠ²Π½ΠΎΠ³ΠΎ сопротивлСния ΠΊΠΎΠ½Ρ‚ΡƒΡ€Π° ΠΎΡ‚ частоты XΞ£ = 1/BΞ£, эта кривая, изобраТённая Π½Π° ΡΠ»Π΅Π΄ΡƒΡŽΡ‰Π΅ΠΌ рисункС, Π² Ρ‚ΠΎΡ‡ΠΊΠ΅ Ο‰ = Ο‰Ρ€ Π±ΡƒΠ΄Π΅Ρ‚ ΠΈΠΌΠ΅Ρ‚ΡŒ Ρ€Π°Π·Ρ€Ρ‹Π² Π²Ρ‚ΠΎΡ€ΠΎΠ³ΠΎ Ρ€ΠΎΠ΄Π°. Π‘ΠΎΠΏΡ€ΠΎΡ‚ΠΈΠ²Π»Π΅Π½ΠΈΠ΅ Ρ€Π΅Π°Π»ΡŒΠ½ΠΎΠ³ΠΎ ΠΏΠ°Ρ€Π°Π»Π»Π΅Π»ΡŒΠ½ΠΎΠ³ΠΎ ΠΊΠΎΠ»Π΅Π±Π°Ρ‚Π΅Π»ΡŒΠ½ΠΎΠ³ΠΎ ΠΊΠΎΠ½Ρ‚ΡƒΡ€Π° (Ρ‚.Π΅ с потСрями), разумССтся, Π½Π΅ Ρ€Π°Π²Π½ΠΎ бСсконСчности β€” ΠΎΠ½ΠΎ Ρ‚Π΅ΠΌ мСньшС, Ρ‡Π΅ΠΌ большС омичСскоС сопротивлСниС ΠΏΠΎΡ‚Π΅Ρ€ΡŒ Π² ΠΊΠΎΠ½Ρ‚ΡƒΡ€Π΅, Ρ‚.Π΅ ΡƒΠΌΠ΅Π½ΡŒΡˆΠ°Π΅Ρ‚ΡΡ прямо ΠΏΡ€ΠΎΠΏΠΎΡ€Ρ†ΠΈΠΎΠ½Π°Π»ΡŒΠ½ΠΎ ΡƒΠΌΠ΅Π½ΡŒΡˆΠ΅Π½ΠΈΡŽ добротности ΠΊΠΎΠ½Ρ‚ΡƒΡ€Π°. Π’ Ρ†Π΅Π»ΠΎΠΌ, физичСский смысл понятий добротности, характСристичСского сопротивлСния ΠΈ рСзонансной частоты ΠΊΠΎΠ»Π΅Π±Π°Ρ‚Π΅Π»ΡŒΠ½ΠΎΠ³ΠΎ ΠΊΠΎΠ½Ρ‚ΡƒΡ€Π°, Π° Ρ‚Π°ΠΊΠΆΠ΅ ΠΈΡ… расчСтныС Ρ„ΠΎΡ€ΠΌΡƒΠ»Ρ‹, справСдливы ΠΊΠ°ΠΊ для ΠΏΠΎΡΠ»Π΅Π΄ΠΎΠ²Π°Ρ‚Π΅Π»ΡŒΠ½ΠΎΠ³ΠΎ, Ρ‚Π°ΠΊ ΠΈ для ΠΏΠ°Ρ€Π°Π»Π»Π΅Π»ΡŒΠ½ΠΎΠ³ΠΎ ΠΊΠΎΠ»Π΅Π±Π°Ρ‚Π΅Π»ΡŒΠ½ΠΎΠ³ΠΎ ΠΊΠΎΠ½Ρ‚ΡƒΡ€Π°.

Для ΠΏΠ°Ρ€Π°Π»Π»Π΅Π»ΡŒΠ½ΠΎΠ³ΠΎ ΠΊΠΎΠ»Π΅Π±Π°Ρ‚Π΅Π»ΡŒΠ½ΠΎΠ³ΠΎ ΠΊΠΎΠ½Ρ‚ΡƒΡ€Π°, Π² ΠΊΠΎΡ‚ΠΎΡ€ΠΎΠΌ ΠΈΠ½Π΄ΡƒΠΊΡ‚ΠΈΠ²Π½ΠΎΡΡ‚ΡŒ, Π΅ΠΌΠΊΠΎΡΡ‚ΡŒ ΠΈ сопротивлСниС Π²ΠΊΠ»ΡŽΡ‡Π΅Π½Ρ‹ ΠΏΠ°Ρ€Π°Π»Π»Π΅Π»ΡŒΠ½ΠΎ, Π΄ΠΎΠ±Ρ€ΠΎΡ‚Π½ΠΎΡΡ‚ΡŒ вычисляСтся:

Π³Π΄Π΅ R, L ΠΈ C β€” сопротивлСниС, ΠΈΠ½Π΄ΡƒΠΊΡ‚ΠΈΠ²Π½ΠΎΡΡ‚ΡŒ ΠΈ Ρ‘ΠΌΠΊΠΎΡΡ‚ΡŒ рСзонансной Ρ†Π΅ΠΏΠΈ, соотвСтствСнно.

Рассмотрим Ρ†Π΅ΠΏΡŒ, ΡΠΎΡΡ‚ΠΎΡΡ‰ΡƒΡŽ ΠΈΠ· Π³Π΅Π½Π΅Ρ€Π°Ρ‚ΠΎΡ€Π° гармоничСских ΠΊΠΎΠ»Π΅Π±Π°Π½ΠΈΠΉ ΠΈ ΠΏΠ°Ρ€Π°Π»Π»Π΅Π»ΡŒΠ½ΠΎΠ³ΠΎ ΠΊΠΎΠ»Π΅Π±Π°Ρ‚Π΅Π»ΡŒΠ½ΠΎΠ³ΠΎ ΠΊΠΎΠ½Ρ‚ΡƒΡ€Π°. Π’ случаС, ΠΊΠΎΠ³Π΄Π° частота ΠΊΠΎΠ»Π΅Π±Π°Π½ΠΈΠΉ Π³Π΅Π½Π΅Ρ€Π°Ρ‚ΠΎΡ€Π° совпадаСт с рСзонансной частотой ΠΊΠΎΠ½Ρ‚ΡƒΡ€Π° Π΅Π³ΠΎ индуктивная ΠΈ Смкостная Π²Π΅Ρ‚Π²ΠΈ ΠΎΠΊΠ°Π·Ρ‹Π²Π°ΡŽΡ‚ Ρ€Π°Π²Π½ΠΎΠ΅ сопротивлСниС ΠΏΠ΅Ρ€Π΅ΠΌΠ΅Π½Π½ΠΎΠΌΡƒ Ρ‚ΠΎΠΊΡƒ, Π² слСдствиС Ρ‡Π΅Π³ΠΎ Ρ‚ΠΎΠΊΠΈ Π² вСтвях ΠΊΠΎΠ½Ρ‚ΡƒΡ€Π° Π±ΡƒΠ΄ΡƒΡ‚ ΠΎΠ΄ΠΈΠ½Π°ΠΊΠΎΠ²Ρ‹ΠΌΠΈ. Π’ этом случаС говорят, Ρ‡Ρ‚ΠΎ Π² Ρ†Π΅ΠΏΠΈ ΠΈΠΌΠ΅Π΅Ρ‚ мСсто рСзонанс Ρ‚ΠΎΠΊΠΎΠ². Как ΠΈ Π² случаС ΠΏΠΎΡΠ»Π΅Π΄ΠΎΠ²Π°Ρ‚Π΅Π»ΡŒΠ½ΠΎΠ³ΠΎ ΠΊΠΎΠ»Π΅Π±Π°Ρ‚Π΅Π»ΡŒΠ½ΠΎΠ³ΠΎ ΠΊΠΎΠ½Ρ‚ΡƒΡ€Π°, рСактивности ΠΊΠ°Ρ‚ΡƒΡˆΠΊΠΈ ΠΈ кондСнсатора ΠΊΠΎΠΌΠΏΠ΅Π½ΡΠΈΡ€ΡƒΡŽΡ‚ Π΄Ρ€ΡƒΠ³ Π΄Ρ€ΡƒΠ³Π°, ΠΈ сопротивлСниС ΠΊΠΎΠ½Ρ‚ΡƒΡ€Π° ΠΏΡ€ΠΎΡ‚Π΅ΠΊΠ°ΡŽΡ‰Π΅ΠΌΡƒ Ρ‡Π΅Ρ€Π΅Π· Π½Π΅Π³ΠΎ Ρ‚ΠΎΠΊΡƒ становится чисто Π°ΠΊΡ‚ΠΈΠ²Π½Ρ‹ΠΌ (рСзистивным). Π’Π΅Π»ΠΈΡ‡ΠΈΠ½Π° этого сопротивлСния, часто Π½Π°Π·Ρ‹Π²Π°Π΅ΠΌΠΎΠ³ΠΎ Π² Ρ‚Π΅Ρ…Π½ΠΈΠΊΠ΅ эквивалСнтным, опрСдСляСтся ΠΏΡ€ΠΎΠΈΠ·Π²Π΅Π΄Π΅Π½ΠΈΠ΅ΠΌ добротности ΠΊΠΎΠ½Ρ‚ΡƒΡ€Π° Π½Π° Π΅Π³ΠΎ характСристичСскоС сопротивлСниС Rэкв = Q·ρ. На частотах, ΠΎΡ‚Π»ΠΈΡ‡Π½Ρ‹Ρ… ΠΎΡ‚ рСзонансной, сопротивлСниС ΠΊΠΎΠ½Ρ‚ΡƒΡ€Π° ΡƒΠΌΠ΅Π½ΡŒΡˆΠ°Π΅Ρ‚ΡΡ ΠΈ ΠΏΡ€ΠΈΠΎΠ±Ρ€Π΅Ρ‚Π°Π΅Ρ‚ Ρ€Π΅Π°ΠΊΡ‚ΠΈΠ²Π½Ρ‹ΠΉ Ρ…Π°Ρ€Π°ΠΊΡ‚Π΅Ρ€ Π½Π° Π±ΠΎΠ»Π΅Π΅ Π½ΠΈΠ·ΠΊΠΈΡ… частотах β€” ΠΈΠ½Π΄ΡƒΠΊΡ‚ΠΈΠ²Π½Ρ‹ΠΉ (ΠΏΠΎΡΠΊΠΎΠ»ΡŒΠΊΡƒ Ρ€Π΅Π°ΠΊΡ‚ΠΈΠ²Π½ΠΎΠ΅ сопротивлСниС индуктивности ΠΏΠ°Π΄Π°Π΅Ρ‚ ΠΏΡ€ΠΈ ΡƒΠΌΠ΅Π½ΡŒΡˆΠ΅Π½ΠΈΠΈ частоты), Π° Π½Π° Π±ΠΎΠ»Π΅Π΅ высоких β€” Π½Π°ΠΎΠ±ΠΎΡ€ΠΎΡ‚, Смкостной (Ρ‚ ΠΊ Ρ€Π΅Π°ΠΊΡ‚ΠΈΠ²Π½ΠΎΠ΅ сопротивлСниС Смкости ΠΏΠ°Π΄Π°Π΅Ρ‚ с ростом частоты).

Π’ процСссС Ρ€Π°Π±ΠΎΡ‚Ρ‹ ΠΊΠΎΠ½Ρ‚ΡƒΡ€Π°, Π΄Π²Π°ΠΆΠ΄Ρ‹ Π·Π° ΠΏΠ΅Ρ€ΠΈΠΎΠ΄ ΠΊΠΎΠ»Π΅Π±Π°Π½ΠΈΠΉ, происходит энСргСтичСский ΠΎΠ±ΠΌΠ΅Π½ ΠΌΠ΅ΠΆΠ΄Ρƒ ΠΊΠ°Ρ‚ΡƒΡˆΠΊΠΎΠΉ ΠΈ кондСнсатором (смотри рисунок). ЭнСргия ΠΏΠΎΠΎΡ‡Π΅Ρ€Π΅Π΄Π½ΠΎ накапливаСтся, Ρ‚ΠΎ Π² Π²ΠΈΠ΄Π΅ энСргии элСктричСского поля заряТСнного кондСнсатора, Ρ‚ΠΎ Π² Π²ΠΈΠ΄Π΅ энСргии ΠΌΠ°Π³Π½ΠΈΡ‚Π½ΠΎΠ³ΠΎ поля ΠΊΠ°Ρ‚ΡƒΡˆΠΊΠΈ индуктивности. ΠŸΡ€ΠΈ этом Π² ΠΊΠΎΠ½Ρ‚ΡƒΡ€Π΅ ΠΏΡ€ΠΎΡ‚Π΅ΠΊΠ°Π΅Ρ‚ собствСнный ΠΊΠΎΠ½Ρ‚ΡƒΡ€Π½Ρ‹ΠΉ Ρ‚ΠΎΠΊ IΠΊ, прСвосходящий ΠΏΠΎ Π²Π΅Π»ΠΈΡ‡ΠΈΠ½Π΅ Ρ‚ΠΎΠΊ Π²ΠΎ внСшнСй Ρ†Π΅ΠΏΠΈ I Π² Q Ρ€Π°Π·. Π’ случаС идСального ΠΊΠΎΠ½Ρ‚ΡƒΡ€Π° (Π±Π΅Π· ΠΏΠΎΡ‚Π΅Ρ€ΡŒ), Π΄ΠΎΠ±Ρ€ΠΎΡ‚Π½ΠΎΡΡ‚ΡŒ ΠΊΠΎΡ‚ΠΎΡ€ΠΎΠ³ΠΎ тСорСтичСски бСсконСчна, Π²Π΅Π»ΠΈΡ‡ΠΈΠ½Π° ΠΊΠΎΠ½Ρ‚ΡƒΡ€Π½ΠΎΠ³ΠΎ Ρ‚ΠΎΠΊΠ° Ρ‚Π°ΠΊΠΆΠ΅ Π±ΡƒΠ΄Π΅Ρ‚ бСсконСчно большой. Но Π½Π° ΠΏΡ€Π°ΠΊΡ‚ΠΈΠΊΠ΅, Ρ‚Π°ΠΊΠΎΠ³ΠΎ Π½Π΅ Π±Ρ‹Π²Π°Π΅Ρ‚. Π’ любом случаС, качСство элСмСнтов ΠΊΠΎΠ½Ρ‚ΡƒΡ€Π°, ΠΈΡ… ΠΏΠ°Ρ€Π°Π·ΠΈΡ‚Π½Ρ‹Π΅ характСристики, элСктричСскиС Ρ†Π΅ΠΏΠΈ, слуТащиС для ΠΏΠΎΠ΄Π²ΠΎΠ΄Π° энСргии ΠΈ ΠΎΡ‚Π±ΠΎΡ€Π° энСргии ΠΈΠ· ΠΊΠΎΠ½Ρ‚ΡƒΡ€Π°, Π½Π΅ позволят ΠΊΠΎΠ½Ρ‚ΡƒΡ€Π½ΠΎΠΌΡƒ Ρ‚ΠΎΠΊΡƒ расти.

Рассмотрим, ΠΊΠ°ΠΊ зависят коэффициСнты ΠΏΠ΅Ρ€Π΅Π΄Π°Ρ‡ΠΈ Ρ‡Π΅Ρ‚Ρ‹Ρ€Π΅Ρ…ΠΏΠΎΠ»ΡŽΡΠ½ΠΈΠΊΠΎΠ² ΠΎΡ‚ частоты, ΠΏΡ€ΠΈ Π²ΠΊΠ»ΡŽΡ‡Π΅Π½ΠΈΠΈ Π² Π½ΠΈΡ… Π½Π΅ ΠΏΠΎΡΠ»Π΅Π΄ΠΎΠ²Π°Ρ‚Π΅Π»ΡŒΠ½Ρ‹Ρ… ΠΊΠΎΠ»Π΅Π±Π°Ρ‚Π΅Π»ΡŒΠ½Ρ‹Ρ… ΠΊΠΎΠ½Ρ‚ΡƒΡ€ΠΎΠ², Π° ΠΏΠ°Ρ€Π°Π»Π»Π΅Π»ΡŒΠ½Ρ‹Ρ….

Π§Π΅Ρ‚Ρ‹Ρ€Π΅Ρ…ΠΏΠΎΠ»ΡŽΡΠ½ΠΈΠΊ, ΠΈΠ·ΠΎΠ±Ρ€Π°ΠΆΠ΅Π½Π½Ρ‹ΠΉ Π½Π° рисункС, Π½Π° рСзонансной частотС ΠΊΠΎΠ½Ρ‚ΡƒΡ€Π° прСдставляСт собой ΠΎΠ³Ρ€ΠΎΠΌΠ½ΠΎΠ΅ сопротивлСниС Ρ‚ΠΎΠΊΡƒ, поэтому ΠΏΡ€ΠΈ Ο‰=Ο‰Ρ€ Π΅Π³ΠΎ коэффициСнт ΠΏΠ΅Ρ€Π΅Π΄Π°Ρ‡ΠΈ Π±ΡƒΠ΄Π΅Ρ‚ Π±Π»ΠΈΠ·ΠΎΠΊ ΠΊ Π½ΡƒΠ»ΡŽ (с ΡƒΡ‡Π΅Ρ‚ΠΎΠΌ омичСских ΠΏΠΎΡ‚Π΅Ρ€ΡŒ). На частотах, ΠΎΡ‚Π»ΠΈΡ‡Π½Ρ‹Ρ… ΠΎΡ‚ рСзонансной, сопротивлСниС ΠΊΠΎΠ½Ρ‚ΡƒΡ€Π° Π±ΡƒΠ΄Π΅Ρ‚ ΡƒΠΌΠ΅Π½ΡŒΡˆΠ°Ρ‚ΡΡ, Π° коэффициСнт ΠΏΠ΅Ρ€Π΅Π΄Π°Ρ‡ΠΈ Ρ‡Π΅Ρ‚Ρ‹Ρ€Π΅Ρ…ΠΏΠΎΠ»ΡŽΡΠ½ΠΈΠΊΠ° β€” Π²ΠΎΠ·Ρ€Π°ΡΡ‚Π°Ρ‚ΡŒ.

Для Ρ‡Π΅Ρ‚Ρ‹Ρ€Π΅Ρ…ΠΏΠΎΠ»ΡŽΡΠ½ΠΈΠΊΠ°, ΠΏΡ€ΠΈΠ²Π΅Π΄Π΅Π½Π½ΠΎΠ³ΠΎ Π½Π° рисункС Π²Ρ‹ΡˆΠ΅, ситуация Π±ΡƒΠ΄Π΅Ρ‚ ΠΏΡ€ΠΎΡ‚ΠΈΠ²ΠΎΠΏΠΎΠ»ΠΎΠΆΠ½ΠΎΠΉ β€” Π½Π° рСзонансной частотС ΠΊΠΎΠ½Ρ‚ΡƒΡ€ Π±ΡƒΠ΄Π΅Ρ‚ ΠΏΡ€Π΅Π΄ΡΡ‚Π°Π²Π»ΡΡ‚ΡŒ собой ΠΎΡ‡Π΅Π½ΡŒ большоС сопротивлСниС ΠΈ практичСски всС Π²Ρ…ΠΎΠ΄Π½ΠΎΠ΅ напряТСниС поступит Π½Π° Π²Ρ‹Ρ…ΠΎΠ΄Π½Ρ‹Π΅ ΠΊΠ»Π΅ΠΌΠΌΡ‹ (Ρ‚.Π΅ коэффициСнт ΠΏΠ΅Ρ€Π΅Π΄Π°Ρ‡ΠΈ Π±ΡƒΠ΄Π΅Ρ‚ максималСн ΠΈ Π±Π»ΠΈΠ·ΠΎΠΊ ΠΊ Π΅Π΄ΠΈΠ½ΠΈΡ†Π΅). ΠŸΡ€ΠΈ Π·Π½Π°Ρ‡ΠΈΡ‚Π΅Π»ΡŒΠ½ΠΎΠΌ ΠΎΡ‚Π»ΠΈΡ‡ΠΈΠΈ частоты Π²Ρ…ΠΎΠ΄Π½ΠΎΠ³ΠΎ воздСйствия ΠΎΡ‚ рСзонансной частоты ΠΊΠΎΠ½Ρ‚ΡƒΡ€Π°, источник сигнала, ΠΏΠΎΠ΄ΠΊΠ»ΡŽΡ‡Π°Π΅ΠΌΡ‹ΠΉ ΠΊ Π²Ρ…ΠΎΠ΄Π½Ρ‹ΠΌ ΠΊΠ»Π΅ΠΌΠΌΠ°ΠΌ Ρ‡Π΅Ρ‚Ρ‹Ρ€Π΅Ρ…ΠΏΠΎΠ»ΡŽΡΠ½ΠΈΠΊΠ°, окаТСтся практичСски Π·Π°ΠΊΠΎΡ€ΠΎΡ‡Π΅Π½Π½ΠΎΠΌ Π½Π°ΠΊΠΎΡ€ΠΎΡ‚ΠΊΠΎ, Π° коэффициСнт ΠΏΠ΅Ρ€Π΅Π΄Π°Ρ‡ΠΈ Π±ΡƒΠ΄Π΅Ρ‚ Π±Π»ΠΈΠ·ΠΎΠΊ ΠΊ Π½ΡƒΠ»ΡŽ.

Π’ΠΈΠ΄Π΅ΠΎ ΠΏΠΎ Ρ‚Π΅ΠΌΠ΅: ΠšΠΎΠ»Π΅Π±Π°Ρ‚Π΅Π»ΡŒΠ½Ρ‹ΠΉ ΠΊΠΎΠ½Ρ‚ΡƒΡ€

Π’ΠΈΠΌΠ΅Ρ€ΠΊΠ°Π΅Π² Борис β€” 68-Π»Π΅Ρ‚Π½ΠΈΠΉ Π΄ΠΎΠΊΡ‚ΠΎΡ€ Ρ„ΠΈΠ·ΠΈΠΊΠΎ-матСматичСских Π½Π°ΡƒΠΊ, профСссор ΠΈΠ· России. Он являСтся Π·Π°Π²Π΅Π΄ΡƒΡŽΡ‰ΠΈΠΌ ΠΊΠ°Ρ„Π΅Π΄Ρ€ΠΎΠΉ ΠΎΠ±Ρ‰Π΅ΠΉ Ρ„ΠΈΠ·ΠΈΠΊΠΈ Π² Казанском Π½Π°Ρ†ΠΈΠΎΠ½Π°Π»ΡŒΠ½ΠΎΠΌ ΠΈΡΡΠ»Π΅Π΄ΠΎΠ²Π°Ρ‚Π΅Π»ΡŒΡΠΊΠΎΠΌ тСхничСском унивСрситСтС ΠΈΠΌΠ΅Π½ΠΈ А. Н. Π’Π£ΠŸΠžΠ›Π•Π’Π β€” КАИ

Π’ ΡΡ‚Π°Ρ‚ΡŒΠ΅ расскаТСм Ρ‡Ρ‚ΠΎ Ρ‚Π°ΠΊΠΎΠ΅ ΠΊΠΎΠ»Π΅Π±Π°Ρ‚Π΅Π»ΡŒΠ½Ρ‹ΠΉ ΠΊΠΎΠ½Ρ‚ΡƒΡ€. ΠŸΠΎΡΠ»Π΅Π΄ΠΎΠ²Π°Ρ‚Π΅Π»ΡŒΠ½Ρ‹ΠΉ ΠΈ ΠΏΠ°Ρ€Π°Π»Π»Π΅Π»ΡŒΠ½Ρ‹ΠΉ ΠΊΠΎΠ»Π΅Π±Π°Ρ‚Π΅Π»ΡŒΠ½Ρ‹ΠΉ ΠΊΠΎΠ½Ρ‚ΡƒΡ€.

ΠšΠΎΠ»Π΅Π±Π°Ρ‚Π΅Π»ΡŒΠ½Ρ‹ΠΉ ΠΊΠΎΠ½Ρ‚ΡƒΡ€ β€” устройство ΠΈΠ»ΠΈ элСктричСская Ρ†Π΅ΠΏΡŒ, содСрТащСС Π½Π΅ΠΎΠ±Ρ…ΠΎΠ΄ΠΈΠΌΡ‹Π΅ радиоэлСктронныС элСмСнты для создания элСктромагнитных ΠΊΠΎΠ»Π΅Π±Π°Π½ΠΈΠΉ. РаздСляСтся Π½Π° Π΄Π²Π° Ρ‚ΠΈΠΏΠ° Π² зависимости ΠΎΡ‚ соСдинСния элСмСнтов: ΠΏΠΎΡΠ»Π΅Π΄ΠΎΠ²Π°Ρ‚Π΅Π»ΡŒΠ½Ρ‹ΠΉ ΠΈ ΠΏΠ°Ρ€Π°Π»Π»Π΅Π»ΡŒΠ½Ρ‹ΠΉ.

Основная радиоэлСмСнтная Π±Π°Π·Π° ΠΊΠΎΠ»Π΅Π±Π°Ρ‚Π΅Π»ΡŒΠ½ΠΎΠ³ΠΎ ΠΊΠΎΠ½Ρ‚ΡƒΡ€Π°: ΠšΠΎΠ½Π΄Π΅Π½ΡΠ°Ρ‚ΠΎΡ€, источник питания ΠΈ ΠΊΠ°Ρ‚ΡƒΡˆΠΊΠ° индуктивности.

ΠŸΠΎΡΠ»Π΅Π΄ΠΎΠ²Π°Ρ‚Π΅Π»ΡŒΠ½Ρ‹ΠΉ ΠΊΠΎΠ»Π΅Π±Π°Ρ‚Π΅Π»ΡŒΠ½Ρ‹ΠΉ ΠΊΠΎΠ½Ρ‚ΡƒΡ€

ΠŸΠΎΡΠ»Π΅Π΄ΠΎΠ²Π°Ρ‚Π΅Π»ΡŒΠ½Ρ‹ΠΉ ΠΊΠΎΠ»Π΅Π±Π°Ρ‚Π΅Π»ΡŒΠ½Ρ‹ΠΉ ΠΊΠΎΠ½Ρ‚ΡƒΡ€ являСтся ΠΏΡ€ΠΎΡΡ‚Π΅ΠΉΡˆΠ΅ΠΉ рСзонансной (ΠΊΠΎΠ»Π΅Π±Π°Ρ‚Π΅Π»ΡŒΠ½ΠΎΠΉ) Ρ†Π΅ΠΏΡŒΡŽ. Бостоит ΠΏΠΎΡΠ»Π΅Π΄ΠΎΠ²Π°Ρ‚Π΅Π»ΡŒΠ½Ρ‹ΠΉ ΠΊΠΎΠ»Π΅Π±Π°Ρ‚Π΅Π»ΡŒΠ½Ρ‹ΠΉ ΠΊΠΎΠ½Ρ‚ΡƒΡ€, ΠΈΠ· ΠΏΠΎΡΠ»Π΅Π΄ΠΎΠ²Π°Ρ‚Π΅Π»ΡŒΠ½ΠΎ Π²ΠΊΠ»ΡŽΡ‡Π΅Π½Π½Ρ‹Ρ… ΠΊΠ°Ρ‚ΡƒΡˆΠΊΠΈ индуктивности ΠΈ кондСнсатора. ΠŸΡ€ΠΈ воздСйствии Π½Π° Ρ‚Π°ΠΊΡƒΡŽ Ρ†Π΅ΠΏΡŒ ΠΏΠ΅Ρ€Π΅ΠΌΠ΅Π½Π½ΠΎΠ³ΠΎ (гармоничСского) напряТСния, Ρ‡Π΅Ρ€Π΅Π· ΠΊΠ°Ρ‚ΡƒΡˆΠΊΡƒ ΠΈ кондСнсатор Π±ΡƒΠ΄Π΅Ρ‚ ΠΏΡ€ΠΎΡ‚Π΅ΠΊΠ°Ρ‚ΡŒ ΠΏΠ΅Ρ€Π΅ΠΌΠ΅Π½Π½Ρ‹ΠΉ Ρ‚ΠΎΠΊ, Π²Π΅Π»ΠΈΡ‡ΠΈΠ½Π° ΠΊΠΎΡ‚ΠΎΡ€ΠΎΠ³ΠΎ вычисляСтся ΠΏΠΎ Π·Π°ΠΊΠΎΠ½Ρƒ Ома: I = U / Π₯Ξ£ , Π³Π΄Π΅ Π₯Ξ£ β€” сумма Ρ€Π΅Π°ΠΊΡ‚ΠΈΠ²Π½Ρ‹Ρ… сопротивлСний ΠΏΠΎΡΠ»Π΅Π΄ΠΎΠ²Π°Ρ‚Π΅Π»ΡŒΠ½ΠΎ Π²ΠΊΠ»ΡŽΡ‡Π΅Π½Π½Ρ‹Ρ… ΠΊΠ°Ρ‚ΡƒΡˆΠΊΠΈ ΠΈ кондСнсатора (ΠΈΡΠΏΠΎΠ»ΡŒΠ·ΡƒΠ΅Ρ‚ΡΡ ΠΌΠΎΠ΄ΡƒΠ»ΡŒ суммы).

Для освСТСния памяти, вспомним ΠΊΠ°ΠΊ зависят Ρ€Π΅Π°ΠΊΡ‚ΠΈΠ²Π½Ρ‹Π΅ сопротивлСния кондСнсатора ΠΈ ΠΊΠ°Ρ‚ΡƒΡˆΠΊΠΈ индуктивности ΠΎΡ‚ частоты ΠΏΡ€ΠΈΠ»ΠΎΠΆΠ΅Π½Π½ΠΎΠ³ΠΎ ΠΏΠ΅Ρ€Π΅ΠΌΠ΅Π½Π½ΠΎΠ³ΠΎ напряТСния. Для ΠΊΠ°Ρ‚ΡƒΡˆΠΊΠΈ индуктивности, эта Π·Π°Π²ΠΈΡΠΈΠΌΠΎΡΡ‚ΡŒ Π±ΡƒΠ΄Π΅Ρ‚ ΠΈΠΌΠ΅Ρ‚ΡŒ Π²ΠΈΠ΄:

Из Ρ„ΠΎΡ€ΠΌΡƒΠ»Ρ‹ Π²ΠΈΠ΄Π½ΠΎ, Ρ‡Ρ‚ΠΎ ΠΏΡ€ΠΈ ΡƒΠ²Π΅Π»ΠΈΡ‡Π΅Π½ΠΈΠΈ частоты, Ρ€Π΅Π°ΠΊΡ‚ΠΈΠ²Π½ΠΎΠ΅ сопротивлСниС ΠΊΠ°Ρ‚ΡƒΡˆΠΊΠΈ индуктивности увСличиваСтся. Для кондСнсатора Π·Π°Π²ΠΈΡΠΈΠΌΠΎΡΡ‚ΡŒ Π΅Π³ΠΎ Ρ€Π΅Π°ΠΊΡ‚ΠΈΠ²Π½ΠΎΠ³ΠΎ сопротивлСния ΠΎΡ‚ частоты Π±ΡƒΠ΄Π΅Ρ‚ Π²Ρ‹Π³Π»ΡΠ΄Π΅Ρ‚ΡŒ ΡΠ»Π΅Π΄ΡƒΡŽΡ‰ΠΈΠΌ ΠΎΠ±Ρ€Π°Π·ΠΎΠΌ:

Π’ ΠΎΡ‚Π»ΠΈΡ‡ΠΈΠΈ ΠΎΡ‚ индуктивности, Ρƒ кондСнсатора всё происходит Π½Π°ΠΎΠ±ΠΎΡ€ΠΎΡ‚ β€” ΠΏΡ€ΠΈ ΡƒΠ²Π΅Π»ΠΈΡ‡Π΅Π½ΠΈΠΈ частоты, Ρ€Π΅Π°ΠΊΡ‚ΠΈΠ²Π½ΠΎΠ΅ сопротивлСниС ΡƒΠΌΠ΅Π½ΡŒΡˆΠ°Π΅Ρ‚ΡΡ. На ΡΠ»Π΅Π΄ΡƒΡŽΡ‰Π΅ΠΌ рисункС графичСски прСдставлСны зависимости Ρ€Π΅Π°ΠΊΡ‚ΠΈΠ²Π½Ρ‹Ρ… сопротивлСний ΠΊΠ°Ρ‚ΡƒΡˆΠΊΠΈ XL ΠΈ кондСнсатора Π₯C ΠΎΡ‚ цикличСской (ΠΊΡ€ΡƒΠ³ΠΎΠ²ΠΎΠΉ) частоты Ο‰, Π° Ρ‚Π°ΠΊΠΆΠ΅ Π³Ρ€Π°Ρ„ΠΈΠΊ зависимости ΠΎΡ‚ частоты Ο‰ ΠΈΡ… алгСбраичСской суммы Π₯Ξ£. Π“Ρ€Π°Ρ„ΠΈΠΊ, ΠΏΠΎ сути, ΠΏΠΎΠΊΠ°Π·Ρ‹Π²Π°Π΅Ρ‚ Π·Π°Π²ΠΈΡΠΈΠΌΠΎΡΡ‚ΡŒ ΠΎΡ‚ частоты ΠΎΠ±Ρ‰Π΅Π³ΠΎ Ρ€Π΅Π°ΠΊΡ‚ΠΈΠ²Π½ΠΎΠ³ΠΎ сопротивлСния ΠΏΠΎΡΠ»Π΅Π΄ΠΎΠ²Π°Ρ‚Π΅Π»ΡŒΠ½ΠΎΠ³ΠΎ ΠΊΠΎΠ»Π΅Π±Π°Ρ‚Π΅Π»ΡŒΠ½ΠΎΠ³ΠΎ ΠΊΠΎΠ½Ρ‚ΡƒΡ€Π°.

Из Π³Ρ€Π°Ρ„ΠΈΠΊΠ° Π²ΠΈΠ΄Π½ΠΎ, Ρ‡Ρ‚ΠΎ Π½Π° Π½Π΅ΠΊΠΎΡ‚ΠΎΡ€ΠΎΠΉ частотС Ο‰=Ο‰Ρ€ , Π½Π° ΠΊΠΎΡ‚ΠΎΡ€ΠΎΠΉ Ρ€Π΅Π°ΠΊΡ‚ΠΈΠ²Π½Ρ‹Π΅ сопротивлСния ΠΊΠ°Ρ‚ΡƒΡˆΠΊΠΈ ΠΈ кондСнсатора Ρ€Π°Π²Π½Ρ‹ ΠΏΠΎ ΠΌΠΎΠ΄ΡƒΠ»ΡŽ (Ρ€Π°Π²Π½Ρ‹ ΠΏΠΎ Π·Π½Π°Ρ‡Π΅Π½ΠΈΡŽ, Π½ΠΎ ΠΏΡ€ΠΎΡ‚ΠΈΠ²ΠΎΠΏΠΎΠ»ΠΎΠΆΠ½Ρ‹ ΠΏΠΎ Π·Π½Π°ΠΊΡƒ), ΠΎΠ±Ρ‰Π΅Π΅ сопротивлСниС Ρ†Π΅ΠΏΠΈ обращаСтся Π² ноль. На этой частотС Π² Ρ†Π΅ΠΏΠΈ Π½Π°Π±Π»ΡŽΠ΄Π°Π΅Ρ‚ΡΡ максимум Ρ‚ΠΎΠΊΠ°, ΠΊΠΎΡ‚ΠΎΡ€Ρ‹ΠΉ ΠΎΠ³Ρ€Π°Π½ΠΈΡ‡Π΅Π½ Ρ‚ΠΎΠ»ΡŒΠΊΠΎ омичСскими потСрями Π² ΠΊΠ°Ρ‚ΡƒΡˆΠΊΠ΅ индуктивности (Ρ‚.Π΅. Π°ΠΊΡ‚ΠΈΠ²Π½Ρ‹ΠΌ сопротивлСниСм ΠΏΡ€ΠΎΠ²ΠΎΠ΄Π° ΠΎΠ±ΠΌΠΎΡ‚ΠΊΠΈ ΠΊΠ°Ρ‚ΡƒΡˆΠΊΠΈ) ΠΈ Π²Π½ΡƒΡ‚Ρ€Π΅Π½Π½ΠΈΠΌ сопротивлСниСм источника Ρ‚ΠΎΠΊΠ° (Π³Π΅Π½Π΅Ρ€Π°Ρ‚ΠΎΡ€Π°). Π’Π°ΠΊΡƒΡŽ частоту, ΠΏΡ€ΠΈ ΠΊΠΎΡ‚ΠΎΡ€ΠΎΠΉ Π½Π°Π±Π»ΡŽΠ΄Π°Π΅Ρ‚ΡΡ рассмотрСнноС явлСниС, Π½Π°Π·Ρ‹Π²Π°Π΅ΠΌΠΎΠ΅ Π² Ρ„ΠΈΠ·ΠΈΠΊΠ΅ рСзонансом, Π½Π°Π·Ρ‹Π²Π°ΡŽΡ‚ рСзонансной частотой ΠΈΠ»ΠΈ собствСнной частотой ΠΊΠΎΠ»Π΅Π±Π°Π½ΠΈΠΉ Ρ†Π΅ΠΏΠΈ. Π’Π°ΠΊΠΆΠ΅ ΠΈΠ· Π³Ρ€Π°Ρ„ΠΈΠΊΠ° Π²ΠΈΠ΄Π½ΠΎ, Ρ‡Ρ‚ΠΎ Π½Π° частотах, Π½ΠΈΠΆΠ΅ частоты рСзонанса Ρ€Π΅Π°ΠΊΡ‚ΠΈΠ²Π½ΠΎΠ΅ сопротивлСниС ΠΏΠΎΡΠ»Π΅Π΄ΠΎΠ²Π°Ρ‚Π΅Π»ΡŒΠ½ΠΎΠ³ΠΎ ΠΊΠΎΠ»Π΅Π±Π°Ρ‚Π΅Π»ΡŒΠ½ΠΎΠ³ΠΎ ΠΊΠΎΠ½Ρ‚ΡƒΡ€Π° носит Смкостной Ρ…Π°Ρ€Π°ΠΊΡ‚Π΅Ρ€, Π° Π½Π° Π±ΠΎΠ»Π΅Π΅ высоких частотах β€” ΠΈΠ½Π΄ΡƒΠΊΡ‚ΠΈΠ²Π½Ρ‹ΠΉ. Π§Ρ‚ΠΎ касаСтся самой рСзонансной частоты, Ρ‚ΠΎ ΠΎΠ½Π° ΠΌΠΎΠΆΠ΅Ρ‚ Π±Ρ‹Ρ‚ΡŒ вычислСна ΠΏΡ€ΠΈ ΠΏΠΎΠΌΠΎΡ‰ΠΈ Ρ„ΠΎΡ€ΠΌΡƒΠ»Ρ‹ Вомсона, ΠΊΠΎΡ‚ΠΎΡ€ΡƒΡŽ ΠΌΡ‹ ΠΌΠΎΠΆΠ΅ΠΌ вывСсти ΠΈΠ· Ρ„ΠΎΡ€ΠΌΡƒΠ» Ρ€Π΅Π°ΠΊΡ‚ΠΈΠ²Π½Ρ‹Ρ… сопротивлСний ΠΊΠ°Ρ‚ΡƒΡˆΠΊΠΈ индуктивности ΠΈ кондСнсатора, приравняв ΠΈΡ… Ρ€Π΅Π°ΠΊΡ‚ΠΈΠ²Π½Ρ‹Π΅ сопротивлСния Π΄Ρ€ΡƒΠ³ ΠΊ Π΄Ρ€ΡƒΠ³Ρƒ:

На рисункС справа, ΠΈΠ·ΠΎΠ±Ρ€Π°ΠΆΠ΅Π½Π° эквивалСнтная схСма ΠΏΠΎΡΠ»Π΅Π΄ΠΎΠ²Π°Ρ‚Π΅Π»ΡŒΠ½ΠΎΠ³ΠΎ рСзонансного ΠΊΠΎΠ½Ρ‚ΡƒΡ€Π° с ΡƒΡ‡Π΅Ρ‚ΠΎΠΌ омичСских ΠΏΠΎΡ‚Π΅Ρ€ΡŒ R, ΠΏΠΎΠ΄ΠΊΠ»ΡŽΡ‡Π΅Π½Π½ΠΎΠ³ΠΎ ΠΊ ΠΈΠ΄Π΅Π°Π»ΡŒΠ½ΠΎΠΌΡƒ Π³Π΅Π½Π΅Ρ€Π°Ρ‚ΠΎΡ€Ρƒ гармоничСского напряТСния с Π°ΠΌΠΏΠ»ΠΈΡ‚ΡƒΠ΄ΠΎΠΉ U. ПолноС сопротивлСниС (импСданс) Ρ‚Π°ΠΊΠΎΠΉ Ρ†Π΅ΠΏΠΈ опрСдСляСтся: Z = √(R 2 +XΞ£ 2 ), Π³Π΄Π΅ XΞ£ = Ο‰ L-1/Ο‰C. На рСзонансной частотС, ΠΊΠΎΠ³Π΄Π° Π²Π΅Π»ΠΈΡ‡ΠΈΠ½Ρ‹ Ρ€Π΅Π°ΠΊΡ‚ΠΈΠ²Π½Ρ‹Ρ… сопротивлСний ΠΊΠ°Ρ‚ΡƒΡˆΠΊΠΈ XL = Ο‰L ΠΈ кондСнсатора Π₯Π‘= 1/Ο‰Π‘ Ρ€Π°Π²Π½Ρ‹ ΠΏΠΎ ΠΌΠΎΠ΄ΡƒΠ»ΡŽ, Π²Π΅Π»ΠΈΡ‡ΠΈΠ½Π° XΞ£ обращаСтся Π² Π½ΡƒΠ»ΡŒ (ΡΠ»Π΅Π΄ΠΎΠ²Π°Ρ‚Π΅Π»ΡŒΠ½ΠΎ, сопротивлСниС Ρ†Π΅ΠΏΠΈ чисто Π°ΠΊΡ‚ΠΈΠ²Π½ΠΎΠ΅), Π° Ρ‚ΠΎΠΊ Π² Ρ†Π΅ΠΏΠΈ опрСдСлятся ΠΎΡ‚Π½ΠΎΡˆΠ΅Π½ΠΈΠ΅ΠΌ Π°ΠΌΠΏΠ»ΠΈΡ‚ΡƒΠ΄Ρ‹ напряТСния Π³Π΅Π½Π΅Ρ€Π°Ρ‚ΠΎΡ€Π° ΠΊ ΡΠΎΠΏΡ€ΠΎΡ‚ΠΈΠ²Π»Π΅Π½ΠΈΡŽ омичСских ΠΏΠΎΡ‚Π΅Ρ€ΡŒ: I= U/R. ΠŸΡ€ΠΈ этом Π½Π° ΠΊΠ°Ρ‚ΡƒΡˆΠΊΠ΅ ΠΈ Π½Π° кондСнсаторС, Π² ΠΊΠΎΡ‚ΠΎΡ€Ρ‹Ρ… запасСна рСактивная элСктричСская энСргия, ΠΏΠ°Π΄Π°Π΅Ρ‚ ΠΎΠ΄ΠΈΠ½Π°ΠΊΠΎΠ²ΠΎΠ΅ напряТСниС UL = UΠ‘ = IXL = IXΠ‘.

На любой Π΄Ρ€ΡƒΠ³ΠΎΠΉ частотС, ΠΎΡ‚Π»ΠΈΡ‡Π½ΠΎΠΉ ΠΎΡ‚ рСзонансной, напряТСния Π½Π° ΠΊΠ°Ρ‚ΡƒΡˆΠΊΠ΅ ΠΈ кондСнсаторС Π½Π΅ΠΎΠ΄ΠΈΠ½Π°ΠΊΠΎΠ²Ρ‹ β€” ΠΎΠ½ΠΈ ΠΎΠΏΡ€Π΅Π΄Π΅Π»ΡΡŽΡ‚ΡΡ Π°ΠΌΠΏΠ»ΠΈΡ‚ΡƒΠ΄ΠΎΠΉ Ρ‚ΠΎΠΊΠ° Π² Ρ†Π΅ΠΏΠΈ ΠΈ Π²Π΅Π»ΠΈΡ‡ΠΈΠ½Π°ΠΌΠΈ ΠΌΠΎΠ΄ΡƒΠ»Π΅ΠΉ Ρ€Π΅Π°ΠΊΡ‚ΠΈΠ²Π½Ρ‹Ρ… сопротивлСний XL ΠΈ XΠ‘.ΠŸΠΎΡΡ‚ΠΎΠΌΡƒ рСзонанс Π² ΠΏΠΎΡΠ»Π΅Π΄ΠΎΠ²Π°Ρ‚Π΅Π»ΡŒΠ½ΠΎΠΌ ΠΊΠΎΠ»Π΅Π±Π°Ρ‚Π΅Π»ΡŒΠ½ΠΎΠΌ ΠΊΠΎΠ½Ρ‚ΡƒΡ€Π΅ принято Π½Π°Π·Ρ‹Π²Π°Ρ‚ΡŒ рСзонансом напряТСний. РСзонансной частотой ΠΊΠΎΠ½Ρ‚ΡƒΡ€Π° Π½Π°Π·Ρ‹Π²Π°ΡŽΡ‚ Ρ‚Π°ΠΊΡƒΡŽ частоту, Π½Π° ΠΊΠΎΡ‚ΠΎΡ€ΠΎΠΉ сопротивлСниС ΠΊΠΎΠ½Ρ‚ΡƒΡ€Π° ΠΈΠΌΠ΅Π΅Ρ‚ чисто Π°ΠΊΡ‚ΠΈΠ²Π½Ρ‹ΠΉ (рСзистивный) Ρ…Π°Ρ€Π°ΠΊΡ‚Π΅Ρ€. УсловиС рСзонанса β€” это равСнство Π²Π΅Π»ΠΈΡ‡ΠΈΠ½ Ρ€Π΅Π°ΠΊΡ‚ΠΈΠ²Π½Ρ‹Ρ… сопротивлСний ΠΊΠ°Ρ‚ΡƒΡˆΠΊΠΈ индуктивности ΠΈ ёмкости.

Одними ΠΈΠ· Π½Π°ΠΈΠ±ΠΎΠ»Π΅Π΅ Π²Π°ΠΆΠ½Ρ‹Ρ… ΠΏΠ°Ρ€Π°ΠΌΠ΅Ρ‚Ρ€ΠΎΠ² ΠΊΠΎΠ»Π΅Π±Π°Ρ‚Π΅Π»ΡŒΠ½ΠΎΠ³ΠΎ ΠΊΠΎΠ½Ρ‚ΡƒΡ€Π° (ΠΊΡ€ΠΎΠΌΠ΅, разумССтся, рСзонансной частоты) ΡΠ²Π»ΡΡŽΡ‚ΡΡ Π΅Π³ΠΎ характСристичСскоС (ΠΈΠ»ΠΈ Π²ΠΎΠ»Π½ΠΎΠ²ΠΎΠ΅) сопротивлСниС ρ ΠΈ Π΄ΠΎΠ±Ρ€ΠΎΡ‚Π½ΠΎΡΡ‚ΡŒ ΠΊΠΎΠ½Ρ‚ΡƒΡ€Π° Q. Π₯арактСристичСским (Π²ΠΎΠ»Π½ΠΎΠ²Ρ‹ΠΌ) сопротивлСниСм ΠΊΠΎΠ½Ρ‚ΡƒΡ€Π° ρ называСтся Π²Π΅Π»ΠΈΡ‡ΠΈΠ½Π° Ρ€Π΅Π°ΠΊΡ‚ΠΈΠ²Π½ΠΎΠ³ΠΎ сопротивлСния Смкости ΠΈ индуктивности ΠΊΠΎΠ½Ρ‚ΡƒΡ€Π° Π½Π° рСзонансной частотС: ρ = Π₯L = Π₯C ΠΏΡ€ΠΈ Ο‰ =Ο‰Ρ€ . Π₯арактСристичСскоС сопротивлСниС ΠΌΠΎΠΆΠ΅Ρ‚ Π±Ρ‹Ρ‚ΡŒ вычислСно ΡΠ»Π΅Π΄ΡƒΡŽΡ‰ΠΈΠΌ ΠΎΠ±Ρ€Π°Π·ΠΎΠΌ: ρ = √(L/C). Π₯арактСристичСскоС сопротивлСниС ρ являСтся количСствСнной ΠΌΠ΅Ρ€ΠΎΠΉ ΠΎΡ†Π΅Π½ΠΊΠΈ энСргии, запасСнной Ρ€Π΅Π°ΠΊΡ‚ΠΈΠ²Π½Ρ‹ΠΌΠΈ элСмСнтами ΠΊΠΎΠ½Ρ‚ΡƒΡ€Π° β€” ΠΊΠ°Ρ‚ΡƒΡˆΠΊΠΎΠΉ (энСргия ΠΌΠ°Π³Π½ΠΈΡ‚Π½ΠΎΠ³ΠΎ поля) WL = (LI 2 )/2 ΠΈ кондСнсатором (энСргия элСктричСского поля) WC=(CU 2 )/2. ΠžΡ‚Π½ΠΎΡˆΠ΅Π½ΠΈΠ΅ энСргии, запасСнной Ρ€Π΅Π°ΠΊΡ‚ΠΈΠ²Π½Ρ‹ΠΌΠΈ элСмСнтами ΠΊΠΎΠ½Ρ‚ΡƒΡ€Π°, ΠΊ энСргии омичСских (рСзистивных) ΠΏΠΎΡ‚Π΅Ρ€ΡŒ Π·Π° ΠΏΠ΅Ρ€ΠΈΠΎΠ΄ принято Π½Π°Π·Ρ‹Π²Π°Ρ‚ΡŒ Π΄ΠΎΠ±Ρ€ΠΎΡ‚Π½ΠΎΡΡ‚ΡŒΡŽ Q ΠΊΠΎΠ½Ρ‚ΡƒΡ€Π°, Ρ‡Ρ‚ΠΎ Π² Π±ΡƒΠΊΠ²Π°Π»ΡŒΠ½ΠΎΠΌ ΠΏΠ΅Ρ€Π΅Π²ΠΎΠ΄Π΅ с английского языка ΠΎΠ±ΠΎΠ·Π½Π°Ρ‡Π°Π΅Ρ‚ «качСство».

Π”ΠΎΠ±Ρ€ΠΎΡ‚Π½ΠΎΡΡ‚ΡŒ ΠΊΠΎΠ»Π΅Π±Π°Ρ‚Π΅Π»ΡŒΠ½ΠΎΠ³ΠΎ ΠΊΠΎΠ½Ρ‚ΡƒΡ€Π° β€” характСристика, ΠΎΠΏΡ€Π΅Π΄Π΅Π»ΡΡŽΡ‰Π°Ρ Π°ΠΌΠΏΠ»ΠΈΡ‚ΡƒΠ΄Ρƒ ΠΈ ΡˆΠΈΡ€ΠΈΠ½Ρƒ АЧΠ₯ рСзонанса ΠΈ ΠΏΠΎΠΊΠ°Π·Ρ‹Π²Π°ΡŽΡ‰Π°Ρ, Π²ΠΎ сколько Ρ€Π°Π· запасы энСргии Π² ΠΊΠΎΠ½Ρ‚ΡƒΡ€Π΅ большС, Ρ‡Π΅ΠΌ ΠΏΠΎΡ‚Π΅Ρ€ΠΈ энСргии Π·Π° ΠΎΠ΄ΠΈΠ½ ΠΏΠ΅Ρ€ΠΈΠΎΠ΄ ΠΊΠΎΠ»Π΅Π±Π°Π½ΠΈΠΉ. Π”ΠΎΠ±Ρ€ΠΎΡ‚Π½ΠΎΡΡ‚ΡŒ ΡƒΡ‡ΠΈΡ‚Ρ‹Π²Π°Π΅Ρ‚ Π½Π°Π»ΠΈΡ‡ΠΈΠ΅ Π°ΠΊΡ‚ΠΈΠ²Π½ΠΎΠ³ΠΎ сопротивлСния Π½Π°Π³Ρ€ΡƒΠ·ΠΊΠΈ R.

Для ΠΏΠΎΡΠ»Π΅Π΄ΠΎΠ²Π°Ρ‚Π΅Π»ΡŒΠ½ΠΎΠ³ΠΎ ΠΊΠΎΠ»Π΅Π±Π°Ρ‚Π΅Π»ΡŒΠ½ΠΎΠ³ΠΎ ΠΊΠΎΠ½Ρ‚ΡƒΡ€Π° Π² RLC цСпях, Π² ΠΊΠΎΡ‚ΠΎΡ€ΠΎΠΌ всС Ρ‚Ρ€ΠΈ элСмСнта Π²ΠΊΠ»ΡŽΡ‡Π΅Π½Ρ‹ ΠΏΠΎΡΠ»Π΅Π΄ΠΎΠ²Π°Ρ‚Π΅Π»ΡŒΠ½ΠΎ, Π΄ΠΎΠ±Ρ€ΠΎΡ‚Π½ΠΎΡΡ‚ΡŒ вычисляСтся:

Π³Π΄Π΅ R, L ΠΈ C β€” сопротивлСниС, ΠΈΠ½Π΄ΡƒΠΊΡ‚ΠΈΠ²Π½ΠΎΡΡ‚ΡŒ ΠΈ Ρ‘ΠΌΠΊΠΎΡΡ‚ΡŒ рСзонансной Ρ†Π΅ΠΏΠΈ, соотвСтствСнно.

Π’Π΅Π»ΠΈΡ‡ΠΈΠ½Ρƒ, ΠΎΠ±Ρ€Π°Ρ‚Π½ΡƒΡŽ добротности d = 1 / Q Π½Π°Π·Ρ‹Π²Π°ΡŽΡ‚ Π·Π°Ρ‚ΡƒΡ…Π°Π½ΠΈΠ΅ΠΌ ΠΊΠΎΠ½Ρ‚ΡƒΡ€Π°. Для опрСдСлСния добротности ΠΎΠ±Ρ‹Ρ‡Π½ΠΎ ΠΏΠΎΠ»ΡŒΠ·ΡƒΡŽΡ‚ΡΡ Ρ„ΠΎΡ€ΠΌΡƒΠ»ΠΎΠΉ Q = ρ / R, Π³Π΄Π΅ R-сопротивлСниС омичСских ΠΏΠΎΡ‚Π΅Ρ€ΡŒ ΠΊΠΎΠ½Ρ‚ΡƒΡ€Π°, Ρ…Π°Ρ€Π°ΠΊΡ‚Π΅Ρ€ΠΈΠ·ΡƒΡŽΡ‰Π΅Π΅ ΠΌΠΎΡ‰Π½ΠΎΡΡ‚ΡŒ рСзистивных (Π°ΠΊΡ‚ΠΈΠ²Π½Ρ‹Ρ… ΠΏΠΎΡ‚Π΅Ρ€ΡŒ) ΠΊΠΎΠ½Ρ‚ΡƒΡ€Π° Π  = I 2 R. Π”ΠΎΠ±Ρ€ΠΎΡ‚Π½ΠΎΡΡ‚ΡŒ Ρ€Π΅Π°Π»ΡŒΠ½Ρ‹Ρ… ΠΊΠΎΠ»Π΅Π±Π°Ρ‚Π΅Π»ΡŒΠ½Ρ‹Ρ… ΠΊΠΎΠ½Ρ‚ΡƒΡ€ΠΎΠ², Π²Ρ‹ΠΏΠΎΠ»Π½Π΅Π½Π½Ρ‹Ρ… Π½Π° дискрСтных ΠΊΠ°Ρ‚ΡƒΡˆΠΊΠ°Ρ… индуктивности ΠΈ кондСнсаторах, составляСт ΠΎΡ‚ Π½Π΅ΡΠΊΠΎΠ»ΡŒΠΊΠΈΡ… Π΅Π΄ΠΈΠ½ΠΈΡ† Π΄ΠΎ сотни ΠΈ Π±ΠΎΠ»Π΅Π΅. Π”ΠΎΠ±Ρ€ΠΎΡ‚Π½ΠΎΡΡ‚ΡŒ Ρ€Π°Π·Π»ΠΈΡ‡Π½Ρ‹Ρ… ΠΊΠΎΠ»Π΅Π±Π°Ρ‚Π΅Π»ΡŒΠ½Ρ‹Ρ… систСм, построСнных Π½Π° ΠΏΡ€ΠΈΠ½Ρ†ΠΈΠΏΠ΅ ΠΏΡŒΠ΅Π·ΠΎΡΠ»Π΅ΠΊΡ‚Ρ€ΠΈΡ‡Π΅ΡΠΊΠΈΡ… ΠΈ Π΄Ρ€ΡƒΠ³ΠΈΡ… эффСктов (Π½Π°ΠΏΡ€ΠΈΠΌΠ΅Ρ€, ΠΊΠ²Π°Ρ€Ρ†Π΅Π²Ρ‹Π΅ Ρ€Π΅Π·ΠΎΠ½Π°Ρ‚ΠΎΡ€Ρ‹) ΠΌΠΎΠΆΠ΅Ρ‚ Π΄ΠΎΡΡ‚ΠΈΠ³Π°Ρ‚ΡŒ Π½Π΅ΡΠΊΠΎΠ»ΡŒΠΊΠΈΡ… тысяч ΠΈ Π±ΠΎΠ»Π΅Π΅.

ЧастотныС свойства Ρ€Π°Π·Π»ΠΈΡ‡Π½Ρ‹Ρ… Ρ†Π΅ΠΏΠ΅ΠΉ Π² Ρ‚Π΅Ρ…Π½ΠΈΠΊΠ΅ принято ΠΎΡ†Π΅Π½ΠΈΠ²Π°Ρ‚ΡŒ с ΠΏΠΎΠΌΠΎΡ‰ΡŒΡŽ Π°ΠΌΠΏΠ»ΠΈΡ‚ΡƒΠ΄Π½ΠΎ-частотных характСристик (АЧΠ₯), ΠΏΡ€ΠΈ этом сами Ρ†Π΅ΠΏΠΈ Ρ€Π°ΡΡΠΌΠ°Ρ‚Ρ€ΠΈΠ²Π°ΡŽΡ‚ ΠΊΠ°ΠΊ Ρ‡Π΅Ρ‚Ρ‹Ρ€Ρ‘Ρ…ΠΏΠΎΠ»ΡŽΡΠ½ΠΈΠΊΠΈ. На рисунках Π½ΠΈΠΆΠ΅ прСдставлСны Π΄Π²Π° ΠΏΡ€ΠΎΡΡ‚Π΅ΠΉΡˆΠΈΡ… Ρ‡Π΅Ρ‚Ρ‹Ρ€Π΅Ρ…ΠΏΠΎΠ»ΡŽΡΠ½ΠΈΠΊΠ°, содСрТащих ΠΏΠΎΡΠ»Π΅Π΄ΠΎΠ²Π°Ρ‚Π΅Π»ΡŒΠ½Ρ‹ΠΉ ΠΊΠΎΠ»Π΅Π±Π°Ρ‚Π΅Π»ΡŒΠ½Ρ‹ΠΉ ΠΊΠΎΠ½Ρ‚ΡƒΡ€ ΠΈ АЧΠ₯ этих Ρ†Π΅ΠΏΠ΅ΠΉ, ΠΊΠΎΡ‚ΠΎΡ€Ρ‹Π΅ ΠΏΡ€ΠΈΠ²Π΅Π΄Π΅Π½Ρ‹ (ΠΏΠΎΠΊΠ°Π·Π°Π½Ρ‹ ΡΠΏΠ»ΠΎΡˆΠ½Ρ‹ΠΌΠΈ линями). По Π²Π΅Ρ€Ρ‚ΠΈΠΊΠ°Π»ΡŒΠ½ΠΎΠΉ оси Π³Ρ€Π°Ρ„ΠΈΠΊΠΎΠ² АЧΠ₯ ΠΎΡ‚Π»ΠΎΠΆΠ΅Π½Π° Π²Π΅Π»ΠΈΡ‡ΠΈΠ½Π° коэффициСнта ΠΏΠ΅Ρ€Π΅Π΄Π°Ρ‡ΠΈ Ρ†Π΅ΠΏΠΈ ΠΏΠΎ Π½Π°ΠΏΡ€ΡΠΆΠ΅Π½ΠΈΡŽ К, ΠΏΠΎΠΊΠ°Π·Ρ‹Π²Π°ΡŽΡ‰Π°Ρ ΠΎΡ‚Π½ΠΎΡˆΠ΅Π½ΠΈΠ΅ Π²Ρ‹Ρ…ΠΎΠ΄Π½ΠΎΠ³ΠΎ напряТСния Ρ†Π΅ΠΏΠΈ ΠΊ Π²Ρ…ΠΎΠ΄Π½ΠΎΠΌΡƒ.

Для пассивных Ρ†Π΅ΠΏΠ΅ΠΉ (Ρ‚.Π΅. Π½Π΅ содСрТащих ΡƒΡΠΈΠ»ΠΈΡ‚Π΅Π»ΡŒΠ½Ρ‹Ρ… элСмСнтов ΠΈ источников энСргии), Π²Π΅Π»ΠΈΡ‡ΠΈΠ½Π° К Π½ΠΈΠΊΠΎΠ³Π΄Π° Π½Π΅ ΠΏΡ€Π΅Π²Ρ‹ΡˆΠ°Π΅Ρ‚ Π΅Π΄ΠΈΠ½ΠΈΡ†Ρƒ. Π‘ΠΎΠΏΡ€ΠΎΡ‚ΠΈΠ²Π»Π΅Π½ΠΈΠ΅ ΠΏΠ΅Ρ€Π΅ΠΌΠ΅Π½Π½ΠΎΠΌΡƒ Ρ‚ΠΎΠΊΡƒ ΠΈΠ·ΠΎΠ±Ρ€Π°ΠΆΡ‘Π½Π½ΠΎΠΉ Π½Π° рисункС Ρ†Π΅ΠΏΠΈ, Π±ΡƒΠ΄Π΅Ρ‚ минимально ΠΏΡ€ΠΈ частотС воздСйствия, Ρ€Π°Π²Π½ΠΎΠΉ рСзонансной частотС ΠΊΠΎΠ½Ρ‚ΡƒΡ€Π°. Π’ этом случаС коэффициСнт ΠΏΠ΅Ρ€Π΅Π΄Π°Ρ‡ΠΈ Ρ†Π΅ΠΏΠΈ Π±Π»ΠΈΠ·ΠΎΠΊ ΠΊ Π΅Π΄ΠΈΠ½ΠΈΡ†Π΅ (опрСдСляСтся омичСскими потСрями Π² ΠΊΠΎΠ½Ρ‚ΡƒΡ€Π΅). На частотах, сильно ΠΎΡ‚Π»ΠΈΡ‡Π°ΡŽΡ‰ΠΈΡ…ΡΡ ΠΎΡ‚ рСзонансной, сопротивлСниС ΠΊΠΎΠ½Ρ‚ΡƒΡ€Π° ΠΏΠ΅Ρ€Π΅ΠΌΠ΅Π½Π½ΠΎΠΌΡƒ Ρ‚ΠΎΠΊΡƒ достаточно Π²Π΅Π»ΠΈΠΊΠΎ, Π° ΡΠ»Π΅Π΄ΠΎΠ²Π°Ρ‚Π΅Π»ΡŒΠ½ΠΎ, ΠΈ коэффициСнт ΠΏΠ΅Ρ€Π΅Π΄Π°Ρ‡ΠΈ Ρ†Π΅ΠΏΠΈ Π±ΡƒΠ΄Π΅Ρ‚ ΠΏΠ°Π΄Π°Ρ‚ΡŒ практичСски Π΄ΠΎ нуля.

ΠŸΡ€ΠΈ рСзонансС Π² этой Ρ†Π΅ΠΏΠΈ, источник Π²Ρ…ΠΎΠ΄Π½ΠΎΠ³ΠΎ сигнала оказываСтся фактичСски Π·Π°ΠΌΠΊΠ½ΡƒΡ‚Ρ‹ΠΌ Π½Π°ΠΊΠΎΡ€ΠΎΡ‚ΠΊΠΎ ΠΌΠ°Π»Ρ‹ΠΌ сопротивлСниСм ΠΊΠΎΠ½Ρ‚ΡƒΡ€Π°, благодаря Ρ‡Π΅ΠΌΡƒ коэффициСнт ΠΏΠ΅Ρ€Π΅Π΄Π°Ρ‡ΠΈ Ρ‚Π°ΠΊΠΎΠΉ Ρ†Π΅ΠΏΠΈ Π½Π° рСзонансной частотС ΠΏΠ°Π΄Π°Π΅Ρ‚ практичСски Π΄ΠΎ нуля (ΠΎΠΏΡΡ‚ΡŒ-Ρ‚Π°ΠΊΠΈ Π² силу наличия ΠΊΠΎΠ½Π΅Ρ‡Π½ΠΎΠ³ΠΎ сопротивлСния ΠΏΠΎΡ‚Π΅Ρ€ΡŒ). Наоборот, ΠΏΡ€ΠΈ частотах Π²Ρ…ΠΎΠ΄Π½ΠΎΠ³ΠΎ воздСйствия, Π·Π½Π°Ρ‡ΠΈΡ‚Π΅Π»ΡŒΠ½ΠΎ отстоящих ΠΎΡ‚ рСзонансной, коэффициСнт ΠΏΠ΅Ρ€Π΅Π΄Π°Ρ‡ΠΈ Ρ†Π΅ΠΏΠΈ оказываСтся Π±Π»ΠΈΠ·ΠΊΠΈΠΌ ΠΊ Π΅Π΄ΠΈΠ½ΠΈΡ†Π΅. Бвойство ΠΊΠΎΠ»Π΅Π±Π°Ρ‚Π΅Π»ΡŒΠ½ΠΎΠ³ΠΎ ΠΊΠΎΠ½Ρ‚ΡƒΡ€Π° Π² Π·Π½Π°Ρ‡ΠΈΡ‚Π΅Π»ΡŒΠ½ΠΎΠΉ стСпСни ΠΈΠ·ΠΌΠ΅Π½ΡΡ‚ΡŒ коэффициСнт ΠΏΠ΅Ρ€Π΅Π΄Π°Ρ‡ΠΈ Π½Π° частотах, Π±Π»ΠΈΠ·ΠΊΠΈΡ… ΠΊ рСзонансной, ΡˆΠΈΡ€ΠΎΠΊΠΎ ΠΈΡΠΏΠΎΠ»ΡŒΠ·ΡƒΠ΅Ρ‚ΡΡ Π½Π° ΠΏΡ€Π°ΠΊΡ‚ΠΈΠΊΠ΅, ΠΊΠΎΠ³Π΄Π° трСбуСтся Π²Ρ‹Π΄Π΅Π»ΠΈΡ‚ΡŒ сигнал с ΠΊΠΎΠ½ΠΊΡ€Π΅Ρ‚Π½ΠΎΠΉ частотой ΠΈΠ· мноТСства Π½Π΅Π½ΡƒΠΆΠ½Ρ‹Ρ… сигналов, располоТСнных Π½Π° Π΄Ρ€ΡƒΠ³ΠΈΡ… частотах. Π’Π°ΠΊ, Π² любом Ρ€Π°Π΄ΠΈΠΎΠΏΡ€ΠΈΠ΅ΠΌΠ½ΠΈΠΊΠ΅ ΠΏΡ€ΠΈ ΠΏΠΎΠΌΠΎΡ‰ΠΈ ΠΊΠΎΠ»Π΅Π±Π°Ρ‚Π΅Π»ΡŒΠ½Ρ‹Ρ… Ρ†Π΅ΠΏΠ΅ΠΉ обСспСчиваСтся настройка Π½Π° частоту Π½ΡƒΠΆΠ½ΠΎΠΉ радиостанции. Бвойство ΠΊΠΎΠ»Π΅Π±Π°Ρ‚Π΅Π»ΡŒΠ½ΠΎΠ³ΠΎ ΠΊΠΎΠ½Ρ‚ΡƒΡ€Π° Π²Ρ‹Π΄Π΅Π»ΡΡ‚ΡŒ ΠΈΠ· мноТСства частот ΠΎΠ΄Π½Ρƒ принято Π½Π°Π·Ρ‹Π²Π°Ρ‚ΡŒ ΡΠ΅Π»Π΅ΠΊΡ‚ΠΈΠ²Π½ΠΎΡΡ‚ΡŒΡŽ ΠΈΠ»ΠΈ ΠΈΠ·Π±ΠΈΡ€Π°Ρ‚Π΅Π»ΡŒΠ½ΠΎΡΡ‚ΡŒΡŽ. ΠŸΡ€ΠΈ этом ΠΈΠ½Ρ‚Π΅Π½ΡΠΈΠ²Π½ΠΎΡΡ‚ΡŒ измСнСния коэффициСнта ΠΏΠ΅Ρ€Π΅Π΄Π°Ρ‡ΠΈ Ρ†Π΅ΠΏΠΈ ΠΏΡ€ΠΈ отстройкС частоты воздСйствия ΠΎΡ‚ рСзонанса принято ΠΎΡ†Π΅Π½ΠΈΠ²Π°Ρ‚ΡŒ ΠΏΡ€ΠΈ ΠΏΠΎΠΌΠΎΡ‰ΠΈ ΠΏΠ°Ρ€Π°ΠΌΠ΅Ρ‚Ρ€Π°, Π½Π°Π·Ρ‹Π²Π°Π΅ΠΌΠΎΠ³ΠΎ полосой пропускания. Π—Π° полосу пропускания принимаСтся Π΄ΠΈΠ°ΠΏΠ°Π·ΠΎΠ½ частот, Π² ΠΏΡ€Π΅Π΄Π΅Π»Π°Ρ… ΠΊΠΎΡ‚ΠΎΡ€ΠΎΠ³ΠΎ ΡƒΠΌΠ΅Π½ΡŒΡˆΠ΅Π½ΠΈΠ΅ (ΠΈΠ»ΠΈ ΡƒΠ²Π΅Π»ΠΈΡ‡Π΅Π½ΠΈΠ΅ β€” Π² зависимости ΠΎΡ‚ Π²ΠΈΠ΄Π° Ρ†Π΅ΠΏΠΈ) коэффициСнта ΠΏΠ΅Ρ€Π΅Π΄Π°Ρ‡ΠΈ ΠΎΡ‚Π½ΠΎΡΠΈΡ‚Π΅Π»ΡŒΠ½ΠΎ Π΅Π³ΠΎ значСния Π½Π° рСзонансной частотС, Π½Π΅ ΠΏΡ€Π΅Π²Ρ‹ΡˆΠ°Π΅Ρ‚ Π²Π΅Π»ΠΈΡ‡ΠΈΠ½Ρ‹ 0,7 (3Π΄Π‘).

ΠŸΡƒΠ½ΠΊΡ‚ΠΈΡ€Π½Ρ‹ΠΌΠΈ линиями Π½Π° Π³Ρ€Π°Ρ„ΠΈΠΊΠ°Ρ… ΠΏΠΎΠΊΠ°Π·Π°Π½Ρ‹ АЧΠ₯ Ρ‚ΠΎΡ‡Π½ΠΎ Ρ‚Π°ΠΊΠΈΡ… ΠΆΠ΅ Ρ†Π΅ΠΏΠ΅ΠΉ, ΠΊΠΎΠ»Π΅Π±Π°Ρ‚Π΅Π»ΡŒΠ½Ρ‹Π΅ ΠΊΠΎΠ½Ρ‚ΡƒΡ€Ρ‹ ΠΊΠΎΡ‚ΠΎΡ€Ρ‹Ρ… ΠΈΠΌΠ΅ΡŽΡ‚ Ρ‚Π°ΠΊΠΈΠ΅ ΠΆΠ΅ рСзонансныС частоты, ΠΊΠ°ΠΊ ΠΈ для случая рассмотрСнного Π²Ρ‹ΡˆΠ΅, Π½ΠΎ ΠΎΠ±Π»Π°Π΄Π°ΡŽΡ‰ΠΈΠ΅ мСньшСй Π΄ΠΎΠ±Ρ€ΠΎΡ‚Π½ΠΎΡΡ‚ΡŒΡŽ (Π½Π°ΠΏΡ€ΠΈΠΌΠ΅Ρ€, ΠΊΠ°Ρ‚ΡƒΡˆΠΊΠ° индуктивности Π½Π°ΠΌΠΎΡ‚Π°Π½Π° ΠΏΡ€ΠΎΠ²ΠΎΠ΄ΠΎΠΌ, ΠΎΠ±Π»Π°Π΄Π°ΡŽΡ‰ΠΈΠΌ большим сопротивлСниСм постоянному Ρ‚ΠΎΠΊΡƒ). Как Π²ΠΈΠ΄Π½ΠΎ ΠΈΠ· рисунков, ΠΏΡ€ΠΈ этом Ρ€Π°ΡΡˆΠΈΡ€ΡΠ΅Ρ‚ΡΡ полоса пропускания Ρ†Π΅ΠΏΠΈ ΠΈ ΡƒΡ…ΡƒΠ΄ΡˆΠ°ΡŽΡ‚ΡΡ Π΅Π΅ сСлСктивныС (ΠΈΠ·Π±ΠΈΡ€Π°Ρ‚Π΅Π»ΡŒΠ½Ρ‹Π΅) свойства. Π˜ΡΡ…ΠΎΠ΄Ρ ΠΈΠ· этого, ΠΏΡ€ΠΈ расчСтС ΠΈ конструировании ΠΊΠΎΠ»Π΅Π±Π°Ρ‚Π΅Π»ΡŒΠ½Ρ‹Ρ… ΠΊΠΎΠ½Ρ‚ΡƒΡ€ΠΎΠ² Π½ΡƒΠΆΠ½ΠΎ ΡΡ‚Ρ€Π΅ΠΌΠΈΡ‚ΡŒΡΡ ΠΊ ΠΏΠΎΠ²Ρ‹ΡˆΠ΅Π½ΠΈΡŽ ΠΈΡ… добротности. Однако, Π² рядС случаСв, Π΄ΠΎΠ±Ρ€ΠΎΡ‚Π½ΠΎΡΡ‚ΡŒ ΠΊΠΎΠ½Ρ‚ΡƒΡ€Π°, Π½Π°ΠΎΠ±ΠΎΡ€ΠΎΡ‚, приходится Π·Π°Π½ΠΈΠΆΠ°Ρ‚ΡŒ (Π½Π°ΠΏΡ€ΠΈΠΌΠ΅Ρ€, Π²ΠΊΠ»ΡŽΡ‡Π°Ρ ΠΏΠΎΡΠ»Π΅Π΄ΠΎΠ²Π°Ρ‚Π΅Π»ΡŒΠ½ΠΎ с ΠΊΠ°Ρ‚ΡƒΡˆΠΊΠΎΠΉ индуктивности рСзистор нСбольшой Π²Π΅Π»ΠΈΡ‡ΠΈΠ½Ρ‹ сопротивлСния), Ρ‡Ρ‚ΠΎ позволяСт ΠΈΠ·Π±Π΅ΠΆΠ°Ρ‚ΡŒ искаТСний ΡˆΠΈΡ€ΠΎΠΊΠΎΠΏΠΎΠ»ΠΎΡΠ½Ρ‹Ρ… сигналов. Π₯отя, Ссли Π½Π° ΠΏΡ€Π°ΠΊΡ‚ΠΈΠΊΠ΅ трСбуСтся Π²Ρ‹Π΄Π΅Π»ΠΈΡ‚ΡŒ достаточно ΡˆΠΈΡ€ΠΎΠΊΠΎΠΏΠΎΠ»ΠΎΡΠ½Ρ‹ΠΉ сигнал, сСлСктивныС Ρ†Π΅ΠΏΠΈ, ΠΊΠ°ΠΊ ΠΏΡ€Π°Π²ΠΈΠ»ΠΎ, строятся Π½Π΅ Π½Π° ΠΎΠ΄ΠΈΠ½ΠΎΡ‡Π½Ρ‹Ρ… ΠΊΠΎΠ»Π΅Π±Π°Ρ‚Π΅Π»ΡŒΠ½Ρ‹Ρ… ΠΊΠΎΠ½Ρ‚ΡƒΡ€Π°Ρ…, Π° Π½Π° Π±ΠΎΠ»Π΅Π΅ слоТных связанных (ΠΌΠ½ΠΎΠ³ΠΎΠΊΠΎΠ½Ρ‚ΡƒΡ€Π½Ρ‹Ρ…) ΠΊΠΎΠ»Π΅Π±Π°Ρ‚Π΅Π»ΡŒΠ½Ρ‹Ρ… систСмах, Π² Ρ‚.Ρ‡. ΠΌΠ½ΠΎΠ³ΠΎΠ·Π²Π΅Π½Π½Ρ‹Ρ… Ρ„ΠΈΠ»ΡŒΡ‚Ρ€Π°Ρ….

ΠŸΠ°Ρ€Π°Π»Π»Π΅Π»ΡŒΠ½Ρ‹ΠΉ ΠΊΠΎΠ»Π΅Π±Π°Ρ‚Π΅Π»ΡŒΠ½Ρ‹ΠΉ ΠΊΠΎΠ½Ρ‚ΡƒΡ€

Π’ Ρ€Π°Π·Π»ΠΈΡ‡Π½Ρ‹Ρ… радиотСхничСских устройствах наряду с ΠΏΠΎΡΠ»Π΅Π΄ΠΎΠ²Π°Ρ‚Π΅Π»ΡŒΠ½Ρ‹ΠΌΠΈ ΠΊΠΎΠ»Π΅Π±Π°Ρ‚Π΅Π»ΡŒΠ½Ρ‹ΠΌΠΈ ΠΊΠΎΠ½Ρ‚ΡƒΡ€Π°ΠΌΠΈ часто (Π΄Π°ΠΆΠ΅ Ρ‡Π°Ρ‰Π΅, Ρ‡Π΅ΠΌ ΠΏΠΎΡΠ»Π΅Π΄ΠΎΠ²Π°Ρ‚Π΅Π»ΡŒΠ½Ρ‹Π΅) ΠΏΡ€ΠΈΠΌΠ΅Π½ΡΡŽΡ‚ ΠΏΠ°Ρ€Π°Π»Π»Π΅Π»ΡŒΠ½Ρ‹Π΅ ΠΊΠΎΠ»Π΅Π±Π°Ρ‚Π΅Π»ΡŒΠ½Ρ‹Π΅ ΠΊΠΎΠ½Ρ‚ΡƒΡ€Ρ‹ На рисункС ΠΏΡ€ΠΈΠ²Π΅Π΄Π΅Π½Π° ΠΏΡ€ΠΈΠ½Ρ†ΠΈΠΏΠΈΠ°Π»ΡŒΠ½Π°Ρ схСма ΠΏΠ°Ρ€Π°Π»Π»Π΅Π»ΡŒΠ½ΠΎΠ³ΠΎ ΠΊΠΎΠ»Π΅Π±Π°Ρ‚Π΅Π»ΡŒΠ½ΠΎΠ³ΠΎ ΠΊΠΎΠ½Ρ‚ΡƒΡ€Π°. Π—Π΄Π΅ΡΡŒ ΠΏΠ°Ρ€Π°Π»Π»Π΅Π»ΡŒΠ½ΠΎ Π²ΠΊΠ»ΡŽΡ‡Π΅Π½Ρ‹ Π΄Π²Π° Ρ€Π΅Π°ΠΊΡ‚ΠΈΠ²Π½Ρ‹Ρ… элСмСнта с Ρ€Π°Π·Π½Ρ‹ΠΌ Ρ…Π°Ρ€Π°ΠΊΡ‚Π΅Ρ€ΠΎΠΌ рСактивности Как извСстно, ΠΏΡ€ΠΈ ΠΏΠ°Ρ€Π°Π»Π»Π΅Π»ΡŒΠ½ΠΎΠΌ Π²ΠΊΠ»ΡŽΡ‡Π΅Π½ΠΈΠΈ элСмСнтов ΡΠΊΠ»Π°Π΄Ρ‹Π²Π°Ρ‚ΡŒ ΠΈΡ… сопротивлСния нСльзя β€” ΠΌΠΎΠΆΠ½ΠΎ лишь ΡΠΊΠ»Π°Π΄Ρ‹Π²Π°Ρ‚ΡŒ проводимости. На рисункС ΠΏΡ€ΠΈΠ²Π΅Π΄Π΅Π½Ρ‹ графичСскиС зависимости Ρ€Π΅Π°ΠΊΡ‚ΠΈΠ²Π½Ρ‹Ρ… проводимостСй ΠΊΠ°Ρ‚ΡƒΡˆΠΊΠΈ индуктивности BL = 1/Ο‰L, кондСнсатора Π’C = -Ο‰C, Π° Ρ‚Π°ΠΊΠΆΠ΅ суммарной проводимости Π’Ξ£, этих Π΄Π²ΡƒΡ… элСмСнтов, ΡΠ²Π»ΡΡŽΡ‰Π°ΡΡΡ Ρ€Π΅Π°ΠΊΡ‚ΠΈΠ²Π½ΠΎΠΉ ΠΏΡ€ΠΎΠ²ΠΎΠ΄ΠΈΠΌΠΎΡΡ‚ΡŒΡŽ ΠΏΠ°Ρ€Π°Π»Π»Π΅Π»ΡŒΠ½ΠΎΠ³ΠΎ ΠΊΠΎΠ»Π΅Π±Π°Ρ‚Π΅Π»ΡŒΠ½ΠΎΠ³ΠΎ ΠΊΠΎΠ½Ρ‚ΡƒΡ€Π°. Аналогично, ΠΊΠ°ΠΊ ΠΈ для ΠΏΠΎΡΠ»Π΅Π΄ΠΎΠ²Π°Ρ‚Π΅Π»ΡŒΠ½ΠΎΠ³ΠΎ ΠΊΠΎΠ»Π΅Π±Π°Ρ‚Π΅Π»ΡŒΠ½ΠΎΠ³ΠΎ ΠΊΠΎΠ½Ρ‚ΡƒΡ€Π°, имССтся нСкоторая частота, называСмая рСзонансной, Π½Π° ΠΊΠΎΡ‚ΠΎΡ€ΠΎΠΉ Ρ€Π΅Π°ΠΊΡ‚ΠΈΠ²Π½Ρ‹Π΅ сопротивлСния (Π° Π·Π½Π°Ρ‡ΠΈΡ‚ ΠΈ проводимости) ΠΊΠ°Ρ‚ΡƒΡˆΠΊΠΈ ΠΈ кондСнсатора ΠΎΠ΄ΠΈΠ½Π°ΠΊΠΎΠ²Ρ‹. На этой частотС суммарная ΠΏΡ€ΠΎΠ²ΠΎΠ΄ΠΈΠΌΠΎΡΡ‚ΡŒ ΠΏΠ°Ρ€Π°Π»Π»Π΅Π»ΡŒΠ½ΠΎΠ³ΠΎ ΠΊΠΎΠ»Π΅Π±Π°Ρ‚Π΅Π»ΡŒΠ½ΠΎΠ³ΠΎ ΠΊΠΎΠ½Ρ‚ΡƒΡ€Π° Π±Π΅Π· ΠΏΠΎΡ‚Π΅Ρ€ΡŒ обращаСтся Π² Π½ΡƒΠ»ΡŒ. Π­Ρ‚ΠΎ Π·Π½Π°Ρ‡ΠΈΡ‚, Ρ‡Ρ‚ΠΎ Π½Π° этой частотС ΠΊΠΎΠ»Π΅Π±Π°Ρ‚Π΅Π»ΡŒΠ½Ρ‹ΠΉ ΠΊΠΎΠ½Ρ‚ΡƒΡ€ ΠΎΠ±Π»Π°Π΄Π°Π΅Ρ‚ бСсконСчно большим сопротивлСниСм ΠΏΠ΅Ρ€Π΅ΠΌΠ΅Π½Π½ΠΎΠΌΡƒ Ρ‚ΠΎΠΊΡƒ.

Если ΠΏΠΎΡΡ‚Ρ€ΠΎΠΈΡ‚ΡŒ Π·Π°Π²ΠΈΡΠΈΠΌΠΎΡΡ‚ΡŒ Ρ€Π΅Π°ΠΊΡ‚ΠΈΠ²Π½ΠΎΠ³ΠΎ сопротивлСния ΠΊΠΎΠ½Ρ‚ΡƒΡ€Π° ΠΎΡ‚ частоты XΞ£ = 1/BΞ£, эта кривая, изобраТённая Π½Π° ΡΠ»Π΅Π΄ΡƒΡŽΡ‰Π΅ΠΌ рисункС, Π² Ρ‚ΠΎΡ‡ΠΊΠ΅ Ο‰ = Ο‰Ρ€ Π±ΡƒΠ΄Π΅Ρ‚ ΠΈΠΌΠ΅Ρ‚ΡŒ Ρ€Π°Π·Ρ€Ρ‹Π² Π²Ρ‚ΠΎΡ€ΠΎΠ³ΠΎ Ρ€ΠΎΠ΄Π°. Π‘ΠΎΠΏΡ€ΠΎΡ‚ΠΈΠ²Π»Π΅Π½ΠΈΠ΅ Ρ€Π΅Π°Π»ΡŒΠ½ΠΎΠ³ΠΎ ΠΏΠ°Ρ€Π°Π»Π»Π΅Π»ΡŒΠ½ΠΎΠ³ΠΎ ΠΊΠΎΠ»Π΅Π±Π°Ρ‚Π΅Π»ΡŒΠ½ΠΎΠ³ΠΎ ΠΊΠΎΠ½Ρ‚ΡƒΡ€Π° (Ρ‚.Π΅ с потСрями), разумССтся, Π½Π΅ Ρ€Π°Π²Π½ΠΎ бСсконСчности β€” ΠΎΠ½ΠΎ Ρ‚Π΅ΠΌ мСньшС, Ρ‡Π΅ΠΌ большС омичСскоС сопротивлСниС ΠΏΠΎΡ‚Π΅Ρ€ΡŒ Π² ΠΊΠΎΠ½Ρ‚ΡƒΡ€Π΅, Ρ‚.Π΅ ΡƒΠΌΠ΅Π½ΡŒΡˆΠ°Π΅Ρ‚ΡΡ прямо ΠΏΡ€ΠΎΠΏΠΎΡ€Ρ†ΠΈΠΎΠ½Π°Π»ΡŒΠ½ΠΎ ΡƒΠΌΠ΅Π½ΡŒΡˆΠ΅Π½ΠΈΡŽ добротности ΠΊΠΎΠ½Ρ‚ΡƒΡ€Π°. Π’ Ρ†Π΅Π»ΠΎΠΌ, физичСский смысл понятий добротности, характСристичСского сопротивлСния ΠΈ рСзонансной частоты ΠΊΠΎΠ»Π΅Π±Π°Ρ‚Π΅Π»ΡŒΠ½ΠΎΠ³ΠΎ ΠΊΠΎΠ½Ρ‚ΡƒΡ€Π°, Π° Ρ‚Π°ΠΊΠΆΠ΅ ΠΈΡ… расчСтныС Ρ„ΠΎΡ€ΠΌΡƒΠ»Ρ‹, справСдливы ΠΊΠ°ΠΊ для ΠΏΠΎΡΠ»Π΅Π΄ΠΎΠ²Π°Ρ‚Π΅Π»ΡŒΠ½ΠΎΠ³ΠΎ, Ρ‚Π°ΠΊ ΠΈ для ΠΏΠ°Ρ€Π°Π»Π»Π΅Π»ΡŒΠ½ΠΎΠ³ΠΎ ΠΊΠΎΠ»Π΅Π±Π°Ρ‚Π΅Π»ΡŒΠ½ΠΎΠ³ΠΎ ΠΊΠΎΠ½Ρ‚ΡƒΡ€Π°.

Для ΠΏΠ°Ρ€Π°Π»Π»Π΅Π»ΡŒΠ½ΠΎΠ³ΠΎ ΠΊΠΎΠ»Π΅Π±Π°Ρ‚Π΅Π»ΡŒΠ½ΠΎΠ³ΠΎ ΠΊΠΎΠ½Ρ‚ΡƒΡ€Π°, Π² ΠΊΠΎΡ‚ΠΎΡ€ΠΎΠΌ ΠΈΠ½Π΄ΡƒΠΊΡ‚ΠΈΠ²Π½ΠΎΡΡ‚ΡŒ, Π΅ΠΌΠΊΠΎΡΡ‚ΡŒ ΠΈ сопротивлСниС Π²ΠΊΠ»ΡŽΡ‡Π΅Π½Ρ‹ ΠΏΠ°Ρ€Π°Π»Π»Π΅Π»ΡŒΠ½ΠΎ, Π΄ΠΎΠ±Ρ€ΠΎΡ‚Π½ΠΎΡΡ‚ΡŒ вычисляСтся:

Π³Π΄Π΅ R, L ΠΈ C β€” сопротивлСниС, ΠΈΠ½Π΄ΡƒΠΊΡ‚ΠΈΠ²Π½ΠΎΡΡ‚ΡŒ ΠΈ Ρ‘ΠΌΠΊΠΎΡΡ‚ΡŒ рСзонансной Ρ†Π΅ΠΏΠΈ, соотвСтствСнно.

Рассмотрим Ρ†Π΅ΠΏΡŒ, ΡΠΎΡΡ‚ΠΎΡΡ‰ΡƒΡŽ ΠΈΠ· Π³Π΅Π½Π΅Ρ€Π°Ρ‚ΠΎΡ€Π° гармоничСских ΠΊΠΎΠ»Π΅Π±Π°Π½ΠΈΠΉ ΠΈ ΠΏΠ°Ρ€Π°Π»Π»Π΅Π»ΡŒΠ½ΠΎΠ³ΠΎ ΠΊΠΎΠ»Π΅Π±Π°Ρ‚Π΅Π»ΡŒΠ½ΠΎΠ³ΠΎ ΠΊΠΎΠ½Ρ‚ΡƒΡ€Π°. Π’ случаС, ΠΊΠΎΠ³Π΄Π° частота ΠΊΠΎΠ»Π΅Π±Π°Π½ΠΈΠΉ Π³Π΅Π½Π΅Ρ€Π°Ρ‚ΠΎΡ€Π° совпадаСт с рСзонансной частотой ΠΊΠΎΠ½Ρ‚ΡƒΡ€Π° Π΅Π³ΠΎ индуктивная ΠΈ Смкостная Π²Π΅Ρ‚Π²ΠΈ ΠΎΠΊΠ°Π·Ρ‹Π²Π°ΡŽΡ‚ Ρ€Π°Π²Π½ΠΎΠ΅ сопротивлСниС ΠΏΠ΅Ρ€Π΅ΠΌΠ΅Π½Π½ΠΎΠΌΡƒ Ρ‚ΠΎΠΊΡƒ, Π² слСдствиС Ρ‡Π΅Π³ΠΎ Ρ‚ΠΎΠΊΠΈ Π² вСтвях ΠΊΠΎΠ½Ρ‚ΡƒΡ€Π° Π±ΡƒΠ΄ΡƒΡ‚ ΠΎΠ΄ΠΈΠ½Π°ΠΊΠΎΠ²Ρ‹ΠΌΠΈ. Π’ этом случаС говорят, Ρ‡Ρ‚ΠΎ Π² Ρ†Π΅ΠΏΠΈ ΠΈΠΌΠ΅Π΅Ρ‚ мСсто рСзонанс Ρ‚ΠΎΠΊΠΎΠ². Как ΠΈ Π² случаС ΠΏΠΎΡΠ»Π΅Π΄ΠΎΠ²Π°Ρ‚Π΅Π»ΡŒΠ½ΠΎΠ³ΠΎ ΠΊΠΎΠ»Π΅Π±Π°Ρ‚Π΅Π»ΡŒΠ½ΠΎΠ³ΠΎ ΠΊΠΎΠ½Ρ‚ΡƒΡ€Π°, рСактивности ΠΊΠ°Ρ‚ΡƒΡˆΠΊΠΈ ΠΈ кондСнсатора ΠΊΠΎΠΌΠΏΠ΅Π½ΡΠΈΡ€ΡƒΡŽΡ‚ Π΄Ρ€ΡƒΠ³ Π΄Ρ€ΡƒΠ³Π°, ΠΈ сопротивлСниС ΠΊΠΎΠ½Ρ‚ΡƒΡ€Π° ΠΏΡ€ΠΎΡ‚Π΅ΠΊΠ°ΡŽΡ‰Π΅ΠΌΡƒ Ρ‡Π΅Ρ€Π΅Π· Π½Π΅Π³ΠΎ Ρ‚ΠΎΠΊΡƒ становится чисто Π°ΠΊΡ‚ΠΈΠ²Π½Ρ‹ΠΌ (рСзистивным). Π’Π΅Π»ΠΈΡ‡ΠΈΠ½Π° этого сопротивлСния, часто Π½Π°Π·Ρ‹Π²Π°Π΅ΠΌΠΎΠ³ΠΎ Π² Ρ‚Π΅Ρ…Π½ΠΈΠΊΠ΅ эквивалСнтным, опрСдСляСтся ΠΏΡ€ΠΎΠΈΠ·Π²Π΅Π΄Π΅Π½ΠΈΠ΅ΠΌ добротности ΠΊΠΎΠ½Ρ‚ΡƒΡ€Π° Π½Π° Π΅Π³ΠΎ характСристичСскоС сопротивлСниС Rэкв = Q·ρ. На частотах, ΠΎΡ‚Π»ΠΈΡ‡Π½Ρ‹Ρ… ΠΎΡ‚ рСзонансной, сопротивлСниС ΠΊΠΎΠ½Ρ‚ΡƒΡ€Π° ΡƒΠΌΠ΅Π½ΡŒΡˆΠ°Π΅Ρ‚ΡΡ ΠΈ ΠΏΡ€ΠΈΠΎΠ±Ρ€Π΅Ρ‚Π°Π΅Ρ‚ Ρ€Π΅Π°ΠΊΡ‚ΠΈΠ²Π½Ρ‹ΠΉ Ρ…Π°Ρ€Π°ΠΊΡ‚Π΅Ρ€ Π½Π° Π±ΠΎΠ»Π΅Π΅ Π½ΠΈΠ·ΠΊΠΈΡ… частотах β€” ΠΈΠ½Π΄ΡƒΠΊΡ‚ΠΈΠ²Π½Ρ‹ΠΉ (ΠΏΠΎΡΠΊΠΎΠ»ΡŒΠΊΡƒ Ρ€Π΅Π°ΠΊΡ‚ΠΈΠ²Π½ΠΎΠ΅ сопротивлСниС индуктивности ΠΏΠ°Π΄Π°Π΅Ρ‚ ΠΏΡ€ΠΈ ΡƒΠΌΠ΅Π½ΡŒΡˆΠ΅Π½ΠΈΠΈ частоты), Π° Π½Π° Π±ΠΎΠ»Π΅Π΅ высоких β€” Π½Π°ΠΎΠ±ΠΎΡ€ΠΎΡ‚, Смкостной (Ρ‚ ΠΊ Ρ€Π΅Π°ΠΊΡ‚ΠΈΠ²Π½ΠΎΠ΅ сопротивлСниС Смкости ΠΏΠ°Π΄Π°Π΅Ρ‚ с ростом частоты).

Π’ процСссС Ρ€Π°Π±ΠΎΡ‚Ρ‹ ΠΊΠΎΠ½Ρ‚ΡƒΡ€Π°, Π΄Π²Π°ΠΆΠ΄Ρ‹ Π·Π° ΠΏΠ΅Ρ€ΠΈΠΎΠ΄ ΠΊΠΎΠ»Π΅Π±Π°Π½ΠΈΠΉ, происходит энСргСтичСский ΠΎΠ±ΠΌΠ΅Π½ ΠΌΠ΅ΠΆΠ΄Ρƒ ΠΊΠ°Ρ‚ΡƒΡˆΠΊΠΎΠΉ ΠΈ кондСнсатором (смотри рисунок). ЭнСргия ΠΏΠΎΠΎΡ‡Π΅Ρ€Π΅Π΄Π½ΠΎ накапливаСтся, Ρ‚ΠΎ Π² Π²ΠΈΠ΄Π΅ энСргии элСктричСского поля заряТСнного кондСнсатора, Ρ‚ΠΎ Π² Π²ΠΈΠ΄Π΅ энСргии ΠΌΠ°Π³Π½ΠΈΡ‚Π½ΠΎΠ³ΠΎ поля ΠΊΠ°Ρ‚ΡƒΡˆΠΊΠΈ индуктивности. ΠŸΡ€ΠΈ этом Π² ΠΊΠΎΠ½Ρ‚ΡƒΡ€Π΅ ΠΏΡ€ΠΎΡ‚Π΅ΠΊΠ°Π΅Ρ‚ собствСнный ΠΊΠΎΠ½Ρ‚ΡƒΡ€Π½Ρ‹ΠΉ Ρ‚ΠΎΠΊ IΠΊ, прСвосходящий ΠΏΠΎ Π²Π΅Π»ΠΈΡ‡ΠΈΠ½Π΅ Ρ‚ΠΎΠΊ Π²ΠΎ внСшнСй Ρ†Π΅ΠΏΠΈ I Π² Q Ρ€Π°Π·. Π’ случаС идСального ΠΊΠΎΠ½Ρ‚ΡƒΡ€Π° (Π±Π΅Π· ΠΏΠΎΡ‚Π΅Ρ€ΡŒ), Π΄ΠΎΠ±Ρ€ΠΎΡ‚Π½ΠΎΡΡ‚ΡŒ ΠΊΠΎΡ‚ΠΎΡ€ΠΎΠ³ΠΎ тСорСтичСски бСсконСчна, Π²Π΅Π»ΠΈΡ‡ΠΈΠ½Π° ΠΊΠΎΠ½Ρ‚ΡƒΡ€Π½ΠΎΠ³ΠΎ Ρ‚ΠΎΠΊΠ° Ρ‚Π°ΠΊΠΆΠ΅ Π±ΡƒΠ΄Π΅Ρ‚ бСсконСчно большой. Но Π½Π° ΠΏΡ€Π°ΠΊΡ‚ΠΈΠΊΠ΅, Ρ‚Π°ΠΊΠΎΠ³ΠΎ Π½Π΅ Π±Ρ‹Π²Π°Π΅Ρ‚. Π’ любом случаС, качСство элСмСнтов ΠΊΠΎΠ½Ρ‚ΡƒΡ€Π°, ΠΈΡ… ΠΏΠ°Ρ€Π°Π·ΠΈΡ‚Π½Ρ‹Π΅ характСристики, элСктричСскиС Ρ†Π΅ΠΏΠΈ, слуТащиС для ΠΏΠΎΠ΄Π²ΠΎΠ΄Π° энСргии ΠΈ ΠΎΡ‚Π±ΠΎΡ€Π° энСргии ΠΈΠ· ΠΊΠΎΠ½Ρ‚ΡƒΡ€Π°, Π½Π΅ позволят ΠΊΠΎΠ½Ρ‚ΡƒΡ€Π½ΠΎΠΌΡƒ Ρ‚ΠΎΠΊΡƒ расти.

Рассмотрим, ΠΊΠ°ΠΊ зависят коэффициСнты ΠΏΠ΅Ρ€Π΅Π΄Π°Ρ‡ΠΈ Ρ‡Π΅Ρ‚Ρ‹Ρ€Π΅Ρ…ΠΏΠΎΠ»ΡŽΡΠ½ΠΈΠΊΠΎΠ² ΠΎΡ‚ частоты, ΠΏΡ€ΠΈ Π²ΠΊΠ»ΡŽΡ‡Π΅Π½ΠΈΠΈ Π² Π½ΠΈΡ… Π½Π΅ ΠΏΠΎΡΠ»Π΅Π΄ΠΎΠ²Π°Ρ‚Π΅Π»ΡŒΠ½Ρ‹Ρ… ΠΊΠΎΠ»Π΅Π±Π°Ρ‚Π΅Π»ΡŒΠ½Ρ‹Ρ… ΠΊΠΎΠ½Ρ‚ΡƒΡ€ΠΎΠ², Π° ΠΏΠ°Ρ€Π°Π»Π»Π΅Π»ΡŒΠ½Ρ‹Ρ….

Π§Π΅Ρ‚Ρ‹Ρ€Π΅Ρ…ΠΏΠΎΠ»ΡŽΡΠ½ΠΈΠΊ, ΠΈΠ·ΠΎΠ±Ρ€Π°ΠΆΠ΅Π½Π½Ρ‹ΠΉ Π½Π° рисункС, Π½Π° рСзонансной частотС ΠΊΠΎΠ½Ρ‚ΡƒΡ€Π° прСдставляСт собой ΠΎΠ³Ρ€ΠΎΠΌΠ½ΠΎΠ΅ сопротивлСниС Ρ‚ΠΎΠΊΡƒ, поэтому ΠΏΡ€ΠΈ Ο‰=Ο‰Ρ€ Π΅Π³ΠΎ коэффициСнт ΠΏΠ΅Ρ€Π΅Π΄Π°Ρ‡ΠΈ Π±ΡƒΠ΄Π΅Ρ‚ Π±Π»ΠΈΠ·ΠΎΠΊ ΠΊ Π½ΡƒΠ»ΡŽ (с ΡƒΡ‡Π΅Ρ‚ΠΎΠΌ омичСских ΠΏΠΎΡ‚Π΅Ρ€ΡŒ). На частотах, ΠΎΡ‚Π»ΠΈΡ‡Π½Ρ‹Ρ… ΠΎΡ‚ рСзонансной, сопротивлСниС ΠΊΠΎΠ½Ρ‚ΡƒΡ€Π° Π±ΡƒΠ΄Π΅Ρ‚ ΡƒΠΌΠ΅Π½ΡŒΡˆΠ°Ρ‚ΡΡ, Π° коэффициСнт ΠΏΠ΅Ρ€Π΅Π΄Π°Ρ‡ΠΈ Ρ‡Π΅Ρ‚Ρ‹Ρ€Π΅Ρ…ΠΏΠΎΠ»ΡŽΡΠ½ΠΈΠΊΠ° β€” Π²ΠΎΠ·Ρ€Π°ΡΡ‚Π°Ρ‚ΡŒ.

Для Ρ‡Π΅Ρ‚Ρ‹Ρ€Π΅Ρ…ΠΏΠΎΠ»ΡŽΡΠ½ΠΈΠΊΠ°, ΠΏΡ€ΠΈΠ²Π΅Π΄Π΅Π½Π½ΠΎΠ³ΠΎ Π½Π° рисункС Π²Ρ‹ΡˆΠ΅, ситуация Π±ΡƒΠ΄Π΅Ρ‚ ΠΏΡ€ΠΎΡ‚ΠΈΠ²ΠΎΠΏΠΎΠ»ΠΎΠΆΠ½ΠΎΠΉ β€” Π½Π° рСзонансной частотС ΠΊΠΎΠ½Ρ‚ΡƒΡ€ Π±ΡƒΠ΄Π΅Ρ‚ ΠΏΡ€Π΅Π΄ΡΡ‚Π°Π²Π»ΡΡ‚ΡŒ собой ΠΎΡ‡Π΅Π½ΡŒ большоС сопротивлСниС ΠΈ практичСски всС Π²Ρ…ΠΎΠ΄Π½ΠΎΠ΅ напряТСниС поступит Π½Π° Π²Ρ‹Ρ…ΠΎΠ΄Π½Ρ‹Π΅ ΠΊΠ»Π΅ΠΌΠΌΡ‹ (Ρ‚.Π΅ коэффициСнт ΠΏΠ΅Ρ€Π΅Π΄Π°Ρ‡ΠΈ Π±ΡƒΠ΄Π΅Ρ‚ максималСн ΠΈ Π±Π»ΠΈΠ·ΠΎΠΊ ΠΊ Π΅Π΄ΠΈΠ½ΠΈΡ†Π΅). ΠŸΡ€ΠΈ Π·Π½Π°Ρ‡ΠΈΡ‚Π΅Π»ΡŒΠ½ΠΎΠΌ ΠΎΡ‚Π»ΠΈΡ‡ΠΈΠΈ частоты Π²Ρ…ΠΎΠ΄Π½ΠΎΠ³ΠΎ воздСйствия ΠΎΡ‚ рСзонансной частоты ΠΊΠΎΠ½Ρ‚ΡƒΡ€Π°, источник сигнала, ΠΏΠΎΠ΄ΠΊΠ»ΡŽΡ‡Π°Π΅ΠΌΡ‹ΠΉ ΠΊ Π²Ρ…ΠΎΠ΄Π½Ρ‹ΠΌ ΠΊΠ»Π΅ΠΌΠΌΠ°ΠΌ Ρ‡Π΅Ρ‚Ρ‹Ρ€Π΅Ρ…ΠΏΠΎΠ»ΡŽΡΠ½ΠΈΠΊΠ°, окаТСтся практичСски Π·Π°ΠΊΠΎΡ€ΠΎΡ‡Π΅Π½Π½ΠΎΠΌ Π½Π°ΠΊΠΎΡ€ΠΎΡ‚ΠΊΠΎ, Π° коэффициСнт ΠΏΠ΅Ρ€Π΅Π΄Π°Ρ‡ΠΈ Π±ΡƒΠ΄Π΅Ρ‚ Π±Π»ΠΈΠ·ΠΎΠΊ ΠΊ Π½ΡƒΠ»ΡŽ.

Π’ΠΈΠ΄Π΅ΠΎ ΠΏΠΎ Ρ‚Π΅ΠΌΠ΅: ΠšΠΎΠ»Π΅Π±Π°Ρ‚Π΅Π»ΡŒΠ½Ρ‹ΠΉ ΠΊΠΎΠ½Ρ‚ΡƒΡ€

Π’ΠΈΠΌΠ΅Ρ€ΠΊΠ°Π΅Π² Борис β€” 68-Π»Π΅Ρ‚Π½ΠΈΠΉ Π΄ΠΎΠΊΡ‚ΠΎΡ€ Ρ„ΠΈΠ·ΠΈΠΊΠΎ-матСматичСских Π½Π°ΡƒΠΊ, профСссор ΠΈΠ· России. Он являСтся Π·Π°Π²Π΅Π΄ΡƒΡŽΡ‰ΠΈΠΌ ΠΊΠ°Ρ„Π΅Π΄Ρ€ΠΎΠΉ ΠΎΠ±Ρ‰Π΅ΠΉ Ρ„ΠΈΠ·ΠΈΠΊΠΈ Π² Казанском Π½Π°Ρ†ΠΈΠΎΠ½Π°Π»ΡŒΠ½ΠΎΠΌ ΠΈΡΡΠ»Π΅Π΄ΠΎΠ²Π°Ρ‚Π΅Π»ΡŒΡΠΊΠΎΠΌ тСхничСском унивСрситСтС ΠΈΠΌΠ΅Π½ΠΈ А. Н. Π’Π£ΠŸΠžΠ›Π•Π’Π β€” КАИ

РасчСт сопротивлСния Π² ΠΌΠ½ΠΎΠ³ΠΎΠΊΠΎΠ½Ρ‚ΡƒΡ€Π½Ρ‹Ρ… цСпях

РасчСт сопротивлСния

ΠŸΠΎΡΠ»Π΅Π΄ΠΎΠ²Π°Ρ‚Π΅Π»ΡŒΠ½Π°Ρ Ρ†Π΅ΠΏΡŒ — это Ρ†Π΅ΠΏΡŒ, состоящая Ρ‚ΠΎΠ»ΡŒΠΊΠΎ ΠΈΠ· ΠΎΠ΄Π½ΠΎΠ³ΠΎ ΠΊΠΎΠ½Ρ‚ΡƒΡ€Π°, ΠΈ Π²Ρ‹Ρ‡ΠΈΡΠ»ΠΈΡ‚ΡŒ сопротивлСниС Π² ΠΏΠΎΡΠ»Π΅Π΄ΠΎΠ²Π°Ρ‚Π΅Π»ΡŒΠ½ΠΎΠΉ Ρ†Π΅ΠΏΠΈ ΠΎΡ‡Π΅Π½ΡŒ просто. Если схСма состоит ΠΈΠ· Ρ‚Ρ€Π΅Ρ… ΠΊΠΎΠΌΠΏΠΎΠ½Π΅Π½Ρ‚ΠΎΠ² ΠΈ ΠΊΠ°ΠΆΠ΄Ρ‹ΠΉ ΠΈΠ· Π½ΠΈΡ… ΠΈΠΌΠ΅Π΅Ρ‚ сопротивлСниС 4 Ом, Ρ‚ΠΎ ΠΏΠΎΠ»Π½ΠΎΠ΅ сопротивлСниС схСмы составляСт 4 + 4 + 4, Ρ‡Ρ‚ΠΎ Ρ€Π°Π²Π½ΠΎ 12 Ом.

Однако с ΠΌΠ½ΠΎΠ³ΠΎΠΊΠΎΠ½Ρ‚ΡƒΡ€Π½Ρ‹ΠΌΠΈ схСмами (Ρ‚Π°ΠΊΠΆΠ΅ извСстными ΠΊΠ°ΠΊ ΠΏΠ°Ρ€Π°Π»Π»Π΅Π»ΡŒΠ½Ρ‹Π΅ схСмы) слоТнСС. Π§Ρ‚ΠΎΠ±Ρ‹ Π²Ρ‹Ρ‡ΠΈΡΠ»ΠΈΡ‚ΡŒ ΠΎΠ±Ρ‰Π΅Π΅ сопротивлСниС Ρ†Π΅ΠΏΠΈ с нСсколькими ΠΊΠΎΠ½Ρ‚ΡƒΡ€Π°ΠΌΠΈ, Π²Π°ΠΌ Π½ΡƒΠΆΠ½ΠΎ ΠΈΡΠΏΠΎΠ»ΡŒΠ·ΠΎΠ²Π°Ρ‚ΡŒ ΡƒΡ€Π°Π²Π½Π΅Π½ΠΈΠ΅.Π­Ρ‚ΠΎ ΡƒΡ€Π°Π²Π½Π΅Π½ΠΈΠ΅ выглядит ΡΠ»Π΅Π΄ΡƒΡŽΡ‰ΠΈΠΌ ΠΎΠ±Ρ€Π°Π·ΠΎΠΌ:

Π£Ρ€Π°Π²Π½Π΅Π½ΠΈΠ΅ ΠΏΠΎΠ»Π½ΠΎΠ³ΠΎ сопротивлСния

Π£Ρ€Π°Π²Π½Π΅Π½ΠΈΠ΅ Π³ΠΎΠ²ΠΎΡ€ΠΈΡ‚, Ρ‡Ρ‚ΠΎ Π²Π΅Π»ΠΈΡ‡ΠΈΠ½Π°, обратная ΠΏΠΎΠ»Π½ΠΎΠΌΡƒ ΡΠΎΠΏΡ€ΠΎΡ‚ΠΈΠ²Π»Π΅Π½ΠΈΡŽ, Ρ€Π°Π²Π½Π° суммС ΠΎΠ±Ρ€Π°Ρ‚Π½Ρ‹Ρ… Π²Π΅Π»ΠΈΡ‡ΠΈΠ½ сопротивлСний ΠΊΠ°ΠΆΠ΄ΠΎΠΉ Π²Π΅Ρ‚Π²ΠΈ. Если Π²Ρ‹ Π²ΠΎΠ·ΡŒΠΌΠ΅Ρ‚Π΅ ΠΎΠ±Ρ€Π°Ρ‚Π½ΡƒΡŽ Π²Π΅Π»ΠΈΡ‡ΠΈΠ½Ρƒ для ΠΎΠ±Π΅ΠΈΡ… сторон, Π²Ρ‹ ΠΏΠΎΠ»ΡƒΡ‡ΠΈΡ‚Π΅ это ΠΎΠΊΠΎΠ½Ρ‡Π°Ρ‚Π΅Π»ΡŒΠ½ΠΎΠ΅ ΡƒΡ€Π°Π²Π½Π΅Π½ΠΈΠ΅:

Π£Ρ€Π°Π²Π½Π΅Π½ΠΈΠ΅ ΠΏΠΎΠ»Π½ΠΎΠ³ΠΎ сопротивлСния ΠΈΠ·ΠΌΠ΅Π½Π΅Π½ΠΎ

Π˜Ρ‚Π°ΠΊ, всС, Ρ‡Ρ‚ΠΎ Π²Π°ΠΌ Π½ΡƒΠΆΠ½ΠΎ ΡΠ΄Π΅Π»Π°Ρ‚ΡŒ, это ΠΏΠΎΠ΄ΡΡ‚Π°Π²ΠΈΡ‚ΡŒ числа ΠΈ Ρ€Π΅ΡˆΠΈΡ‚ΡŒ.

ΠŸΡ€ΠΈΠΌΠ΅Ρ€

Допустим, Ρƒ вас Π΅ΡΡ‚ΡŒ Ρ†Π΅ΠΏΡŒ, содСрТащая ΡΠ΅Ρ€ΠΈΡŽ Π»Π°ΠΌΠΏΠΎΡ‡Π΅ΠΊ. Π‘Ρ…Π΅ΠΌΠ° ΠΈΠΌΠ΅Π΅Ρ‚ Ρ‚Ρ€ΠΈ ΠΏΠ°Ρ€Π°Π»Π»Π΅Π»ΡŒΠ½Ρ‹Ρ… ΠΊΠΎΠ½Ρ‚ΡƒΡ€Π°, ΠΏΠΎΠ΄ΠΊΠ»ΡŽΡ‡Π΅Π½Π½Ρ‹Ρ… ΠΊ Π±Π°Ρ‚Π°Ρ€Π΅Π΅. На ΠΊΠ°ΠΆΠ΄ΠΎΠΌ ΡˆΠ»Π΅ΠΉΡ„Π΅ находится Π»Π°ΠΌΠΏΠΎΡ‡ΠΊΠ° сопротивлСниСм 6 Ом. КакоС ΠΏΠΎΠ»Π½ΠΎΠ΅ сопротивлСниС Ρ†Π΅ΠΏΠΈ?

Π§Ρ‚ΠΎΠ±Ρ‹ ΠΏΠΎΠ½ΡΡ‚ΡŒ это, всС, Ρ‡Ρ‚ΠΎ Π½Π°ΠΌ Π½ΡƒΠΆΠ½ΠΎ ΡΠ΄Π΅Π»Π°Ρ‚ΡŒ, это ΠΏΠΎΠ΄ΡΡ‚Π°Π²ΠΈΡ‚ΡŒ числа Π² ΡƒΡ€Π°Π²Π½Π΅Π½ΠΈΠ΅. ΠŸΠΎΡΠΊΠΎΠ»ΡŒΠΊΡƒ каТдая Π²Π΅Ρ‚Π²ΡŒ ΠΈΠΌΠ΅Π΅Ρ‚ ΠΎΠ΄ΠΈΠ½Π°ΠΊΠΎΠ²ΠΎΠ΅ сопротивлСниС 6 Ом, ΠΏΠΎΠ»Π½ΠΎΠ΅ сопротивлСниС Ρ†Π΅ΠΏΠΈ Ρ€Π°Π²Π½ΠΎ ΠΎΠ΄Π½ΠΎΠΉ ΡˆΠ΅ΡΡ‚ΠΎΠΉ плюс ΠΎΠ΄Π½Π° ΡˆΠ΅ΡΡ‚Π°Ρ плюс ΠΎΠ΄Π½Π° ΡˆΠ΅ΡΡ‚Π°Ρ.Π’Ρ‹ ΠΌΠΎΠΆΠ΅Ρ‚Π΅ ввСсти это Π² ΠΊΠ°Π»ΡŒΠΊΡƒΠ»ΡΡ‚ΠΎΡ€ ΠΈΠ»ΠΈ ΠΈΡΠΏΠΎΠ»ΡŒΠ·ΠΎΠ²Π°Ρ‚ΡŒ Π½Π΅ΠΊΠΎΡ‚ΠΎΡ€Ρ‹Π΅ ΠΏΡ€Π°Π²ΠΈΠ»Π° для Π΄Ρ€ΠΎΠ±Π΅ΠΉ, Ρ‡Ρ‚ΠΎΠ±Ρ‹ ΡƒΠΏΡ€ΠΎΡΡ‚ΠΈΡ‚ΡŒ Π΅Π³ΠΎ, Π½ΠΎ Π² ΠΈΡ‚ΠΎΠ³Π΅ Π²Ρ‹ ΠΏΠΎΠ»ΡƒΡ‡ΠΈΡ‚Π΅ ΠΎΠ±Ρ€Π°Ρ‚Π½ΡƒΡŽ Π²Π΅Π»ΠΈΡ‡ΠΈΠ½Ρƒ: 3 Π½Π° 6, Ρ‚ΠΎ Π΅ΡΡ‚ΡŒ 6 Π½Π° 3. И это Ρ€Π°Π²Π½ΠΎ 2. Π’Π°ΠΊΠΈΠΌ ΠΎΠ±Ρ€Π°Π·ΠΎΠΌ, ΠΎΠ±Ρ‰Π΅Π΅ сопротивлСниС Ρ†Π΅ΠΏΠΈ составляСт 2 Ом.

Π’ΠΎΡ‚ Ρ„ΠΎΡ€ΠΌΡƒΠ»Π° уравнСния Π·Π°Π΄Π°Ρ‡ΠΈ:

ΠŸΡ€ΠΈΠΌΠ΅Ρ€ Ρ€Π΅ΡˆΠ΅Π½ΠΈΡ

ΠšΡ€Π°Ρ‚ΠΊΠΎΠ΅ содСрТаниС ΡƒΡ€ΠΎΠΊΠ°

ΠŸΡ€ΠΎΠ²ΠΎΠ΄Π½ΠΈΠΊ — это ΠΌΠ°Ρ‚Π΅Ρ€ΠΈΠ°Π», Ρ‡Π΅Ρ€Π΅Π· ΠΊΠΎΡ‚ΠΎΡ€Ρ‹ΠΉ ΠΏΡ€ΠΎΡ…ΠΎΠ΄ΠΈΡ‚ элСктричСство (элСктричСский Ρ‚ΠΎΠΊ). ЭлСктричСскоС сопротивлСниС — это ΠΎΡΠΎΠ±Π΅Π½Π½ΠΎΡΡ‚ΡŒ ΠΏΡ€ΠΎΠ²ΠΎΠ΄Π½ΠΈΠΊΠ°, ΠΊΠΎΡ‚ΠΎΡ€Ρ‹ΠΉ сдСрТиваСт ΠΏΡ€ΠΎΡ…ΠΎΠΆΠ΄Π΅Π½ΠΈΠ΅ элСктричСского Ρ‚ΠΎΠΊΠ°. Π‘ΠΎΠΏΡ€ΠΎΡ‚ΠΈΠ²Π»Π΅Π½ΠΈΠ΅ измСряСтся Π² Ом.

ΠŸΡ€ΠΈ вычислСнии ΠΎΠ±Ρ‰Π΅Π³ΠΎ сопротивлСния Ρ†Π΅ΠΏΠΈ с ΠΎΠ΄Π½ΠΈΠΌ ΠΊΠΎΠ½Ρ‚ΡƒΡ€ΠΎΠΌ Π²Ρ‹ просто складываСтС сопротивлСния ΠΊΠ°ΠΆΠ΄ΠΎΠ³ΠΎ ΠΊΠΎΠΌΠΏΠΎΠ½Π΅Π½Ρ‚Π°. ΠŸΡ€ΠΈ расчСтС ΠΎΠ±Ρ‰Π΅Π³ΠΎ сопротивлСния Ρ†Π΅ΠΏΠΈ с нСсколькими ΠΊΠΎΠ½Ρ‚ΡƒΡ€Π°ΠΌΠΈ (ΠΏΠ°Ρ€Π°Π»Π»Π΅Π»ΡŒΠ½Π°Ρ Ρ†Π΅ΠΏΡŒ) Π²Ρ‹ Π΄ΠΎΠ»ΠΆΠ½Ρ‹ ΠΈΡΠΏΠΎΠ»ΡŒΠ·ΠΎΠ²Π°Ρ‚ΡŒ ΡƒΡ€Π°Π²Π½Π΅Π½ΠΈΠ΅:

ΠžΠ±Ρ€Π°Ρ‚Π½ΠΎΠ΅ Π·Π½Π°Ρ‡Π΅Π½ΠΈΠ΅ ΠΎΠ±Ρ‰Π΅Π³ΠΎ сопротивлСния = сумма ΠΎΠ±Ρ€Π°Ρ‚Π½Ρ‹Ρ… сопротивлСний ΠΊΠ°ΠΆΠ΄ΠΎΠΉ Π²Π΅Ρ‚Π²ΠΈ

ВсС, Ρ‡Ρ‚ΠΎ Π²Π°ΠΌ Π½ΡƒΠΆΠ½ΠΎ do — это ΠΏΠΎΠ΄ΡΡ‚Π°Π²ΠΈΡ‚ΡŒ числа Π² ΡƒΡ€Π°Π²Π½Π΅Π½ΠΈΠ΅ ΠΈ Ρ€Π΅ΡˆΠΈΡ‚ΡŒ.

Π˜Π·ΠΌΠ΅Ρ€Π΅Π½ΠΈΠ΅ ΠΈ испытаниС сопротивлСния ΠΊΠΎΠ½Ρ‚ΡƒΡ€Π° постоянного Ρ‚ΠΎΠΊΠ°

Π‘ΠΎΠΏΡ€ΠΎΡ‚ΠΈΠ²Π»Π΅Π½ΠΈΠ΅ ΠΊΠΎΠ½Ρ‚ΡƒΡ€Π° постоянного Ρ‚ΠΎΠΊΠ° — это ΠΏΠΎΠ»Π½ΠΎΠ΅ сопротивлСниС Π΄Π²ΡƒΡ… ΠΏΡ€ΠΎΠ²ΠΎΠ΄Π½ΠΈΠΊΠΎΠ², Π·Π°ΠΌΠΊΠ½ΡƒΡ‚Ρ‹Ρ… Π½Π° ΠΎΠ΄Π½ΠΎΠΌ ΠΊΠΎΠ½Ρ†Π΅ Π·Π²Π΅Π½Π°. ΠžΠ±Ρ‹Ρ‡Π½ΠΎ это функция Π΄ΠΈΠ°ΠΌΠ΅Ρ‚Ρ€Π° ΠΏΡ€ΠΎΠ²ΠΎΠ΄Π½ΠΈΠΊΠ° ΠΈ зависит Ρ‚ΠΎΠ»ΡŒΠΊΠΎ ΠΎΡ‚ расстояния. Иногда это ΠΈΠ·ΠΌΠ΅Ρ€Π΅Π½ΠΈΠ΅ проводится для Ρ‚ΠΎΠ³ΠΎ, Ρ‡Ρ‚ΠΎΠ±Ρ‹ ΡƒΠ±Π΅Π΄ΠΈΡ‚ΡŒΡΡ Π² отсутствии ΡΠ΅Ρ€ΡŒΠ΅Π·Π½Ρ‹Ρ… Π½Π΅ΠΏΡ€Π°Π²ΠΈΠ»ΡŒΠ½Ρ‹Ρ… ΠΏΠΎΠ΄ΠΊΠ»ΡŽΡ‡Π΅Π½ΠΈΠΉ, ΠΊΠΎΡ‚ΠΎΡ€Ρ‹Π΅ ΠΌΠΎΠ³ΡƒΡ‚ Π΄ΠΎΠ±Π°Π²ΠΈΡ‚ΡŒ Π·Π½Π°Ρ‡ΠΈΡ‚Π΅Π»ΡŒΠ½ΠΎΠΌΡƒ ΡΠΎΠΏΡ€ΠΎΡ‚ΠΈΠ²Π»Π΅Π½ΠΈΡŽ Π»ΠΈΠ½ΠΈΠΈ. ΠžΠ±Ρ€Π°Ρ‚ΠΈΡ‚Π΅ Π²Π½ΠΈΠΌΠ°Π½ΠΈΠ΅, Ρ‡Ρ‚ΠΎ ΠΏΡ€ΠΎΠ²Π΅Ρ€ΠΊΠ° схСмы Ρ€Π°Π·Π²ΠΎΠ΄ΠΊΠΈ автоматичСски ΠΈΠ·ΠΎΠ»ΠΈΡ€ΡƒΠ΅Ρ‚ ΠΎΠ±Ρ€Ρ‹Π²Ρ‹, Π½ΠΎ Π½Π΅ соСдинСния с высоким сопротивлСниСм.

Π‘ΠΎΠΏΡ€ΠΎΡ‚ΠΈΠ²Π»Π΅Π½ΠΈΠ΅ постоянному Ρ‚ΠΎΠΊΡƒ часто ΠΏΡƒΡ‚Π°ΡŽΡ‚ с импСдансом, Ρ‚Π΅Ρ€ΠΌΠΈΠ½ΠΎΠΌ, ΠΎΠΏΠΈΡΡ‹Π²Π°ΡŽΡ‰ΠΈΠΌ динамичСскоС сопротивлСниС ΠΏΠΎΡ‚ΠΎΠΊΡƒ сигнала, ΠΎΠ±Ρ‹Ρ‡Π½ΠΎ Π½Π° ΠΎΠΏΡ€Π΅Π΄Π΅Π»Π΅Π½Π½ΠΎΠΉ частотС.Оба ΠΈΠ·ΠΌΠ΅Ρ€ΡΡŽΡ‚ΡΡ Π² ΠΎΠΌΠ°Ρ…, ΠΏΠΎΡ‚ΠΎΠΌΡƒ Ρ‡Ρ‚ΠΎ ΠΎΠ½ΠΈ ΠΎΠΏΡ€Π΅Π΄Π΅Π»ΡΡŽΡ‚ Ρ€Π°Π·Π½Ρ‹Π΅ Ρ‚ΠΈΠΏΡ‹ противодСйствия элСктричСскому Ρ‚ΠΎΠΊΡƒ. Π‘ΠΎΠΏΡ€ΠΎΡ‚ΠΈΠ²Π»Π΅Π½ΠΈΠ΅ постоянному Ρ‚ΠΎΠΊΡƒ увСличиваСтся ΠΏΡ€ΠΎΠΏΠΎΡ€Ρ†ΠΈΠΎΠ½Π°Π»ΡŒΠ½ΠΎ Π΄Π»ΠΈΠ½Π΅ тСстируСмого кабСля, Π² Ρ‚ΠΎ врСмя ΠΊΠ°ΠΊ импСданс остаСтся «довольно» постоянным нСзависимо ΠΎΡ‚ Π΄Π»ΠΈΠ½Ρ‹.

Π‘ Ρ‚ΠΎΡ‡ΠΊΠΈ зрСния сигнала Π·Π°Ρ‚ΡƒΡ…Π°Π½ΠΈΠ΅ (ΠΈΠ½ΠΎΠ³Π΄Π° Π½Π°Π·Ρ‹Π²Π°Π΅ΠΌΠΎΠ΅ вносимыми потСрями) Ρ‚Π΅ΠΏΠ΅Ρ€ΡŒ являСтся Π±ΠΎΠ»Π΅Π΅ ΠΏΠΎΠ»Π΅Π·Π½Ρ‹ΠΌ ΠΈΠ·ΠΌΠ΅Ρ€Π΅Π½ΠΈΠ΅ΠΌ, Π° сопротивлСниС постоянному Ρ‚ΠΎΠΊΡƒ стало ΠΌΠ΅Π½Π΅Π΅ Π²Π°ΠΆΠ½Ρ‹ΠΌ. Π§Ρ‚ΠΎ ΠΆ, Π½Π΅ совсСм Ρ‚Π°ΠΊ, ΠΊΠΎΠ³Π΄Π° становится популярным VoIP, ΠΊΠΎΡ‚ΠΎΡ€Ρ‹ΠΉ обСспСчиваСт ΠΏΠΈΡ‚Π°Π½ΠΈΠ΅ ΠΏΠΎ кабСлю.

Π˜Π½Ρ‚Π΅Ρ€ΠΏΡ€Π΅Ρ‚Π°Ρ†ΠΈΡ Ρ€Π΅Π·ΡƒΠ»ΡŒΡ‚Π°Ρ‚ΠΎΠ²

Различия Π² сопротивлСнии ΠΊΠΎΠ½Ρ‚ΡƒΡ€Π° ΠΌΠ΅ΠΆΠ΄Ρƒ ΠΏΠ°Ρ€Π°ΠΌΠΈ часто ΠΌΠΎΠ³ΡƒΡ‚ Π±Ρ‹Ρ‚ΡŒ быстрым ΠΏΡ€ΠΈΠ·Π½Π°ΠΊΠΎΠΌ нСисправности кабСля.Π’ тСстовой срСдС с ΠΊΠΎΡ€ΠΎΡ‚ΠΊΠΎΠΉ ΠΏΠ΅Ρ‚Π»Π΅ΠΉ ΠΎΠΆΠΈΠ΄Π°Π΅ΠΌΠΎΠ΅ Π·Π½Π°Ρ‡Π΅Π½ΠΈΠ΅ просто Π²Π΄Π²ΠΎΠ΅ ΠΏΡ€Π΅Π²Ρ‹ΡˆΠ°Π΅Ρ‚ сумму Π·Π½Π°Ρ‡Π΅Π½ΠΈΠΉ, ΠΎΠΆΠΈΠ΄Π°Π΅ΠΌΡ‹Ρ… для Π΄Π°Π½Π½ΠΎΠΉ Π΄Π»ΠΈΠ½Ρ‹. Π­Ρ‚ΠΎ простой тСст для любого ΠΎΠΏΡ‹Ρ‚Π½ΠΎΠ³ΠΎ ΠΏΠΎΠ»Π΅Π²ΠΎΠ³ΠΎ тСстСра.

ЗначСния Π±ΡƒΠ΄ΡƒΡ‚ Ρ€Π°Π·Π½Ρ‹ΠΌΠΈ для ΠΊΠ°ΠΆΠ΄ΠΎΠΉ ΠΊΠΎΠΌΠ±ΠΈΠ½Π°Ρ†ΠΈΠΈ ΠΏΠ°Ρ€ ΠΈΠ·-Π·Π° Ρ€Π°Π·Π½ΠΎΠΉ скорости скручивания ΠΌΠ΅ΠΆΠ΄Ρƒ ΠΏΠ°Ρ€Π°ΠΌΠΈ. Глядя Π½Π° ΠΏΡ€ΠΈΠ²Π΅Π΄Π΅Π½Π½Ρ‹ΠΉ Π²Ρ‹ΡˆΠ΅ Ρ€Π΅Π·ΡƒΠ»ΡŒΡ‚Π°Ρ‚, ΠΌΡ‹ ΠΌΠΎΠΆΠ΅ΠΌ ΡΠ΄Π΅Π»Π°Ρ‚ΡŒ Π²Ρ‹Π²ΠΎΠ΄, Ρ‡Ρ‚ΠΎ ΠΏΠ°Ρ€Π° 1,2 ΠΈΠΌΠ΅Π΅Ρ‚ самый ΠΊΡ€ΡƒΡ‚ΠΎΠΉ ΠΏΠΎΠ²ΠΎΡ€ΠΎΡ‚, Π° ΠΏΠ°Ρ€Π° 7,8 — наимСньший. Π­Ρ‚ΠΎ Π½ΠΎΡ€ΠΌΠ°Π»ΡŒΠ½ΠΎ ΠΈ этого слСдовало ΠΎΠΆΠΈΠ΄Π°Ρ‚ΡŒ.

Π Π΅ΠΊΠΎΠΌΠ΅Π½Π΄Π°Ρ†ΠΈΠΈ ΠΏΠΎ поиску ΠΈ ΡƒΡΡ‚Ρ€Π°Π½Π΅Π½ΠΈΡŽ нСисправностСй

Π’ случаС Π½Π΅ΠΎΠΆΠΈΠ΄Π°Π½Π½ΠΎ высокого сопротивлСния постоянному Ρ‚ΠΎΠΊΡƒ сравнитС Π²Ρ‹ΡˆΠ΅Π΄ΡˆΡƒΡŽ ΠΈΠ· строя ΠΏΠ°Ρ€Ρƒ с Π΄Ρ€ΡƒΠ³ΠΈΠΌΠΈ ΠΏΠ°Ρ€Π°ΠΌΠΈ кабСля.Π­Ρ‚ΠΎ ΠΏΠΎΠ·Π²ΠΎΠ»ΠΈΡ‚ ΠΎΠΏΡ€Π΅Π΄Π΅Π»ΠΈΡ‚ΡŒ, связана Π»ΠΈ ΠΏΡ€ΠΎΠ±Π»Π΅ΠΌΠ° с ΠΎΠ΄Π½ΠΎΠΉ нСисправной ΠΏΠ°Ρ€ΠΎΠΉ ΠΈΠ»ΠΈ с ΠΏΡ€ΠΎΠ±Π»Π΅ΠΌΠΎΠΉ, Π·Π°Ρ‚Ρ€Π°Π³ΠΈΠ²Π°ΡŽΡ‰Π΅ΠΉ вСсь кабСль. Если нСисправна ΠΎΠ΄Π½Π° ΠΏΠ°Ρ€Π°, ΠΏΡ€ΠΎΠ²Π΅Ρ€ΡŒΡ‚Π΅ Ρ‚ΠΎΡ‡ΠΊΠΈ ΠΏΠΎΠ΄ΠΊΠ»ΡŽΡ‡Π΅Π½ΠΈΡ Π½Π° Π½Π°Π»ΠΈΡ‡ΠΈΠ΅ ΠΏΠ»ΠΎΡ…ΠΎ Π²Ρ‹ΠΏΠΎΠ»Π½Π΅Π½Π½Ρ‹Ρ… ΠΈΠ»ΠΈ окислСнных соСдинСний.

Если всС Ρ‡Π΅Ρ‚Ρ‹Ρ€Π΅ ΠΏΠ°Ρ€Ρ‹ ΠΈΠΌΠ΅ΡŽΡ‚ Π½Π΅ΠΎΠΆΠΈΠ΄Π°Π½Π½ΠΎ высокоС сопротивлСниС постоянному Ρ‚ΠΎΠΊΡƒ, ΠΏΡ€ΠΎΠ²Π΅Ρ€ΡŒΡ‚Π΅ свои прСдполоТСния. Π’Ρ‹ ΡƒΡ‡Π»ΠΈ ΡƒΠ΄Π²ΠΎΠ΅Π½ΠΈΠ΅ сопротивлСния для Π²ΠΊΠ»ΡŽΡ‡Π΅Π½ΠΈΡ ΠΏΠ΅Ρ‚Π»ΠΈ? ΠŸΡ€Π°Π²ΠΈΠ»ΡŒΠ½ΠΎ Π»ΠΈ ΠΏΡ€Π΅Π΄ΠΏΠΎΠ»ΠΎΠΆΠ΅Π½ΠΈΠ΅ ΠΎ сопротивлСнии для ΠΈΡΠΏΠΎΠ»ΡŒΠ·ΡƒΠ΅ΠΌΠΎΠ³ΠΎ ΠΊΠ°Π»ΠΈΠ±Ρ€Π° ΠΏΡ€ΠΎΠ²ΠΎΠ΄Π°? 26 ΠΊΠ°Π»ΠΈΠ±Ρ€Π° ΠΈΠΌΠ΅Π΅Ρ‚ Π±ΠΎΠ»Π΅Π΅ высокоС сопротивлСниС Π½Π° Ρ„ΡƒΡ‚, Ρ‡Π΅ΠΌ 24 ΠΊΠ°Π»ΠΈΠ±Ρ€Π°. Π£ вас Π² Π·Π²Π΅Π½Π΅ Π½Π΅ΠΎΠ±Ρ‹Ρ‡Π½Ρ‹ΠΉ ΠΏΠ°Ρ‚Ρ‡-ΠΊΠΎΡ€Π΄, ΠΊΠΎΡ‚ΠΎΡ€Ρ‹ΠΉ ΠΌΠΎΠΆΠ΅Ρ‚ ΠΈΠΌΠ΅Ρ‚ΡŒ высокоС сопротивлСниС? Π˜Ρ‰ΠΈΡ‚Π΅ Ρ‡Ρ‚ΠΎ-Π½ΠΈΠ±ΡƒΠ΄ΡŒ Π½Π΅ΠΎΠ±Ρ‹Ρ‡Π½ΠΎΠ΅, особСнно Ссли сосСдниС ΠΊΠ°Π±Π΅Π»ΠΈ каТутся Π½ΠΎΡ€ΠΌΠ°Π»ΡŒΠ½Ρ‹ΠΌΠΈ.

Loop Resistance — ΠΎΠ±Π·ΠΎΡ€

5.5 ΠœΡƒΠ»ΡŒΡ‚ΠΈΠΏΠ»Π΅ΠΊΡ, сСти шин Π΄Π°Π½Π½Ρ‹Ρ… ΠΈ адрСсныС систСмы

Multiplex ΠΏΡ€Π΅Π΄Π»Π°Π³Π°Π΅Ρ‚ высокий ΡƒΡ€ΠΎΠ²Π΅Π½ΡŒ бСзопасности ΠΈ надСТности ΠΏΡ€ΠΈ ΠΎΠ΄Π½ΠΎΠ²Ρ€Π΅ΠΌΠ΅Π½Π½ΠΎΠΌ сниТСнии Π·Π°Ρ‚Ρ€Π°Ρ‚ Π½Π° установку ΠΈ обслуТиваниС. ΠŸΡ€ΠΈ использовании ΠΎΠ±Ρ‹Ρ‡Π½Ρ‹Ρ… систСм ΠΏΡ€ΠΎΠ²ΠΎΠ΄ΠΊΠΈ Π±Ρ‹Π»ΠΎ Π·Π°ΠΌΠ΅Ρ‡Π΅Π½ΠΎ, Ρ‡Ρ‚ΠΎ каТдая Ρ†Π΅ΠΏΡŒ Π΄Π΅Ρ‚Π΅ΠΊΡ‚ΠΎΡ€Π° ΠΏΡ€ΠΎΡ…ΠΎΠ΄ΠΈΡ‚ ΠΏΠΎ ΠΏΠ°Ρ€Π΅ ΠΏΡ€ΠΎΠ²ΠΎΠ΄ΠΎΠ² для обнаруТСния Ρ‚Ρ€Π΅Π²ΠΎΠ³ΠΈ; ΠΎΠ΄Π½Π°ΠΊΠΎ ΠΌΠ΅Ρ‚ΠΎΠ΄ ΠΌΡƒΠ»ΡŒΡ‚ΠΈΠΏΠ»Π΅ΠΊΡΠΈΡ€ΠΎΠ²Π°Π½ΠΈΡ ΠΈΡΠΏΠΎΠ»ΡŒΠ·ΡƒΠ΅Ρ‚ основныС магистрали ΠΏΠ΅Ρ€Π΅Π΄Π°Ρ‡ΠΈ Π΄Π°Π½Π½Ρ‹Ρ… ΠΈΠ»ΠΈ отвСтвлСния ΠΎΡ‚ Π³Π»Π°Π²Π½ΠΎΠΉ ΠΏΠ°Π½Π΅Π»ΠΈ управлСния. Π­Ρ‚ΠΎ ΠΎΠ·Π½Π°Ρ‡Π°Π΅Ρ‚, Ρ‡Ρ‚ΠΎ достигаСтся ΠΎΠ³Ρ€ΠΎΠΌΠ½ΠΎΠ΅ сокращСниС ΠΏΡ€ΠΎΠΊΠ»Π°Π΄ΠΊΠΈ кабСля.ΠŸΡ€ΠΈ использовании ΠΌΡƒΠ»ΡŒΡ‚ΠΈΠΏΠ»Π΅ΠΊΡΠ½ΠΎΠΉ Ρ‚Π΅Ρ…Π½ΠΎΠ»ΠΎΠ³ΠΈΠΈ каТдая Π²Π΅Ρ‚Π²ΡŒ ΠΌΠΎΠΆΠ΅Ρ‚ ΡΠΎΠ΄Π΅Ρ€ΠΆΠ°Ρ‚ΡŒ ΠΊΠ°ΠΊ ΠΌΠΈΠ½ΠΈΠΌΡƒΠΌ Ρ‡Π΅Ρ‚Ρ‹Ρ€Π΅Ρ…ΠΆΠΈΠ»ΡŒΠ½Ρ‹ΠΉ кабСль Π΄Π»ΠΈΠ½ΠΎΠΉ порядка 2 ΠΊΠΌ ΠΈ, ΠΊΠ°ΠΊ ΠΏΡ€Π°Π²ΠΈΠ»ΠΎ, Π²ΠΌΠ΅Ρ‰Π°Ρ‚ΡŒ Π΄ΠΎ 16 ΠΌΠΎΠ΄ΡƒΠ»Π΅ΠΉ Π»ΠΈΠ½Π΅ΠΉΠ½ΠΎΠ³ΠΎ интСрфСйса (LIM). Π­Ρ‚ΠΈ ΠΌΠΎΠ΄ΡƒΠ»ΠΈ LIM ΠΌΠΎΠ³ΡƒΡ‚ ΠΈΠΌΠ΅Ρ‚ΡŒ ΠΏΡΡ‚ΡŒ Π΄Π²ΡƒΡ…ΠΏΠΎΠ»ΡŽΡΠ½Ρ‹Ρ… Ρ†Π΅ΠΏΠ΅ΠΉ, ΠΊΠΎΡ‚ΠΎΡ€Ρ‹Π΅ ΠΌΠΎΠΆΠ½ΠΎ ΠΈΠ½Π΄ΠΈΠ²ΠΈΠ΄ΡƒΠ°Π»ΡŒΠ½ΠΎ Π·Π°ΠΏΡ€ΠΎΠ³Ρ€Π°ΠΌΠΌΠΈΡ€ΠΎΠ²Π°Ρ‚ΡŒ Π½Π° Ρ‚ΠΈΠΏ Ρ†Π΅ΠΏΠΈ с ΡΠΎΠΎΡ‚Π²Π΅Ρ‚ΡΡ‚Π²ΡƒΡŽΡ‰ΠΈΠΌΠΈ Π°Ρ‚Ρ€ΠΈΠ±ΡƒΡ‚Π°ΠΌΠΈ. Π­Ρ‚ΠΈ ΠΏΠ°Ρ€Π°ΠΌΠ΅Ρ‚Ρ€Ρ‹ Π½Π΅ Π±ΡƒΠ΄ΡƒΡ‚ ΠΎΡ‚Π»ΠΈΡ‡Π°Ρ‚ΡŒΡΡ ΠΎΡ‚ своих Π°Π½Π°Π»ΠΎΠ³ΠΎΠ² для ΠΎΠ±Ρ‹Ρ‡Π½ΠΎΠΉ ΠΏΡ€ΠΎΠ²ΠΎΠ΄ΠΊΠΈ.

Рассмотрим Π² качСствС ΠΏΡ€ΠΈΠΌΠ΅Ρ€Π° систСму с Ρ‡Π΅Ρ‚Ρ‹Ρ€ΡŒΠΌΡ вСтвями обнаруТСния, которая Π±ΡƒΠ΄Π΅Ρ‚ ΠΈΠΌΠ΅Ρ‚ΡŒ Π΅ΠΌΠΊΠΎΡΡ‚ΡŒ для 320 Π΄Π²ΡƒΡ…ΠΏΠΎΠ»ΡŽΡΠ½Ρ‹Ρ… Ρ†Π΅ΠΏΠ΅ΠΉ, каТдая ΠΈΠ· ΠΊΠΎΡ‚ΠΎΡ€Ρ‹Ρ… ΠΈΠ½Π΄ΠΈΠ²ΠΈΠ΄ΡƒΠ°Π»ΡŒΠ½ΠΎ рСгистрируСтся ΠΈ контролируСтся (рисунок 5.16). На рис. 5.17 ΠΏΠΎΠΊΠ°Π·Π°Π½Ρ‹ соСдинСния основной ΠΏΠ»Π°Ρ‚Ρ‹ с ΠΏΡ€ΠΎΠ²ΠΎΠ΄ΠΊΠΎΠΉ ΠΊ модулям LIM ΠΈ Π΄Π΅Ρ‚Π΅ΠΊΡ‚ΠΎΡ€Π°ΠΌ, Π²ΠΊΠ»ΡŽΡ‡Π°Ρ ΠΎΡ‚Π΄Π΅Π»ΡŒΠ½Ρ‹Π΅ ΡˆΠ»Π΅ΠΉΡ„Ρ‹ Ρ‚Π°ΠΌΠΏΠ΅Ρ€Π° (A / T).

Рисунок 5.16. ΠœΠ΅Ρ‚ΠΎΠ΄ ΠΌΡƒΠ»ΡŒΡ‚ΠΈΠΏΠ»Π΅ΠΊΡΠΈΡ€ΠΎΠ²Π°Π½ΠΈΡ

Рисунок 5.17. Основная ΠΏΠ»Π°Ρ‚Π° ΠΌΡƒΠ»ΡŒΡ‚ΠΈΠΏΠ»Π΅ΠΊΡΠΎΡ€Π°

Учащийся Π·Π°ΠΌΠ΅Ρ‚ΠΈΡ‚, Ρ‡Ρ‚ΠΎ Π²Π΅Ρ‚Π²ΡŒ 2 ΠΈΠΌΠ΅Π΅Ρ‚ ΡˆΠ΅ΡΡ‚ΠΆΠΈΠ»ΡŒΠ½Ρ‹ΠΉ кабСль, Ρ‚Π°ΠΊ ΠΊΠ°ΠΊ Π΄Π΅Ρ‚Π΅ΠΊΡ‚ΠΎΡ€Ρ‹ Π² этом LIM Π΄ΠΎΠ»ΠΆΠ½Ρ‹ Π±Ρ‹Ρ‚ΡŒ Π·Π°ΠΏΠΈΡ‚Π°Π½Ρ‹, ΠΈΠ»ΠΈ ΠΌΠΎΠΆΠ΅Ρ‚ ΠΏΠΎΡ‚Ρ€Π΅Π±ΠΎΠ²Π°Ρ‚ΡŒΡΡ удалСнная ΠΊΠ»Π°Π²ΠΈΠ°Ρ‚ΡƒΡ€Π°. Π‘Π°ΠΌΠΈ ΠΌΠΎΠ΄ΡƒΠ»ΠΈ LIM ΠΌΠΎΠ³ΡƒΡ‚ Π±Ρ‹Ρ‚ΡŒ ΡƒΠΏΠ°ΠΊΠΎΠ²Π°Π½Ρ‹ ΠΈ ΡƒΠΊΠΎΠΌΠΏΠ»Π΅ΠΊΡ‚ΠΎΠ²Π°Π½Ρ‹, с ΠΊΡ€Ρ‹ΡˆΠΊΠΎΠΉ ΠΏΠ΅Ρ‡Π°Ρ‚Π½ΠΎΠΉ ΠΏΠ»Π°Ρ‚Ρ‹ ΠΈ Π·Π°Ρ‰ΠΈΡ‚ΠΎΠΉ ΠΎΡ‚ нСсанкционированного доступа, ΠΈΠ»ΠΈ ΠΏΠΎΡΡ‚Π°Π²Π»ΡΡ‚ΡŒΡΡ Π³ΠΎΠ»Ρ‹ΠΌΠΈ для установки Π² Π±Π»ΠΎΠΊΠ΅ питания с устройством Π·Π°Ρ‰ΠΈΡ‚Ρ‹ ΠΎΡ‚ нСсанкционированного доступа.Π­Ρ‚ΠΈ источники питания ΠΈΡΠΏΠΎΠ»ΡŒΠ·ΡƒΡŽΡ‚ΡΡ для ΠΏΠΎΠ²Ρ‹ΡˆΠ΅Π½ΠΈΡ напряТСния Π² Π»ΠΈΠ½ΠΈΠΈ отвСтвлСния, ΠΊΠΎΠ³Π΄Π° ΠΎΠ½ΠΎ ΠΏΠ°Π΄Π°Π΅Ρ‚ Π½ΠΈΠΆΠ΅ 12 Π’.

Π˜Π½ΠΆΠ΅Π½Π΅Ρ€ Π·Π°ΠΌΠ΅Ρ‚ΠΈΡ‚, Ρ‡Ρ‚ΠΎ Π² этом ΠΏΡ€ΠΈΠΌΠ΅Ρ€Π΅ ΠΌΡƒΠ»ΡŒΡ‚ΠΈΠΏΠ»Π΅ΠΊΡΠΈΡ€ΠΎΠ²Π°Π½ΠΈΡ схСмы обнаруТСния Π½Π΅ ΠΈΠΌΠ΅ΡŽΡ‚ ΠΎΠΊΠΎΠ½Π΅Ρ‡Π½Ρ‹Ρ… устройств. Π‘Π»Π΅Π΄ΡƒΡŽΡ‰ΠΈΠΌ этапом являСтся рассмотрСниС Π²Π°Ρ€ΠΈΠ°Π½Ρ‚Π° сСтСвой адрСсной систСмы с шиной Π΄Π°Π½Π½Ρ‹Ρ…, которая ΠΈΡΠΏΠΎΠ»ΡŒΠ·ΡƒΠ΅Ρ‚ нСсколько ΠΈΠ½ΠΎΠΉ ΠΏΠΎΠ΄Ρ…ΠΎΠ΄ ΠΈ называСтся Ρ€Π°ΡΡˆΠΈΡ€ΡΠ΅ΠΌΠΎΠΉ (рис. 5.18).

Рисунок 5.18. Π Π°ΡΡˆΠΈΡ€ΡΠ΅ΠΌΠ°Ρ систСма с ΡƒΠ·Π»ΠΎΠ²ΠΎΠΉ ΡΠ΅Ρ‚ΡŒΡŽ

Π­Ρ‚Π° систСма ΠΈΠΌΠ΅Π΅Ρ‚ восСмь Π·ΠΎΠ½ Π½Π° Π³Π»Π°Π²Π½ΠΎΠΉ ΠΏΠ»Π°Ρ‚Π΅ управлСния ΠΈ Π΄Π²Π΅ Π·ΠΎΠ½Ρ‹ Π½Π° ΠΊΠ°ΠΆΠ΄ΠΎΠΉ ΡƒΠ΄Π°Π»Π΅Π½Π½ΠΎΠΉ ΠΊΠ»Π°Π²ΠΈΠ°Ρ‚ΡƒΡ€Π΅.Π­Ρ‚ΠΈ Π·ΠΎΠ½Ρ‹ ΠΌΠΎΠ³ΡƒΡ‚ Π±Ρ‹Ρ‚ΡŒ Π·Π°ΠΌΠΊΠ½ΡƒΡ‚Ρ‹ΠΌΠΈ ΠΈΠ»ΠΈ ΠΈΠΌΠ΅Ρ‚ΡŒ ΠΎΠΊΠΎΠ½Π΅Ρ‡Π½Ρ‹Π΅ рСзисторы. КаТдая удалСнная ΠΊΠ»Π°Π²ΠΈΠ°Ρ‚ΡƒΡ€Π° Ρ‚Π°ΠΊΠΆΠ΅ ΠΈΠΌΠ΅Π΅Ρ‚ ΠΏΡ€ΠΎΠ³Ρ€Π°ΠΌΠΌΠΈΡ€ΡƒΠ΅ΠΌΡ‹ΠΉ Π²Ρ‹Ρ…ΠΎΠ΄. Если Ρ‚Ρ€Π΅Π±ΡƒΡŽΡ‚ΡΡ Π΄ΠΎΠΏΠΎΠ»Π½ΠΈΡ‚Π΅Π»ΡŒΠ½Ρ‹Π΅ Π²Ρ‹Ρ…ΠΎΠ΄Ρ‹, Ρ‚ΠΎ Π² ΡΠ΅Ρ‚ΡŒ ΡƒΠ΄Π°Π»Π΅Π½Π½Ρ‹Ρ… ΠΊΠ»Π°Π²ΠΈΠ°Ρ‚ΡƒΡ€ ΠΌΠΎΠΆΠ½ΠΎ ΡƒΡΡ‚Π°Π½ΠΎΠ²ΠΈΡ‚ΡŒ Π»ΠΎΠΊΠ°Π»ΡŒΠ½Ρ‹Π΅ ΠΊΠ°Ρ€Ρ‚Ρ‹ Ρ€Π°ΡΡˆΠΈΡ€Π΅Π½ΠΈΡ (LEC), Ρ‡Ρ‚ΠΎΠ±Ρ‹ ΠΎΠ±Π΅ΡΠΏΠ΅Ρ‡ΠΈΡ‚ΡŒ Π΄Π²Π΅ Π΄ΠΎΠΏΠΎΠ»Π½ΠΈΡ‚Π΅Π»ΡŒΠ½Ρ‹Π΅ Π·ΠΎΠ½Ρ‹ ΠΈ ΠΎΠ΄ΠΈΠ½ Π²Ρ‹Ρ…ΠΎΠ΄. Π‘Π»ΠΎΠΊ управлСния управляСт двумя ΠΎΡ‚Π΄Π΅Π»ΡŒΠ½Ρ‹ΠΌΠΈ сСтями шин Π΄Π°Π½Π½Ρ‹Ρ…, ΠΎΠ΄Π½ΠΎΠΉ для ΡƒΠ΄Π°Π»Π΅Π½Π½Ρ‹Ρ… ΠΊΠ»Π°Π²ΠΈΠ°Ρ‚ΡƒΡ€ ΠΈ Π»ΠΎΠΊΠ°Π»ΡŒΠ½Ρ‹Ρ… ΠΏΠ»Π°Ρ‚ Ρ€Π°ΡΡˆΠΈΡ€Π΅Π½ΠΈΡ, Π° Ρ‚Π°ΠΊΠΆΠ΅ Π΄Ρ€ΡƒΠ³ΠΎΠΉ ΡΠ΅Ρ‚ΡŒΡŽ шин Π΄Π°Π½Π½Ρ‹Ρ… для ΡƒΠ·Π»ΠΎΠ². Π­Ρ‚ΠΈΡ… ΡƒΠ·Π»ΠΎΠ² ΠΌΠΎΠΆΠ΅Ρ‚ Π±Ρ‹Ρ‚ΡŒ Π΄ΠΎ пяти Π² сСти, ΠΈ ΠΎΠ½ΠΈ сами ΠΎΠ±Π΅ΡΠΏΠ΅Ρ‡ΠΈΠ²Π°ΡŽΡ‚ восСмь Π·ΠΎΠ½ ΠΈ Π΄Π²Π° Π²Ρ‹Ρ…ΠΎΠ΄Π°.Π”ΠΎΠΏΠΎΠ»Π½ΠΈΡ‚Π΅Π»ΡŒΠ½Ρ‹ΠΉ ΡƒΠ·Π΅Π» ID (ΠΈΠ½Ρ‚Π΅Π»Π»Π΅ΠΊΡ‚ΡƒΠ°Π»ΡŒΠ½ΠΎΠ΅ устройство) позволяСт ΠΈΡΠΏΠΎΠ»ΡŒΠ·ΠΎΠ²Π°Ρ‚ΡŒ с панСлью Π΄Π΅Ρ‚Π΅ΠΊΡ‚ΠΎΡ€Ρ‹, совмСстимыС с ID. Π­Ρ‚ΠΎΡ‚ ΡƒΠ·Π΅Π» ΠΈΠ΄Π΅Π½Ρ‚ΠΈΡ„ΠΈΠΊΠ°Ρ†ΠΈΠΈ обСспСчиваСт ΠΎΠ΄ΠΈΠ½ ΠΊΠΎΠ½Ρ‚ΡƒΡ€ Π΄Π΅Ρ‚Π΅ΠΊΡ‚ΠΎΡ€Π° ΠΈΠ΄Π΅Π½Ρ‚ΠΈΡ„ΠΈΠΊΠ°Ρ‚ΠΎΡ€Π° для ΠΏΠΎΠ΄ΠΊΠ»ΡŽΡ‡Π΅Π½ΠΈΡ ΠΊ 30 устройствам ΠΈΠ΄Π΅Π½Ρ‚ΠΈΡ„ΠΈΠΊΠ°Ρ†ΠΈΠΈ ΠΈ восьми ΠΏΡ€ΠΎΠ³Ρ€Π°ΠΌΠΌΠΈΡ€ΡƒΠ΅ΠΌΡ‹ΠΌ Π²Ρ‹Ρ…ΠΎΠ΄Π°ΠΌ.

Π­Ρ‚Π° систСма ΠΌΠΎΠΆΠ΅Ρ‚ Π±Ρ‹Ρ‚ΡŒ Ρ€Π°ΡΡˆΠΈΡ€Π΅Π½Π° с 10 Π΄ΠΎ 56 Π·ΠΎΠ½. Π£Π·Π΅Π» Π°Π»ΡŒΡ‚Π΅Ρ€Π½Π°Ρ‚ΠΈΠ²Π½ΠΎΠ³ΠΎ ΠΈΠ΄Π΅Π½Ρ‚ΠΈΡ„ΠΈΠΊΠ°Ρ‚ΠΎΡ€Π° позволяСт ΠΈΡΠΏΠΎΠ»ΡŒΠ·ΠΎΠ²Π°Ρ‚ΡŒ Π°Π»ΡŒΡ‚Π΅Ρ€Π½Π°Ρ‚ΠΈΠ²Π½Ρ‹ΠΉ ΠΌΠ΅Ρ‚ΠΎΠ΄ настройки Π·ΠΎΠ½ ΠΈ ΠΌΠΎΠΆΠ΅Ρ‚ Π²ΠΌΠ΅ΡΡ‚ΠΈΡ‚ΡŒ любоС Π½ΠΎΡ€ΠΌΠ°Π»ΡŒΠ½ΠΎ Π·Π°ΠΊΡ€Ρ‹Ρ‚ΠΎΠ΅ устройство обнаруТСния Π² сочСтании с дискрСтным Π΄Π°Ρ‚Ρ‡ΠΈΠΊΠΎΠΌ ΠΈΠ΄Π΅Π½Ρ‚ΠΈΡ„ΠΈΠΊΠ°Ρ‚ΠΎΡ€Π°. Π”Π°Ρ‚Ρ‡ΠΈΠΊΠΈ ID ΠΏΠΎΠ΄ΠΊΠ»ΡŽΡ‡Π°ΡŽΡ‚ΡΡ ΠΏΠ°Ρ€Π°Π»Π»Π΅Π»ΡŒΠ½ΠΎ Ρ‡Π΅Ρ€Π΅Π· ΡΠ΅Π½ΡΠΎΡ€Π½ΡƒΡŽ линию, которая ΠΌΠΎΠΆΠ΅Ρ‚ Π±Ρ‹Ρ‚ΡŒ Π’-ΠΎΠ±Ρ€Π°Π·Π½ΠΎΠΉ, Π·Π²Π΅Π·Π΄ΠΎΠΎΠ±Ρ€Π°Π·Π½ΠΎΠΉ ΠΈΠ»ΠΈ ΠΊΠΎΠ»ΡŒΡ†Π΅Π²ΠΎΠΉ Ρ„ΠΎΡ€ΠΌΡ‹.КабСль являСтся Π΄Π²ΡƒΡ…ΠΆΠΈΠ»ΡŒΠ½Ρ‹ΠΌ Ρ‚ΠΎΠ»ΡŒΠΊΠΎ для измСрСния ΠΈΠ»ΠΈ Ρ‡Π΅Ρ‚Ρ‹Ρ€Π΅Ρ…ΠΆΠΈΠ»ΡŒΠ½Ρ‹ΠΌ для измСрСния ΠΈ ΠΏΠΎΠ΄Π°Ρ‡ΠΈ напряТСния. ВСхнология ID — это ΠΌΠ΅Ρ‚ΠΎΠ΄ обнаруТСния ΠΈ ΠΏΠ΅Ρ€Π΅Π΄Π°Ρ‡ΠΈ сигналов с использованиСм ΡƒΡΠΎΠ²Π΅Ρ€ΡˆΠ΅Π½ΡΡ‚Π²ΠΎΠ²Π°Π½Π½ΠΎΠ³ΠΎ ΠΊΡ€Π΅ΠΌΠ½ΠΈΠ΅Π²ΠΎΠ³ΠΎ ΠΌΠΈΠΊΡ€ΠΎΡ‡ΠΈΠΏΠ° Π² адрСсных Ρ‚ΠΎΡ‡ΠΊΠ°Ρ… обнаруТСния. Он обСспСчиваСт ΠΈΠ½Π΄ΠΈΠ²ΠΈΠ΄ΡƒΠ°Π»ΡŒΠ½ΡƒΡŽ ΠΈΠ΄Π΅Π½Ρ‚ΠΈΡ„ΠΈΠΊΠ°Ρ†ΠΈΡŽ ΠΊΠ°ΠΆΠ΄ΠΎΠ³ΠΎ извСщатСля Π² сСти, ΠΈΡΠΏΠΎΠ»ΡŒΠ·ΡƒΡ Ρ‚ΠΎΠ»ΡŒΠΊΠΎ Ρ‡Π΅Ρ‚Ρ‹Ρ€Π΅Ρ…ΠΆΠΈΠ»ΡŒΠ½Ρ‹ΠΉ кабСль для Π·Π°Ρ‰ΠΈΡ‚Ρ‹ ΠΎΡ‚ нСсанкционированного доступа. Π­Ρ‚ΠΈ ID-Π΄Π΅Ρ‚Π΅ΠΊΡ‚ΠΎΡ€Ρ‹ ΠΌΠΎΠ³ΡƒΡ‚ ΠΈΡΠΏΠΎΠ»ΡŒΠ·ΠΎΠ²Π°Ρ‚ΡŒΡΡ Π½Π° рядС ID-совмСстимых ΠΏΠ°Π½Π΅Π»Π΅ΠΉ управлСния, ΠΊΠΎΡ‚ΠΎΡ€Ρ‹Π΅ ΠΌΠΎΠ³ΡƒΡ‚ Π°Π½Π°Π»ΠΈΠ·ΠΈΡ€ΠΎΠ²Π°Ρ‚ΡŒ сигналы ΠΎΡ‚ любого устройства благодаря ΠΊΠΎΠ½Ρ„ΠΈΠ³ΡƒΡ€Π°Ρ†ΠΈΠΈ ΠΏΠ°Ρ€Π°Π»Π»Π΅Π»ΡŒΠ½ΠΎΠΉ ΠΏΡ€ΠΎΠ²ΠΎΠ΄ΠΊΠΈ.

Π’ настоящСС врСмя ΠΈΠ΄Π΅Π½Ρ‚ΠΈΡ„ΠΈΠΊΠ°Ρ†ΠΈΠΎΠ½Π½Ρ‹Π΅ Π΄Π°Ρ‚Ρ‡ΠΈΠΊΠΈ ΠΈΡΠΏΠΎΠ»ΡŒΠ·ΡƒΡŽΡ‚ΡΡ ΠΌΠ΅Π½Π΅Π΅ ΡˆΠΈΡ€ΠΎΠΊΠΎ, Ρ‡Π΅ΠΌ Ρ‚Ρ€Π°Π΄ΠΈΡ†ΠΈΠΎΠ½Π½Ρ‹Π΅ Π΄Π΅Ρ‚Π΅ΠΊΡ‚ΠΎΡ€Ρ‹, Π½ΠΎ ΠΎΠ½ΠΈ ΠΎΠ±Π»Π°Π΄Π°ΡŽΡ‚ большим ΠΏΠΎΡ‚Π΅Π½Ρ†ΠΈΠ°Π»ΠΎΠΌ Π½Π° Π±ΡƒΠ΄ΡƒΡ‰Π΅Π΅. Π’ любом случаС сущСствуСт огромная Π³ΠΈΠ±ΠΊΠΎΡΡ‚ΡŒ Π² стандартном ΠΌΠ΅Ρ‚ΠΎΠ΄Π΅ ΠΏΠΎΠ΄ΠΊΠ»ΡŽΡ‡Π΅Π½ΠΈΡ ΡƒΠ·Π»ΠΎΠ² (рис. 5.19).

Рисунок 5.19. Π°) схСма ΡƒΠ·Π»ΠΎΠ²ΠΎΠΉ систСмы; (b) Узловая ΠΊΠ»Π΅ΠΌΠΌΠ°, Π΄Π΅Ρ‚Π°Π»ΡŒ

КабСль ΠΌΠΎΠΆΠ΅Ρ‚ ΠΏΡ€Π΅Π΄ΡΡ‚Π°Π²Π»ΡΡ‚ΡŒ собой Π³ΠΈΡ€Π»ΡΠ½Π΄Π½ΡƒΡŽ Ρ†Π΅ΠΏΡŒ ΠΈ / ΠΈΠ»ΠΈ Π·Π²Π΅Π·Π΄ΠΎΠΎΠ±Ρ€Π°Π·Π½ΡƒΡŽ схСму с использованиСм Ρ‡Π΅Ρ‚Ρ‹Ρ€Π΅Ρ…ΠΆΠΈΠ»ΡŒΠ½ΠΎΠ³ΠΎ кабСля с ΠΏΠΎΠ΄ΠΊΠ»ΡŽΡ‡Π΅Π½Π½Ρ‹ΠΌΠΈ Π΄Π΅Ρ‚Π΅ΠΊΡ‚ΠΎΡ€Π°ΠΌΠΈ, ΠΊΠ°ΠΊ ΠΏΠΎΠΊΠ°Π·Π°Π½ΠΎ Π½Π° рисункС 5.20, с использованиСм ΠΎΠΊΠΎΠ½Π΅Ρ‡Π½ΠΎΠΉ ΠΈΠ»ΠΈ Π΄Π²ΡƒΡ…ΠΏΠΎΠ»ΡŽΡΠ½ΠΎΠΉ ΠΏΡ€ΠΎΠ²ΠΎΠ΄ΠΊΠΈ. Π­Ρ‚ΠΎ Π² Ρ€Π°Π²Π½ΠΎΠΉ стСпСни относится ΠΊ ΠΏΠ°Π½Π΅Π»ΠΈ управлСния, ΡƒΠ΄Π°Π»Π΅Π½Π½ΠΎΠΉ ΠΊΠ»Π°Π²ΠΈΠ°Ρ‚ΡƒΡ€Π΅ ΠΈΠ»ΠΈ ΠΊΠ»Π΅ΠΌΠΌΠ°ΠΌ LEC.

Рисунок 5.20. Π‘Ρ…Π΅ΠΌΠ° ΠΏΠΎΠ΄ΠΊΠ»ΡŽΡ‡Π΅Π½ΠΈΡ Π΄Π΅Ρ‚Π΅ΠΊΡ‚ΠΎΡ€Π° ΡƒΠ·Π»Π°

ЗамСчания ΠΏΠΎ ΠΏΠΎΠ΄ΠΊΠ»ΡŽΡ‡Π΅Π½ΠΈΡŽ ΠΎΠΊΠΎΠ½Π΅Ρ‡Π½ΠΎΠ³ΠΎ рСзистора:

β€’

Π½Π° Ρ‚Ρ€Π΅Π²ΠΎΠΆΠ½Ρ‹Ρ… ΠΊΠΎΠ½Ρ‚Π°ΠΊΡ‚Π°Ρ… Π΄Π΅Ρ‚Π΅ΠΊΡ‚ΠΎΡ€Π° Π΄ΠΎΠ»ΠΆΠ΅Π½ Π±Ρ‹Ρ‚ΡŒ установлСн ΡˆΡƒΠ½Ρ‚ΠΈΡ€ΡƒΡŽΡ‰ΠΈΠΉ рСзистор 4 кОм;

β€’

ΠΎΠΊΠΎΠ½Π΅Ρ‡Π½Ρ‹ΠΉ рСзистор 2 кОм Π΄ΠΎΠ»ΠΆΠ΅Π½ Π±Ρ‹Ρ‚ΡŒ установлСн Π² самой дальнСй Ρ‚ΠΎΡ‡ΠΊΠ΅;

β€’

сопротивлСниС ΠΊΠΎΠ½Ρ‚ΡƒΡ€Π° с Π·Π°ΠΊΠΎΡ€ΠΎΡ‡Π΅Π½Π½Ρ‹ΠΌ ΠΎΠΊΠΎΠ½Π΅Ρ‡Π½Ρ‹ΠΌ рСзистором Π΄ΠΎΠ»ΠΆΠ½ΠΎ Π±Ρ‹Ρ‚ΡŒ ΠΌΠ΅Π½Π΅Π΅ 100 Ом;

β€’

максимальноС количСство ΠΈΠ·Π²Π΅Ρ‰Π°Ρ‚Π΅Π»Π΅ΠΉ, Ρ€Π°Π·Ρ€Π΅ΡˆΠ΅Π½Π½ΠΎΠ΅ Π² ΠΎΠ΄Π½ΠΎΠΉ Ρ†Π΅ΠΏΠΈ, составляСт 10.

Π‘Ρ‚Π°Π½Π΄Π°Ρ€Ρ‚Π½Ρ‹Π΅ британскиС стандартныС условныС обозначСния ΠΏΡ€ΠΈΠΌΠ΅Π½ΡΡŽΡ‚ΡΡ ΠΊ цСпям. Π’Π°ΠΊΠΈΠΌ ΠΎΠ±Ρ€Π°Π·ΠΎΠΌ, ΠΊ ΠΊΠ°ΠΆΠ΄ΠΎΠΉ Ρ†Π΅ΠΏΠΈ ΠΌΠΎΠΆΠ½ΠΎ ΠΏΠΎΠ΄ΠΊΠ»ΡŽΡ‡ΠΈΡ‚ΡŒ Π΄ΠΎ 10 Π΄Π²Π΅Ρ€Π½Ρ‹Ρ… ΠΊΠΎΠ½Ρ‚Π°ΠΊΡ‚ΠΎΠ², Π½ΠΎ Ρ‚ΠΎΠ»ΡŒΠΊΠΎ ΠΎΠ΄ΠΈΠ½ Π΄Π°Ρ‚Ρ‡ΠΈΠΊ двиТСния, Ρ‚Π°ΠΊΠΎΠΉ ΠΊΠ°ΠΊ PIR.

Из-Π·Π° Ρ€Π°Π·Π»ΠΈΡ‡Π½Ρ‹Ρ… ΡƒΡ€ΠΎΠ²Π½Π΅ΠΉ оборудования ΠΈ Ρ€Π°Π·Π»ΠΈΡ‡Π½Ρ‹Ρ… срСдств сбора ΠΈΠ½Ρ„ΠΎΡ€ΠΌΠ°Ρ†ΠΈΠΈ ΠΈΠ· сСти идСнтификация Ρ†Π΅ΠΏΠΈ Ρ‚Π°ΠΊΠΆΠ΅ ΠΎΠ±ΡΠ·Π°Ρ‚Π΅Π»ΡŒΠ½ΠΎ Π΄ΠΎΠ»ΠΆΠ½Π° ΠΈΠ·ΠΌΠ΅Π½ΠΈΡ‚ΡŒΡΡ. Рассмотрим ΠΏΡ€ΠΈΠΌΠ΅Ρ€ с использованиСм LIM. КаТдой Ρ†Π΅ΠΏΠΈ назначаСтся ΡƒΠ½ΠΈΠΊΠ°Π»ΡŒΠ½Ρ‹ΠΉ Ρ‡Π΅Ρ‚Ρ‹Ρ€Π΅Ρ…Π·Π½Π°Ρ‡Π½Ρ‹ΠΉ ΠΈΠ΄Π΅Π½Ρ‚ΠΈΡ„ΠΈΠΊΠ°Ρ†ΠΈΠΎΠ½Π½Ρ‹ΠΉ Π½ΠΎΠΌΠ΅Ρ€, Π½ΠΎΠΌΠ΅Ρ€ LIM ΠΈ Π½ΠΎΠΌΠ΅Ρ€ Ρ†Π΅ΠΏΠΈ ΡΠ»Π΅Π΄ΡƒΡŽΡ‰ΠΈΠΌ ΠΎΠ±Ρ€Π°Π·ΠΎΠΌ:

1-я Ρ†ΠΈΡ„Ρ€Π° НомСр отдСлСния (1, 2, 3 ΠΈΠ»ΠΈ 4)
2-я ΠΈ 3-я Ρ†ΠΈΡ„Ρ€Ρ‹ НомСр LIM (ΠΎΡ‚ 01 Π΄ΠΎ 16)
4-я Ρ†ΠΈΡ„Ρ€Π° НомСр Ρ†Π΅ΠΏΠΈ (ΠΎΡ‚ 1 Π΄ΠΎ 5)

ΠŸΡ€ΠΈΠΌΠ΅Ρ€ .ЦСпь 3 Π½Π° Ρ‡Π΅Ρ‚Π²Π΅Ρ€Ρ‚ΠΎΠΌ LIM Π²Π΅Ρ‚Π²ΠΈ 2 ΠΎΠ±ΠΎΠ·Π½Π°Ρ‡Π΅Π½Π° ΠΊΠ°ΠΊ 2043.

LIM Π΄ΠΎΠ»ΠΆΠ½Ρ‹ Π±Ρ‹Ρ‚ΡŒ ΠΏΡ€ΠΎΠ½ΡƒΠΌΠ΅Ρ€ΠΎΠ²Π°Π½Ρ‹ ΠΏΠΎΡΠ»Π΅Π΄ΠΎΠ²Π°Ρ‚Π΅Π»ΡŒΠ½ΠΎ для ΠΊΠ°ΠΆΠ΄ΠΎΠ³ΠΎ отвСтвлСния, Ρ‚. Π•. ΠŸΠ΅Ρ€Π²Ρ‹ΠΉ LIM Π½Π° ΠΊΠ°ΠΆΠ΄ΠΎΠΌ ΠΎΡ‚Π²Π΅Ρ‚Π²Π»Π΅Π½ΠΈΠΈ ΠΈΠΌΠ΅Π΅Ρ‚ Π½ΠΎΠΌΠ΅Ρ€ 01. На ΠΏΠ΅Ρ‡Π°Ρ‚Π½ΠΎΠΉ ΠΏΠ»Π°Ρ‚Π΅ LIM Π΄ΠΎΠ»ΠΆΠ½Ρ‹ ΡΠΎΠ±Π»ΡŽΠ΄Π°Ρ‚ΡŒΡΡ Π²Ρ…ΠΎΠ΄Π½Ρ‹Π΅ ΠΈ Π²Ρ‹Ρ…ΠΎΠ΄Π½Ρ‹Π΅ соСдинСния. для достиТСния этой Ρ†Π΅Π»ΠΈ.

ΠŸΡ€ΠΈ использовании ΡƒΠ·Π»ΠΎΠ² сСтСвыС ΠΊΠ°Π±Π΅Π»ΠΈ ΡƒΠ·Π»ΠΎΠ² Π΄ΠΎΠ»ΠΆΠ½Ρ‹ Π±Ρ‹Ρ‚ΡŒ ΠΏΠΎΠ΄ΠΊΠ»ΡŽΡ‡Π΅Π½Ρ‹ ΠΊ ΡΠΎΠΎΡ‚Π²Π΅Ρ‚ΡΡ‚Π²ΡƒΡŽΡ‰ΠΈΠΌ ΠΊΠ»Π΅ΠΌΠΌΠ°ΠΌ, Π° Ρ†Π΅ΠΏΠΈ обнаруТСния Π΄ΠΎΠ»ΠΆΠ½Ρ‹ Π±Ρ‹Ρ‚ΡŒ ΠΏΠΎΠ΄ΠΊΠ»ΡŽΡ‡Π΅Π½Ρ‹ ΠΊ ΠΎΠ±ΠΎΠ·Π½Π°Ρ‡Π΅Π½Π½ΠΎΠΉ ΡΠΎΠΎΡ‚Π²Π΅Ρ‚ΡΡ‚Π²ΡƒΡŽΡ‰ΠΈΠΌ ΠΎΠ±Ρ€Π°Π·ΠΎΠΌ схСмС (рисунок 5.20). Π—Π°Ρ‚Π΅ΠΌ Π½Π΅ΠΎΠ±Ρ…ΠΎΠ΄ΠΈΠΌΠΎ ΡƒΡΡ‚Π°Π½ΠΎΠ²ΠΈΡ‚ΡŒ сСлСкторный ΠΏΠ΅Ρ€Π΅ΠΊΠ»ΡŽΡ‡Π°Ρ‚Π΅Π»ΡŒ ΡƒΠ·Π»Π° I / D.Π­Ρ‚ΠΎ Π΄ΠΎΠ»ΠΆΠ½ΠΎ Π±Ρ‹Ρ‚ΡŒ установлСно ΠΏΠΎΡΠ»Π΅Π΄ΠΎΠ²Π°Ρ‚Π΅Π»ΡŒΠ½ΠΎ, Ρ‡Ρ‚ΠΎΠ±Ρ‹ ΠΎΠ±Π»Π΅Π³Ρ‡ΠΈΡ‚ΡŒ поиск нСисправностСй ΠΈ ΠΏΡ€ΠΎΠ³Ρ€Π°ΠΌΠΌΠΈΡ€ΠΎΠ²Π°Π½ΠΈΠ΅. Π—Π°Ρ‚Π΅ΠΌ схСма ΠΌΠΎΠΆΠ΅Ρ‚ Π±Ρ‹Ρ‚ΡŒ установлСна ​​со ссылкой Π½Π° Ρ‚Π°Π±Π»ΠΈΡ†Ρƒ, Ρ‚Π°ΠΊΡƒΡŽ ​​как Π’Π°Π±Π»ΠΈΡ†Π° 5.6.

Π’Π°Π±Π»ΠΈΡ†Π° 5.6. ЦСпь ΠΈΠ΄Π΅Π½Ρ‚ΠΈΡ„ΠΈΠΊΠ°Ρ†ΠΈΠΈ ΡƒΠ·Π»Π°

9001

11 900

I / D ΡƒΠ·Π»Π° Π¦Π΅ΠΏΠΈ обнаруТСния
A B C D E F G 17 18 19 20 21 22 23 24
2 25 26 27 28 29 30 31 32
3 33 34 35 36 37 38 39 40
4 41 42 43 44 45 46 47 48
5 49 50 51 52 53 54 55 56

ПослС ΠΏΠ΅Ρ€Π²ΠΎΠ½Π°Ρ‡Π°Π»ΡŒΠ½ΠΎΠ³ΠΎ Π²ΠΊΠ»ΡŽΡ‡Π΅Π½ΠΈΡ питания ΠΈΠ»ΠΈ Π² нСустановлСнном состоянии ΠΈΠ½ΠΆΠ΅Π½Π΅Ρ€ систСмы ΠΎΡ…Ρ€Π°Π½Π½ΠΎΠΉ сигнализации ΠΌΠΎΠΆΠ΅Ρ‚ ΠΏΡ€ΠΎΠ»ΠΈΡΡ‚Π°Ρ‚ΡŒ мСню ΠΈΠ½ΠΆΠ΅Π½Π΅Ρ€Π°, ввСдя ΠΊΠΎΠ΄ ΠΈΠ½ΠΆΠ΅Π½Π΅Ρ€Π°.Π—Π°Ρ‚Π΅ΠΌ ΠΌΠΎΠΆΠ½ΠΎ Π·Π°ΠΏΡ€ΠΎΠ³Ρ€Π°ΠΌΠΌΠΈΡ€ΠΎΠ²Π°Ρ‚ΡŒ Ρ‚ΠΈΠΏΡ‹ Ρ†Π΅ΠΏΠ΅ΠΉ ΠΈ Π°Ρ‚Ρ€ΠΈΠ±ΡƒΡ‚Ρ‹ Π² соотвСтствии с Π·Π°Ρ‰ΠΈΡ‰Π°Π΅ΠΌΡ‹ΠΌ ΠΏΠΎΠΌΠ΅Ρ‰Π΅Π½ΠΈΠ΅ΠΌ.

Π’ ΠΏΠΎΠ»Π΅Π²Ρ‹Ρ… условиях доступСн ΡˆΠΈΡ€ΠΎΠΊΠΈΠΉ спСктр оборудования, ΡƒΠ΄ΠΎΠ²Π»Π΅Ρ‚Π²ΠΎΡ€ΡΡŽΡ‰Π΅Π³ΠΎ Ρ€Π°Π·Π»ΠΈΡ‡Π½Ρ‹ΠΌ ΠΌΠ΅Ρ‚ΠΎΠ΄Π°ΠΌ ΠΌΡƒΠ»ΡŒΡ‚ΠΈΠΏΠ»Π΅ΠΊΡΠΈΡ€ΠΎΠ²Π°Π½ΠΈΡ, ΠΈ ΠΈΡΠΏΠΎΠ»ΡŒΠ·ΡƒΠ΅ΠΌΡ‹Π΅ ΠΌΠΎΠ΄ΡƒΠ»ΠΈ Π±ΡƒΠ΄ΡƒΡ‚ ΠΎΠ±ΠΎΠ·Π½Π°Ρ‡Π°Ρ‚ΡŒΡΡ Ρ€Π°Π·Π½Ρ‹ΠΌΠΈ Ρ‚Π΅Ρ€ΠΌΠΈΠ½Π°ΠΌΠΈ ΠΈ ΠΎΠΏΡ€Π΅Π΄Π΅Π»ΡΡ‚ΡŒΡΡ ΠΏΠΎ-Ρ€Π°Π·Π½ΠΎΠΌΡƒ. ΠŸΡ€ΠΈΠΌΠ΅Ρ€Π°ΠΌΠΈ ΡΠ²Π»ΡΡŽΡ‚ΡΡ ΠΌΠΎΠ΄ΡƒΠ»ΠΈ ΠΌΡƒΠ»ΡŒΡ‚ΠΈΠΏΠ»Π΅ΠΊΡΠΈΡ€ΠΎΠ²Π°Π½Π½ΠΎΠ³ΠΎ Π²Π²ΠΎΠ΄Π° ΠΈ Π²Ρ‹Π²ΠΎΠ΄Π° (MIOM) ΠΈ ΠΌΠΎΠ΄ΡƒΠ»ΠΈ ΡƒΠ΄Π°Π»Π΅Π½Π½ΠΎΠ³ΠΎ Π²Π²ΠΎΠ΄Π° / Π²Ρ‹Π²ΠΎΠ΄Π° (RIO). ΠœΠ΅Ρ‚ΠΎΠ΄Ρ‹ сбора Π΄Π°Π½Π½Ρ‹Ρ… ΠΈ возмоТности Π±ΡƒΠ΄ΡƒΡ‚ Ρ€Π°Π·Π»ΠΈΡ‡Π°Ρ‚ΡŒΡΡ, Π½ΠΎ концСпция использования магистралСй Π΄Π°Π½Π½Ρ‹Ρ… распространяСтся ΠΏΠΎΠ²ΡΡŽΠ΄Ρƒ. Однако Π²Π°ΠΆΠ½ΠΎ ΠΎΠ±Ρ€Π°Ρ‰Π°Ρ‚ΡŒ Π²Π½ΠΈΠΌΠ°Π½ΠΈΠ΅ Π½Π° Π΄Π°Π½Π½Ρ‹Π΅ ΠΏΡ€ΠΎΠΈΠ·Π²ΠΎΠ΄ΠΈΡ‚Π΅Π»Π΅ΠΉ Π² ΠΎΡ‚Π½ΠΎΡˆΠ΅Π½ΠΈΠΈ кабСля, ΠΊΠΎΡ‚ΠΎΡ€Ρ‹ΠΉ Π±ΡƒΠ΄Π΅Ρ‚ ΠΈΡΠΏΠΎΠ»ΡŒΠ·ΠΎΠ²Π°Ρ‚ΡŒΡΡ, ΠΏΠΎΡΠΊΠΎΠ»ΡŒΠΊΡƒ ΠΏΡ€ΠΎΠ²ΠΎΠ΄ΠΊΠ° RS-485 для ΠΏΠΎΠ΄ΠΊΠ»ΡŽΡ‡Π΅Π½ΠΈΡ ΠΊΠ»Π°Π²ΠΈΠ°Ρ‚ΡƒΡ€Ρ‹ ΠΈ Ρ‚. Π”.ΠΈΠ»ΠΈ идСнтификационная ΠΏΡ€ΠΎΠ²ΠΎΠ΄ΠΊΠ° ΠΌΠΎΠΆΠ΅Ρ‚ Π²Ρ‹Π·Ρ‹Π²Π°Ρ‚ΡŒ ΠΎΠΏΡ€Π΅Π΄Π΅Π»Π΅Π½Π½Ρ‹ΠΉ Ρ‚ΠΈΠΏ, ΠΈ ΠΌΠΎΠΆΠ΅Ρ‚ ΠΎΠΊΠ°Π·Π°Ρ‚ΡŒΡΡ Π½Π΅Π²ΠΎΠ·ΠΌΠΎΠΆΠ½Ρ‹ΠΌ ΠΈΡΠΏΠΎΠ»ΡŒΠ·ΠΎΠ²Π°Ρ‚ΡŒ ΡΡ‚Π°Π½Π΄Π°Ρ€Ρ‚Π½ΡƒΡŽ ΡΠΈΠ³Π½Π°Π»ΡŒΠ½ΡƒΡŽ ΠΏΡ€ΠΎΠ²ΠΎΠ΄ΠΊΡƒ.

Tech Talk Π‘ΠΎΠΏΡ€ΠΎΡ‚ΠΈΠ²Π»Π΅Π½ΠΈΠ΅ ΠΊΠΎΠ½Ρ‚ΡƒΡ€Π° постоянного Ρ‚ΠΎΠΊΠ°
Π£ … — Π£Ρ‡Π΅Π±Π½Ρ‹ΠΉ институт CATV

Tech Talk
Π‘ΠΎΠΏΡ€ΠΎΡ‚ΠΈΠ²Π»Π΅Π½ΠΈΠ΅ ΠΊΠΎΠ½Ρ‚ΡƒΡ€Π° постоянного Ρ‚ΠΎΠΊΠ°
Π—Π½Π°Π΅Ρ‚Π΅ Π»ΠΈ Π²Ρ‹, ΠΊΠ°ΠΊ ΠΎΠΏΡ€Π΅Π΄Π΅Π»ΠΈΡ‚ΡŒ Π΄Π»ΠΈΠ½Ρƒ кабСля, ΠΈΡΠΏΠΎΠ»ΡŒΠ·ΡƒΡ Ρ‚ΠΎΠ»ΡŒΠΊΠΎ сопротивлСниС ΠΊΠΎΠ½Ρ‚ΡƒΡ€Π°?
Если Π½Π΅Ρ‚, я Π½Π°ΡƒΡ‡Ρƒ тСбя, ΠΊΠ°ΠΊ это Π΄Π΅Π»Π°Ρ‚ΡŒ.
Π‘ΠΎΠΏΡ€ΠΎΡ‚ΠΈΠ²Π»Π΅Π½ΠΈΠ΅ ΠΊΠΎΠ½Ρ‚ΡƒΡ€Π° постоянного Ρ‚ΠΎΠΊΠ°
Π‘ΠΎΠΏΡ€ΠΎΡ‚ΠΈΠ²Π»Π΅Π½ΠΈΠ΅ ΠΊΠΎΠ½Ρ‚ΡƒΡ€Π° постоянного Ρ‚ΠΎΠΊΠ° складываСтся ΠΈΠ· совокупного сопротивлСния постоянному Ρ‚ΠΎΠΊΡƒ ΠΊΠ°ΠΊ Π²Π½ΡƒΡ‚Ρ€Π΅Π½Π½Π΅Π³ΠΎ, Ρ‚Π°ΠΊ ΠΈ внСшнСго ΠΏΡ€ΠΎΠ²ΠΎΠ΄Π½ΠΈΠΊΠΎΠ².
Π˜Ρ‚Π°ΠΊ, для этого ΠΏΡ€ΠΈΠΌΠ΅Ρ€Π° ΠΌΡ‹ Π±ΡƒΠ΄Π΅ΠΌ ΠΈΡΠΏΠΎΠ»ΡŒΠ·ΠΎΠ²Π°Ρ‚ΡŒ кусок кабСля RG 6 ΠΎΡ‚ Commscope.
Π‘ΠΎΠΏΡ€ΠΎΡ‚ΠΈΠ²Π»Π΅Π½ΠΈΠ΅ ΠΊΠΎΠ½Ρ‚ΡƒΡ€Π° постоянного Ρ‚ΠΎΠΊΠ° ΠΎΠ±Ρ‹Ρ‡Π½ΠΎ измСряСтся Π² Ом / 1kft.
Π˜Ρ‚Π°ΠΊ, Ссли Ρƒ нас Π΅ΡΡ‚ΡŒ сопротивлСниС 37,17 Ом / 1k ft. Если ΠΌΡ‹ Ρ€Π°Π·Π΄Π΅Π»ΠΈΠΌ это Π½Π° Π΄Π΅ΡΡΡ‚ΡŒ, ΠΌΡ‹ ΠΏΠΎΠ»ΡƒΡ‡ΠΈΠΌ 3,717, это сопротивлСниС 100-Ρ„ΡƒΡ‚ΠΎΠ²ΠΎΠ³ΠΎ ΠΎΡ‚Ρ€Π΅Π·ΠΊΠ° rg 6, Ρ‚Π°ΠΊ Ρ‡Ρ‚ΠΎ Ρ‚Π΅ΠΏΠ΅Ρ€ΡŒ, ΠΊΠΎΠ³Π΄Π° ΠΌΡ‹ Π—Π½Π°ΠΉΡ‚Π΅, Ρ‡Ρ‚ΠΎ ΠΌΡ‹ ΠΌΠΎΠΆΠ΅ΠΌ Ρ€Π°ΡΡΡ‡ΠΈΡ‚Π°Ρ‚ΡŒ Π΄Π»ΠΈΠ½Ρƒ кабСля, просто ΠΈΠ·ΠΌΠ΅Ρ€ΠΈΠ² внСшний ΠΈ Π²Π½ΡƒΡ‚Ρ€Π΅Π½Π½ΠΈΠΉ ΠΏΡ€ΠΎΠ²ΠΎΠ΄Π½ΠΈΠΊΠΈ. Π’Π°ΠΊ, Π½Π°ΠΏΡ€ΠΈΠΌΠ΅Ρ€, Ссли Ρƒ вас Π΅ΡΡ‚ΡŒ сопротивлСниС 83 Ом, Π²Ρ‹ Π²Ρ‹Ρ‡Ρ‚ΠΈΡ‚Π΅ 75 ΠΈΠ· 83 (83-75) = 8, Π° Π·Π°Ρ‚Π΅ΠΌ Π²ΠΎΠ·ΡŒΠΌΠΈΡ‚Π΅ 8, Ρ€Π°Π·Π΄Π΅Π»Π΅Π½Π½Ρ‹Π΅ Π½Π° 3.7, ΠΏΠΎΡΠΊΠΎΠ»ΡŒΠΊΡƒ это сопротивлСниС ΠΎΡ‚Ρ€Π΅Π·ΠΊΠ° кабСля Π΄Π»ΠΈΠ½ΠΎΠΉ 100 Ρ„ΡƒΡ‚ΠΎΠ² 8/3.7 = 2,16 X100 = 216 Ρƒ вас Π±ΡƒΠ΄Π΅Ρ‚ ΠΎΡ‚Ρ€Π΅Π·ΠΎΠΊ кабСля Π΄Π»ΠΈΠ½ΠΎΠΉ 216 Ρ„ΡƒΡ‚ΠΎΠ². Однако сопротивлСниС измСняСтся Π² зависимости ΠΎΡ‚ Ρ‚Π΅ΠΌΠΏΠ΅Ρ€Π°Ρ‚ΡƒΡ€Ρ‹, Π΅ΡΡ‚ΡŒ Ρ‚Π΅ΠΌΠΏΠ΅Ρ€Π°Ρ‚ΡƒΡ€Π½Ρ‹ΠΉ коэффициСнт 1 + 0,0022 (T-68)

Π’ΠΎΡ‚ ΠΊΠ°ΠΊ Π²Ρ‹ ΠΌΠΎΠ³Π»ΠΈ Π±Ρ‹ ΠΎΠΏΡ€Π΅Π΄Π΅Π»ΠΈΡ‚ΡŒ ΠΏΠ°Π΄Π΅Π½ΠΈΠ΅ напряТСния Π½Π° ΠΊΠ°Π±Π΅Π»Π΅, Ссли Π±Ρ‹ ΡΠΎΠ±ΠΈΡ€Π°Π»ΠΈΡΡŒ ΡƒΡΡ‚Π°Π½ΠΎΠ²ΠΈΡ‚ΡŒ Π½ΠΎΠ²Ρ‹ΠΉ источник питания
Π‘ΠΎΠΏΡ€ΠΎΡ‚ΠΈΠ²Π»Π΅Π½ΠΈΠ΅ ΠΊΠΎΠ½Ρ‚ΡƒΡ€Π° постоянного Ρ‚ΠΎΠΊΠ°

Π‘ΠΎΠΏΡ€ΠΎΡ‚ΠΈΠ²Π»Π΅Π½ΠΈΠ΅ ΠΊΠΎΠ½Ρ‚ΡƒΡ€Π° постоянного Ρ‚ΠΎΠΊΠ° складываСтся ΠΈΠ· совокупного сопротивлСния постоянному Ρ‚ΠΎΠΊΡƒ ΠΊΠ°ΠΊ Π²Π½ΡƒΡ‚Ρ€Π΅Π½Π½Π΅Π³ΠΎ, Ρ‚Π°ΠΊ ΠΈ внСшнСго ΠΏΡ€ΠΎΠ²ΠΎΠ΄Π½ΠΈΠΊΠΎΠ². Π­Ρ‚ΠΎ

, ΠΎΠ±Ρ‹Ρ‡Π½ΠΎ ΡƒΠΊΠ°Π·Ρ‹Π²Π°Π΅ΠΌΠΎΠ΅ ΠΊΠ°ΠΊ Ом Π½Π° 1000 Ρ„ΡƒΡ‚ΠΎΠ², рассчитанноС ΠΏΡ€ΠΈ 680 F

Π‘ΠΎΠ»ΡŒΡˆΠΈΠ½ΡΡ‚Π²ΠΎ систСм кабСльного тСлСвидСния ΠΈ ΡˆΠΈΡ€ΠΎΠΊΠΎΠΏΠΎΠ»ΠΎΡΠ½ΠΎΠ³ΠΎ доступа Ρ€Π°Π±ΠΎΡ‚Π°ΡŽΡ‚ ΠΏΠΎ схСмС источника питания 60/90 Π’.Как Π²Π½ΡƒΡ‚Ρ€Π΅Π½Π½ΠΈΠΉ, Ρ‚Π°ΠΊ ΠΈ внСшний ΠΏΡ€ΠΎΠ²ΠΎΠ΄Π½ΠΈΠΊΠΈ

ΠΊΠΎΠ°ΠΊΡΠΈΠ°Π»ΡŒΠ½Ρ‹Ρ… ΠΊΠ°Π±Π΅Π»Π΅ΠΉ ΠΈΡΠΏΠΎΠ»ΡŒΠ·ΡƒΡŽΡ‚ΡΡ для ΠΏΠ΅Ρ€Π΅Π΄Π°Ρ‡ΠΈ этой энСргии. ΠŸΠΎΡΠΊΠΎΠ»ΡŒΠΊΡƒ это расстояниС ΠΌΠΎΠΆΠ΅Ρ‚ ΡΡ‚Π°Ρ‚ΡŒ довольно большим, сопротивлСниС ΠΎΠ±ΠΎΠΈΡ… ΠΏΡ€ΠΎΠ²ΠΎΠ΄ΠΎΠ²

способствуСт падСнию напряТСния. Богласно Π·Π°ΠΊΠΎΠ½Ρƒ Ома, Ссли извСстный Ρ‚ΠΎΠΊ ΠΏΡ€ΠΎΡ…ΠΎΠ΄ΠΈΡ‚ Ρ‡Π΅Ρ€Π΅Π· Ρ†Π΅ΠΏΡŒ с извСстным сопротивлСниСм

, ΠΊΠΎΠ½Π΅Ρ‡Π½Ρ‹ΠΌ Ρ€Π΅Π·ΡƒΠ»ΡŒΡ‚Π°Ρ‚ΠΎΠΌ являСтся ΠΏΠ°Π΄Π΅Π½ΠΈΠ΅ напряТСния. Рассматривая Π·Π°ΠΊΠΎΠ½ Ома, ΠΌΡ‹ Π½Π°Ρ…ΠΎΠ΄ΠΈΠΌ, Ρ‡Ρ‚ΠΎ:

E

I = — 1 = Ρ‚ΠΎΠΊ Π² Π°ΠΌΠΏΠ΅Ρ€Π°Ρ…

RE = напряТСниС Π² Π²ΠΎΠ»ΡŒΡ‚Π°Ρ…

R = сопротивлСниС Π² ΠΎΠΌΠ°Ρ…

ИспользованиС этой Ρ„ΠΎΡ€ΠΌΡƒΠ»Ρ‹ ΠΏΠΎΠΊΠ°Π·Ρ‹Π²Π°Π΅Ρ‚, Ρ‡Ρ‚ΠΎ ΠΊΠΎΠ³Π΄Π° 60 Π²ΠΎΠ»ΡŒΡ‚ ΠΏΡ€ΠΈΠ»ΠΎΠΆΠ΅Π½Π½Ρ‹ΠΉ ΠΊ рСзистору 75 Ом, ΠΎΠ½ Π±ΡƒΠ΄Π΅Ρ‚ ΠΏΡ€ΠΎΠΈΠ·Π²ΠΎΠ΄ΠΈΡ‚ΡŒ Ρ‚ΠΎΠΊ 0.80 Π°ΠΌΠΏΠ΅Ρ€.

Если ΠΊ ΠΊΠ°ΠΆΠ΄ΠΎΠΌΡƒ ΡΠΎΠ΅Π΄ΠΈΠ½ΠΈΡ‚Π΅Π»ΡŒΠ½ΠΎΠΌΡƒ ΠΏΡ€ΠΎΠ²ΠΎΠ΄Ρƒ Π² Ρ†Π΅ΠΏΠΈ (ΠΌΠ΅ΠΆΠ΄Ρƒ источником напряТСния ΠΈ источником питания) Π΄ΠΎΠ±Π°Π²ΠΈΡ‚ΡŒ сопротивлСниС 4 Ом, Ρ‚ΠΎ Π»Π΅Π³ΠΊΠΎ Π²Ρ‹Ρ‡ΠΈΡΠ»ΠΈΡ‚ΡŒ ΠΏΠ°Π΄Π΅Π½ΠΈΠ΅ напряТСния,

ΠžΠΏΡΡ‚ΡŒ ΠΆΠ΅, ΠΈΡΠΏΠΎΠ»ΡŒΠ·ΡƒΡ Ρ„ΠΎΡ€ΠΌΡƒΠ»Ρƒ Π·Π°ΠΊΠΎΠ½Π° Ома для расчСта Ρ‚ΠΎΠΊΠ°, ΠΌΡ‹ Ρ‚Π΅ΠΏΠ΅Ρ€ΡŒ производят Ρ‚ΠΎΠΊ 0,85 Π°ΠΌΠΏΠ΅Ρ€Π°. Π˜ΡΠΏΠΎΠ»ΡŒΠ·ΡƒΡ Ρ„ΠΎΡ€ΠΌΡƒΠ»Ρƒ напряТСния Π·Π°ΠΊΠΎΠ½Π° Ома, ΠΌΡ‹ Π½Π°Ρ…ΠΎΠ΄ΠΈΠΌ, Ρ‡Ρ‚ΠΎ:

E = 1 x R ΠΈΠ»ΠΈ 0,85 x 4 = 3,40 Π²ΠΎΠ»ΡŒΡ‚

НапряТСниС, ΠΏΡ€ΠΈΠ»ΠΎΠΆΠ΅Π½Π½ΠΎΠ΅ ΠΊ 75-ΠΎΠΌΠ½ΠΎΠΉ Π½Π°Π³Ρ€ΡƒΠ·ΠΊΠ΅, Π±ΡƒΠ΄Π΅Ρ‚ Ρ€Π°Π²Π½ΠΎ 3.На 4 Π²ΠΎΠ»ΡŒΡ‚Π° мСньшС, Ρ‡Π΅ΠΌ напряТСниС источника 60 Π²ΠΎΠ»ΡŒΡ‚, ΠΈΠ»ΠΈ 56,6 Π²ΠΎΠ»ΡŒΡ‚.
ΠŸΡ€Π΅Π΄Ρ‹Π΄ΡƒΡ‰Π°Ρ информация ΠΏΡ€Π΅Π΄Π»Π°Π³Π°Π΅Ρ‚ основы, Π½Π΅ΠΎΠ±Ρ…ΠΎΠ΄ΠΈΠΌΡ‹Π΅ для расчСта падСния напряТСния Π² Ρ‚Π΅Π»Π΅ΠΊΠΎΠΌΠΌΡƒΠ½ΠΈΠΊΠ°Ρ†ΠΈΠΎΠ½Π½ΠΎΠΉ систСмС Π½Π° 60 Π²ΠΎΠ»ΡŒΡ‚. Π’ Ρ€Π΅Π°Π»ΡŒΠ½ΠΎΠΉ систСмС Π±ΡƒΠ΄Π΅Ρ‚ нСсколько Π½Π°Π³Ρ€ΡƒΠ·ΠΎΠΊ ΠΈ нСсколько Π΄Π»ΠΈΠ½ ΠΊΠ°Π±Π΅Π»Π΅ΠΉ с Ρ€Π°Π·Π»ΠΈΡ‡Π½Ρ‹ΠΌ сопротивлСниСм ΠΊΠΎΠ½Ρ‚ΡƒΡ€Π° постоянного Ρ‚ΠΎΠΊΠ° для ΠΊΠ°ΠΆΠ΄ΠΎΠ³ΠΎ мСстополоТСния.
Π§Ρ‚ΠΎΠ±Ρ‹ Ρ€Π°ΡΡΡ‡ΠΈΡ‚Π°Ρ‚ΡŒ систСму питания, Π½Π΅ΠΎΠ±Ρ…ΠΎΠ΄ΠΈΠΌΠΎ Π·Π½Π°Ρ‚ΡŒ сопротивлСниС ΠΊΠΎΠ½Ρ‚ΡƒΡ€Π° постоянного Ρ‚ΠΎΠΊΠ° для кабСля. Но ΠΎΠΏΡΡ‚ΡŒ ΠΆΠ΅, ΠΈΡΠΏΠΎΠ»ΡŒΠ·ΡƒΡ Π·Π°ΠΊΠΎΠ½ Ома, Π»Π΅Π³ΠΊΠΎ Ρ€Π°ΡΡΡ‡ΠΈΡ‚Π°Ρ‚ΡŒ напряТСниС.

Π§Ρ‚ΠΎ Ρ‚Π°ΠΊΠΎΠ΅ ΠΏΡ€ΠΎΠ²Π΅Ρ€ΠΊΠ° сопротивлСния ΠΊΠΎΠ½Ρ‚ΡƒΡ€Π°?

ΠžΠ±Π·ΠΎΡ€

По Π΄Π°Π½Π½Ρ‹ΠΌ Π€Π΅Π΄Π΅Ρ€Π°Π»ΡŒΠ½ΠΎΠ³ΠΎ управлСния граТданской Π°Π²ΠΈΠ°Ρ†ΠΈΠΈ БША, ΠΏΡ€ΠΈΠΌΠ΅Ρ€Π½ΠΎ Ρ€Π°Π· Π² Π³ΠΎΠ΄ (ΠΊΠ°ΠΆΠ΄Ρ‹Π΅ 1000 часов ΠΏΠΎΠ»Π΅Ρ‚Π°) Π² самолСт ΠΏΠΎΡ€Π°ΠΆΠ°Π΅Ρ‚ молния.Π­Ρ‚ΠΎ происходит Ρ‡Π°Ρ‰Π΅, Ρ‡Π΅ΠΌ Π΄ΡƒΠΌΠ°Π΅Ρ‚ Π±ΠΎΠ»ΡŒΡˆΠΈΠ½ΡΡ‚Π²ΠΎ людСй, учитывая Π²ΠΎΠ·ΠΌΠΎΠΆΠ½ΠΎΡΡ‚ΡŒ катастрофы.

Π₯ΠΎΡ€ΠΎΡˆΠ°Ρ Π½ΠΎΠ²ΠΎΡΡ‚ΡŒ Π·Π°ΠΊΠ»ΡŽΡ‡Π°Π΅Ρ‚ΡΡ Π² Ρ‚ΠΎΠΌ, Ρ‡Ρ‚ΠΎ стандартныС самолСты ΠΏΡ€Π΅Π΄Π½Π°Π·Π½Π°Ρ‡Π΅Π½Ρ‹ для Π·Π°Ρ‰ΠΈΡ‚Ρ‹ ΠΎΡ‚ ΡƒΠ΄Π°Ρ€ΠΎΠ² ΠΌΠΎΠ»Π½ΠΈΠΉ. Π’Ρ‰Π°Ρ‚Π΅Π»ΡŒΠ½Π°Ρ конструкция обСспСчиваСт ΠΏΡƒΡ‚ΡŒ с Π½ΠΈΠ·ΠΊΠΈΠΌ сопротивлСниСм, позволяя Ρ‚ΠΎΠΊΡƒ Ρ‚Π΅Ρ‡ΡŒ ΠΎΡ‚ Ρ‚ΠΎΡ‡ΠΊΠΈ ΡƒΠ΄Π°Ρ€Π° Π΄ΠΎ хвоста, ΠΎΡ‚ΠΊΡƒΠ΄Π° ΠΎΠ½ ΠΌΠΎΠΆΠ΅Ρ‚ бСзопасно Π²Ρ‹ΠΉΡ‚ΠΈ.

ΠžΠ΄ΠΈΠ½ΠΎΡ‡Π½ΠΎΠ΅ соСдинСниС с высоким сопротивлСниСм становится фокусом Ρ‚ΠΎΠΊΠ° ΡƒΠ΄Π°Ρ€Π° ΠΌΠΎΠ»Π½ΠΈΠΈ, ΠΊΠΎΡ‚ΠΎΡ€Ρ‹ΠΉ пытаСтся ΡƒΠΉΡ‚ΠΈ, Ρ‡Ρ‚ΠΎ ΠΌΠΎΠΆΠ΅Ρ‚ привСсти ΠΊ катастрофС.

Π˜Ρ‚Π°ΠΊ, ΠΏΡ€ΠΈΠ½Ρ†ΠΈΠΏ, Π»Π΅ΠΆΠ°Ρ‰ΠΈΠΉ Π² основС конструкции, достаточно прост.Однако самолСты ΠΏΡ€Π΅Π΄ΡΡ‚Π°Π²Π»ΡΡŽΡ‚ собой слоТныС сборки мСханичСских ΠΈ элСктричСских ΠΊΠΎΠΌΠΏΠΎΠ½Π΅Π½Ρ‚ΠΎΠ²; Π‘ΡƒΡ‰Π΅ΡΡ‚Π²ΡƒΡŽΡ‚ тысячи Ρ‚ΠΎΡ‡Π΅ΠΊ соСдинСния ΠΈ Π·Π°Π·Π΅ΠΌΠ»ΡΡŽΡ‰ΠΈΡ… Ρ†Π΅ΠΏΠ΅ΠΉ, ΠΊΠΎΡ‚ΠΎΡ€Ρ‹Π΅ Π½Π΅ΠΎΠ±Ρ…ΠΎΠ΄ΠΈΠΌΠΎ ΠΏΡ€ΠΎΠ²Π΅Ρ€ΠΈΡ‚ΡŒ, Ρ‡Ρ‚ΠΎΠ±Ρ‹ Π³Π°Ρ€Π°Π½Ρ‚ΠΈΡ€ΠΎΠ²Π°Ρ‚ΡŒ Π½ΠΈΠ·ΠΊΠΎΠ΅ сопротивлСниС ΠΏΡƒΡ‚ΠΈ.

Π­Π»Π΅ΠΌΠ΅Π½Ρ‚Ρ‹, ΡΠΎΡΡ‚Π°Π²Π»ΡΡŽΡ‰ΠΈΠ΅ Ρ†Π΅ΠΏΡŒ соСдинСния, ΠΏΡ€Π΅Π΄ΡΡ‚Π°Π²Π»ΡΡŽΡ‚ собой ΠΊΠΎΠΌΠ±ΠΈΠ½Π°Ρ†ΠΈΡŽ сСкций аэроструктуры, корпусов Π»Π΅Ρ‚Π½ΠΎΠ³ΠΎ оборудования, экранов ΠΆΠ³ΡƒΡ‚ΠΎΠ² ΠΊΠ°Π±Π΅Π»Π΅ΠΉ, систСм Ρ‚Ρ€ΡƒΠ±ΠΎΠΏΡ€ΠΎΠ²ΠΎΠ΄ΠΎΠ² ΠΈ ΡΠΎΠ΅Π΄ΠΈΠ½ΠΈΡ‚Π΅Π»ΡŒΠ½Ρ‹Ρ… Π»Π΅Π½Ρ‚. Π­Ρ‚ΠΈ элСмСнты Π² совокупности ΠΎΠ±Π΅ΡΠΏΠ΅Ρ‡ΠΈΠ²Π°ΡŽΡ‚ ΠΏΡƒΡ‚ΡŒ с Π½ΠΈΠ·ΠΊΠΈΠΌ сопротивлСниСм, ΠΏΠΎ ΠΊΠΎΡ‚ΠΎΡ€ΠΎΠΌΡƒ ΠΌΠΎΠΆΠ΅Ρ‚ бСзопасно ΠΏΡ€ΠΎΡ…ΠΎΠ΄ΠΈΡ‚ΡŒ Ρ‚ΠΎΠΊ ΠΎΡ‚ ΠΌΠΎΠ»Π½ΠΈΠΈ.ИмСнно ΡΠ»ΠΎΠΆΠ½ΠΎΡΡ‚ΡŒ этих элСмСнтов Π΄Π΅Π»Π°Π΅Ρ‚ Ρ‚Π°ΠΊ Π²Π°ΠΆΠ½Ρ‹ΠΌΠΈ ΡΡ„Ρ„Π΅ΠΊΡ‚ΠΈΠ²Π½ΠΎΡΡ‚ΡŒ ΠΌΠ΅Ρ‚ΠΎΠ΄Π° тСстирования ΠΈ ΠΏΠΎΠ΄Ρ‚Π²Π΅Ρ€ΠΆΠ΄Π΅Π½ΠΈΠ΅ Ρ€Π΅Π·ΡƒΠ»ΡŒΡ‚Π°Ρ‚ΠΎΠ² тСстирования.

Как это ΠΌΠΎΠΆΠ΅Ρ‚ ΠΏΠΎΠΉΡ‚ΠΈ Π½Π΅ Ρ‚Π°ΠΊ?

ΠŸΡ€Π°Π²ΠΈΠ»ΡŒΠ½ΠΎΠ΅ элСктричСскоС соСдинСниС ΠΈΠΌΠ΅Π΅Ρ‚ Ρ€Π΅ΡˆΠ°ΡŽΡ‰Π΅Π΅ Π·Π½Π°Ρ‡Π΅Π½ΠΈΠ΅ для обСспСчСния бСзопасности самолСтов ΠΈ пассаТиров. Одно соСдинСниС с высоким сопротивлСниСм — это всС, Ρ‡Ρ‚ΠΎ Π½ΡƒΠΆΠ½ΠΎ, Ρ‡Ρ‚ΠΎΠ±Ρ‹ ΡΠ΄Π΅Π»Π°Ρ‚ΡŒ схСму Π·Π°Ρ‰ΠΈΡ‚Ρ‹ ΠΎΡ‚ ΡƒΠ΄Π°Ρ€Π° ΠΌΠΎΠ»Π½ΠΈΠΈ бСсполСзной.

Π₯ΡƒΠΆΠ΅ Ρ‚ΠΎΠ³ΠΎ, соСдинСниС с высоким сопротивлСниСм становится фокусом Ρ‚ΠΎΠΊΠ° ΠΌΠΎΠ»Π½ΠΈΠΈ Π΄ΠΎ 200 кА, ΠΊΠΎΡ‚ΠΎΡ€Ρ‹ΠΉ пытаСтся ΡƒΠΉΡ‚ΠΈ, Ρ‡Ρ‚ΠΎ ΠΌΠΎΠΆΠ΅Ρ‚ привСсти ΠΊ катастрофС.

БоСдинСния с высоким сопротивлСниСм ΠΌΠΎΠ³ΡƒΡ‚ Π±Ρ‹Ρ‚ΡŒ Π²Ρ‹Π·Π²Π°Π½Ρ‹ ΡΠ»Π΅Π΄ΡƒΡŽΡ‰ΠΈΠΌΠΈ ΠΏΡ€ΠΈΡ‡ΠΈΠ½Π°ΠΌΠΈ — ΠΈ ΠΈΡ… Π³ΠΎΡ€Π°Π·Π΄ΠΎ большС:

  • ЗагрязнСниС повСрхности
  • ΠΠ΅ΠΏΡ€Π°Π²ΠΈΠ»ΡŒΠ½ΠΎ ΠΏΠΎΠ΄Π³ΠΎΡ‚ΠΎΠ²Π»Π΅Π½Π½Ρ‹Π΅ склСиваСмыС повСрхности
  • ΠžΡ‚ΡΡƒΡ‚ΡΡ‚Π²ΡƒΡŽΡ‰ΠΈΠ΅ ΠΊΠΎΠΌΠΏΠΎΠ½Π΅Π½Ρ‚Ρ‹
  • НСисправныС ΠΌΠ°Ρ‚Π΅Ρ€ΠΈΠ°Π»Ρ‹
  • ΠžΡΠ»Π°Π±Π»Π΅Π½Π½Ρ‹Π΅ Π·Π°ΠΆΠΈΠΌΡ‹
  • ΠžΡΠ»Π°Π±Π»Π΅Π½Π½Ρ‹Π΅ ΠΊΠΎΠ»ΡŒΡ†Π΅Π²Ρ‹Π΅ ΠΊΠ»Π΅ΠΌΠΌΡ‹
  • ΠΠ΅ΠΏΡ€Π°Π²ΠΈΠ»ΡŒΠ½ΠΎ рассчитанныС ΡΠΎΠ΅Π΄ΠΈΠ½ΠΈΡ‚Π΅Π»ΡŒΠ½Ρ‹Π΅ Π»Π΅Π½Ρ‚Ρ‹

Как Π²Ρ‹ ΠΏΡ€ΠΎΠ²ΠΎΠ΄ΠΈΡ‚Π΅ испытаниС сопротивлСния соСдинСния ΠΈ ΠΏΠ΅Ρ‚Π»ΠΈ?

Как ΠΌΡ‹ ΠΈΠ΄Π΅Π½Ρ‚ΠΈΡ„ΠΈΡ†ΠΈΡ€ΡƒΠ΅ΠΌ эти соСдинСния с высоким сопротивлСниСм ΠΈ ΠΏΡ€Π΅Π΄ΠΎΡ‚Π²Ρ€Π°Ρ‰Π°Π΅ΠΌ ΠΈΡ… Π²Π½Π΅Π΄Ρ€Π΅Π½ΠΈΠ΅ Π² самолСт? ΠŸΡƒΡ‚Π΅ΠΌ внСдрСния эффСктивных ΠΌΠ΅Ρ‚ΠΎΠ΄ΠΎΠ² тСстирования с использованиСм ΡΠΎΠΎΡ‚Π²Π΅Ρ‚ΡΡ‚Π²ΡƒΡŽΡ‰Π΅Π³ΠΎ оборудования ΠΈ Ρ‚Π΅Ρ…Π½ΠΎΠ»ΠΎΠ³ΠΈΠΉ.

ΠŸΡ€ΠΎΠ²Π΅Ρ€ΠΊΠ° сопротивлСния ΠΊΠΎΠ½Ρ‚ΡƒΡ€Π° — Π²Π°ΠΆΠ½Ρ‹ΠΉ этап Π² производствС ΠΈ обслуТивании бСзопасных Ρ†Π΅ΠΏΠ΅ΠΉ соСдинСния; это Π½Π΅ Π΄ΠΎΠ»ΠΆΠ½ΠΎ Π±Ρ‹Ρ‚ΡŒ Π½Π΅ΡƒΠ΄ΠΎΠ±Π½ΠΎΠΉ Π·Π°Π΄Π΅Ρ€ΠΆΠΊΠΎΠΉ с Π²Π²ΠΎΠ΄ΠΎΠΌ Π²ΠΎΠ·Π΄ΡƒΡˆΠ½ΠΎΠ³ΠΎ судна Π² Π°ΠΊΡ‚ΠΈΠ²Π½ΠΎΠ΅ использованиС.

ΠŸΡ€ΠΎΡΡ‚Ρ‹Π΅ элСктричСскиС связи ΠΌΠ΅ΠΆΠ΄Ρƒ двумя дискрСтными элСмСнтами ΠΎΡ‚Π½ΠΎΡΠΈΡ‚Π΅Π»ΡŒΠ½ΠΎ Π»Π΅Π³ΠΊΠΎ ΠΏΡ€ΠΎΠ²Π΅Ρ€ΠΈΡ‚ΡŒ. ΠŸΡ€ΠΈΠΌΠ΅Π½ΡΡ ΠΏΡ€ΠΈΠ½Ρ†ΠΈΠΏ измСрСния КСльвина, ΠΈΠ·ΠΌΠ΅Ρ€ΠΈΡ‚Π΅Π»ΠΈ связи Π²Ρ‹Π·Ρ‹Π²Π°ΡŽΡ‚ ΠΏΡ€ΠΎΡ…ΠΎΠΆΠ΄Π΅Π½ΠΈΠ΅ Ρ‚ΠΎΠΊΠ° ΠΌΠ΅ΠΆΠ΄Ρƒ двумя элСмСнтами, ΠΈΠ·ΠΌΠ΅Ρ€ΡΡŽΡ‚ ΠΏΠ°Π΄Π΅Π½ΠΈΠ΅ напряТСния Π½Π° связи ΠΈ ΡΠΎΠΎΠ±Ρ‰Π°ΡŽΡ‚ ΠΎ сопротивлСнии.

ΠŸΠΎΠ΄Ρ…ΠΎΠ΄ΠΈΡ‚ Π»ΠΈ ΠΏΡ€ΠΈΠ½Ρ†ΠΈΠΏ измСрСния КСльвина для всСх Ρ†Π΅ΠΏΠ΅ΠΉ?

Π­Ρ‚ΠΎΡ‚ ΠΌΠ΅Ρ‚ΠΎΠ΄ Π½Π΅ ΠΏΠΎΠ΄Ρ…ΠΎΠ΄ΠΈΡ‚ для тСстирования Ρ†Π΅ΠΏΠ΅ΠΉ, содСрТащих ΠΏΠ°Ρ€Π°Π»Π»Π΅Π»ΡŒΠ½Ρ‹Π΅ ΠΏΡƒΡ‚ΠΈ; Π’ этом сцСнарии часто ΠΈΡΠΏΠΎΠ»ΡŒΠ·ΡƒΡŽΡ‚ΡΡ Π½Π΅ΠΏΡ€Π°Π²ΠΈΠ»ΡŒΠ½Ρ‹Π΅ ΠΌΠ΅Ρ‚ΠΎΠ΄Ρ‹ тСстирования.

Π’ качСствС ΠΏΡ€ΠΈΠΌΠ΅Ρ€Π° возьмСм ΠΈΠ»Π»ΡŽΡΡ‚Ρ€Π°Ρ†ΠΈΡŽ Π½ΠΈΠΆΠ΅ (рис. 1). Π”Π²Π΅ части конструкции самолСта соСдинСны сСриСй ΡΠΊΡ€Π΅ΠΏΠ»ΡΡŽΡ‰ΠΈΡ… Ρ€Π΅ΠΌΠ½Π΅ΠΉ; ΠΎΠ΄Π½Π° ΠΈΠ· Ρ‚ΠΎΡ‡Π΅ΠΊ соСдинСния ΠΏΠ»ΠΎΡ…ΠΎ смонтирована ΠΈ прСдставляСт собой Ρ€Π°Π·Ρ€Ρ‹Π² Ρ†Π΅ΠΏΠΈ. Если этот ΡƒΠ·Π΅Π» испытываСтся с использованиСм измСритСля сцСплСния, описанного Π²Ρ‹ΡˆΠ΅, ΠΏΠ°Ρ€Π°Π»Π»Π΅Π»ΡŒΠ½Ρ‹Π΅ ΠΏΡƒΡ‚ΠΈ сопротивлСния ΠΏΠΎΠ·Π²ΠΎΠ»ΡΡŽΡ‚ Ρ‚ΠΎΠΊΡƒ Ρ‚Π΅Ρ‡ΡŒ ΠΌΠ΅ΠΆΠ΄Ρƒ Π·ΠΎΠ½Π΄Π°ΠΌΠΈ измСритСля.

ПадСниС напряТСния измСряСтся, Π½ΠΎ это ΠΏΠ°Π΄Π΅Π½ΠΈΠ΅ фактичСски происходит Ρ‡Π΅Ρ€Π΅Π· ΠΏΠ°Ρ€Π°Π»Π»Π΅Π»ΡŒΠ½Ρ‹Π΅ ΡΠΎΠ΅Π΄ΠΈΠ½ΠΈΡ‚Π΅Π»ΡŒΠ½Ρ‹Π΅ ΠΏΠ΅Ρ€Π΅ΠΌΡ‹Ρ‡ΠΊΠΈ. Π’Π°ΠΊΠΈΠΌ ΠΎΠ±Ρ€Π°Π·ΠΎΠΌ, ΠΏΠΎΠ»ΡƒΡ‡Π΅Π½Π½ΠΎΠ΅ Π² Ρ€Π΅Π·ΡƒΠ»ΡŒΡ‚Π°Ρ‚Π΅ ΠΈΠ·ΠΌΠ΅Ρ€Π΅Π½Π½ΠΎΠ΅ сопротивлСниС являСтся суммой ΠΏΠ°Ρ€Π°Π»Π»Π΅Π»ΡŒΠ½Ρ‹Ρ… Ρ†Π΅ΠΏΠ΅ΠΉ сопротивлСния, Ρ‡Ρ‚ΠΎ позволяСт тСсту случайно Π·Π°Ρ€Π΅Π³ΠΈΡΡ‚Ρ€ΠΈΡ€ΠΎΠ²Π°Ρ‚ΡŒ Ρ€Π΅Π·ΡƒΠ»ΡŒΡ‚Π°Ρ‚ прохоТдСния тСста.

Π’ΠΎΠΊ разряда ΠΌΠΎΠ»Π½ΠΈΠΈ, ΠΏΡ€ΠΎΡ‚Π΅ΠΊΠ°ΡŽΡ‰ΠΈΠΉ Ρ‡Π΅Ρ€Π΅Π· эту систСму, поступаСт Π² Ρ€Π°Π·ΠΎΠΌΠΊΠ½ΡƒΡ‚ΡƒΡŽ Ρ†Π΅ΠΏΡŒ ΠΈΠ»ΠΈ соСдинСниС с высоким сопротивлСниСм, ΠΏΡ‹Ρ‚Π°ΡΡΡŒ Π²Ρ‹Π·Π²Π°Ρ‚ΡŒ силу Π΄ΠΎ 200 кА Π² соСдинСнии. Π Π΅Π·ΡƒΠ»ΡŒΡ‚Π°Ρ‚Ρ‹ ΠΌΠΎΠ³ΡƒΡ‚ Π±Ρ‹Ρ‚ΡŒ катастрофичСскими.

Рисунок 1: Π‘ΠΎΠ΅Π΄ΠΈΠ½ΠΈΡ‚Π΅Π»ΡŒΠ½Ρ‹Π΅ Π»Π΅Π½Ρ‚Ρ‹ с Ρ€Π°Π·ΠΎΠΌΠΊΠ½ΡƒΡ‚ΠΎΠΉ Ρ†Π΅ΠΏΡŒΡŽ

Рисунок 2: Π˜ΡΠΏΡ‹Ρ‚Π°Π½ΠΈΠ΅ ΠΏΠ΅Ρ‚Π»ΠΈ ΡΠΊΡ€Π΅ΠΏΠ»ΡΡŽΡ‰Π΅Π³ΠΎ рСмня

Π‘ΠΏΠ΅Ρ†ΠΈΠ°Π»ΡŒΠ½ΠΎ Ρ€Π°Π·Ρ€Π°Π±ΠΎΡ‚Π°Π½Π½Ρ‹ΠΉ тСстСр сопротивлСния соСдинСний ΠΈ ΠΊΠΎΠ½Ρ‚ΡƒΡ€ΠΎΠ²

Π‘Π°ΠΌΡ‹ΠΉ эффСктивный ΠΌΠ΅Ρ‚ΠΎΠ΄ ΠΏΡ€ΠΎΠ²Π΅Ρ€ΠΊΠΈ ΠΏΠ΅Ρ‚Π΅Π»ΡŒ соСдинСния — это использованиС ΡΠΏΠ΅Ρ†ΠΈΠ°Π»ΡŒΠ½ΠΎ Ρ€Π°Π·Ρ€Π°Π±ΠΎΡ‚Π°Π½Π½ΠΎΠΉ систСмы ΠΏΡ€ΠΎΠ²Π΅Ρ€ΠΊΠΈ ΠΏΠ΅Ρ‚Π΅Π»ΡŒ, ΠΊΠΎΡ‚ΠΎΡ€Ρ‹Ρ… Π½Π° Ρ€Ρ‹Π½ΠΊΠ΅ сущСствуСт нСсколько.Π­Ρ‚ΠΎΡ‚ ΠΌΠ΅Ρ‚ΠΎΠ΄ Π²ΠΊΠ»ΡŽΡ‡Π°Π΅Ρ‚ Π² сСбя ΠΏΠΎΠ΄Π°Ρ‡Ρƒ Ρ‚ΠΎΠΊΠ° Π² ΠΏΠ΅Ρ‚Π»ΡŽ с ΠΏΠΎΠΌΠΎΡ‰ΡŒΡŽ Π·Π°ΠΆΠΈΠΌΠΎΠ². Π’Π²ΠΎΠ΄ΠΈΠΌΡ‹ΠΉ Ρ‚ΠΎΠΊ Π·Π°Ρ‚Π΅ΠΌ измСряСтся ΠΏΠΎ ΠΌΠ΅Ρ€Π΅ Π΅Π³ΠΎ прохоТдСния Ρ‡Π΅Ρ€Π΅Π· ΠΊΠΎΠ½Ρ‚ΡƒΡ€. ΠšΠΎΠ½Ρ‚Ρ€ΠΎΠ»ΠΈΡ€ΡƒΠ΅Ρ‚ΡΡ напряТСниС, Π½Π΅ΠΎΠ±Ρ…ΠΎΠ΄ΠΈΠΌΠΎΠ΅ для протСкания Ρ‚ΠΎΠΊΠ°, ΠΈ рассчитываСтся ΠΏΠΎΠ»Π½ΠΎΠ΅ сопротивлСниС ΠΊΠΎΠ½Ρ‚ΡƒΡ€Π°.

Ѐазовая коррСкция примСняСтся для изоляции рСзистивного элСмСнта ΠΈ для опрСдСлСния сопротивлСния ΠΊΠ°ΠΆΠ΄ΠΎΠΉ ΠΎΡ‚Π΄Π΅Π»ΡŒΠ½ΠΎΠΉ ΠΏΠ΅Ρ‚Π»ΠΈ. Π’ ΠΏΡ€ΠΈΠΌΠ΅Ρ€Π΅, ΠΏΠΎΠΊΠ°Π·Π°Π½Π½ΠΎΠΌ Π²Ρ‹ΡˆΠ΅ (рисунок 2), систСма тСстирования ΠΊΠΎΠ½Ρ‚ΡƒΡ€Π° сообщит ΠΎ ΠΊΠΎΠ½Ρ‚ΡƒΡ€Π΅ ΠΊΠ°ΠΊ ΠΎΠ± Ρ€Π°Π·ΠΎΠΌΠΊΠ½ΡƒΡ‚ΠΎΠΉ Ρ†Π΅ΠΏΠΈ, Ρ‡Ρ‚ΠΎ ΠΏΠΎΠ·Π²ΠΎΠ»ΠΈΡ‚ ΠΈΠ½ΠΆΠ΅Π½Π΅Ρ€Π°ΠΌ ΠΈΡΠΏΡ€Π°Π²ΠΈΡ‚ΡŒ Π½Π΅ΠΈΡΠΏΡ€Π°Π²Π½ΠΎΡΡ‚ΡŒ.

Π”ΠΎΠΏΠΎΠ»Π½ΠΈΡ‚Π΅Π»ΡŒΠ½Π°Ρ Π»ΠΈΡ‚Π΅Ρ€Π°Ρ‚ΡƒΡ€Π°

ΠœΡ‹ ΠΏΡ€Π΅Π΄Π»Π°Π³Π°Π΅ΠΌ ряд инструмСнтов для ΠΏΡ€ΠΎΠ²Π΅Ρ€ΠΊΠΈ сцСплСния, подходящих для Ρ€Π°Π·Π»ΠΈΡ‡Π½Ρ‹Ρ… областСй примСнСния. Если Π²Ρ‹ Ρ…ΠΎΡ‚ΠΈΡ‚Π΅ ΡƒΠ·Π½Π°Ρ‚ΡŒ, ΠΊΠ°ΠΊ наши ΠΌΠΎΠ΄Π΅Π»ΠΈ ΡΡ€Π°Π²Π½ΠΈΠ²Π°ΡŽΡ‚ΡΡ со стандартным LRT Β«ΠΆΠ΅Π»Ρ‚ΠΎΠ³ΠΎ ящика», Ρ‚ΠΎ стоит ΠΏΡ€ΠΎΡ‡ΠΈΡ‚Π°Ρ‚ΡŒ Π½Π°ΡˆΡƒ ΡΡ€Π°Π²Π½ΠΈΡ‚Π΅Π»ΡŒΠ½ΡƒΡŽ ΡΡ‚Π°Ρ‚ΡŒΡŽ. Π•ΡΡ‚ΡŒ Ρ‚Π°ΠΊΠΆΠ΅ нСсколько статСй Π² Π±Π»ΠΎΠ³Π°Ρ…, Π² ΠΊΠΎΡ‚ΠΎΡ€Ρ‹Ρ… ΡΡ€Π°Π²Π½ΠΈΠ²Π°ΡŽΡ‚ΡΡ наши ExLRT ΠΈ LRT с Ρ‚ΠΎΡ‡ΠΊΠΈ зрСния Ρ€Π°Π·ΠΌΠ΅Ρ€Π° ΠΈ вСса, тСхничСских возмоТностСй ΠΈ соотвСтствия трСбованиям искробСзопасности.

Π€ΠΎΡ€ΠΌΡƒΠ»Π° ΠΏΡ€Π°Π²ΠΈΠ»Π° ΠΏΠ΅Ρ‚Π»ΠΈ ΠšΠΈΡ€Ρ…Π³ΠΎΡ„Π°

Π’ любой Β«ΠΏΠ΅Ρ‚Π»Π΅Β» Π·Π°ΠΌΠΊΠ½ΡƒΡ‚ΠΎΠΉ Ρ†Π΅ΠΏΠΈ ΠΌΠΎΠΆΠ΅Ρ‚ Π±Ρ‹Ρ‚ΡŒ любоС количСство элСмСнтов схСмы, Ρ‚Π°ΠΊΠΈΡ… ΠΊΠ°ΠΊ Π±Π°Ρ‚Π°Ρ€Π΅ΠΈ ΠΈ рСзисторы.Π‘ΡƒΠΌΠΌΠ° разностСй напряТСний Π½Π° всСх этих элСмСнтах схСмы Π΄ΠΎΠ»ΠΆΠ½Π° Π±Ρ‹Ρ‚ΡŒ Ρ€Π°Π²Π½Π° Π½ΡƒΠ»ΡŽ. Π­Ρ‚ΠΎ извСстно ΠΊΠ°ΠΊ ΠΏΡ€Π°Π²ΠΈΠ»ΠΎ ΠΏΠ΅Ρ‚Π»ΠΈ ΠšΠΈΡ€Ρ…Π³ΠΎΡ„Π°. Π Π°Π·Π½ΠΈΡ†Π° напряТСний измСряСтся Π² Π²ΠΎΠ»ΡŒΡ‚Π°Ρ… (Π’). Когда Ρ‚ΠΎΠΊ I Π² ΠΊΠΎΠ½Ρ‚ΡƒΡ€Π΅ ΡƒΠΊΠ°Π·Π°Π½ Π² АмпСрах (А), Π° сопротивлСниС элСмСнтов схСмы ΡƒΠΊΠ°Π·Π°Π½ΠΎ Π² Ом (Ом), Ρ€Π°Π·Π½ΠΎΡΡ‚ΡŒ напряТСний Π½Π° рСзисторС ΠΌΠΎΠΆΠ΅Ρ‚ Π±Ρ‹Ρ‚ΡŒ Π½Π°ΠΉΠ΄Π΅Π½Π° ΠΏΠΎ Ρ„ΠΎΡ€ΠΌΡƒΠ»Π΅.

Π’ = Ρ€Π°Π·Π½ΠΎΡΡ‚ΡŒ напряТСний (Π’ΠΎΠ»ΡŒΡ‚, Π’)

Π€ΠΎΡ€ΠΌΡƒΠ»Π° ΠΏΡ€Π°Π²ΠΈΠ»Π° ΠΏΠ΅Ρ‚Π»ΠΈ ΠšΠΈΡ€Ρ…Π³ΠΎΡ„Π° Вопросы:

1) ЦСпь Π½Π° рисункС Π½ΠΈΠΆΠ΅ состоит ΠΈΠ· Ρ‚Ρ€Π΅Ρ… рСзисторов ΠΈ источника напряТСния (Π±Π°Ρ‚Π°Ρ€Π΅ΠΈ).Π’ΠΎΠΊ Π² ΠΊΠΎΠ½Ρ‚ΡƒΡ€Π΅ I = +4,00 А ΠΏΠΎ часовой стрСлкС. БатарСя ΠΏΠΎΠ΄Π°Π΅Ρ‚ напряТСниС v b = 100,0 Π’. ЗначСния сопротивлСния для Π΄Π²ΡƒΡ… ΠΈΠ· Ρ‚Ρ€Π΅Ρ… рСзисторов ΠΏΡ€ΠΈΠ²Π΅Π΄Π΅Π½Ρ‹ Π½Π° рисункС. КакоС Π·Π½Π°Ρ‡Π΅Π½ΠΈΠ΅ ΠΈΠΌΠ΅Π΅Ρ‚ рСзистор R 3 ?

ΠžΡ‚Π²Π΅Ρ‚: ΠŸΡ€Π°Π²ΠΈΠ»ΠΎ ΠΏΠ΅Ρ‚Π»ΠΈ ΠšΠΈΡ€Ρ…Π³ΠΎΡ„Π° гласит, Ρ‡Ρ‚ΠΎ сумма разностСй напряТСний Π²ΠΎΠΊΡ€ΡƒΠ³ ΠΏΠ΅Ρ‚Π»ΠΈ Π΄ΠΎΠ»ΠΆΠ½Π° Π±Ρ‹Ρ‚ΡŒ Ρ€Π°Π²Π½Π° Π½ΡƒΠ»ΡŽ. Π§Ρ‚ΠΎΠ±Ρ‹ Π½Π°ΠΉΡ‚ΠΈ сумму, Π½ΡƒΠΆΠ½ΠΎ Π²Ρ‹Π±Ρ€Π°Ρ‚ΡŒ Π½Π°ΠΏΡ€Π°Π²Π»Π΅Π½ΠΈΠ΅ двиТСния. НаправлСниС ΠΏΠΎΠ»ΠΎΠΆΠΈΡ‚Π΅Π»ΡŒΠ½ΠΎΠ³ΠΎ Ρ‚ΠΎΠΊΠ° задаСтся ΠΏΠΎ часовой стрСлкС, поэтому ΠΏΡ€ΠΎΡ‰Π΅ всСго ΠΈΡΠΏΠΎΠ»ΡŒΠ·ΠΎΠ²Π°Ρ‚ΡŒ Π΅Π³ΠΎ ΠΊΠ°ΠΊ Π½Π°ΠΏΡ€Π°Π²Π»Π΅Π½ΠΈΠ΅ двиТСния, Ρ‡Ρ‚ΠΎΠ±Ρ‹ Π½Π°ΠΉΡ‚ΠΈ сумму.Π˜ΡΡ‚ΠΎΡ‡Π½ΠΈΠΊ напряТСния ΠΈΠ»ΠΈ батарСя слСва Π½Π° рисункС ΠΈΠΌΠ΅Π΅Ρ‚ ΠΏΠΎΠ»ΠΎΠΆΠΈΡ‚Π΅Π»ΡŒΠ½ΠΎΠ΅ Π·Π½Π°Ρ‡Π΅Π½ΠΈΠ΅ напряТСния ΠΏΠΎ часовой стрСлкС. Π’Ρ€ΠΈ рСзистора Π²Ρ‹Π·Ρ‹Π²Π°ΡŽΡ‚ ΠΏΠ°Π΄Π΅Π½ΠΈΠ΅ напряТСния Π² этом Π½Π°ΠΏΡ€Π°Π²Π»Π΅Π½ΠΈΠΈ. Π’Π΅Π»ΠΈΡ‡ΠΈΠ½Π° ΠΏΠ°Π΄Π΅Π½ΠΈΠΉ напряТСния Ρ€Π°Π²Π½Π° ΡΠΎΠΏΡ€ΠΎΡ‚ΠΈΠ²Π»Π΅Π½ΠΈΡŽ, ΡƒΠΌΠ½ΠΎΠΆΠ΅Π½Π½ΠΎΠΌΡƒ Π½Π° Ρ‚ΠΎΠΊ Π² ΠΊΠΎΠ½Ρ‚ΡƒΡ€Π΅. Π‘ΡƒΠΌΠΌΠ° разностСй напряТСний:

Π—Π½Π°Ρ‡Π΅Π½ΠΈΠ΅ Ρ‚Ρ€Π΅Ρ‚ΡŒΠ΅Π³ΠΎ рСзистора ΠΌΠΎΠΆΠ½ΠΎ Π½Π°ΠΉΡ‚ΠΈ, пСрСставив Ρ„ΠΎΡ€ΠΌΡƒΠ»Ρƒ Π²Ρ‹ΡˆΠ΅:

НоминальноС сопротивлСниС рСзистора R 3 составляСт (Ом).

2) ЦСпь Π½Π° рисункС Π½ΠΈΠΆΠ΅ состоит ΠΈΠ· Ρ‚Ρ€Π΅Ρ… рСзисторов ΠΈ источника напряТСния (Π±Π°Ρ‚Π°Ρ€Π΅ΠΈ). Π’ΠΎΠΊ Π² ΠΊΠΎΠ½Ρ‚ΡƒΡ€Π΅ I = +10,0 мА (ΠΌΠΈΠ»Π»ΠΈΠ°ΠΌΠΏΠ΅Ρ€) ΠΏΡ€ΠΎΡ‚ΠΈΠ² часовой стрСлки. ЗначСния для Ρ‚Ρ€Π΅Ρ… рСзисторов ΠΏΡ€ΠΈΠ²Π΅Π΄Π΅Π½Ρ‹ Π½Π° рисункС Π² Π΅Π΄ΠΈΠ½ΠΈΡ†Π°Ρ… ΠΊΠΈΠ»ΠΎΠΎΠΌ (Π½ΠΎΠΌΠΈΠ½Π°Π» рСзистора R 3 Ρ€Π°Π²Π΅Π½). КакоС напряТСниС ( Π’ b ) Π΄ΠΎΠ»ΠΆΠ½ΠΎ ΠΏΠΎΠ΄Π°Π²Π°Ρ‚ΡŒΡΡ Π½Π° аккумулятор?

ΠžΡ‚Π²Π΅Ρ‚: ΠŸΡ€Π°Π²ΠΈΠ»ΠΎ ΠΏΠ΅Ρ‚Π»ΠΈ ΠšΠΈΡ€Ρ…Π³ΠΎΡ„Π° гласит, Ρ‡Ρ‚ΠΎ сумма разностСй напряТСний Π²ΠΎΠΊΡ€ΡƒΠ³ ΠΏΠ΅Ρ‚Π»ΠΈ Π΄ΠΎΠ»ΠΆΠ½Π° Π±Ρ‹Ρ‚ΡŒ Ρ€Π°Π²Π½Π° Π½ΡƒΠ»ΡŽ.Π§Ρ‚ΠΎΠ±Ρ‹ Π½Π°ΠΉΡ‚ΠΈ сумму, Π½ΡƒΠΆΠ½ΠΎ Π²Ρ‹Π±Ρ€Π°Ρ‚ΡŒ Π½Π°ΠΏΡ€Π°Π²Π»Π΅Π½ΠΈΠ΅ двиТСния. НаправлСниС ΠΏΠΎΠ»ΠΎΠΆΠΈΡ‚Π΅Π»ΡŒΠ½ΠΎΠ³ΠΎ Ρ‚ΠΎΠΊΠ° задаСтся ΠΏΡ€ΠΎΡ‚ΠΈΠ² часовой стрСлки, поэтому Π΅Π³ΠΎ ΠΏΡ€ΠΎΡ‰Π΅ всСго ΠΈΡΠΏΠΎΠ»ΡŒΠ·ΠΎΠ²Π°Ρ‚ΡŒ Π² качСствС направлСния двиТСния, Ρ‡Ρ‚ΠΎΠ±Ρ‹ Π½Π°ΠΉΡ‚ΠΈ сумму. Π˜ΡΡ‚ΠΎΡ‡Π½ΠΈΠΊ напряТСния ΠΈΠ»ΠΈ батарСя слСва Π½Π° рисункС ΠΈΠΌΠ΅Π΅Ρ‚ ΠΏΠΎΠ»ΠΎΠΆΠΈΡ‚Π΅Π»ΡŒΠ½ΠΎΠ΅ Π·Π½Π°Ρ‡Π΅Π½ΠΈΠ΅ напряТСния Π² Π½Π°ΠΏΡ€Π°Π²Π»Π΅Π½ΠΈΠΈ ΠΏΡ€ΠΎΡ‚ΠΈΠ² часовой стрСлки. Π’Ρ€ΠΈ рСзистора Π²Ρ‹Π·Ρ‹Π²Π°ΡŽΡ‚ ΠΏΠ°Π΄Π΅Π½ΠΈΠ΅ напряТСния Π² этом Π½Π°ΠΏΡ€Π°Π²Π»Π΅Π½ΠΈΠΈ. Π’Π΅Π»ΠΈΡ‡ΠΈΠ½Π° ΠΏΠ°Π΄Π΅Π½ΠΈΠΉ напряТСния Ρ€Π°Π²Π½Π° ΡΠΎΠΏΡ€ΠΎΡ‚ΠΈΠ²Π»Π΅Π½ΠΈΡŽ, ΡƒΠΌΠ½ΠΎΠΆΠ΅Π½Π½ΠΎΠΌΡƒ Π½Π° Ρ‚ΠΎΠΊ Π² ΠΊΠΎΠ½Ρ‚ΡƒΡ€Π΅, поэтому ΠΈΡ… сумма Π΄ΠΎΠ»ΠΆΠ½Π° Π±Ρ‹Ρ‚ΡŒ Ρ‚ΠΎΠΉ ΠΆΠ΅ Π²Π΅Π»ΠΈΡ‡ΠΈΠ½Ρ‹, Ρ‡Ρ‚ΠΎ ΠΈ напряТСниС ΠΎΡ‚ Π±Π°Ρ‚Π°Ρ€Π΅ΠΈ.Π‘ΡƒΠΌΠΌΠ° разностСй напряТСний Π² Π²Ρ‹Π±Ρ€Π°Π½Π½ΠΎΠΌ Π½Π°ΠΏΡ€Π°Π²Π»Π΅Π½ΠΈΠΈ двиТСния составляСт:

Π§Ρ‚ΠΎΠ±Ρ‹ ΡƒΠΌΠ½ΠΎΠΆΠΈΡ‚ΡŒ значСния Π² ΠΏΡ€ΠΈΠ²Π΅Π΄Π΅Π½Π½ΠΎΠΉ Π²Ρ‹ΡˆΠ΅ Ρ„ΠΎΡ€ΠΌΡƒΠ»Π΅, Π½Π΅ΠΎΠ±Ρ…ΠΎΠ΄ΠΈΠΌΠΎ ΠΏΡ€Π΅ΠΎΠ±Ρ€Π°Π·ΠΎΠ²Π°Ρ‚ΡŒ значСния Ρ‚ΠΎΠΊΠ° ΠΈ сопротивлСния Π² Π±Π°Π·ΠΎΠ²Ρ‹Π΅ Π΅Π΄ΠΈΠ½ΠΈΡ†Ρ‹. Для Ρ‚ΠΎΠΊΠ° 1000 ΠΌΠΈΠ»Π»ΠΈΠ°ΠΌΠΏΠ΅Ρ€ равняСтся 1 Π°ΠΌΠΏΠ΅Ρ€Ρƒ (1000 мА = 1 А), Π° для сопротивлСния 1 ΠΊΠΈΠ»ΠΎΠΎΠΌ Ρ€Π°Π²Π΅Π½ 1000 Ом (). Π€ΠΎΡ€ΠΌΡƒΠ»Π° ΠΏΡ€ΠΈΠ½ΠΈΠΌΠ°Π΅Ρ‚ ΡΠ»Π΅Π΄ΡƒΡŽΡ‰ΠΈΠΉ Π²ΠΈΠ΄:

НапряТСниС ΠΎΡ‚ Π±Π°Ρ‚Π°Ρ€Π΅ΠΈ ΠΌΠΎΠΆΠ½ΠΎ Π½Π°ΠΉΡ‚ΠΈ, пСрСставив Ρ„ΠΎΡ€ΠΌΡƒΠ»Ρƒ Π²Ρ‹ΡˆΠ΅:

НапряТСниС, ΠΏΠΎΠ΄Π°Π²Π°Π΅ΠΌΠΎΠ΅ Π±Π°Ρ‚Π°Ρ€Π΅Π΅ΠΉ V b , составляСт 240 Π’ (Π’ΠΎΠ»ΡŒΡ‚).

Π”ΠΎΠ±Π°Π²Π»Π΅Π½ΠΈΠ΅ сопротивлСния Π² ΠΎΠ΄Π½Ρƒ ΠΏΠ΅Ρ‚Π»ΡŽ

ЭлСктричСская Ρ†Π΅ΠΏΡŒ

ЭлСктричСство ΠΈ ΠΌΠ°Π³Π½Π΅Ρ‚ΠΈΠ·ΠΌ

Π”ΠΎΠ±Π°Π²Π»Π΅Π½ΠΈΠ΅ сопротивлСния Π² ΠΎΠ΄Π½Ρƒ ΠΏΠ΅Ρ‚Π»ΡŽ

ΠŸΠΎΠ²Π΅ΡΡ‚Π²ΠΎΠ²Π°Π½ΠΈΠ΅ ΠΎ Ρ„ΠΈΠ·ΠΈΠΊΠ΅
для 14-16

Π‘ΠΎΠ»ΡŒΡˆΠ΅ рСзисторов Π² ΠΎΠ΄Π½ΠΎΠΌ ΡˆΠ»Π΅ΠΉΡ„Π΅ — ΠΏΠΎΡΡ‚Π°Ρ€Π°ΠΉΡ‚Π΅ΡΡŒ ΠΈΠ΄Π΅Π½Ρ‚ΠΈΡ„ΠΈΡ†ΠΈΡ€ΠΎΠ²Π°Ρ‚ΡŒ ΠΊΠ°ΠΆΠ΄Ρ‹ΠΉ

Π‘ΠΎΠΏΡ€ΠΎΡ‚ΠΈΠ²Π»Π΅Π½ΠΈΠ΅ прСпятствуСт Ρ‚ΠΎΠΊΡƒ, поэтому Π΄ΠΎΠ±Π°Π²Π»Π΅Π½ΠΈΠ΅ большСго сопротивлСния ΠΏΡ€ΠΈΠ²ΠΎΠ΄ΠΈΡ‚ ΠΊ ΠΌΠ΅Π½ΡŒΡˆΠ΅ΠΌΡƒ Ρ‚ΠΎΠΊΡƒ.Π’Π°ΠΊ Ρ‡Ρ‚ΠΎ, Π²ΠΎΠ·ΠΌΠΎΠΆΠ½ΠΎ, ΠΊΠ°ΠΊ ΠΈ слСдовало ΠΎΠΆΠΈΠ΄Π°Ρ‚ΡŒ, Π΄ΠΎΠ±Π°Π²Π»Π΅Π½ΠΈΠ΅ Π²Ρ‚ΠΎΡ€ΠΎΠ³ΠΎ рСзистора ΠΊ ΠΎΠ΄Π½ΠΎΠΌΡƒ ΠΊΠΎΠ½Ρ‚ΡƒΡ€Ρƒ всСгда ΡƒΠ²Π΅Π»ΠΈΡ‡ΠΈΠ²Π°Π΅Ρ‚ сопротивлСниС ΠΊΠΎΠ½Ρ‚ΡƒΡ€Π° ΠΈ, Ρ‚Π°ΠΊΠΈΠΌ ΠΎΠ±Ρ€Π°Π·ΠΎΠΌ, ΡƒΠΌΠ΅Π½ΡŒΡˆΠ°Π΅Ρ‚ Ρ‚ΠΎΠΊ (ΠΏΡ€ΠΈ условии, Ρ‡Ρ‚ΠΎ Ρ€Π°Π·Π½ΠΎΡΡ‚ΡŒ ΠΏΠΎΡ‚Π΅Π½Ρ†ΠΈΠ°Π»ΠΎΠ² остаСтся ΠΏΡ€Π΅ΠΆΠ½Π΅ΠΉ — Π²Ρ‹ Π½Π΅ мСняли Π±Π°Ρ‚Π°Ρ€Π΅ΡŽ).

Π‘ΠΎΠΏΡ€ΠΎΡ‚ΠΈΠ²Π»Π΅Π½ΠΈΠ΅ Π² ΠΏΠ΅Ρ‚Π»Π΅ ΠΌΠΎΠΆΠ½ΠΎ ΠΈΠ·ΠΌΠ΅Π½ΡΡ‚ΡŒ, добавляя Π² ΠΏΠ΅Ρ‚Π»ΡŽ всС большС ΠΈ большС кусочков ΠΌΠ°Ρ‚Π΅Ρ€ΠΈΠ°Π»Π° (см. ΠŸΠΎΠ΄Ρ…ΠΎΠ΄Ρ‹ ΠΊ ΠΎΠ±ΡƒΡ‡Π΅Π½ΠΈΡŽ), Π½ΠΎ Π²Ρ‹, скорСС всСго, ΠΈΠ·ΠΌΠ΅Π½ΠΈΡ‚Π΅ сопротивлСниС, Π΄ΠΎΠ±Π°Π²ΠΈΠ² Π΄ΠΎΠΏΠΎΠ»Π½ΠΈΡ‚Π΅Π»ΡŒΠ½Ρ‹Π΅ рСзисторы. ΠœΡ‹ всСгда Π±ΡƒΠ΄Π΅ΠΌ Π½Π°Π·Ρ‹Π²Π°Ρ‚ΡŒ ΠΈΡ… R 1 ΠΈ R 2 , Π·Π°Ρ€Π΅Π·Π΅Ρ€Π²ΠΈΡ€ΠΎΠ²Π°Π² R для ΠΎΠ±Ρ‰Π΅Π³ΠΎ сопротивлСния Π² Ρ†Π΅ΠΏΠΈ.

Но Ρ‡Ρ‚ΠΎ происходит с Ρ€Π°Π·Π½ΠΎΡΡ‚ΡŒΡŽ ΠΏΠΎΡ‚Π΅Π½Ρ†ΠΈΠ°Π»ΠΎΠ²? ΠžΠΏΡΡ‚ΡŒ ΠΆΠ΅, Π²Π°ΠΌ Π½ΡƒΠΆΠ½ΠΎ Π±Ρ‹Ρ‚ΡŒ остороТным, ΠΏΠΎΡ‚ΠΎΠΌΡƒ Ρ‡Ρ‚ΠΎ Ρ‚Π΅ΠΏΠ΅Ρ€ΡŒ Π²Ρ‹ ΠΌΠΎΠΆΠ΅Ρ‚Π΅ ΠΈΠ·ΠΌΠ΅Ρ€ΠΈΡ‚ΡŒ Ρ‚Ρ€ΠΈ ΠΏΠΎΡ‚Π΅Π½Ρ†ΠΈΠ°Π»ΡŒΠ½Ρ‹Ρ… Ρ€Π°Π·Π½ΠΈΡ†Ρ‹: V 1 ΠΏΠΎ R 1 , V 2 ΠΏΠΎ R 2 ΠΈ V для разности ΠΏΠΎΡ‚Π΅Π½Ρ†ΠΈΠ°Π»ΠΎΠ² Π²ΠΎ всСм, ΠΈ, ΡΠ»Π΅Π΄ΠΎΠ²Π°Ρ‚Π΅Π»ΡŒΠ½ΠΎ, разности ΠΏΠΎΡ‚Π΅Π½Ρ†ΠΈΠ°Π»ΠΎΠ², обСспСчиваСмой аккумулятором.

Π’ΠΎΠΊ Ρ‚ΠΎΠ»ΡŒΠΊΠΎ ΠΎΠ΄ΠΈΠ½ — ΠΎΠ½ Π²Π΅Π·Π΄Π΅ ΠΎΠ΄ΠΈΠ½Π°ΠΊΠΎΠ²Ρ‹ΠΉ Π² ΠΎΠ΄Π½ΠΎΠΌ ΡˆΠ»Π΅ΠΉΡ„Π΅.

Анализ Ρ†Π΅ΠΏΠΈ Ρ‚ΠΎΠ»ΡŒΠΊΠΎ с ΠΏΠΎΡΠ»Π΅Π΄ΠΎΠ²Π°Ρ‚Π΅Π»ΡŒΠ½Ρ‹ΠΌΠΈ соСдинСниями

Π’ΠΎΡ‚ ΠΊΠ°ΠΊ Ρ€Π°Π·ΠΎΠ±Ρ€Π°Ρ‚ΡŒ Ρ‚Π°ΠΊΡƒΡŽ ​​схСму, идСализируя Π΅Π΅ Ρ‚Π°ΠΊ, Ρ‡Ρ‚ΠΎΠ±Ρ‹ Π½ΠΈ ΡΠΎΠ΅Π΄ΠΈΠ½ΠΈΡ‚Π΅Π»ΡŒΠ½Ρ‹Π΅ ΠΏΡ€ΠΎΠ²ΠΎΠ΄Π°, Π½ΠΈ аккумулятор Π½Π΅ ΠΈΠΌΠ΅Π»ΠΈ сопротивлСния. ΠžΠ±Ρ‰Π΅Π΅ сопротивлСниС Π² Ρ†Π΅ΠΏΠΈ — это просто сумма ΠΎΡ‚Π΄Π΅Π»ΡŒΠ½Ρ‹Ρ… сопротивлСний: R = R 1 & plus; Π  2 .

Π—Π°Ρ‚Π΅ΠΌ Π²Ρ‹ ΠΌΠΎΠΆΠ΅Ρ‚Π΅ Ρ€Π°ΡΡΡ‡ΠΈΡ‚Π°Ρ‚ΡŒ Ρ‚ΠΎΠΊ Π² ΠΊΠΎΠ½Ρ‚ΡƒΡ€Π΅, Π·Π°ΠΌΠ΅Π½ΠΈΠ² Π΄Π²Π° рСзистора ΠΎΠ΄Π½ΠΈΠΌ эквивалСнтным рСзистором (Π»ΠΈΠ±ΠΎ пСрСрисуйтС схСму, Π»ΠΈΠ±ΠΎ ΠΏΡ€Π΅Π΄ΡΡ‚Π°Π²ΡŒΡ‚Π΅, Ρ‡Ρ‚ΠΎ это дСлаСтся):

I = Π’ R

Π’Π΅ΠΏΠ΅Ρ€ΡŒ вСрнСмся ΠΊ исходной схСмС.Π’Ρ‹ ΠΌΠΎΠΆΠ΅Ρ‚Π΅ Ρ€Π°ΡΡΡ‡ΠΈΡ‚Π°Ρ‚ΡŒ Ρ€Π°Π·Π½ΠΎΡΡ‚ΡŒ ΠΏΠΎΡ‚Π΅Π½Ρ†ΠΈΠ°Π»ΠΎΠ² Π½Π° ΠΊΠ°ΠΆΠ΄ΠΎΠΌ рСзисторС, ΠΈΡΠΏΠΎΠ»ΡŒΠ·ΡƒΡ ΡΠΎΠΎΡ‚Π½ΠΎΡˆΠ΅Π½ΠΈΠ΅ ΠΌΠ΅ΠΆΠ΄Ρƒ Π’ 1 , R 1 ΠΈ I ΠΈΠ»ΠΈ Π’ 2 , R 2 ΠΈ I .

Π’ 1 = R 1 Γ— I

Π’ 2 = R 2 Γ— I

Π­Ρ‚ΠΈ Π΄Π²Π° Π½Π°Π±ΠΎΡ€Π° Π²Π΅Π»ΠΈΡ‡ΠΈΠ½ связаны ΠΌΠ΅ΠΆΠ΄Ρƒ собой ΠΎΠΏΡ€Π΅Π΄Π΅Π»Π΅Π½Π½Ρ‹ΠΌ ΠΎΠ±Ρ€Π°Π·ΠΎΠΌ, ΠΊΠ°ΠΊ ΠΏΠΎΠΊΠ°Π·Π°Π½ΠΎ Π² этой ΠΏΠ°Ρ€Π΅ ΡƒΡ€Π°Π²Π½Π΅Π½ΠΈΠΉ.Они Π²Π·Π°ΠΈΠΌΠ½ΠΎ ΠΎΠ³Ρ€Π°Π½ΠΈΡ‡Π΅Π½Ρ‹. Π’Π°ΠΊΠΈΠΌ ΠΎΠ±Ρ€Π°Π·ΠΎΠΌ, ΠΌΠΎΠΆΠ½ΠΎ Π½Π°Π·Π²Π°Ρ‚ΡŒ эти Π²ΠΈΠ΄Ρ‹ ΠΎΡ‚Π½ΠΎΡˆΠ΅Π½ΠΈΠΉ ΠΎΠ³Ρ€Π°Π½ΠΈΡ‡Π΅Π½ΠΈΠΉ ΠΎΡ‚Π½ΠΎΡˆΠ΅Π½ΠΈΠΉ. Они Π²Π΅Ρ€Π½Ρ‹ Π² любой ΠΌΠΎΠΌΠ΅Π½Ρ‚, нСзависимо ΠΎΡ‚ истории Ρ†Π΅ΠΏΠΈ. ΠžΡ‚Π½ΠΎΡˆΠ΅Π½ΠΈΡ Π½Π΅ ΠΏΡ€Π΅Π΄ΠΏΠΎΠ»Π°Π³Π°ΡŽΡ‚ ΡΠ²ΠΎΠ»ΡŽΡ†ΠΈΠΈ с Ρ‚Π΅Ρ‡Π΅Π½ΠΈΠ΅ΠΌ Π²Ρ€Π΅ΠΌΠ΅Π½ΠΈ.

Раскраска ΠΏΠΎΠΌΠΎΠΆΠ΅Ρ‚ Π²Ρ‹ΡΠ²ΠΈΡ‚ΡŒ различия

Разности ΠΏΠΎΡ‚Π΅Π½Ρ†ΠΈΠ°Π»ΠΎΠ² ΠΏΠΎΠΊΠ°Π·Ρ‹Π²Π°ΡŽΡ‚, ΠΊΡƒΠ΄Π° Π±ΡƒΠ΄Π΅Ρ‚ ΡΠ΄Π²ΠΈΠ³Π°Ρ‚ΡŒΡΡ энСргия Π² Ρ€Π΅Π·ΡƒΠ»ΡŒΡ‚Π°Ρ‚Π΅ дСйствия Ρ‚ΠΎΠΊΠ° Π² этой части Ρ†Π΅ΠΏΠΈ. Π£Π²ΠΈΠ΄Π΅Ρ‚ΡŒ различия ΠΌΠΎΠΆΠ½ΠΎ, раскрасив всС ΠΏΡ€ΠΎΠ²ΠΎΠ΄Π°.

Π Π°Π·Π½ΠΈΡ†Π° высот, прСдставлСнная Π½Π° ΠΊΠ°Ρ€Ρ‚Π°Ρ…, выполняСт Π°Π½Π°Π»ΠΎΠ³ΠΈΡ‡Π½ΡƒΡŽ Ρ„ΡƒΠ½ΠΊΡ†ΠΈΡŽ для пСрСмСщСния энСргии ΠΈΠ· вашСго химичСского Ρ…Ρ€Π°Π½ΠΈΠ»ΠΈΡ‰Π° Π² Π³Ρ€Π°Π²ΠΈΡ‚Π°Ρ†ΠΈΠΎΠ½Π½Ρ‹ΠΉ, ΠΊΠΎΠ³Π΄Π° Π²Ρ‹ ΠΏΠΎΠ΄Π½ΠΈΠΌΠ°Π΅Ρ‚Π΅ΡΡŒ Π½Π° Ρ…ΠΎΠ»ΠΌ. ΠŸΡ€Π΅Π΄ΡΠΊΠ°Π·Π°Ρ‚ΡŒ, Π³Π΄Π΅ находятся эти различия, ΠΏΠΎΠΌΠΎΠ³Π°Π΅Ρ‚ Ρ€Π°ΡΠΊΡ€Π°ΡˆΠΈΠ²Π°Π½ΠΈΠ΅ ΠΊΠ°Ρ€Ρ‚ ΠΏΠΎ высотС. ΠžΠΏΡΡ‚ΡŒ ΠΆΠ΅, Ρ‚ΠΎΠ»ΡŒΠΊΠΎ Ρ€Π°Π·Π½ΠΈΡ†Π° Π² высотС ΠΈΠΌΠ΅Π΅Ρ‚ ΠΊΠ°ΠΊΠΎΠ΅-Π»ΠΈΠ±ΠΎ влияниС Π½Π° ΡΠΌΠ΅Ρ‰Π΅Π½Π½ΡƒΡŽ ΡΠ½Π΅Ρ€Π³ΠΈΡŽ.

Раскраска ΠΏΠΎ высотС ΠΏΠΎΠ»Π΅Π·Π½Π° Ρ‚ΠΎΠ»ΡŒΠΊΠΎ ΠΏΠΎΡΡ‚ΠΎΠ»ΡŒΠΊΡƒ, ΠΏΠΎΡΠΊΠΎΠ»ΡŒΠΊΡƒ ΠΎΠ½Π° ΠΏΠΎΠΊΠ°Π·Ρ‹Π²Π°Π΅Ρ‚, Π³Π΄Π΅ происходят измСнСния высоты. Π’Π°ΠΊΠΈΠΌ ΠΎΠ±Ρ€Π°Π·ΠΎΠΌ, Ρƒ вас Π±ΡƒΠ΄Π΅Ρ‚ Π²ΠΈΠ·ΡƒΠ°Π»ΡŒΠ½ΠΎΠ΅ прСдставлСниС ΠΎ Ρ‚ΠΎΠΌ, ΠΊΠ°ΠΊ измСняСтся высота ΠΏΠΎ ΠΌΠ΅Ρ€Π΅ продвиТСния ΠΏΠΎ ΠΏΡƒΡ‚ΠΈ.Π­Ρ‚ΠΈ измСнСния ΠΏΡ€Π΅Π΄ΡƒΠΏΡ€Π΅ΠΆΠ΄Π°ΡŽΡ‚ вас ΠΎ Ρ‚ΠΎΠΌ, Ρ‡Ρ‚ΠΎ вас ΠΆΠ΄Π΅Ρ‚ Π²ΠΏΠ΅Ρ€Π΅Π΄ΠΈ, поэтому Π²Ρ‹ ΠΌΠΎΠΆΠ΅Ρ‚Π΅ ΠΏΡ€Π΅Π΄ΡΠΊΠ°Π·Π°Ρ‚ΡŒ, ΠΊΡƒΠ΄Π° ΠΈ сколько Π±ΡƒΠ΄Π΅Ρ‚ смСщСна энСргия.

ΠžΠΏΡ€Π΅Π΄Π΅Π»Π΅Π½ΠΈΠ΅ Ρ€Π°Π·Π½ΠΈΡ†Ρ‹ ΠΏΠΎΡ‚Π΅Π½Ρ†ΠΈΠ°Π»ΠΎΠ²

Π Π°ΡΠΊΡ€Π°ΡˆΠΈΠ²Π°Π½ΠΈΠ΅ Ρ€Π°Π·Π»ΠΈΡ‡Π½Ρ‹Ρ… участков ΠΏΡ€ΠΈΠ½Ρ†ΠΈΠΏΠΈΠ°Π»ΡŒΠ½Ρ‹Ρ… схСм ΠΏΠΎΠ·Π²ΠΎΠ»ΠΈΡ‚ Π²Π°ΠΌ ΡƒΠ²ΠΈΠ΄Π΅Ρ‚ΡŒ, Π° Π·Π°Ρ‚Π΅ΠΌ Π²Ρ‹Ρ‡ΠΈΡΠ»ΠΈΡ‚ΡŒ различия. По этим разностям ΠΏΠΎΡ‚Π΅Π½Ρ†ΠΈΠ°Π»ΠΎΠ² ΠΌΠΎΠΆΠ½ΠΎ Ρ€Π°ΡΡΡ‡ΠΈΡ‚Π°Ρ‚ΡŒ ΡΠΌΠ΅Ρ‰Π΅Π½Π½ΡƒΡŽ ΡΠ½Π΅Ρ€Π³ΠΈΡŽ. Π­Ρ‚ΠΈ ΡˆΠ°Π±Π»ΠΎΠ½Ρ‹ ΠΏΠΎΠΊΠ°Π·Ρ‹Π²Π°ΡŽΡ‚, Ρ‡Ρ‚ΠΎ Π±ΡƒΠ΄Π΅Ρ‚ Π΄Π΅Π»Π°Ρ‚ΡŒ схСма.

Π”ΠΎΠ±Π°Π²ΠΈΡ‚ΡŒ ΠΊΠΎΠΌΠΌΠ΅Π½Ρ‚Π°Ρ€ΠΈΠΉ

Π’Π°Ρˆ адрСс email Π½Π΅ Π±ΡƒΠ΄Π΅Ρ‚ ΠΎΠΏΡƒΠ±Π»ΠΈΠΊΠΎΠ²Π°Π½. ΠžΠ±ΡΠ·Π°Ρ‚Π΅Π»ΡŒΠ½Ρ‹Π΅ поля ΠΏΠΎΠΌΠ΅Ρ‡Π΅Π½Ρ‹ *